You are on page 1of 412

1

Geometry 2

TERMINOLOGY

Congruent: Two figures are congruent if they have the are equal and corresponding sides are in the
same size and shape. They are identical in every way same ratio
Polygon: A polygon is a closed plane figure with Vertex: A vertex is a corner of a figure (vertices is plural,
straight sides meaning more than one vertex)
Similar: Two figures are similar if they have the same
shape but a different size. Corresponding angles
Chapter 1 Geometry 2 3

INTRODUCTION
YOU STUDIED GEOMETRY IN the Preliminary Course. In this
chapter, you will revise this work and extend it to include some
more general applications of geometrical properties involving
polygons.
You will also use the Preliminary topic on straight-line
graphs to explore coordinate methods in geometry.

Plane Figure Geometry


Here is a summary of the geometry you studied in the Preliminary Course.

Vertically opposite angles

+AEC and +DEB are called vertically opposite angles. +AED and +CEB are
also vertically opposite angles.

Vertically opposite angles are equal.

Parallel lines

If the lines are parallel, then alternate angles are equal.

If the lines are parallel, then corresponding angles are equal.


4 Maths In Focus Mathematics HSC Course

If the lines are parallel, cointerior angles are supplementary (i.e. their
sum is 180).

TESTS FOR PARALLEL LINES


If alternate angles are equal, then the lines are parallel.

If +AEF = +EFD, then


AB || CD.

If corresponding angles are equal, then the lines are parallel.

If +BEF = +DFG, then


AB || CD.

If cointerior angles are supplementary, then the lines are parallel.

If +BEF + +DFE = 180c, then


AB || CD.

Angle sum of a triangle

The sum of the interior angles in any triangle is 180,


that is, a + b + c = 180
Chapter 1 Geometry 2 5

Exterior angle of a triangle

The exterior angle in any triangle is equal to the sum of the two
opposite interior angles. That is,
x+y=z

Congruent triangles

Two triangles are congruent if they are the same shape and size. All pairs of
corresponding sides and angles are equal.
For example:

We write ABC / XYZ.

TESTS
To prove that two triangles are congruent, we only need to prove that certain
combinations of sides or angles are equal.

Two triangles are congruent if


SSS: all three pairs of corresponding sides are equal
SAS: two pairs of corresponding sides and their included angles are equal The included angle
is the angle between
AAS: two pairs of angles and one pair of corresponding sides are equal the 2 sides.
RHS: both have a right angle, their hypotenuses are equal and one
other pair of corresponding sides are equal

Similar triangles

Triangles, for example ABC and XYZ, are similar if they are the same shape but
different sizes.
As in the example, all three pairs of corresponding angles are equal.
All three pairs of corresponding sides are in proportion (in the same ratio).
6 Maths In Focus Mathematics HSC Course

We write: ABC <; XYZ


XYZ is three times larger than ABC
XY 6
= =3
AB 2
XZ 12
= =3
AC 4
This shows that all 3 pairs of YZ 15
sides are in proportion. = =3
BC 5
XY XZ YZ
` = =
AB AC BC

TESTS
There are three tests for similar triangles.

If 2 pairs of angles are equal


then the third pair must also Two triangles are similar if:
be equal. three pairs of corresponding angles are equal
three pairs of corresponding sides are in proportion
two pairs of sides are in proportion and their included angles
are equal

Ratios of intercepts

When two (or more) transversals cut a series of parallel lines, the ratios of
their intercepts are equal.
That is, AB: BC = DE: EF
AB DE
or =
BC EF

Pythagoras theorem

The square on the hypotenuse in any right angled triangle is equal to the
sum of the squares on the other two sides.
That is, c2 = a2 + b2
or c = a2 + b2
Chapter 1 Geometry 2 7

Quadrilaterals

A quadrilateral is any four-sided figure

In any quadrilateral the sum of the interior angles is 360

PARALLELOGRAM

A parallelogram is a quadrilateral with opposite sides parallel

Properties of a parallelogram:
opposite sides of a parallelogram are equal
opposite angles of a parallelogram are equal
diagonals in a parallelogram bisect each other These properties can all be
each diagonal bisects the parallelogram into two congruent triangles proved.

TESTS
A quadrilateral is a parallelogram if:
both pairs of opposite sides are equal
both pairs of opposite angles are equal
one pair of sides is both equal and parallel
the diagonals bisect each other
8 Maths In Focus Mathematics HSC Course

RECTANGLE

If one angle is a right angle,


then you can prove all angles
are right angles. A rectangle is a parallelogram with one angle a right angle

Properties of a rectangle:
the same as for a parallelogram, and also
diagonals are equal

TEST
A quadrilateral is a rectangle if its diagonals are equal

Application

Builders use the property of equal diagonals to check if a rectangle is accurate.


For example, a timber frame may look rectangular, but may be slightly slanting.
Checking the diagonals makes sure that a building does not end up like the
Leaning Tower of Pisa!

RHOMBUS

It can be proved that all


sides are equal. A rhombus is a parallelogram with a pair of adjacent sides equal

Properties of a rhombus:
the same as for parallelogram, and also
diagonals bisect at right angles
diagonals bisect the angles of the rhombus
Chapter 1 Geometry 2 9

TESTS
A quadrilateral is a rhombus if:
all sides are equal
diagonals bisect each other at right angles

SQUARE

A square is a rectangle with a pair of adjacent sides equal

Properties of a square:
the same as for rectangle, and also
diagonals are perpendicular
diagonals make angles of 45 with the sides

TRAPEZIUM

A trapezium is a quadrilateral with one pair of sides parallel

KITE

A kite is a quadrilateral with two pairs of adjacent sides equal


10 Maths In Focus Mathematics HSC Course

Polygons

A polygon is a plane figure with straight sides

A regular polygon has all sides and all interior angles equal

The sum of the interior angles of an n-sided polygon is


given by
S = (n 2) 180

The sum of the exterior angles of any polygon is 360

Areas

Most areas of plane figures come from the area of a rectangle.

RECTANGLE

A = lb

SQUARE

A square is a special
rectangle.
A = x2

TRIANGLE

The area of a triangle is half


the area of a rectangle.
1
A= bh
2
Chapter 1 Geometry 2 11

PARALLELOGRAM

The area of a parallelogram


is the same as the area of
two triangles.
A = bh

RHOMBUS

1
A= xy
2
(x and y are lengths of diagonals)

TRAPEZIUM

1
A= h(a + b)
2

CIRCLE

You will study the circle in


more detail. See Chapter 5.

A = r2

The following examples and exercises use these results to prove properties of
plane figures.
12 Maths In Focus Mathematics HSC Course

EXAMPLES

1. Prove +A = +C in parallelogram ABCD.

Solution
Let +A = x
Then +B = 180 x (+A, +B cointerior angles, AD < BC)
+C = 180 (180 x) (+B, +C cointerior angles, AB < DC)
= 180 180 + x
= x
` +A = +C

2. Triangle ABC below is isosceles with AB = AC. Prove that the base
angles of ABC are equal by showing that ABD and ACD are congruent.

Solution
+ADB = +ADC = 90 (given)
AB = AC (given)
AD is common
\ ABD / ACD (RHS)
So +ABD = +ACD (corresponding angles in congruent s)
\ base angles are equal
Chapter 1 Geometry 2 13

3. Prove that opposite sides in a parallelogram are equal.

Solution

Let ABCD be any parallelogram and draw in diagonal AC.


+DAC = +ACB (alternate +s, AD < BC)
+BAC = +ACD (alternate +s, AB < DC)
AC is common.
` ABC / ADC (AAS)
` AB = DC (corresponding sides in congruent s)
AD = BC (similarly)
` opposite sides in a parallelogram are equal

1.1 Exercises
1. 3. Prove VW < XY.

DE bisects acute angle +ABC 4.


so that +ABD = +CBD. Prove
that DE also bisects reflex
angle +ABC. That is, prove
+ABE = +CBE.
Given x + y = 180, prove that
2. ABCD is a parallelogram.

Prove that CD bisects +AFE.


14 Maths In Focus Mathematics HSC Course

5. BD bisects +ABC. Prove that 9. CE and BD are altitudes of ABC,


The altitude is ABD / CBD. and ABC is isosceles (AB = AC).
perpendicular to the Prove that CE = BD.
other side of the
triangle

6. (a) Show that ABC / AED.


(b) Hence prove that ACD is
isosceles. 10. ABCD is a kite where AB = AD
and BC = DC. Prove that diagonal
AC bisects both +DAB and
+DCB.

7. ABCD is a square. Lines are


drawn from C to M and N,
the midpoints of AD and AB 11. MNOP is a rhombus with
respectively. Prove that MC = NC. MN = NO.
Show that
(a) MNO is congruent to MPO
(b) +PMQ = +NMQ
(c) PMQ is congruent to NMQ
(d) +MQN = 90

8. OC is drawn so that it is
perpendicular to chord AB and
O is the centre of the circle.
Prove that OAC and OBC are 12. Show that a diagonal cuts a
congruent, and hence that OC parallelogram into two congruent
bisects AB. triangles.

13. Prove that opposite angles are


equal in any parallelogram.
Chapter 1 Geometry 2 15

14. ABCD is a parallelogram with 19. Prove that the diagonals in any
BM = DN. Prove that AMCN is rectangle are equal.
also a parallelogram.
20. Prove that if one angle in a
rectangle is 90 then all the
angles are 90.

21. ABCD is a rhombus with


AD = CD. Prove that all sides of
the rhombus are equal.

15. ABCD and BCEF are parallelograms.


Show that AFED is a parallelogram.

22. ABCD is an isosceles trapezium.


Prove the base angles +ADC and
+BCD are equal.

16. ABCD is a parallelogram with


DE = DC. Prove that CE bisects
+BCD.

23. Prove that +ADC = +ABC in kite


ABCD.

17. In quadrilateral ABCD, AB = CD


and +BAC = +DCA. Prove ABCD
is a parallelogram.

24. In rectangle ABCD, E is the


midpoint of CD. Prove AE = BE.

18. ABCD is a parallelogram with


+AEB = 90. Prove
(a) AB = BC
(b) +ABE = +CBE
16 Maths In Focus Mathematics HSC Course

25. ABCD is a rhombus.


(a) Prove ADB and BCD are
congruent.
(b) Hence show +ABE = +CBE.
(c) Prove ABE and CBE are
congruent.
(d) Prove +AEB = 90.

Surface Areas and Volumes


Areas are used in finding the surface area and volume of solids. Here is a
You will need some of these summary of some of the most common ones.
formulae when you study maxima
and minima problems in Chapter 2.

SURFACE AREA VOLUME

S = 2(lb + bh + lh) V = lbh

S = 6x 2 V = x3

S = 2 r (r + h) V = r2 h

4 3
S = 4 r 2 V= r
3
Chapter 1 Geometry 2 17

1 2
S = r (r + l) V= r h
3

In general, the volume of any prism is given by


V = Ah
where A is the area of the base and h is its height

In general, the volume of any pyramid is given by


1
V= Ah
3
Where A is the area of the base and h is its height
18 Maths In Focus Mathematics HSC Course

EXAMPLE

Find the surface area of a cone whose height is twice the radius, in
terms of r.

Solution

h = 2r
l2 = r2 + h2
= r 2 + ( 2 r) 2
= r 2 + 4r 2
= 5r 2
` l= 5r 2
= 5r
Surface area S = r (r + l) where l is slant height
= r (r + 5 r )

While surface area and volume is not a part of the geometry in the HSC
syllabus, the topic in Chapter 2 uses calculus to find maximum or minimum
areas, perimeters, surface areas or volumes. So you will need to know these
formulae in order to answer the questions in the next chapter. Here are some
questions to get you started.

1.2 Exercises
1. A rectangular prism has 2. A sphere has a volume of 120 m3.
dimensions 12.5 mm, 84 mm and Find the exact value of r.
64 mm. Find its
(a) surface area and 3. A rectangular prism has
(b) volume. dimensions x, x + 2 and 2x 1.
Find its volume in terms of x.
Chapter 1 Geometry 2 19

4. A cylinder has a volume of 250 cm3. 11. The area of the base of a prism
If its base has radius r and its is given by 3h + 2, where h is
250 the height of the prism. Find a
height is h, show that r = .
h formula for the volume of the
5. Find the volume of a cylinder prism.
in terms of r if its height is five
12. The area of the base of a pyramid
times the size of its radius.
is 6h + 15 where h is the height of
6. The ratio of the length to the the pyramid. Find the volume of
breadth of a certain rectangle is 3:2. the pyramid in terms of h.
If the breadth of this rectangle is
13. A rectangular pyramid has base
b units, find a formula for the area
dimensions x 3 and 3x + 5,
of the rectangle in terms of b.
and a height of 2x + 1. Write a
7. Find the volume of a cube with formula for the volume of the
sides (x + 2) cm. pyramid in terms of x.

8. What would the surface area of 14. The height of a rectangular


a cylinder be in terms of h if its prism is twice the length of its
height is a third of its radius? base. If the width of the base is x
and the length is 3x 1, find an
9. A square piece of metal with sides expression for the
3 m has a square of side x cut out (a) volume and
of each corner. The metal is then (b) surface area of the prism.
folded up to form a rectangular
prism. Find its volume in terms 15. Find a formula for the slant
of x. height of a cone in terms of its
radius r and height h.

16. A page measuring x by y is curved


around to make an open cylinder
with height y. Find the volume of
the cylinder in terms of x and y.

17. The volume of a cylinder is


10. A cone-shaped vessel has a height 400 cm3. Find the height of the
of twice its radius. If I fill the cylinder in terms of its radius r.
vessel with water to a depth of
18. A cylinder has a surface area of
10 cm, find the volume of water
1500 cm2. Find a formula for its
to the nearest cm3.
height h in terms of r.

19. The surface area of a cone is given


by S = r (r + l) where l is the
slant height. Find a formula for
the slant height of a cone with
surface area 850 cm2 in terms of r.
20 Maths In Focus Mathematics HSC Course

20. A rectangular timber post is cut


out of a log with diameter d as
shown. If the post has length x
and breadth y, write y in terms of d
x when d = 900 mm.

DID YOU KNOW?

REGULAR SOLIDS
There are only five solids with each face the same size and shape. These are called platonic
solids. Research these on the internet.
Chapter 1 Geometry 2 21

The skeletons opposite are those of radiolarians. These are tiny sea animals, with their
skeletons shaped like regular solids.
A salt crystal is a cube. A diamond crystal is an octahedron.

Diamond crystal

Coordinate Methods in Geometry


Problems in plane geometry can be solved by using the number plane.
You studied straight-line graphs in the Preliminary Course. Some of the
main results that you learned will be used in this section. You may need to
revise that work before studying this section.
Here is a summary of the main formulae.

Distance

The distance between two points (x1, y1) and (x2, y2) is given by

d= _ x2 x1 i + _ y2 y1 i
2 2

Midpoint

The midpoint of two points (x1, y1) and (x2, y2) is given by
x1 + x2 y1 + y2
P=e , o
2 2
Gradient

The gradient of the line between (x1, y1) and (x2, y2) is given by
y2 y1
m= x x
2 1

The gradient of a straight line is given by


m = tan
where is the angle the line makes with the x-axis in the positive direction.
22 Maths In Focus Mathematics HSC Course

The gradient of the line ax + by + c = 0 is given by


a
m=
b
Equation of a straight line

The equation of a straight line is given by


y y1 = m(x x1)
where (x1, y1) lies on the line with gradient m.

Parallel lines

If two lines are parallel, then they have the same gradient. That is,
m1 = m2
Perpendicular lines

If two lines with gradients m1 and m2 respectively are perpendicular, then


1
m1m2 = 1 or m2 = m
1

Perpendicular distance

The perpendicular distance from (x1, y1) to the line ax + by + c = 0 is given by


| ax1 + by1 + c |
d=
a2 + b2

EXAMPLES

1. Show that triangle ABC is right angled, where A = (3, 4), B = (1, 1)
and C = ( 2, 8).

Solution
Method 1:
d = (x2 x1) 2 + (y2 y1) 2
AB = (1 3) 2 + (1 4) 2
= (4) 2 + (5) 2
= 16 + 25
= 41
AC = (2 3) 2 + (8 4) 2
= (5) 2 + 42
= 25 + 16
= 41
BC = (1 (2)) 2 + (1 8) 2
= 12 + (9) 2
= 1 + 81
= 82
Chapter 1 Geometry 2 23

AB2 + AC2 = 41 + 41
= 82
= BC2
Since Pythagoras theorem is true, the triangle ABC is right angled.
Method 2:
y2 y1
m=x x
2 1
1 4
m AB =
1 3
5
=
4
5
=
4
84
m AC =
2 3
4
=
5
4
=
5
5 4
m ABm AC =
4 5
= 1
So AB and AC are perpendicular
So triangle ABC is right angled at A.

2. Prove that points A ^ 1, 1 h, B ^ 2, 1 h and C (4, 3) are collinear.

Solution
Collinear points lie on the same straight line, so they will have the same
gradient.
y2 y1
m= x x
2 1

m AB =
11
2 1

= 2
3
=2
3
3 (1)
m BC =
4 ( 2)
=4
6
=2
3
m AB = m BC
So the points are collinear.
24 Maths In Focus Mathematics HSC Course

1.3 Exercises
1. Show that points A(1, 0), 9. (a) Prove that OAB and
B(0, 4), C(7, 0) and D(6, 4) are OCB are congruent given
the vertices of a parallelogram. A(3, 4), B(5, 0), C(2, 4) and O the
origin.
2. Prove that A(1, 5), B(4, 6) and
(b) Show that OABC is a
C( 3, 2) are vertices of a right
parallelogram.
angled triangle.
10. The points A(0, 0), B(2, 0), C(2, 2)
3. Given ABC with vertices
and D(0, 2) are the vertices of a
A(0, 8), B(3, 0) and C( 3, 0)
square. Prove that its diagonals
(a) show that ABC is isosceles
make angles of 45 with the sides
(b) find the length of the altitude
of the square.
from A
(c) find the area of the triangle. 11. Prove that P (2, 0), Q (0, 5),
R(10, 1) and S(8, 4) are the
4. Show that the points X ^ 3, 2 h,
vertices of a rectangle.
Y (2, 1) and Z (8, 3) are collinear.
12. The points A( 5, 0), B(1, 4) and
5. (a) Show that the points A ^ 2, 5 h,
C(3, 0) form the vertices of a
B(1, 0), C(7, 4) and D(3, 4) are
triangle.
the vertices of a kite.
(a) Find X and Y, the midpoints
(b) Prove that the diagonals of
of AB and AC respectively.
the kite are perpendicular.
(b) Show that XY and BC are
(c) Show that CE = 2AE where E
parallel.
is the point of intersection of the
(c) Show that BC = 2XY.
diagonals.
13. Show that the diagonals of
6. Find the radius of the circle that
a square are perpendicular
has its centre at the origin and a
bisectors, given the vertices
tangent with equation given by
of square ABCD where
4x 3y 5 = 0.
A = ^ a, 0 h, B = ^ a, a h, C = ^ 0, a h
7. (a) Find the equation of the and D = (0, 0).
perpendicular bisector of the line
14. (a) Show that points X ^ 3, 2 h and
joining A(3, 2) and B(1, 8).
Y (1, 0) are equal distances from
(b) Show that the point C(7, 9)
the line 4x 3y 1 = 0.
lies on the perpendicular bisector.
(b) Find Z, the x-intercept of the
(c) What type of triangle is ABC?
line.
8. Show that OAB and OCD are (c) What is the area of triangle
similar where (0, 7), (2, 0), (0, 14) XYZ?
and (4, 0) are the points A, B, C
15. ABCD is a quadrilateral with
and D respectively and O is the
A^ 3, 1 h, B^ 1, 4 h, C ^ 5, 2 h
origin.
and D ^ 4, 3 h . Show that the
midpoints of each side are the
vertices of a parallelogram.
Chapter 1 Geometry 2 25

Test Yourself 1
1. Triangle ABC is isosceles, with AB = AC. 6. In the figure AEF
D is the midpoint of AB and E is the CB CD
(a) prove =
midpoint of AC. BA DE
(b) find the length of AE.

(a) Prove that BEC is congruent to 7. Given A = (1, 3), B = (2, 4), C = (5, 4)
BDC. and D = (6, 3), prove ABCD is a
(b) Prove BE = DC. parallelogram.
2. If the diagonals of a rhombus are x and y, 8. A parallelogram has sides 5 cm and
show that the length of its side is 12 cm, with diagonal 13 cm.
x2 + y2
.
2

3. If A = (4, 1), B = (7, 5) and C = (1, 3),


prove that triangle ABC is isosceles.
Show that the parallelogram is a
4. The surface area of a closed cylinder rectangle.
is 100 m2. Write the height h of the
9. Prove that PQR and WXY are similar.
cylinder in terms of its radius r.

5. ABCD is a parallelogram with


+C = 45, AE perpendicular to ED and
CD = DE.

10. In quadrilateral ABCE, AD = ED = DC


(a) Show that ADE is isosceles. and +ACB = 90. Also, AC bisects+BAD.
(b) If AE = y, show that the area of ABCE
3y2
is .
2
26 Maths In Focus Mathematics HSC Course

(a) Prove ABC and ADC are congruent.


(b) Prove ABE and ADE are congruent.
(c) Prove AC is the perpendicular
bisector of BD.

14. A(1, 2), B(3, 3) and C(5, 1) are points on


a number plane.
Prove ABCE is a parallelogram. (a) Show that AB is perpendicular to BC.
11. If A = (1, 5), B = (4, 2) and C = (2, 3), find (b) Find the coordinates of D such that
the coordinates of D such that ABCD is a ABCD is a rectangle.
parallelogram. (c) Find the point where the diagonals of
the rectangle intersect.
12. (a) Find the equation of AB if A = (2, 3) (d) Calculate the length of the diagonal.
and B = (4, 5). 15. The surface area of a box is 500 cm2. Its
(b) Find the perpendicular distance from length is twice its breadth x.
C (1, 3) to line AB. (a) Show that the height h of the box is
(c) Find the area of ABC. 250 2x2
given by h = .
3x
13. ABCD is a kite.
(b) Show that the volume of the box is
500x 4x3
V= .
3

Challenge Exercise 1
1. In the figure, BD is the perpendicular (a) DE is parallel to BC
bisector of AC. Prove that ABC is 1
(b) DE = BC.
isosceles. 2

3. Prove that the diagonals in a rhombus


2. Given E and D are midpoints of AC and bisect the angles they make with the sides.
AB respectively, prove that
Chapter 1 Geometry 2 27

4. Paper comes in different sizes, called A0,


A1, A2, A3, A4 and so on. The largest size
is A0, which has an area of one square
metre. If the ratio of its length to breadth
is 2 :1, find the dimensions of its sides
in millimetres, to the nearest millimetre.

5. The volume of a prism with a square base


of side x is 1000 cm3. Find its surface area 11. A pair of earrings is made with a wire
in terms of x. surround holding a circular stone, as
shown. Find the total length of wire
6. Prove that in any regular n-sided polygon needed for the earrings.
360
the size of each angle is b 180 n l .

7. A parallelogram ABCD has AB produced


to E and diagonal AC produced to F so
that EF < BC. Prove that AEF is similar
to ADC.

8. ABCD is a rhombus with A(0, 0), B(a, b),


C(2a, 0) and D(a, b). Show
(a) the diagonals bisect each other at
right angles
(b) all sides are equal
12. The sides of a quadrilateral ABCD have
(c) AC bisects +BCD.
midpoints P, Q, R and S, as shown below.
9. In the parallelogram ABCD, AC is
perpendicular to BD. Prove that AB = AD.

(a) Show that DPS is similar to DAC.


10. Triangle ABC has P, Q and R as midpoints (b) Show PS < QR.
of the sides, as shown in the diagram (c) Show that PQRS is a parallelogram.
below. Prove that PQR / CPR.
28 Maths In Focus Mathematics HSC Course

13. A plastic frame for a pair of glasses is designed as below. Find the length of plastic needed
for the frame, to the nearest centimetre.
2
Geometrical
Applications
of Calculus
TERMINOLOGY

Anti-differentiation: The process of finding a primitive Minimum turning point: A local stationary point (where
(original) function from the derivative. It is the inverse the first derivative is zero) and where the curve is
operation to differentiation concave up. The gradient of the tangent is zero
Concavity: The shape of a curve as it bends around (it can Monotonic increasing or decreasing function: A function is
be concave up or concave down) always increasing or decreasing
Differentiation: The process of finding the gradient of a Point of inexion: A point at which the curve is neither
tangent to a curve or the derivative concave upwards nor downwards, but where the
concavity changes
Gradient of a secant: The gradient (slope) of a line
between two points that lie close together on a curve Primitive function: The original function found by
working backwards from the derivative. Found by anti-
Gradient of a tangent: The gradient (slope) of a line that
differentiation
is a tangent to a curve at a point on a function. It is the
derivative of the function Rate of change: The rate at which the dependent variable
changes as the independent variable changes
Horizontal point of inexion: A stationary point (where
the first derivative is zero) where the concavity of the Stationary (turning) point: A local point at which the
curve changes gradient of the tangent is zero and the tangent is
horizontal. The first derivative is zero
Instantaneous rate of change: The derivative of a function
Maximum turning point: A local stationary point (where
the first derivative is zero) and where the curve is
concave down. The gradient of the tangent is zero
Chapter 2 Geometrical Applications of Calculus 31

INTRODUCTION
YOU LEARNED ABOUT differentiation in the Preliminary Course. This is the
process of nding the gradient of a tangent to a curve. This chapter looks at
how the gradient of a tangent can be used to describe the shape of a curve.
Knowing this will enable us to sketch various curves and nd their maximum
and minimum values. The theory also allows us to solve various problems
involving maximum and minimum values.

DID YOU KNOW?

Although Newton and Leibniz are said to have discovered calculus, elements of calculus were
around before then. It was Newton and Leibniz who perfected the processes and developed the
notation associated with calculus.
Pierre de Fermat (160165) used coordinate geometry to find maximum and minimum
values of functions. His method is very close to calculus. He also worked out a way of finding the
tangent to a curve.
The 17th-century mathematicians who developed calculus knew that it worked, but it was
not fully understood. Limits were not introduced into calculus until the nineteenth century.

Gradient of a Curve
To learn about the shape of a curve, we rst need to revise what we know
about the gradient of a tangent. The gradient (slope) of a straight line measures
the rate of change of y with respect to the change in x.

Since the gradient of a curve varies, we nd the gradient of the tangent at


each point along the curve.
32 Maths In Focus Mathematics HSC Course

EXAMPLES

1.

2.

3.

4.
Chapter 2 Geometrical Applications of Calculus 33

In the examples on the previous page, where the gradient is positive, the
curve is going up, or increasing (reading from left to right).
Where the gradient is negative, the curve is going downwards, or
decreasing.
The gradient is zero at particular points of the curves. At these points
the curve isnt increasing or decreasing. We say the curve is stationary at these
points.

If f l] x g > 0, the curve is increasing


If f l] x g < 0, the curve is decreasing
If f l] x g = 0, the curve is stationary

A curve is monotonic increasing or decreasing if it is always increasing or


decreasing; that is,
if f l] x g > 0 for all x (monotonic increasing)
or f l] x g < 0 for all x (monotonic decreasing)

EXAMPLES

1. Find all x values for which the curve f ] x g = x2 4x + 1 is increasing.

Solution
f l] x g = 2x 4
f l] x g > 0 for increasing curve
i.e. 2x 4 > 0
2x > 4
x>2
So the curve is increasing for x > 2.

This function is a
parabola.

CONTINUED
34 Maths In Focus Mathematics HSC Course

2. Find the stationary point on the parabola y = x2 6x + 3.

Solution
dy
= 2x 6
dx
dy
For stationary points, =0
dx
i.e. 2x 6 = 0
2x = 6
x=3
When x = 3, y = 3 6 3 g + 3
2 ]
= 6
So the stationary point is ^ 3, 6 h.

3. Find any stationary points on the curve y = x3 48x 7.

Solution
yl = 3x2 48
For stationary points, yl = 0
i.e. 3x2 48 = 0
3x2 = 48
x2 = 16
` x = 4
When x = 4, y = 43 48 (4) 7
= 135
When x = 4, y = ] 4 g3 48 ( 4) 7
You will use stationary points
to sketch curves later in this
= 121
chapter. So the stationary points are ^ 4, 135 h and ^ 4, 121 h.
Chapter 2 Geometrical Applications of Calculus 35

PROBLEM

What is wrong with this working out?


Find the stationary point on the curve y = 2x2 + x 1.

Solution
yl = 4x + 1
For stationary points, yl = 0
i.e. 4x + 1 = 0
4x = 1
x = 0.25
When x = 0.25, y = 4 ( 0.25) + 1
= 1 + 1
=0 Can you find the
correct answer?
So the stationary point is ^ 0.25, 0 h.

2.1 Exercises

1. Find the parts of each curve (d)


where the gradient of the tangent
is positive, negative or zero. Label
each curve with +, or 0.

(a)

2. Find all values of x for which the


curve y = 2x 2 x is decreasing.

3. Find the domain over which


(b) the function f ] x g = 4 x 2 is
increasing.

4. Find values of x for which the


curve y = x 2 3x 4 is
(a) decreasing
(b) increasing
(c) stationary.
(c)
5. Show that the function
f ] x g = 2x 7 is always
(monotonic) decreasing.
36 Maths In Focus Mathematics HSC Course

6. Prove that y = x 3 is monotonic 17. Sketch a function with f l] x g > 0


increasing for all x 0. for x < 2, f l] 2 g = 0 and f l] x g < 0
when x > 2.
7. Find the stationary point on the
curve f (x) = x 3. 18. Draw a sketch showing a curve
8. Find all x values for which the dy dy
with < 0 for x < 4, = 0 when
curve y = 2x 3 + 3x 2 36x + 9 is dx dx
dy
stationary. x = 4 and > 0 for x > 4.
dx
9. Find all stationary points on the dy
curve 19. Sketch a curve with > 0 for all
dx
(a) y = x 2 2x 3 dy
(b) f ] x g = 9 x 2 x 1 and = 0 when x = 1.
dx
(c) y = 2x 3 9x 2 + 12x 4
20. Draw a sketch of a function
(d) y = x 4 2x 2 + 1.
that has f l] x g > 0 for x < 2,
10. Find any stationary points on the x > 5, f l] x g = 0 for x = 2, 5 and
curve y = ] x 2 g4. f l] x g < 0 for 2 < x < 5.

11. Find all values of x for which 21. A function has f (3) = 2 and
the curve f (x) = x 3 3x + 4 is f l] 3 g < 0. Show this information
decreasing. on a sketch.

12. Find the domain over which the 22. The derivative is positive at
curve y = x 3 + 12x 2 + 45x 30 is the point (2, 1). Show this
increasing. information on a graph.

13. Find any values of x for which the 23. Find the stationary points on the
curve y = 2x 3 21x 2 + 60x 3 is curve y = ] 3x 1 g ] x 2 g4.
(a) stationary
24. Differentiate y = x x + 1 . Hence
(b) decreasing
nd the stationary point on the
(c) increasing.
curve, giving the exact value.
14. The function f ] x g = 2x 2 + px + 7
25. The curve
has a stationary point at x = 3.
f (x) = ax 4 2x 3 + 7x 2 x + 5
Evaluate p.
has a stationary point at x = 1.
15. Evaluate a and b if Find the value of a.
y = x 3 ax 2 + bx 3 has stationary
26. Show that f (x) = x has no
points at x = 1 and x = 2.
stationary points.
16. (a) Find the derivative of
1
y = x 3 3x 2 + 27x 3. 27. Show that f ] x g = has no
x3
(b) Show that the curve is stationary points.
monotonic increasing for all
values of x.
Chapter 2 Geometrical Applications of Calculus 37

Types of Stationary Points


There are three types of stationary points.

Local minimum point

The curve is decreasing on the left and increasing on the right of the
minimum turning point.

x LHS Minimum RHS


f l] x g <0 0 >0

Local maximum point

The curve is increasing on the left and decreasing on the right of the
maximum turning point.

x LHS Maximum RHS


f l] x g >0 0 <0

Local maximum and minimum points are also called turning points,
as the curve turns around at these points. They can also be called relative
maxima or minima.

Point of horizontal inexion

The derivative has the


same sign on both sides
of the inflexion.

The curve is either increasing on both sides of the inexion or it is decreasing


on both sides. It is not called a turning point as the curve does not turn
around at this point.
38 Maths In Focus Mathematics HSC Course

x LHS Inexion RHS


f l] x g >0 0 >0
or <0 0 <0

The stationary points are important to the shape of a curve. A reasonably


accurate sketch of the curve can be made by nding these points, together
with the intercepts on the axes if possible.

EXAMPLES

1. Find the stationary point on the curve y = x 3 and determine which


type it is.

Solution
dy
= 3x 2
dx
dy
For stationary points, =0
dx
i.e. 3x 2 = 0
x=0
When x = 0, y = 0 3
=0
So the stationary point is ^ 0, 0 h.
To determine its type, check the curve on the LHS and the RHS.

x 1 0 1
dy dy
Substitute x = 1 into
dx
. 3 0 3
dx

Since the curve is increasing on both sides, (0, 0) is a point of inexion.


Chapter 2 Geometrical Applications of Calculus 39

2. Find any stationary points on the curve f ] x g = 2x 3 15x 2 + 24x 7 and


distinguish between them.

Solution
f l] x g = 6x 2 30x + 24
For stationary points, f l(x) = 0
i.e. 6x 2 30x + 24 = 0
x 2 5x + 4 = 0
(x 1) (x 4) = 0
` x = 1 or 4
f (1) = 2 ] 1 g3 15 ] 1 g2 + 24 ] 1 g 7
=4
So ^ 1, 4 h is a stationary point.
Substitute x = 0 and x = 2
into f(x). Take care that
x 0 1 2 there is no other stationary
f l] x g 24 0 12 point between the x values
you choose.

` ^ 1, 4 h is a maximum stationary point


f (4) = 2 ] 4 g3 15 ] 4 g2 + 24 (4) 7
= 23
So ^ 4, 23 h is a stationary point.

x 2 4 5
f l] x g 12 0 24

` ^ 4, 23 h is a minimum stationary point.


40 Maths In Focus Mathematics HSC Course

2.2 Exercises

1. Find the stationary point on the 12. The curve y = x5 + mx3 7x + 5


curve y = x2 1 and show that it has a stationary point at x = 2.
is a minimum point by checking Find the value of m.
the derivative on both sides of it.
13. For a certain function,
2. Find the stationary point on the f l] x g = 3 + x. For what value
curve y = x4 and determine its of x does the function have a
type. stationary point? What type of
stationary point is it?
3. Find the stationary point on the
curve y = x3 + 2 and determine its 14. A curve has f l] x g = x (x + 1) . For
nature. what x values does the curve have
stationary points? What type are
4. The function f ] x g = 7 4x x2
they?
has one stationary point. Find its
coordinates and show that it is a 15. For a certain curve,
maximum turning point. dy
= (x 1) 2 (x 2) . Find the x
dx
5. Find the turning point on
values of its stationary points and
the curve y = 3x 2 + 6x + 1 and
determine their nature.
determine its nature.
50
6. For the curve y = ] 4 x g2 nd the 16. (a) Differentiate P = 2x + x with
turning point and determine its respect to x.
nature. (b) Find any stationary points
on the curve and determine their
7. The curve y = x3 6x2 + 5 has 2 nature.
turning points. Find them and
use the derivative to determine h 2 2h + 5
17. For the function A = ,
8
their nature.
nd any stationary points and
8. Show that the curve f ] x g = x5 + 1 determine their nature.
has a point of inexion at ^ 0, 1 h.
18. Find any stationary points for
9. Find the turning points on the function V = 40r r 3 and
the curve y = x3 3x2 + 5 and determine their nature (correct to
determine their nature. 2 decimal places).

10. Find any stationary points on the 19. Find any stationary points on the
curve f ] x g = x4 2x2 3. What 120
curve S = 2r + r (correct to 2
type of stationary points are they?
decimal places) and determine
11. The curve y = x3 3x + 2 has their nature.
2 stationary points. Find their
20. (a) Differentiate A = x 3600 x 2 .
coordinates and determine
(b) Find any stationary points for
their type.
A = x 3600 x 2 (to 1 decimal
place) and determine their nature.
Chapter 2 Geometrical Applications of Calculus 41

Higher Derivatives
A function can be differentiated several times:
differentiating f ] x g gives f l] x g
differentiating f l] x g gives f m] x g
Only the first 2 derivatives
differentiating f m] x g gives f n ] x g, and so on are used in sketching
d2 y d3 y graphs of curves.
the other notation is , and so on
dx2 dx3

EXAMPLES

1. Find the rst 4 derivatives of f (x) = x3 4x2 + 3x 2.

Solution
f l(x) = 3x2 8x + 3
f m(x) = 6x 8
f n(x) = 6 Differentiating further will
only give 0.
f mm(x) = 0

2. Find the second derivative of y = ] 2x + 5 g7.

Solution
dy
= 7 ] 2x + 5 g6 $ 2
dx
= 14 ] 2x + 5 g6
d2 y
= 14 $ 6 ] 2x + 5 g5 $ 2
dx2
= 168 ] 2x + 5 g5

3. Find f m] 1 g if f ] x g = x 4 1

Solution
f l(x) = 4x3
f m(x) = 12x2
` f m(1) = 12 ] 1 g2
= 12
42 Maths In Focus Mathematics HSC Course

2.3 Exercises
1. Find the rst 4 derivatives of 12. Find all values of x for
x 7 2x 5 + x 4 x 3. which f m] x g > 0 given that
f ] x g = x3 x2 + x + 9.
2. If f ] x g = x 9 5, nd f m] x g .
13. Differentiate ] 4x 3 g5 twice.
3. Find f l] x g and f m] x g if
f ] x g = 2x 5 x 3 + 1 . 14. Find f l] x g and f m] x g if
f ]xg = 2 x .
4. Find f l] 1 g and f m] 2 g, given
f ] t g = 3t 4 2t 3 + 5t 4. 15. Find the rst and second
x+5
derivatives of f ] x g = .
5. Find the rst 3 derivatives of 3x 1
x 7 2x 6 + 4x 4 7.
d2 v
16. Find if v = ] t + 3 g (2t 1) 2 .
6. Differentiate y = 2x 2 3x + 3 dt2
twice. 17. Find the value of b in
d2 y
y = bx 3 2x 2 + 5x + 4 if = 2
7. If f ] x g = x 4 x 3 + 2x 2 5x 1, nd dx2
f l] 1 g and f m] 2 g. 1
when x = .
2
8. Differentiate x 4 twice.
18. Find the exact value of f m] 2 g if
9. If g ] x g = x , nd g m] 4 g . f (x) = x 3x 4 .

10. Given h = 5t 3 2t 2 + t + 5, nd 19. Find f m] 1 g if f ] t g = t (2t 1) 7.


d2 h
when t = 1. 20. Find the value of b if
dt2
f ] x g = 5bx 2 4x 3 and f m] 1 g = 3.
11. Find the value of x for which
d2 y
= 3 given y = 3x 3 2x 2 + 5x.
dx2

Sign of the Second Derivative


The second derivative gives extra information about a curve that helps us to
nd its shape.
Since f m] x g is the derivative of f l] x g, then f m] x g and f l] x g have the same
relationship as f l] x g and f ] x g.

That is if f m(x) > 0 then f l(x) is increasing


if f m(x) < 0 then f l(x) is decreasing
if f m(x) = 0 then f l(x) is stationary
Chapter 2 Geometrical Applications of Calculus 43

Concavity

If f m] x g > 0 then f l] x g is increasing. This means that the gradient of the


tangent is increasing, that is, the curve is becoming steeper.

f (x) is increasing since a


1
gradient of is greater
2
than a gradient of 6.

Notice the upward shape of these curves. The curve lies above the
tangents. We say that the curve is concave upwards.
If f m] x g < 0 then f l] x g is decreasing. This means that the gradient of the
tangent is decreasing. That is, the curve is becoming less steep.

f (x) is decreasing since


1
4 is less then .
2

Notice the downward shape of these curves. The curve lies below the
tangents. We say that the curve is concave downwards.
If f m] x g = 0 then f l] x g is stationary. That is, it is neither increasing nor
decreasing. This happens when the curve goes from being concave upwards to The point where concavity
changes is called an
concave downwards, or when the curve changes from concave downwards to inflexion.
concave upwards. We say that the curve is changing concavity.

These points of inflexion


are called horizontal
inflexions, as the tangent
is horizontal.

The curve has a point of inexion as long as concavity changes.


44 Maths In Focus Mathematics HSC Course

Class Investigation

How would you check that concavity changes?

If f m] x g > 0, the curve is concave upwards.


If f m] x g < 0, the curve is concave downwards.
If f m] x g = 0 and concavity changes, there is a
point of inexion.

EXAMPLES

1. Does the curve y = x 4 have a point of inexion?

Solution
dy
= 4x 3
dx
d2 y
= 12x2
dx2
d2 y
For inflexions, =0
dx2
i.e. 12x2 = 0
x=0
When x = 0, y = 04
=0
The sign must change for So ^ 0, 0 h is a possible point of inexion.
concavity to change.
Check that concavity changes:

x 1 0 1
d y
2
12 0 12
dx2
What type of point is it?
Since concavity doesnt change, (0, 0) is not a point of inexion.
Chapter 2 Geometrical Applications of Calculus 45

2. Find all values of x for which the curve f ] x g = 2x 3 7x 2 5x + 4 is


concave downwards.

Solution

f l(x) = 6x2 14x 5


f m(x) = 12x 14
For concave downwards, f m(x) < 0
12x 14 < 0
12x < 14
1
` x<1
6

3. Find the point of inexion on the curve y = x 3 6x 2 + 5x + 9.

Solution
yl = 3x2 12x + 5
y m = 6x 12
For inflexions, y m = 0
6x 12 = 0
6x = 12
` x=2
When x = 2, y = 2 3 6 ] 2 g2 + 5 (2) + 9
=3
` ^ 2, 3 h is a possible point of inexion
Check that concavity changes:
x 1 2 3
d y
2
6 0 6
dx 2
Since concavity changes, ^ 2, 3 h is a point of inexion.

2.4 Exercises
1. For what values of x is the curve 4. Show that the curve y = x2 is
y = x3 + x2 2x 1 concave always concave upwards.
upwards?
5. Find the domain over which
2. Find all values of x for which the curve f ] x g = x3 7x2 + 1 is
the curve y = ] x 3 g3 is concave concave downwards.
downwards.
6. Find any points of inexion on
3. Prove that the curve the curve g ] x g = x3 3x2 + 2x + 9.
y = 8 6x 4x2 is always concave
downwards. 7. Find the points of inexion on
the curve y = x4 6x2 + 12x 24.
46 Maths In Focus Mathematics HSC Course

8. Find the stationary point on the (b) Find which of these points
curve y = x 3 2. Show that it is an are horizontal points of inexion
inexion. (stationary points).

9. Find all values of x for 16. (a) Show that the curve
which the function y = x4 + 12x2 20x + 3 has no
f ] x g = x 4 + 2x 3 12x 2 + 12x 1 points of inexion.
is concave downwards. (b) Describe the concavity of the
curve.
10. Determine whether there are any
points of inexion on the curve 17. If y = ax3 12x2 + 3x 5 has
(a) y = x 6 a point of inexion at x = 2,
(b) y = x 7 evaluate a.
(c) y = x 5
18. Evaluate p if
(d) y = x 9
f ] x g = x4 6px2 20x + 11 has a
(e) y = x 12
point of inexion at x = 2.
11. Sketch a curve that is always
19. The curve
concave up.
y = 2ax4 + 4bx3 72x2 + 4x 3
12. Sketch a curve where f m] x g < 0 for has points of inexion at x = 2
x > 1 and f m] x g > 0 for x < 1. and x = 1. Find the values of a
and b.
13. Find any points of inexion on the
curve y = x4 8x3 + 24x2 4x 9. 20. The curve y = x6 3x5 + 21x 8
2 has two points of inexion.
14. Show that f ] x g = is concave
x2 (a) Find these points.
upwards for all x 0. (b) Show that these points of
inexion are not stationary
15. For the function
points.
f ] x g = 3x5 10x3 + 7
(a) Find any points of inexion.

If we combine the information from the rst and second derivatives, this
will tell us about the shape of the curve.

EXAMPLES

1. For a particular curve, f ] 2 g = 1, f l] 2 g > 0 and f m] 2 g < 0. Sketch the


curve at this point, showing its shape.

Solution
f ] 2 g = 1 means that the point ^ 2, 1 h lies on the curve.
If f l] 2 g > 0, the curve is increasing at this point.
If f m ] 2 g < 0, the curve is concave downwards at this point.
Chapter 2 Geometrical Applications of Calculus 47

2. The curve below shows the number of unemployed people P over


time t months.

dP d2 P
(a) Describe the sign of and 2 .
dt dt Remember that the gradient,
(b) How is the number of unemployed people changing over time? or first derivative, measures
the rate of change.
(c) Is the unemployment rate increasing or decreasing?

Solution
dP
(a) The curve is decreasing, so < 0 and the curve is concave upwards,
dt
d2 P
so > 0.
dt2
(b) As the curve is decreasing, the number of unemployed people is
decreasing.
(c) Since the curve is concave upwards, the gradient is increasing. This
means that the unemployment rate is increasing.
48 Maths In Focus Mathematics HSC Course

2.5 Exercises

1. For each curve, describe the sign 2. The curve below shows the
dy d2 y population of a colony of seals.
of and 2 .
dx dx
(a)

(a) Describe the sign of the rst


(b) and second derivatives.
(b) How is the population rate
changing?

3. Ination is increasing, but the


rate of increase is slowing. Draw a
graph to show this trend.

4. Draw a sketch to show the shape


(c) of each curve:
(a) f l] x g < 0 and f m ] x g < 0
(b) f l] x g > 0 and f m] x g < 0
(c) f l] x g < 0 and f m] x g > 0
(d) f l] x g > 0 and f m] x g > 0

5. The size of classes at a local TAFE


college is decreasing and the rate
(d) at which this is happening is
decreasing. Draw a graph to show
this.

6. As an iceblock melts, the rate at


which it melts increases. Draw a
graph to show this information.

(e) 7. The graph shows the decay of a


radioactive substance.

dM d2 M
Describe the sign of and .
dt dt2
Chapter 2 Geometrical Applications of Calculus 49

8. The population P of sh in a Describe how the level of


certain lake was studied over education has changed over this
time, and at the start the number period of time. Include mention
of sh was 2500. of the rate of change.
dP
(a) During the study, < 0. 10. The graph shows the number of
dt
What does this say about the students in a high school over
number of sh during the study? several years.
d2 P
(b) If at the same time, 2 > 0,
d
what can you say about the
population rate?
(c) Sketch the graph of the
population P against t.

9. The graph shows the level of


education of youths in a certain
Describe how the school
rural area over the past 100 years.
population is changing over time,
including the rate of change.

Here is a summary of the shape of a curve given the rst and second
derivatives.

dy dy dy
>0 <0 =0
dx dx dx

d2 y
>0
dx2

d2 y
<0
dx2

d2 y
=0
dx2
50 Maths In Focus Mathematics HSC Course

Class Investigation

There are three mistakes in this argument. Can you nd all of them?
1
y = 2 x = 2x 2
dy 1 1 1
= 2c x 2 m =
dx 2 2 x
dy
For stationary points, =0
dx
1
i.e. =0
2 x
1 = x
` 1 =x
So there are stationary points at x = 1 and x = 1.

Determining Types of Stationary Points


We can determine the type of a stationary point by looking at the rst and
second derivatives together.
If f l] x g = 0 and f m] x g > 0, there is a minimum turning point.

The curve is concave


upwards at this point.

If f l] x g = 0 and f m] x g < 0, there is a maximum turning point.

The curve is concave


downwards at this point.

If f l] x g = 0, f m] x g = 0 and concavity changes, then there is a horizontal


point of inexion.

Minimum turning point: f l] x g = 0, f m] x g > 0.


Maximum turning point: f l] x g = 0, f m] x g < 0.
Horizontal inexion: f l] x g = 0, f m] x g = 0
and concavity changes.
Chapter 2 Geometrical Applications of Calculus 51

EXAMPLES

1. Find the stationary points on the curve f ] x g = 2x3 3x2 12x + 7 and
distinguish between them.

Solution
f l] x g = 6x2 6x 12
For stationary points, f l] x g = 0
i.e. 6x2 6x 12 = 0
x2 x 2 = 0
(x 2) (x + 1) = 0
` x = 2 or 1
f (2) = 2 ] 2 g 3 3 ] 2 g 2 12 (2) + 7
= 13
So (2, 13) is a stationary point.
f (1) = 2 ] 1 g 3 3 ] 1 g 2 12 (1) + 7
= 14
So ^ 1, 14 h is a stationary point.
Now f m(x) = 12x 6
f m(2) = 12 (2) 6
= 18
>0 (concave upwards)
So ^ 2, 13 h is a minimum turning point.
f m(1) = 12 (1) 6
= 18
<0 (concave downwards)
So ^ 1, 14 h is a maximum turning point.

2. Find the stationary point on the curve y = 2x5 3 and determine its
nature.

Solution
yl = 10x 4
For stationary points, yl = 0
i.e. 10x4 = 0
` x=0
CONTINUED
52 Maths In Focus Mathematics HSC Course

When x = 0, y = 2 ] 0 g5 3
= 3
So ^ 0, 3 h is a stationary point.
Now y m = 40x 3
When x = 0, y m = 40 ] 0 g3
=0
So ^ 0, 3 h is a possible point of inexion.
Check concavity on the LHS and RHS:

x 1 0 1

d2 y
40 0 40
dx 2

Since concavity changes, ^ 0, 3 h is a horizontal point of inexion.

2.6 Exercises
1. Find the stationary point on 8. Find the stationary points on
the curve y = x2 2x + 1 and the curve y = 3x4 4x3 12x2 + 1
determine its nature. and determine whether they are
maximum or minimum turning
2. Find the stationary point on the
points.
curve y = 3x4 + 1 and determine
what type of point it is. 9. Find any stationary points on the
curve y = ] 4x2 1 g4 and determine
3. Find the stationary point on the
their nature.
curve y = 3x2 12x + 7 and show
that it is a minimum point. 10. (a) Find any stationary points on
the curve y = 2x3 27x2 + 120x
4. Determine the stationary point
and distinguish between them.
on the curve y = x x2 and show
(b) Find any points of inexion
that it is a maximum turning
on the curve.
point.
11. Find any stationary points on
5. Show that the curve f ] x g = 2x3 5
the curve y = ] x 3 g 4 x and
has an inexion and nd its
determine their nature.
coordinates.
12. Find any stationary points on the
6. Does the function f ] x g = 2x5 + 3
curve y = x4 + 8x3 + 16x2 1 and
have a stationary point? If it
determine their nature.
does, determine its nature.

7. Find any stationary points on the


curve f ] x g = 2x3 + 15x2 + 36x 50
and determine their nature.
Chapter 2 Geometrical Applications of Calculus 53

13. The curve y = ax2 4x + 1 has a 14. The curve y = x3 mx2 + 8x 7


stationary point where x = 3. has a stationary point where
(a) Find the value of a. x = 1. Find the value of m.
(b) Hence, or otherwise,
15. The curve y = ax3 + bx2 x + 5 has
determine the nature of the
a point of inexion at ^ 1, 2 h .
stationary point.
Find the values of a and b.

Curve Sketching
We can sketch curves by nding all of their important features, such as
stationary points, points of inexion and intercepts. Here is a summary of
strategies for sketching a curve.

1. Find stationary points ^ yl= 0 h .


2. Find points of inexion ^ y m= 0 h .
3. Find intercepts on axes. We cannot always
For x-intercept, y = 0 find the x-intercept.

For y-intercept, x = 0
4. Find domain and range.
5. Find any asymptotes or limits.
6. Use symmetry, odd or even functions.
7. Draw up a table of values.

EXAMPLES

1. Find any stationary points and points of inexion on the curve


f ] x g = x3 3x2 9x + 1 and hence sketch the curve.

Solution
f l(x) = 3x2 6x 9
and f m(x) = 6x 6
First, nd the stationary points.
For stationary points, f l(x) = 0
i.e. 3x 2 6x 9 = 0
x 2 2x 3 = 0
(x 3) (x + 1) = 0
` x = 1 or 3
CONTINUED
54 Maths In Focus Mathematics HSC Course

f (3) = ] 3 g 3 3 ] 3 g 2 9 (3) + 1
= 26
So (3, 26) is a stationary point.
f ( 1) = ] 1 g 3 3 ] 1 g 2 9 ( 1) + 1
=6
So ^ 1, 6 h is a stationary point.

We use the second derivative to determine their type.


f m(3) = 6 (3) 6
= 12
>0 (concave upwards)
` ^ 3, 26 h is a minimum turning point
f m(1) = 6 (1) 6
= 12
<0 (concave downwards)
` ^ 1, 6 h is a maximum turning point

Next, nd any points of inexion.


For inflexions, f m(x) = 0
i.e. 6x 6 = 0
6x = 6
x=1

f (1) = 1 3 3 ] 1 g 2 9 ] 1 g + 1
= 10

x 0 1 2

f m ]xg 6 0 6

Since concavity changes, ^ 1, 10 h is a point of inexion.


Next, try to nd intercepts on the axes.

For y -intercept, x = 0:
f (0) = 0 3 3 ] 0 g 2 9 (0) + 1
=1
For x-intercept, y = 0:
i.e. x 3 3x 2 9x + 1 = 0
This is too hard to solve.
Chapter 2 Geometrical Applications of Calculus 55

Now sketch the graph using an appropriate scale so that all stationary
points and points of inexion t on the graph.

2. Sketch the curve y = 2x 3 + 1, showing any important features.

Solution
dy
= 6x 2
dx
dy
For stationary points, =0
dx
i.e. 6x 2 = 0
x=0
When x = 0, y = 2 ] 0 g 3 + 1
=1
So (0, 1) is a stationary point.
d2 y
= 12x
dx 2
d2 y
At (0, 1) 2 = 12 (0)
dx
=0
So (0, 1) is a possible point of inexion.

x 1 0 1

d2 y
12 0 12
dx2

Since concavity changes, (0, 1) is a horizontal inexion.


For x-intercept, y = 0
i.e. 2x 3 + 1 = 0
2x 3 = 1
x 3 = 0 .5
` x = 0 .8
For y-intercept, x = 0

CONTINUED
56 Maths In Focus Mathematics HSC Course

We already know this point. It is (0, 1), the inexion. We can also nd a
point on either side of the inexion.

When x = 1, y = 2 ] 1 g 3 + 1
= 1
When x = 1, y = 2 ] 1 g 3 + 1
=3

2.7 Exercises

1. Find the stationary point on 8. (a) Determine any stationary


the curve f ] x g = x2 3x 4 and points on the curve
determine its type. Find the y = x 3 + 6x 2 7.
intercepts on the axes and sketch (b) Find any points of inexion
the curve. on the curve.
(c) Sketch the curve.
2. Sketch y = 6 2x x2, showing
the stationary point. 9. Find any stationary points
and inexions on the curve
3. Find the stationary point on the
y = x 3 6x 2 + 3 and hence sketch
curve y = ] x 1 g 3 and determine
the curve.
its nature. Hence sketch the
curve. 10. Find any stationary points
and inexions on the curve
4. Sketch y = x 4 + 3, showing any
y = 2 + 9x 3x2 x3. Hence sketch
stationary points.
the curve.
5. Find the stationary point on the
11. Sketch the function
curve y = x 5 and show that it is a
f ] x g = 3x4 + 4x3 12x2 1,
point of inexion. Hence sketch
showing all stationary points.
the curve.
12. Find the stationary points on
6. Sketch f ] x g = x 7.
the curve y = ] x 4 g (x + 2) 2 and
7. Find any stationary points on the hence sketch the curve.
curve y = 2x3 9x2 24x + 30 and
sketch its graph.
Chapter 2 Geometrical Applications of Calculus 57

13. Find all stationary points 15. Find any stationary points
and inflexions on the curve x2
on the curve y = . By also
y = ] 2x + 1 g (x 2)4. Sketch the x2
curve. considering the domain of the
curve, sketch its graph.
2
14. Show that the curve y =
1+x
has no stationary points. By
considering the domain and
range of the function, sketch the
curve.

Maximum and Minimum Values


A curve may have maximum and minimum turning points, but they are not
necessarily the maximum or minimum values of the function.

EXAMPLE

You sketched this curve on


page 55.

This curve has a maximum turning point at ^ 1, 6 h . We call it a The relative maximum
relative maximum point, since it does not give the maximum value of or minimum point
is also called a
the graph. Similarly, the curve has a relative minimum turning point at local maximum or
^ 3, 26 h . The curve does not have any absolute maximum or minimum minimum point.
value since it increases to infinity and decreases to negative infinity.
If we restrict the domain of the curve to, say, 4 x 4, then the
curve will have an absolute maximum and minimum value. The absolute
maximum is the greatest value of the curve in the domain. The absolute
minimum is the least value of the curve in the domain.
In order to find the maximum or minimum value of a curve, find
the values of the function at the endpoints of the domain as well as its
turning points.

CONTINUED
58 Maths In Focus Mathematics HSC Course

When x = 4, y = ] 4 g 3 3 ] 4 g 2 9 ( 4) + 1
= 75
When x = 4, y = 43 3 ] 4 g 2 9 (4) + 1
= 19

The highest part of the


curve is at y = 6 and the
lowest part is at y = 75.

By restricting the curve to 4 x 4, the maximum value is 6 and the


minimum value is 75.

EXAMPLES

1. Find the maximum and minimum values of y for the function


f ] x g = x2 4x + 3 in the domain 4 x 3.

Solution
f l] x g = 2x 4
For stationary points, f l(x) = 0
i.e. 2x 4 = 0
2x = 4
x=2
f (2) = 2 2 4 (2) + 3
= 1
So ^ 2, 1 h is a stationary point.
f m(x) = 2
>0 (concave upwards)
` ^ 2, 1 h is a minimum turning point
Chapter 2 Geometrical Applications of Calculus 59

Endpoints: f ] 4 g = ] 4 g 2 4 ( 4) + 3
= 35
f (3) = 3 2 4 (3) + 3
=0

The maximum value of y is 35 and the minimum value is l in the


domain 4 x 3.

2. Find the maximum and minimum values of the curve y = x 4 2x 2 + 1


for 2 x 3.

Solution
yl = 4x 3 4x
For stationary points, yl = 0
i.e. 4x 3 4x = 0
4x (x 2 1) = 0
4x (x + 1) (x 1) = 0
` x = 0 or 1
When x = 1, y = 14 2 (1) 2 + 1
=0
So (1, 0) is a stationary point.
When x = 1, y = ] 1 g 4 2 ] 1 g 2 + 1
=0
So ^ 1, 0 h is a stationary point.
When x = 0, y = ] 0 g 4 2 ] 0 g 2 + 1
=1
So (0, 1) is a stationary point.
Now y m = 12x2 4
When x = 1, y m = 12 ] 1 g 2 4
=8
>0 (concave upwards)

CONTINUED
60 Maths In Focus Mathematics HSC Course

So (1, 0) is a minimum turning point.


When x = 1, y m = 12 ] 1 g 2 4
=8
>0 (concave upwards)
So (1, 0) is a minimum turning point.
When x = 0, y m = 12 ] 0 g 2 4
= 4
<0 (concave downwards)
So (0, 1) is a maximum turning point.
Endpoints: When x = 2, y = ] 2 g 4 2 ] 2 g 2 + 1
=9
When x = 3, y = 34 2 ] 3 g 2 + 1
= 64

The maximum value is 64 and the minimum value is 0 in the domain


2 x 3.

2.8 Exercises

1. Sketch y = x2 + x 2 for 2 x 2 3. Find the maximum value of the


and nd the maximum value of y. curve y = x2 4x + 4 for 3 x 3.

2. Sketch f ] x g = 9 x2 over the 4. Sketch f ] x g = 2x3 + 3x2 36x + 5


domain 4 x 2. Hence nd the for 3 x 3, showing any
maximum and minimum values stationary points. Find the
of the curve over this domain. maximum and minimum values
of the function for 3 x 3.
Chapter 2 Geometrical Applications of Calculus 61

5. Find the maximum value of y for 8. Sketch y = x3 + 2 over the domain


the curve y = x5 3 for 2 x 1. 3 x 3 and nd its minimum
and maximum values in that
6. Sketch the curve
domain.
f ] x g = 3x2 16x + 5 for 0 x 4
and nd the maximum and 9. Sketch y = x + 5 for 4 x 4
minimum values of the function and nd its maximum and
over this domain. minimum values.

7. Find the relative and absolute 1


10. Show that y = has no
x2
maximum and minimum
stationary points. Find the
values of the function
maximum and minimum values
f ] x g = 3x4 + 4x3 12x2 3 for
of the curve for 3 x 3.
2 x 2.

Problems Involving Maxima and Minima


Often a problem involves nding maximum or minimum values. For example,
a salesperson wants to maximise prot; a warehouse manager wants to
maximise storage; a driver wants to minimise petrol consumption; a farmer
wants to maximise paddock size.

PROBLEM

One disc 20 cm in diameter and one 10 cm in diameter are cut from a


disc of cardboard 30 cm in diameter. Can you nd the largest disc that
can be cut from the remainder of the cardboard?

EXAMPLES

1. The equation for the expense per year (in units of ten thousand
dollars) of running a certain business is given by E = x 2 6x + 12, where
x is the number (in hundreds) of items manufactured.
(a) Find the expense of running the business if no items are
manufactured.
(b) Find the number of items needed to minimise the expense of the
business.
(c) Find the minimum expense of the business.

CONTINUED
62 Maths In Focus Mathematics HSC Course

Solution
E is the expense in units of (a) When x = 0, E = 0 2 6 (0) + 12
ten thousand dollars.
= 12
So the expense of running the business when no items are
manufactured is 12 $10 000, or $120 000 per year.
dE
(b) = 2x 6
dx
This will give any maximum
dE
or minimum values of E. For stationary points, =0
dx
i.e. 2x 6 = 0
2x = 6
x=3
d2 E
=2>0 (concave upwards)
dx 2
x is the number of items in
hundreds. ` x = 3 gives a minimum value
So 300 items manufactured each year will give the minimum expenses.
(c) When x = 3, E = 32 6 ] 3 g + 12
=3
So the minimum expense per year is $30 000.

2. The formula for the volume of a pond is given by


V = 2h 3 12h 2 + 18h + 50, where h is the depth of the pond in metres.
Find the maximum volume of the pond.

Solution
Vl = 6h 2 24h + 18
For stationary points, Vl = 0
i.e. 6h 2 24h + 18 = 0
h 2 4h + 3 = 0
(h 1) (h 3) = 0
So h = 1 or 3
V m = 12h 24
When h = 1, V m = 12 (1) 24
= 12
<0 (concave downwards)

So h = 1 gives a maximum V.
When h = 3, V m = 12 (3) 24
= 12
>0 (concave upwards)
Chapter 2 Geometrical Applications of Calculus 63

So h = 3 gives a minimum V.
When h = 1, V = 2 ] 1 g 3 12 ] 1 g 2 + 18 ] 1 g + 50
= 58
So the maximum volume is 58 m3.

In the above examples, the equation is given as part of the question.


However, we often need to work out an equation before we can start
answering the question. Working out the equation is often the hardest part!

EXAMPLES

1. A rectangular prism has a base with length twice its breadth. The volume
900
is to be 300 cm3. Show that the surface area is given by S = 4x2 + x .

Solution

Volume: V = lbh
= 2x x h
300 = 2x 2 h
300
` =h (1)
2x 2
Surface area:
S = 2 (lb + bh + lh)
= 2 (2x 2 + xh + 2xh)
= 2 (2x 2 + 3xh)
= 4x 2 + 6xh
Now we substitute (1) into this equation.
300
S = 4x 2 + 6x $
2x 2
900
= 4x 2 + x

2. ABCD is a rectangle with AB = 10 cm and BC = 8 cm. Length AE = x cm


and CF = y cm.

CONTINUED
64 Maths In Focus Mathematics HSC Course

(a) Show that triangles AEB and CBF are similar.


(b) Show that xy = 80.
320
(c) Show that triangle EDF has area given by A = 80 + 5x + x .

Solution
(a) +EAB = +BCF = 90 (given)
+BFC = +EBA (corresponding +s, AB < DC )
+BEA = +FBC (similarly, AD < BC )
` AEB and CBF are similar (AAA)
10 x
(b) `
y =8 (similar triangles have sides in proportion)
xy = 80

(c) Side FD = y + 10 and side ED = x + 8


1
A= bh
2
1
= (y + 10) (x + 8)
2
1
= (xy + 8y + 10x + 80)
2
We need to eliminate y from this equation.
If xy = 80,
80
then y = x

80
Substituting xy = 80 and y = x into the area equation gives:

1
A= (xy + 8y + 10x + 80)
2
1 80
= (80 + 8 $ x + 10x + 80)
2
1 640
= (160 + x + 10x)
2
320
= 80 + x + 5x
Chapter 2 Geometrical Applications of Calculus 65

2.9 Exercises
1. The area of a rectangle is to be 6. A timber post with a rectangular
50 m2. cross-sectional area is to be cut
out of a log with a diameter of
280 mm as shown.

Show that the equation of its


100
perimeter is given by P = 2x + x .

2. A rectangular paddock on a
farm is to have a fence with a
120 m perimeter. Show that the
(a) Show that y = 78400 x 2 .
area of the paddock is given by
(b) Show that the cross-sectional
A = 60x x 2 .
area is given by A = x 78400 x 2 .
3. The product of two numbers is
7. A 10 cm 7 cm rectangular piece
20. Show that the sum of the
of cardboard has equal square
20
numbers is S = x + x . corners with side x cut out and
folded up to make an open box.
4. A closed cylinder is to have a
volume of 400 cm3.

Show that its surface area is


800
S = 2 r 2 + r .

5. A 30 cm length of wire is cut into


2 pieces and each piece bent to
Show that the volume of the box
form a square.
is V = 70x 34x2 + 4x3.

8. A travel agency calculates the


expense E of organising a holiday
per person in a group of x people
as E = 200 + 400x. The cost C for
each person taking a holiday is
(a) Show that y = 30 x.
C = 900 100x. Show that the
(b) Show that the total area
prot to the travel agency on a
of the 2 squares is given by
holiday with a group of x people
x2 30x + 450
A= . is given by P = 700x 500x 2 .
8
66 Maths In Focus Mathematics HSC Course

9. Joel is 700 km north of a town, 10. Sean wants to swim from point
travelling towards it at an average A to point B across a 500 m wide
speed of 75 km/h. Nick is 680 km river, then walk along the river
east of the town, travelling bank to point C. The distance
towards it at 80 km/h. along the river bank is 7 km.

Show that after t hours, the If he swims at 5 km/h and walks


distance between Joel and at 4 km/h, show that the time
Nick is given by taken to reach point C is given by
d = 952400 213800t + 12025t 2 . x2 + 0.25 7 x
t= + .
5 4

When you have found the equation, you can use calculus to nd the
maximum or minimum value. The process is the same as for nding stationary
points on curves.
Always check that an answer gives a maximum or minimum value. Use
the second derivative to nd its concavity, or if the second derivative is too
hard to nd, check the rst derivative either side of this value.

EXAMPLES

1. The council wanted to make a rectangular swimming area at


the beach using a straight cliff on one side and a length of 300 m of
sharkproof netting for the other three sides. What are the dimensions of
the rectangle that encloses the greatest area?

Solution
Let the length of the rectangle be y and the width be x.

There are many differently


shaped rectangles that could
have a perimeter of 300 m.
Chapter 2 Geometrical Applications of Calculus 67

Perimeter: 2x + y = 300 m
` y = 300 2x (1)
Area A = xy
= x (300 2x) 7 substituting ] 1 g A
= 300x 2x 2
dA
= 300 4x
dx
dA
For stationary points, =0
dx
i.e. 300 4x = 0
300 = 4x
75 = x
d2 A
= 4 < 0 (concave downwards)
dx2
So x = 75 gives maximum A.
When x = 75, y = 300 2 (75)
= 150
So the dimensions that give the maximum area are 150 m 75 m.

2. Trinh and Soi set out from two towns. They travel on roads that meet
at right angles, and they walk towards the intersection. Trinh is initially
15 km from the intersection and walks at 3 km/h. Soi is initially 10 km
from the intersection and walks at 4 km/h.
(a) Show that their distance apart after t hours is given by
D2 = 25t2 170t + 325.
(b) Hence nd how long it takes them to reach their minimum distance
apart.
(c) Find their minimum distance apart.

Solution
(a) After t hours, Trinh has walked 3t km. She is now 15 3t km from
the intersection.
After t hours, Soi has walked 4t km. He is now 10 4t km from the
intersection.

CONTINUED
68 Maths In Focus Mathematics HSC Course

By Pythagoras theorem:

D2 = ] 15 3t g 2 + ] 10 4t g 2
= 225 90t + 9t2 + 100 80t + 16t2
= 25t2 170t + 325
dD 2
(b) = 50t 170
Dt
dD 2
For stationary points, =0
dt
i.e. 50t 170 =0
50t = 170
t = 3 .4

d2 D2
= 50 > 0 (concave upwards)
dt 2
So t = 3.4 gives minimum D 2 .
Now 0.4 60 minutes = 24 minutes
` 3.4 hours = 3 hours and 24 minutes
So Trinh and Soi are a minimum distance apart after 3 hours and
24 minutes.
(c) When t = 3.4, D 2 = 25 ] 3.4 g 2 170 (3.4) + 325
= 36
D = 36
=6
So the minimum distance apart is 6 km.

2.10 Exercises
1. The height, in metres, of a 3. The perimeter of a rectangle
ball is given by the equation is 60 m and its length is x m.
h = 16t 4t 2, where t is time in Show that the area of the
seconds. Find when the ball will rectangle is given by the equation
reach its maximum height, and A = 30x x 2 . Hence nd the
what the maximum height will be. maximum area of the rectangle.

2. The cost per hour of a bike 4. A farmer wants to make a


ride is given by the formula rectangular paddock with an area
C = x 2 15x + 70, where x is the of 4000 m2. However, fencing
distance travelled in km. Find the costs are high and she wants the
distance that gives the minimum paddock to have a minimum
cost. perimeter.
Chapter 2 Geometrical Applications of Calculus 69

(a) Show that the perimeter is 9. A box is made from an 80 cm by


given by the equation 30 cm rectangle of cardboard by
8000 cutting out 4 equal squares of
P = 2x + x . side x cm from each corner. The
(b) Find the dimensions of edges are turned up to make an
the rectangle that will give the open box.
minimum perimeter, correct to
1 decimal place.
(c) Calculate the cost of fencing
the paddock, at $48.75 per metre.

5. Bill wants to put a small


rectangular vegetable garden in (a) Show that the volume of the
his backyard using two existing box is given by the equation
walls as part of its border. He V = 4x 3 220x 2 + 2400x.
has 8 m of garden edging for the (b) Find the value of x that gives
border on the other two sides. the box its greatest volume.
Find the dimensions of the garden (c) Find the maximum volume of
bed that will give the greatest area. the box.

10. The formula for the surface


area of a cylinder is given by
S = 2 r ] r + h g, where r is the
radius of its base and h is its
height. Show that if the cylinder
holds a volume of 54 m3, the
surface area is given by the
108
equation S = 2 r 2 + r .
6. Find two numbers whose sum Hence find the radius that gives
is 28 and whose product is a the minimum surface area.
maximum.
11. A silo in the shape of a cylinder
7. The difference of two numbers is required to hold 8600 m3 of
is 5. Find these numbers if their wheat.
product is to be minimum.

8. A piece of wire 10 m long is


broken into two parts, which are
bent into the shape of a rectangle
and a square as shown. Find the
dimensions x and y that make the
total area a maximum.
70 Maths In Focus Mathematics HSC Course

(a) Find an equation for the 16. A half-pipe is to be made from


surface area of the silo in terms of a rectangular piece of metal of
the base radius. length x m. The perimeter of the
(b) Find the minimum surface rectangle is 30 m.
area required to hold this amount
of wheat, to the nearest square
metre.

12. A rectangle is cut from a circular


disc of radius 6 cm. Find the area
of the largest rectangle that can
be produced. (a) Find the dimensions of
the rectangle that will give the
13. A poster consists of a photograph
maximum surface area.
bordered by a 5 cm margin. The
(b) Find the height from the
area of the poster is to be 400 cm2.
ground up to the top of the half-
pipe with this maximum area,
correct to 1 decimal place.

17. Find the least surface area, to the


nearest cm2, of a closed cylinder
that will hold a volume of
400 cm3.

18. The picture frame shown below


(a) Show that the area of the has a border of 2 cm at the top
photograph is given by the and bottom and 3 cm at the
4000
equation A = 500 10x x sides. If the total area of the
(b) Find the maximum area border is to be 100 cm2, nd
possible for the photograph. the maximum area of the frame.

14. The sum of the dimensions of a


box with a square base is 60 cm.
Find the dimensions that will
give the box a maximum volume.

15. A surfboard is in the shape of


a rectangle and semicircle, as
shown. The perimeter is to be 19. A 3 m piece of wire is cut into
4 m. Find the maximum area two pieces and bent around
of the surfboard, correct to to form a square and a circle.
2 decimal places. Find the size of the two lengths,
correct to 2 decimal places, that
will make the total area of the
square and circle a minimum.
Chapter 2 Geometrical Applications of Calculus 71

20. Two cars are travelling along (a) Show that


roads that intersect at right 2400 2y y
x= .
angles to one another. One starts 4
200 km away and travels towards (b) Show that the area of the
the intersection at 80 kmh-1, enclosure is given by
while the other starts at 120 km 4800y 4y2 y2
away and travels towards the A= .
8
intersection at 60 kmh-1. (c) Find the dimensions x and
Show that their distance y (to the nearest metre) that
apart after t hours is given by maximises the area.
d 2 = 10 000t 2 46 400t + 54 400
and hence nd their minimum 23. Points A ^ a, 0 h, B ^ 0, b h and
distance apart. O ^ 0, 0 h form a triangle as shown
and AB always passes through the
21. X is a point on the curve point ^ 1, 2 h .
y = x 2 2x + 5. Point Y lies
y
directly below X and is on the
curve y = 4x x 2. B (0, b)
y
X

x
A (-a, 0)
x

Y
2a
(a) Show that b = .
a1
(b) Find values of a and b that
(a) Show that the distance d give the minimum area of
between X and Y is given by triangle OAB.
d = 2x 2 6x + 5.
(b) Find the minimum distance 24. Grant is at point A on one side
between X and Y. of a 20 m wide river and needs to
get to point B on the other side
22. A park is to have a dog-walking 80 m along the bank as shown.
enclosure in the shape of a
A
rectangle with a semi-circle at the
end as shown, and a perimeter of 20 m x

1200 m.
x 80 m B

Grant swims to any point on the


other bank and then runs along
y
the side of the river to point B. If
he can swim at 7 km/h and run
at 11 km/h, nd the distance he
72 Maths In Focus Mathematics HSC Course

swims (x) to minimise the time 25. A truck travels 1500 km at an


taken to reach point B. Answer to hourly cost given by
the nearest metre. s 2 + 9000 cents where s is the
average speed of the truck.
(a) Show that the cost for
the trip is given by
9000 l
C = 1500 b s + s .

(b) Find the speed that minimises


the cost of the trip.
(c) Find the cost of the trip to the
nearest dollar.

Class Challenge

Can you solve either of these problems?

1. Herons problem
One boundary of a farm is a straight river bank, and on the farm stands
a house and some distance away, a shed; each is sited away from the
river bank. Each morning the farmer takes a bucket from his house to
the river, lls it with water, and carries the water to the shed.
Find the position on the river bank that will allow him to walk the
shortest distance from house to river to shed. Further, describe how the
farmer could solve the problem on the ground with the aid of a few
stakes for sighting.

2. Lewis Carrolls problem


After a battle at least 95% of the combatants had lost a tooth, at least
90% had lost an eye, at least 80% had lost an arm, and at least 75% had
lost a leg. At least how many had lost all four?

Primitive Functions
This chapter uses differentiation to nd the gradient of tangents and
stationary points of functions.
Chapter 2 Geometrical Applications of Calculus 73

Sometimes you may know f l] x g and need to nd the original function,


] g
f x . This process is called anti-differentiation, and the original function is
called the primitive function.

EXAMPLE

Sketch the primitive function (the original function) given the derivative
function below.

Solution
Reversing what you would do to sketch the derivative function, the
parts at the x-axis have a zero gradient, so show stationary points on the
original function. There are stationary points at x1 and x2.
The parts of the graph above the x-axis show a positive gradient, so
the original function is increasing. This happens to the left of x1 and to
the right of x2.
The parts of the graph below the x-axis show a negative gradient, so
the original function is decreasing. This happens between x1 and x2.
Sketching this information gives:

CONTINUED
74 Maths In Focus Mathematics HSC Course

The only problem is, we do not have enough information to know


where the curve turns around. There are many choices of where we could
put the graph:

The primitive function gives


a family of curves.

The examples below use what we know about differentiation to nd out


how to reverse this to nd the primitive function.

EXAMPLES

1. Differentiate x2. Hence nd a primitive function of 2x.

Solution
The derivative of x2 is 2x
` a primitive function of 2x is x2

2. Differentiate x2 + 5. Hence nd a primitive function of 2x.

Solution
The derivative of x2 + 5 is 2x
` a primitive function of 2x is x2 + 5

3. Differentiate x 2 3. Hence nd a primitive function of 2x.

Solution
The derivative of x2 3 is 2x
` a primitive function of 2x is x2 3
Chapter 2 Geometrical Applications of Calculus 75

Thus 2x has many different primitive functions. In general, the primitive


of 2x is x 2 + C, where C is a constant (real number).

EXAMPLES

1. Find the primitive of x.

Solution
The derivative of x2 is 2x.
x2 x2
The derivative of is x. So the primitive of x is + C.
2 2

2. Find the primitive of x2.

Solution
The derivative of x3 is 3x2.
x3
The derivative of is x2.
3
x3
So the primitive of x2 is + C.
3

Continuing this pattern gives the general primitive function of x n.

dy xn + 1
If = xn then y = +C
dx n+1
where C is a constant

Proof
d xn + 1 ] n + 1 g xn + 1 1
c + Cm =
dx n + 1 n+1
= xn

EXAMPLES
dy
1. The gradient of a curve is given by = 6x2 + 8x. If the curve passes
dx
through the point ^ 1, 3 h, nd the equation of the curve.

Solution
dy
= 6x 2 + 8x
dx
x3 x2
y = 6c m + 8c m + C
3 2
` y = 2x 3 + 4x 2 + C
CONTINUED
76 Maths In Focus Mathematics HSC Course

The curve passes through ^ 1, 3 h


` 3 = 2 ]1 g3 + 4 ]1 g2 + C
=2+4+C
9 = C
Equation is y = 2x 3 + 4x 2 9.

2. If f m] x g = 6x + 2 and f l] 1 g = f ] 2 g = 0, nd f ] 3 g .

Solution

f m(x) = 6x + 2
x2
f l(x) = 6 c m + 2x + C
2
= 3x 2 + 2x + C
Now f l(1) = 0
So 0 = 3 ] 1 g 2 + 2 (1) + C
5 = C
` f l(x) = 3x2 + 2x 5
x3 x2
f (x) = 3 c m + 2 c m 5x + C
3 2
= x 3 + x 2 5x + C
Now f ( 2) = 0
So 0 = ] 2 g 3 + ] 2 g 2 5 (2) + C
= 8 + 4 + 10 + C
6 = C
` f (x) = x 3 + x 2 5x 6
f (3) = 3 3 + 3 2 5 (3) 6
= 27 + 9 15 6
= 15

2.11 Exercises
1. Find the primitive function of 2. Find f (x) if
(a) 2x 3 (a) f l(x) = 6x 2 x
(b) x2 + 8x + 1 (b) f l(x) = x4 3x2 + 7
(c) x5 4x3 (c) f l(x) = x 2
(d) ] x 1 g 2 (d) f l(x) = (x + 1) (x 3)
(e) 6 1
(e) f l(x) = x 2
Chapter 2 Geometrical Applications of Calculus 77

3. Express y in terms of x if d2 y dy
dy 11. If = 12x + 6 and = 1 at the
dx 2
dx
(a) = 5x 4 9
dx point ^ 1, 2 h, nd the equation
dy of the curve.
(b) = x 4 2x 2
dx
12. If f m(x) = 6x 2 and
dyx3 f l] 2 g = f ] 2 g = 7, nd the
(c) = x2
dx 5 function f ] x g .
dy 2
(d) = 13. Given f m(x) = 5x 4, f l(0) = 3 and
dx x2
f ] 1 g = 1, nd f ] 2 g .
dy 2x
(e) = x3 +1
dx 3 d2 y
14. If = 2x + 1 and there is a
dx2
4. Find the primitive function of
stationary point at (3, 2), nd the
(a) x
equation of the curve.
(b) x 3
1 d2 y
(c) 8 15. A curve has = 8x and the
x dx2
1 2

(d) x 2 + 2x 3
tangent at (2, 5) makes an angle
(e) x 7
2x 2 of 45 with the x-axis. Find the
equation of the curve.
dy
5. If = x3 3x2 + 5 and y = 4 when 16. The tangent to a curve with
dx
d2 y
x = 1, nd an equation for y in = 2x 4 makes an angle
dx2
terms of x.
of 135 with the x-axis in the
6. If f l(x) = 4x 7 and f (2) = 5, nd positive direction at the point
f (x) . ^ 2, 4 h . Find its equation.

7. Given f l] x g = 3x 2 + 4x 2 and 17. A function has a tangent parallel


f ( 3) = 4, nd f ] 1 g. to the line 4x y 2 = 0
at the point ^ 0, 2 h and
8. Given that the gradient of the
f m(x) = 12x 2 6x + 4. Find the
tangent to a curve is given
equation of the function.
dy
by = 2 6x and the curve
dx d2 y
passes through ^ 2, 3 h, nd the 18. A curve has = 6 and the
dx2
equation of the curve. tangent at ^ 1, 3 h is perpendicular
to the line 2x + 4y 3 = 0. Find
dx ]
9. If = t 3 g 2 and x = 7 when the equation of the curve.
dt
t = 0, nd x when t = 4. 19. A function has f l(1) = 3 and
f (1) = 5. Evaluate f (2) given
d2 y dy f m] x g = 6x + 18.
10. Given = 8 and
= 0 and
dx dx
2

y = 3 when x = 1, nd the d2 y
20. A curve has = 12x 24 and
equation of y in terms of x. dx2
a stationary point at ^ 1, 4 h .
Evaluate y when x = 2.
78 Maths In Focus Mathematics HSC Course

Test Yourself 2
1. Find the stationary points on the curve (a) show that the surface area of a can is
y = x 3 + 6x 2 + 9x 11 and determine their 750
given by S = 2 r2 + r
nature.
(b) nd the radius of the can that gives
2. Find all x-values for which the curve the minimum surface area.
y = 2x 3 7x 2 3x + 1 is concave upwards.
12. For the curve y = 3x 4 + 8x 3 + 6x 2,
dy (a) nd any stationary points
3. A curve has = 6x2 + 12x 5. If the (b) determine their nature
dx
curve passes through the point (2, 3), (c) sketch the curve for 3 x 3.
nd the equation of the curve.
13. A rectangular prism with a square base is
4. If f (x) = 3x 5 2x 4 + x 3 2, nd to have a surface area of 250 cm2.
(a) f (1) (a) Show that the volume is given by
(b) f l(1) 125x x3
V= .
(c) f m] 1 g 2
(b) Find the dimensions that will give
5. The height in metres of an object thrown
the maximum volume.
up into the air is given by h = 20t 2t 2
where t is time in seconds. Find the 14. Find all x-values for which the curve
maximum height that the object reaches. y = 3x 2 18x + 4 is decreasing.

6. Find the stationary point on the curve


d2 y
y = 2x 2 and determine its nature. 15. If = 6x + 6, and there is a stationary
dx2
7. Find the domain over which the curve point at (0, 3), nd the equation of the
y = 5 6x 3x 2 is decreasing. curve.

8. For the curve y = 3x 4 + 8x 3 48x 2 + 1 16. The cost to a business of manufacturing


(a) nd any stationary points and x products a week is given by
determine their nature C = x 2 300x + 9000. Find the number
(b) sketch the curve. of products that will give the minimum
cost each week.
9. Find the point of inexion on the curve
y = 2x 3 3x 2 + 3x 2. 17. Show that y = x 3 is monotonic
decreasing for all x 0.
10. If f m(x) = 15x + 12, and f (2) = f l(2) = 5,
nd f (x) .

11. A soft drink manufacturer wants to


minimise the amount of aluminium in
its cans while still holding 375 mL of soft
drink. Given that 375 mL has a volume
of 375 cm3,
Chapter 2 Geometrical Applications of Calculus 79

18. Sketch a primitive function for each (a) Show that the area of the garden is
graph. 1
A = x 25 x2 .
2
(a) (b) Find the greatest possible area of the
garden bed.

20. For the curve y = x 3 + 3x 2 72x + 5,


(a) nd any stationary points on the
curve and determine their nature
(b) nd any points of inexion
(c) sketch the curve.

21. Find the domain over which the


(b) curve y = x 3 3x 2 + 7x 5 is concave
downwards.

22. A function has f l(3) = 5 and f (3) = 2. If


f m(x) = 12x 6, nd the equation of the
function.

23. Find any points of inexion on the curve


y = x 4 6x 3 + 2x + 1 .
(c)
24. Find the maximum value of the curve
y = x 3 + 3x 2 24x 1 in the domain
5 x 6.

25. A function has f l(2) < 0 and f m(2) < 0.


Sketch the shape of the function near
x = 2.

19. A 5 m length of timber is used to border


a triangular garden bed, with the other
sides of the garden against the house
walls.
80 Maths In Focus Mathematics HSC Course

Challenge Exercise 2
1. Find the rst and second derivatives of 10. Find the radius and height, correct to
5x 2 decimal places, of a cylinder that holds
.
] 4x2 + 1 g3 200 cm3, if its surface area is to be a
minimum.
2. Sketch the curve y = x ] x 2 g3, showing
any stationary points and inexions. 11. A curve passes through the point (0, 1)
and the gradient at any point is given by
3. Find all values of x for which the curve ] x + 3 g ] x 5 g. Find the equation of the
y = 4x 3 21x 2 24x + 5 is increasing. curve.
4. Find the maximum possible area if a 12. The cost of running a car at an average
straight 8 m length of fencing is placed v2
speed of km/h is given by c = 150 +
across a corner to enclose a triangular 80
space. cents per hour. Find the average speed, to
the nearest km/h, at which the cost of a
500 km trip is a minimum.

13. Find the equation of a curve that is


always concave upwards with a stationary
point at (1, 2) and y-intercept 3.

14. Given f l(x) = x (x 3) (x + 1) and f (0) = 0


(a) nd the equation for f (x)
5. Find the greatest and least values of the
(b) nd any stationary points on the
function f (x) = 4x 3 3x 2 18x in the
function
domain 2 x 3.
(c) sketch the function.
6. Show that the function f (x) = 2 (5x 3)3
15. The volume of air in a cubic room on
has a horizontal point of inexion at
a submarine is given by the formula
^ 0 .6 , 0 h .
V = 4x 3 + 27x 2 24x + 2, where x is
7. Two circles have radii r and s such that the side of the room in metres. Find the
r + s = 25. Show that the sum of areas of dimensions of the room that will give
the circles is least when r = s. the maximum volume of air.

8. The rate of change of V with respect to t 16. If f m(x) = x 9 and f l(1) = f (2) = 7,
dV ] nd f (3) .
is given by = 2t 1 g 2 . If V = 5 when
dt dy 1
t=
1
, nd V when t = 3. 17. Given = and y = 1 when x = 4,
2 dx ^ x 5 h2
nd
9. (a) Show that the curve y = x 1 has
d2 y
no stationary points. (a) when x = 6
(b) Find the domain and range of the dx2
curve. (b) y when x = 6.
(c) Hence sketch the curve.
Chapter 2 Geometrical Applications of Calculus 81

18. Show that y = x n has a stationary point at (b) What values of k give a minimum
(0, 0) where n is a positive integer. turning point?
(a) If n is even, show that (0, 0) is a
21. Find the minimum and maximum values
minimum turning point.
x+3
(b) If n is odd, show that (0, 0) is a point of y = 2 in the domain 2 x 2.
x 9
of inexion.
22. The cost of running a car at an average
19. Find the maximum possible volume if a V2
rectangular prism with a length twice its speed of V km/h is given by c = 100 +
75
breadth has a surface area of 48 cm2. cents per hour. Find the average speed (to
the nearest km/h) at which the cost of a
20. (a) Find the stationary point on the
1000 km trip is a minimum.
curve y = x k + 1 where k is a positive
integer.
3
Integration

TERMINOLOGY

Denite integral: The integral or primitive function Indenite integral: General primitive function represented
restricted to a lower and upper boundary. It has the
by # f (x) dx
#a
b
notation f (x) dx and geometrically represents the area
Integration: The process of finding a primitive function
between the curve y = f (x), the x-axis and the ordinates Odd function: A function where f ( x) = f (x). An odd
x = a and x = b function has rotational symmetry about the origin
Even function: A function where f ( x) = f (x). It is
symmetrical about the y-axis
Chapter 3 Integration 83

INTRODUCTION
INTEGRATION IS THE PROCESS of finding an area under a curve.
It is an important process in many areas of knowledge, such as
surveying, physics and the social sciences. In this chapter, you will
look at approximation methods of integrating, as well as shorter
methods that lead to finding areas and volumes. You will learn
how integration and differentiation are related.

DID YOU KNOW?


Integration has been of interest to mathematicians since very early times. Archimedes (287212 BC)
found the area of enclosed curves by cutting them into very thin layers and finding their sum.
He found the formula for the volume of a sphere this way. He also found an estimation of ,
correct to 2 decimal places.

DID YOU KNOW?

Box text...

Area within curve Archimedes

Approximation Methods
Mathematicians used rectangles in order to find the approximate area between
a curve and the x-axis.
84 Maths In Focus Mathematics HSC Course

NOTATION
The area of each rectangle is f (x) x where f (x) is the height and x is
the width of each rectangle. As x " 0, sum of rectangles " exact area.
The symbol # comes Area = lim f (x) x
x"0
from S, the sum of
rectangles. = # f (x) dx

Now there are other, more accurate ways to nd the area under a
curve. However, the notation is still used.

Trapezoidal rule

The trapezoidal rule uses a trapezium for the approximate area under a
curve. A trapezium generally gives a better approximation to the area than a
rectangle.

1
#a
b
f (x) dx Z (b a) [ f (a) + f (b)]
2

Proof
1
A= h (a + b)
2
1
= (b a) [ f (a) + f (b)]
2

EXAMPLES

dx
#1
4
1. Find an approximation for x using the trapezoidal rule.
Solution
dx 4 1
#1 x = #1 x dx
4

1
#a
b
f (x) dx Z(b a) [ f (a) + f (b)]
2
#1 dx 1
4

x Z 2 (4 1) [ f (1) + f (4)]
Chapter 3 Integration 85

1 1 1
= (3) c + m 1
The function f (x) is x .
2 1 4
3 5
=
2 4
= 1.875

#0
1
2. Find an approximation for x 3 dx using the trapezoidal rule with
2 subintervals.

Solution

Two subintervals mean


2 trapezia.

1
#a
b
f (x) dx Z (b a) [ f (a) + f (b)] The function is x3.
2
#0 #0 x3 dx + # x3 dx
1 0.5 1
x3 dx Z
0.5

1 1
= (0.5 0) [ f (0) + f (0.5)] + (1 0.5) [ f (0.5) + f (1)]
2 2
1 1
= (0.5) (0 3 + 0.5 3) + (0.5) (0.5 3 + 1 3)
2 2
= 0.25 (0.125) + 0.25 (1.125)
= 0.3125

There is a more general formula for n subintervals. Several trapezia give a


more accurate area than one. However, you could use the rst formula several
times if you prefer using it.
86 Maths In Focus Mathematics HSC Course

h
#a
b
f (x) dx Z [(y + yn ) + 2 (y1 + y2 + y3 + . . . + yn 1 )]
2 0
ba
where h = n ^ width of each trapezium h

Proof
h h h h
#a
b
f (x) dx Z ( y + y 1 ) + ( y 1 + y 2 ) + ( y 2 + y 3 ) + . . . + (y n 1 + y n )
2 0 2 2 2
h
= (y + 2y 1 + 2y 2 + 2y 3 + . . . + 2y n 1 + y n )
2 0
h
= [(y 0 + y n ) + 2 (y 1 + y 2 + y 3 + . . . + y n 1 )]
2

EXAMPLES

#0
14
1. Find an approximation for (t 2 + 3) dt, using the trapezoidal rule
with 7 subintervals.

Solution

Seven subintervals
mean 7 trapezia.
Chapter 3 Integration 87

h
#a
b
f (x) dx Z
[(y + yn ) + 2 (y1 + y2 + y3 + . . . + yn 1)]
2 0
#0 (t2 + 3) dt Z h2 [(y0 + y7) + 2 (y1 + y2 + y3 + y4 + y5 + y6 )]
14

14 0
where h=
=2
7
#0 (t2 + 3) dt Z 22 " [(02 + 3) + (142 + 3)] + 2 [(22 + 3) + (42 + 3)
14

+ (62 + 3) + (82 + 3) + (102 + 3) + (122 + 3)]}


= 966

2
#2
3
2. Find an approximation for dx, using the trapezoidal rule
x 1
with 4 subintervals, correct to 3 decimal places.

Solution

There are 4
trapezia.

h
#a
b
f (x) dx Z [(y + y 4 ) + 2 (y 1 + y 2 + y 3 )]
2 0
ba
where h= n
32
=
4
= 0.25
2 0.25 2 2 2 2 2
#2
3
dx Z <c + m + 2c + + mF
x1 2 21 31 2.25 1 2.5 1 2.75 1
Z 0.125[(2 + 1) + 2 (1.6 + 1.3333 + 1.1429)]
= 1.394

Application

When surveyors need to find the area of an irregular piece of land, they measure
regular strips and use an approximation method such as the trapezoidal rule.

CONTINUED
88 Maths In Focus Mathematics HSC Course

EXAMPLE

The table below gives the measurements of a certain piece of land:

xm 0 1 2 3 4
ym 3.7 5.9 6.4 5.1 4.9
Using the trapezoidal rule to find its area, correct to 2 decimal places:

# h
b
[(y + y 4) + 2 (y 1 + y 2 + y 3 )]
f (x) dx Z
a 2 0
ba
Where h = n
40
=
4
=1

# 1
4
f (x) dx Z [(y + y 4) + 2 (y 1 + y 2 + y 3 )]
0 2 0
1
= [(3.7 + 4.9) + 2 (5.9 + 6.4 + 5.1)]
2
= 21.7

So the area of the land is approximately 21.7 m2.

3.1 Exercises
Use the trapezoidal rule to nd an approximation for

#1 #1
2 3
1. x2 dx using 1 subinterval. 5. x3 dx using
(a) 1 subinterval
#0
2
2. (x 3 + 1) dx using 1 subinterval.
(b) 2 subintervals.
dx
#1
5
#2
3
3. x using 1 subinterval. 6. log x dx using 2 subintervals.

dx
#1 dx
2
#0
2
4. using 1 subinterval. 7. using 2 strips.
x+3 x+4
Chapter 3 Integration 89

#1 #1
4 4
8. log x dx using 3 subintervals. 14. f (t) dt where values of f (t) are
given in the table:
#0
2
9. (x2 x) dx using 4 trapezia.
t 1 2 3 4
#0
1
10. x dx using 5 subintervals. f (t) 8.9 6.5 4.1 2.9
1
#1
5
#2
14
11. dx using 4 subintervals. 15. f (x) dx where values of f (x)
x2
are given in the table:
1
12. #
6
dx using 6 trapezia.
3 x 1
x 2 4 6 8 10 12 14
#1
9
13. f (x) dx where values of f (x) are f (x) 25.1 37.8 52.3 89.3 67.8 45.4 39.9
given in the table:

x 1 3 5 7 9
f (x) 3.2 5.9 8.4 11.6 20.1

Simpsons rule

This is generally more accurate than the trapezoidal rule, since it makes use of
parabolic arcs instead of straight lines.

A parabola is drawn through points A, B and C to give the formula

ba a+b
#a
b
f (x) dx Z < f (a) + 4f c m + f (b ) F
6 2

Proof
This proof is difcult and is only included for completion of the topic. It uses
results that will be studied later in the chapter.
90 Maths In Focus Mathematics HSC Course

a+b
Let f (a) = y0, f c m = y1 and f (b) = y2
2
Let the parabola passing through points A ( h, y 0), B (0, y 1) and C (h, y 2) be
given by y = ax 2 + bx + c
Then y0 = a ] h g 2 + b ] h g + c
(1)
y0 = ah 2 bh + c
y 1 = a ] 0 g 2 + b (0) + c
(2)
y1 = c
y 2 = a ] h g 2 + b ( h) + c
(3)
y 2 = ah 2 + bh + c
y 0 + 4y 1 + y 2 = ah 2 bh + c + 4c + ah 2 + bh + c
(4)
= 2ah 2 + 6c
h
# h ] ax2 + bx + c g dx = ; ax3 bx2
h 3
+ + cx E
2 h
You will study this
ah3 bh2 ah3 bh2
later in the chapter. =c + + ch m c + ch m
3 2 3 2
2ah3
= + 2ch
3
h
= (2ah2 + 6c)
3
h
= ( y 0 + 4 y 1 + y 2)
3

ba a+b
Now h = , y0 = f (a), y1 = f c m and y2 = f (b)
2 2
ba
2 a+b
#a
b
` f (x) dx = < f (a) + 4f c m + f (b) F
3 2
ba a+b
= < f (a) + 4f c m + f (b) F
6 2

One application of Simpsons rule uses 3 function values (ordinates).

Ordinates are y-values. You first


met this name in Chapter 5 of
the Preliminary Course book.
Chapter 3 Integration 91

Two applications use 5 function values.

Three applications use 7 function values.

How many function


values would you need
for 4 applications of
Simpsons rule?

EXAMPLES

1. Use Simpsons rule with 3 function values to nd an approximation for


#2 x dx.
3

Solution

CONTINUED
92 Maths In Focus Mathematics HSC Course

ba a+b
#a
b
f (x) dx Z < f (a) + 4f c m + f ( b) F
6 2
32 2+3
#2
3
x dx Z < f (2) + 4 f c m + f (3) F
6 2
1
= ( 2 + 4 2. 5 + 3 )
6
Z 1.58

2. Use Simpsons rule with 5 function values to nd an approximation


dx
for #
2
.
0 x + 1

Solution

Five function values mean 2


applications of Simpsons rule.

ba a+b
#a
b
f (x) dx Z < f (a) + 4f c m + f (b) F
6 2
#0 x dx =#
dx
+#
dx
2 1 2

+1 0 x+1 1 x+1
10 0+1 21 1+2
Z < f (0) + 4f c m + f ( 1) F + < f (1) + 4f c m + f (2 ) F
6 2 6 2
1 1 1 1 1 1 1 1
= < + 4c m+ F+ < + 4c m+ F
6 0+1 0 .5 + 1 1+1 6 1+1 1. 5 + 1 2+1
= 1 .1

There is a general formula for n equal subintervals. There are different versions
of this formula. This one uses the function values y0 to yn.

The n + 1 function values give


n subintervals.
Chapter 3 Integration 93

h
#a
b
f (x) dx Z
[(y + yn) + 4 (y1 + y3 + y5 + . . .) + 2 (y2 + y4 + . . .)]
3 0
ba
where h = n for n subintervals (n + 1 function values)

Proof
2h 2h 2h
#a
b
f (x) dx = (y0 + 4y1 + y2) + (y2 + 4y3 + y4) + (y + 4y5 + y6)
6 6 6 4
2h
+ ... + (y + 4yn 1 + yn)
6 n2
h
= (y0 + 4y1 + y2 + y2 + 4y3 + y4 + y4 + 4y5 + y6 + . . . + yn 2 + 4yn 1 + yn)
3
h
= [(y0 + yn) + 4 (y I + y3 + y5 + . . .) + 2 (y2 + y4 + . . .)]
3

You could also remember this rule as


#a f (x) dx Z h3 [( y 0 + y n ) + 4 (odds) + 2 (evens)].
b

EXAMPLES

1. Use Simpsons rule with 7 ordinates to nd an approximation for


#1
4
log x dx.

Solution

Seven ordinates or function


values give 6 subintervals.

h
#a
b
f (x) dx Z
[(y + y6) + 4 (y1 + y3 + y5) + 2 (y2 + y4)]
3 0
ba
where h = n
41
=
6
= 0 .5
Use the log key on your
0 .5
#1
4
log x dx Z [ log 1 + log 4 + 4 (log 1.5 + log 2.5 + log 3.5) calculator.
3
+ 2 (log 2 + log 3)]
Z 1.105

CONTINUED
94 Maths In Focus Mathematics HSC Course

2. Use Simpsons rule with 9 function values to nd an approximation


5 dx
for # 2 .
1 x

Solution

h
#a
b
f (x) dx Z [(y + y8 ) + 4 (y1 + y3 + y5 + y7 ) + 2 (y2 + y4 + y6 )]
3 0
ba
where h= n
5 1
=
8
= 0.5
1 0.5 1 1 1 1 1 1 1 1 1
#1
5
dx Z < + + 4c + + + m + 2 c 2 + 2 + 2 mF
x2 3 12 52 1.52 2.52 3.52 4.52 2 3 4
Z 0.8

#1
4
3. Use Simpsons rule to nd an approximation for f (x) dx using the
values in the table below:
x 1 1.5 2 2.5 3 3.5 4
f(x) 8.6 11.9 23.7 39.8 56.7 71.4 93.2

Solution

#
4
h
f (x) dx Z [( y 0 + y 6 ) + 4 ( y 1 + y 3 + y 5 ) + 2 ( y 2 + y 4 )]
1 3
41
where h=
6
= 0.5

#
4
0 .5
f (x) dx Z [(8.6 + 93.2) + 4 (11.9 + 39.8 + 71.4) + 2 (23.7 + 56.7)]
1 3
Z 125.83
Chapter 3 Integration 95

3.2 Exercises
Use Simpsons rule to nd an approximation for

#1 1
3
#1
7
1. x4 dx using 3 function values. 11. dx using 7 function values.
x3
#3
5
1
12. # 2
5
2. (x 2 1) dx using 3 ordinates. dx using 6 subintervals.
2 x 1

dx
#1
3
#0
4
3. x using 2 subintervals. 13. f (x) dx where values of f(x) are
dx given in the table:
#0
1
4. using 3 function values.
x+2
x 0 1 2 3 4
#2
4
5. x dx using
f(x) 0.7 1.3 5.4 0.5 3.8
(a) 3 function values
(b) 5 function values.
#2
4
14. f (t) dt where values of f(t) are
#3
7
6. log x dx using 5 function given in the table:
values.
t 2 2.5 3 3.5 4
dx
#2
5
7. using 6 subintervals. f(t) 3.7 1.2 9.8 4.1 2.7
x+1

#3
6
#0
3
8. log x dx using 7 function 15. f (x) dx where values of f(x) are
values. given in the table:
#0
4
9. (x 3 + x) dx using 9 ordinates.
x 0 0.5 1 1.5 2 2.5 3

#0
4
10. x dx using 5 function values. f(x) 15.3 29.2 38.1 56.2 69.9 94.8 102.5

Computer Application

There are computer application packages that can calculate the area under curves
by using rectangles, the trapezoidal rule or Simpsons rule.

A graphics calculator will also do this.

Integration and the Primitive Function


Mathematicians found a link between nding areas under a curve and the
primitive function. This made possible a simple method for nding exact
areas.
96 Maths In Focus Mathematics HSC Course

EXAMPLE

The graph below shows the speed at which an object travels over time.

(a) Find the distance it travels in


(i) 1 s
(ii) 2 s
(iii) 3 s
(b) Find the area under the line between
(i) t = 0 and 1
(ii) t = 0 and 2
(iii) t = 0 and 3

Solution
(a) The speed is a constant 30 metres per second (ms 1)
(i) The object travels 30 metres in 1 s
(ii) The object travels 60 metres in 2 s
(iii) The object travels 90 metres in 3 s
(b) (i) The area is 30 1 = 30 units2
Notice that the area (ii) The area is 30 2 = 60 units2
gives the distances.
(iii) The area is 30 3 = 90 units2

In this example, the line graph gives a rate of change. The area under
the curve gives the information about the original data for this rate of
change.
In the same way, the area under a rate of change curve will give the
original data. This original data is the primitive function of the curve.
Chapter 3 Integration 97

DID YOU KNOW?

Many mathematicians in the 17th century were interested in the problem of finding areas
under a curve. Isacc Barrow (163077) is said to be the first to discover that differentiation
and integration are inverse operations. This discovery is called the fundamental theorem of
calculus.
Barrow was an Englishman who was an outstanding Greek scholar as well as making
contributions in the areas of mathematics, theology, astronomy and physics. However, when he
was a schoolboy, he was so often in trouble that his father was overheard saying to God in his
prayers that if He decided to take one of his children, he could best spare Isaac.
Sir Isaac Newton (16431727), another English mathematician, and scientist and
astronomer, helped to discover calculus. He was not interested in his school work, but spent
most of his time inventing things, such as a water clock and sundial.
He left school at 14 to manage the estate after his stepfather died. However, he spent so
much time reading that he was sent back to school. He went on to university and developed his
theories in mathematics and science which have made him famous today.

Fundamental theorem of calculus

The area enclosed by the curve y = f (x), the x-axis and the lines x = a and
x = b is given by

#a
b
f (x) dx = F (b) F (a) where F(x) is the primitive of function f(x)

Proof
Consider a continuous curve y = f (x) for all values of x > a
98 Maths In Focus Mathematics HSC Course

Let area ABCD be A(x)


Let area ABGE be A (x + h)
Then area DCGE is A (x + h) A (x)

Area DCFE < area DCGE < area DHGE


i.e. f (x) . h < A(x + h) A(x) < f (x + h) . h
A(x + h) A(x)
This is the formula for f (x) < < f (x + h)
differentiation from first principles. h
You first saw this in Chapter 8 of A (x + h) A (x)
The Preliminary Course book. lim f (x) < lim < lim f (x + h)
h "0 h "0 h h "0

f (x) < Al(x) < f (x)


` Al(x) = f (x)
i.e. A(x) is a primitive function of f(x)
A (x) = F(x) + C where F(x) is the primitive function of f(x) (1)

Now A(x) is the area between a and x


` A(a) = 0
Substitute in (1):

A(a) = F(a) + C
0 = F(a) + C
F(a) = C
` A(x) = F(x) F(a)
If x = b where b > a,
A(b) = F(b) F(a)

Denite Integrals
By the fundamental theorem of calculus,
# f (x) dx = F(b) F(a) where F(x) is the primitive function of f(x).
b

xn + 1
The primitive function of x n is + C.
n+1
Putting these pieces of information together, we can nd areas under
simple curves.
Chapter 3 Integration 99

xn + 1 b
#a
b
xn = < F
n+1 a
bn + 1 an + 1
=
n+1 n+1

EXAMPLES

Evaluate
#3
4
1. (2x + 1) dx

Solution

#3
4 4
(2x + 1) dx = 8 x2 + x B 3
Constant C will cancel out. That
= ( 4 2 + 4 ) ( 3 2 + 3) is, (4 2 + 4 + C ) (3 2 + 3 + C )
= 20 12 = 20 12
= 8.
=8

The graph shows the area


that the definite integral
calculates.

#0
5
2. 3x2 dx

Solution

#0
5 5
3x 2 dx = 8 x 3 B 0
= (5 3) (0 3)
= 125
CONTINUED
100 Maths In Focus Mathematics HSC Course

#0
2
3. 3x2 dx

Solution

#0
2 2
3x2 dx = : x3 D
0

= (23) (03)
= 8

Can you see why there is a


negative solution?

# 1 x3 dx
1
4.

Solution
4 1
# 1 x 3 dx = ; x4 E 1
1

1 4 ( 1)
4
=
4 4
1 1
=
4 4
The definite integral gives an area
of zero. Can you see why? =0
Chapter 3 Integration 101

3.3 Exercises
Evaluate
#0 # 1 (3x2 2x) dx #3
2 4 4
1. 4x dx 9. 18. (2 x) 2 dx

#1 #1 # 2 4t3 dt
3 3 2
2. (2x + 1) dx 10. (4x2 + 6x 3) dx 19.

x2
#0 # 1 x2 dx #2
5 1 4
3. 3x2 dx 11. 20. dx
3
Simplify first, then find the
5x 4
#1 # 2 (x3 + 1) dx #0
2 3 3
4. (4t 7) dt 12. 21. x dx
definite integral.

3x
# 2 x
4
# 1 (6y + 5) dy # 2 x5 dx
1 2 1
5. 13. 22. x dx

4x 3 + x 2 + 5 x
#0 #1 #0
3 4 2
6. 6x2 dx 14. x dx 23. x dx

x 3 2x 2 + 3x
#1 #0 #0
2 1 1
7. (x2 + 1) dx 15. (x3 3x2 + 4x) dx 24. x dx

x2 + x + 3
#0 #1 #3
2 2 4
8. 4x3 dx 16. (2x 1) 2 dx 25. dx
3x5

# 1 (y3 + y) dy
1
17.

Class Investigation
Look at the results of denite integrals in the examples and exercises.
Sketch the graphs where possible and shade in the areas found.
Can you see why the denite integral sometimes gives a
negative answer?
Can you see why it will sometimes be zero?

Indenite Integrals
Sometimes it is necessary to nd a general or indenite integral (primitive
function).

# f (x) dx = F (x) + C where F(x) is a primitive function of f (x)

In Chapter 2 you learned the result for the primitive function of x n. This
result is the same as the integral of x n.

n+1
# x n dx = nx + 1 + C
102 Maths In Focus Mathematics HSC Course

EXAMPLES

Find each indenite integral (primitive function)

1. # (x4 3x2 + 4x 7) dx
Solution

# (x4 3x2 + 4x 7) dx = x5 x3 x2
5
3: +4: 7x + C
3 2
x5
= x 3 + 2x 2 7 x + C
5

2. # 5x9 dx
Solution

# 5x9 dx = 5 c x10 m + C
10

# 5x9 dx is the same as 5 # x9 dx.


x10
= +C
2

+ 7x2 3x l
# b 2x
5
3. x dx

Solution
7x 2 3 x
# b 2xx
5
+ x x l dx = # (2x4 + 7x 3) dx
2 x 5 7x 2
= + 3x + C
5 2

4. # c x13 + x m dx

Solution
1
# c x13 + x m dx = # (x 3 + x 2 ) dx
3
x 2 x 2
= + +C
2 3
2
1 2 x3
= 2 + +C
2x 3
Chapter 3 Integration 103

DID YOU KNOW?

John Wallis (16161703) found that the area under the curve y = 1 + x + x2 + x3 + . . . is
given by
x2 x3 x4
x+ + + + ....
2 3 4

He found this result independently of the fundamental theorem of calculus.

3.4 Exercises
Find each indenite integral (primitive function)

1. # x2 dx 19. # (6x3 + 5x2 4) dx


2. # 3x5 dx 20. # (3x 4 + x 3 + 2x 2) dx
3. # 2x4 dx 21. # dx
x8
4. # (m + 1) dm 22.
1
# x 3 dx
5. # (t2 7) dt 23. #x
6
3 x 5 + 2x 4
dx
x3
6. # (h2 + 5) dh 24. # (1 2x)2 dx
7. # (y 3) dy 25. # (x 2) (x + 5) dx
8. # (2x + 4) dx 26. # x32 dx
9. # (b2 + b) db
27. # dx
x3
10. # (a3 a 1) da x 5 3x 2 + 7
# 4x
3
28. dx
11. # (x2 + 2x + 5) dx x5
29. # (y2 y 7 + 5) dy
12. # (4x3 3x2 + 8x 1) dx
30. # (t2 4) (t 1) dt
13. # (6x5 + x4 + 2x3) dx
31. # x dx
14. # (x7 3x6 9) dx
32. # t25 dt
15. # (2x3 + x2 x 2) dx
33. #3 x dx
16. # (x5 + x3 + 4) dx
34. #x x dx
17. # (4x2 5x 8) dx
1
35. # x e1 + o dx
18. # (3x4 2x3 + x) dx x
104 Maths In Focus Mathematics HSC Course

Function of a function rule

EXAMPLE

Differentiate (2x + 3)5 .


Hence nd # 10 (2x + 3) 4 dx.
Find # (2x + 3) 4 dx.
Solution

d ]
2x + 3 g5 = 5 ] 2x + 3 g4 $ 2
dx
= 10 ] 2x + 3 g4
Remember: Integration is the
reverse of differentiation. ` # 10 ] 2x + 3 g4 dx = ] 2x + 3 g5 + C
1
# ] 2x + 3 g4 dx = 10 # 10 ] 2x + 3 g4 dx
1 ]
= 2x + 3 g5 + C
10

(ax + b) n + 1
# (ax + b) dx =
n

a ( n + 1)
+C

Proof
d ]
ax + b gn + 1 = ] n + 1 g ] ax + b gn $ a
dx
= a ] n + 1 g ] ax + b gn
` # a ] n + 1 g ] ax + b gn dx = ] ax + b gn + 1 + C
# ] ax + b gn dx = ] 1 g # a ] n + 1 g ] ax + b gn dx
a n+1
1 ] ax + b gn + 1 + C
=
a n + 1g
]
] ax + b gn + 1
= +C
a ]n + 1g
Chapter 3 Integration 105

EXAMPLES

Find
1. # (5x 9)3 dx

Solution
(5x 9)4
# (5x 9)3 dx =
54
+C
(5x 9)4

= +C
20

2. # (3 x)8 dx
Solution
(3 x)9
# (3 x)8 dx = 1 9
+C
(3 x)9
= +C
9

3. # 4x + 3 dx

Solution
3
1 (4x + 3) 2
# (4x + 3) dx =
2
3
+C
4
2
(4x + 3) 3
= +C
6

3.5 Exercises
Find each indenite integral (primitive function)

1. (a) # (3x 4)2 dx by (f) # (7x + 8)12 dx


(i) expanding
(g) # (1 x)6 dx
(ii) the function of a
function rule (h) # 2x 5 dx
(b) # (x + 1)4 dx (i) # 2 (3x + 1) 4 dx
(c) # (5x 1)9 dx (j) # 3 (x + 7) 2 dx
(d) # (3y 2)7 dy (k) # 2 (4x1 5) 3 dx
(e) # (4 + 3x)4 dx
(l) #3 4x + 3 dx
106 Maths In Focus Mathematics HSC Course

(3x 1) 2
#0
1 1
(m) # (2 x) 2 dx (e) dx
6
(n) # (t + 3)3 dt
#4
5
(f) (5 x)6 dx
(o) # (5x + 2)5 dx
#3
6
(g) x 2 dx
2. Evaluate
#1
2
dx
#0
1
(a) (2x + 1)4 dx (h)
(3x 2) 4
#0
1
(b) (3y 2)3 dy 5
#0
2
(i) dn
(2n + 1) 3
#1
2
(c) (1 x)7 dx
2
#1
4
(j) dx
#0
2
(d) (3 2x)5 dx (5x 4)3

Class Exercise
3x 2
Differentiate (x + 1) 2x 3 . Hence nd # dx.
2x 3

Areas Enclosed by the x-axis


The denite integral gives the signed area under a curve.
Areas above the x-axis give a positive denite integral.
Chapter 3 Integration 107

Areas below the x-axis give a negative denite integral.

We normally think of areas as positive. So to nd areas below the x-axis,


take the absolute value of the denite integral. That is,

#a
b
Area = f (x) dx

Always sketch the area to be found.

EXAMPLES

1. Find the area enclosed by the curve y = 2 + x x2 and the x-axis.

Solution
Sketch y = 2 + x x 2 .

The integral will be


positive as the area is
above the x-axis.

Area = # (2 + x x2) dx
2

x2 x3 2
= < 2x + F
2 3 1
2 2
23 (1) 2 (1) 3
= c 2 ( 2) + m d 2 (1) + n
2 3 2 3
8 1 1
= c 4 + 2 m c 2 + + m
3 2 3
1
=4
2
So the area is 4 1 units2.
2

CONTINUED
108 Maths In Focus Mathematics HSC Course

2. Find the area bounded by the curve y = x 2 4 and the x-axis.

Solution
The integral will be Sketch y = x 2 4.
negative as the area is
below the x-axis.

2
# 2 (x2 4)dx = < x3
2 3
4x F
2

2 3 (2)3
=c 4 ( 2) m d 4 (2) n
3 3
8 8
= c 8m c + 8m
3 3
2
= 10
3
2
So the area is 10 units2.
3

3. Find the area enclosed between the curve y = x 3, the x-axis and the
lines x = 1 and x = 3.

Solution
Sketch y = x 3.

Some of the area is below


the x-axis, so its integral will
be negative.
Chapter 3 Integration 109

We need to nd the sum of the areas between x = 1 and 0, and x = 0


and 3.
4 0
# 1 x 3 dx = ; x4 E 1
0

0 4 (1)
4
=
4 4
1
=
4
1
So A 1 = units2.
4
x4 3
#0
3
x3 dx = ; E
4 0
34 04
=
4 4
81
=
4
81
So A2 =
units 2 .
4
1 81
A1 + A2 = +
4 4
1
= 20 units 2
2

Even and odd functions

Some functions have special properties.

For odd functions,


# a f (x) dx = 0
a

For even functions,


# a f (x) dx = 2 #0
a a
f (x) dx
110 Maths In Focus Mathematics HSC Course

EXAMPLES

1. Find the area between the curve y = x 3, the x-axis and the lines x = 2
and x = 2.

Solution

# 2 x 3 dx = 0
2

y = x 3 is an odd function
Area = 2 # x 3 dx
2
since f ( x) = f (x) .
0

x4 2
= 2; E
4 0
24 04
= 2c m
4 4
=8
So the area is 8 units2.

2. Find the area between the curve y = x 2, the x-axis and the lines x = 4
and x = 4.

Solution
y = x 2 is an even function
# 4 x 2 dx = 2 #0
4 4
since f ( x) = f (x) . x 2 dx
x3 4
= 2< F
3 0
43 03
= 2c m
3 3
2
= 42
3
So the area is 42 2 units2.
3

3.6 Exercises
1. Find the area enclosed between 5. Find the area bounded by the
the curve y = 1 x 2 and the x-axis. curve y = x 2 + 9x 20 and the
x-axis.
2. Find the area bounded by the
curve y = x 2 9 and the x-axis. 6. Find the area enclosed between
the curve y = 2x 2 5x + 3 and
3. Find the area enclosed between
the x-axis.
the curve y = x 2 + 5x + 4 and the
x-axis. 7. Find the area enclosed between
the curve y = x 3, the x-axis and
4. Find the area enclosed between
the lines x = 0 and x = 2.
the curve y = x 2 2x 3 and the
x-axis.
Chapter 3 Integration 111

8. Find the area enclosed between 15. Find the area enclosed between
the curve y = x 4, the x-axis and 1
the curve y = 2 , the x-axis and
the lines x = 1 and x = 1. x
the lines x = 1 and x = 3.
9. Find the area enclosed between
16. Find the area enclosed between
the curve y = x 3, the x-axis and
2
the lines x = 2 and x = 2. the curve y = , the x-axis
(x 3) 2
and the lines x = 0 and x = 1.
10. Find the area enclosed between
the curve y = x 3, the x-axis and 17. Find the area bounded by the
the lines x = 3 and x = 2. curve y = x , the x-axis and the
line x = 4.
11. Find the area bounded between
the curve y = 3x 2, the x-axis and 18. Find the area bounded by the
the lines x = 1 and x = 1. curve y = x + 2 , the x-axis and
the line x = 7.
12. Find the area enclosed between
the curve y = x 2 + 1, the x-axis 19. Find the area bounded by the
and the lines x = 2 and x = 2. curve y = 4 x 2 , the x-axis and
the y-axis in the rst quadrant.
13. Find the area enclosed between
the curve y = x 2, the x-axis and 20. Find the area bounded by the
the lines x = 3 and x = 2. x-axis, the curve y = x 3 and the
lines x = a and x = a.
14. Find the area enclosed between the
curve y = x 2 + x, and the x-axis.

Areas Enclosed by the y-axis

To nd the area between a curve and the y-axis, we change the subject of the
equation of the curve to x. That is,
x = f (y) .
The denite integral is given by

#a f (y) dy or # x dy
b b

Since x is positive on the right-hand side of the y-axis, the denite


integral is positive. Since x is negative on the left-hand side of the y-axis, the
denite integral is negative.
112 Maths In Focus Mathematics HSC Course

EXAMPLES

1. Find the area enclosed by the curve x = y2, the y-axis and the lines
y = 1 and y = 3.

Solution
Area = # y 2 dy
3

y3 3
== G
3 1
33 13
=
3 3
2
= 8 units 2
The integral is positive 3
since the area is to the
right of the y-axis.

2. Find the area enclosed by the curve y = x2, the y-axis and the lines
y = 0 and y = 4 in the rst quadrant.

Solution

Change the subject of the equation to x.


y = x2
` y =x
In the rst quadrant,
y =x
1
i.e. y 2 = x
1
Area = # y 2 dy
4

0
R 3 V4
S y2 W
=S W
SS 3 WW
T 2 X0 4
2 y3
=> H
3 0
2 43 2 03
=
3 3
1
= 5 units 2
3
Chapter 3 Integration 113

3. Find the area enclosed between the curve y = x + 1 , the y-axis and
the lines y = 0 and y = 3.

Solution

Some of the area is on


the left of the y-axis.

y= x+1
y2 = x + 1
y2 1 = x
#0 ^ y 2 1 h dy + # ^ y 2 1 h dy
1 3
Area =
1
1 3
y3 y3
= = yG + = yG
3 0 3 1

13 03 33 13
= c 1m c 0m + c 3m c 1m
3 3 3 3
2 2
= +6 +
3 3
2 2
= +6
3 3
1
= 7 units 2
3

3.7 Exercises
1. Find the area bounded by the 4. Find the area between the lines
y-axis, the curve x = y2 and the y = x 1, y = 0 and y = 1 and the
lines y = 0 and y = 4. y-axis.

2. Find the area enclosed between 5. Find the area bounded by the line
the curve x = y3, the y-axis and y = 2x + 1, the y-axis and the lines
the lines y = 1 and y = 3. y = 3 and y = 4.

3. Find the area enclosed between 6. Find the area bounded by the
the curve y = x2, the y-axis and curve y = x , the y-axis and the
the lines y = 1 and y = 4 in the lines y = 1 and y = 2.
rst quadrant.
114 Maths In Focus Mathematics HSC Course

7. Find the area bounded by the 12. Find the area enclosed between
curve x = y 2 - 2y - 3 and the y-axis. the curve y = x3 - 2 and the y-axis
between y = -1 and y = 25.
8. Find the area bounded by the
curve x = -y 2 - 5y - 6 and the 13. Find the area enclosed between
y-axis. the lines y = 4 and y = 1 - x in the
second quadrant.
9. Find the area enclosed by the
curve y = 3x 5 , the y-axis and 14. Find the area enclosed between the
the lines y = 2 and y = 3. y-axis and the curve x = y(y - 2).

10. Find the area enclosed between 15. Find the area bounded by the
1 curve y = x4 + 1, the y-axis and the
the curve y = 2 , the y-axis and
x lines y = 1 and y = 3 in the rst
the lines y = 1 and y = 4 in the
quadrant, correct to 2 signicant
rst quadrant.
gures.
11. Find the area enclosed between
the curve y = x3, the y-axis and
the lines y = 1 and y = 8.

Sums and Differences of Areas

EXAMPLES

1. Find the area enclosed between the curve y = x2, the y-axis and the
lines y = 0 and y = 4 in the rst quadrant.

Solution

A = Area of rectangle # x 2 dx
2

x3 2
= (4 2) < F
3 0
8 0
= 8 c m
3 3
This example was done before 1
by finding the area between = 5 units 2
3
the curve and the y-axis.
Chapter 3 Integration 115

2. Find the area enclosed between the curves y = x2, y = (x 4)2 and the x-axis.

Solution
Solve simultaneous equations to nd the intersection of the curves.
y = x2 (1)
y = ]x 4 g2 (2 )
` x2 = ] x 4 g 2
= x 2 8x + 16
0 = 8x + 16
8x = 16
x=2

Area = # x2 dx + # ] x 4 g2 dx
2 4

0 2
4
x3 2 < ] x 4 g 3 F
=< F +
3 0 3 2

8 0 0 8
= c m+c m
3 3 3 3
1
= 5 units2
3

3. Find the area enclosed between the curve y = x2 and the line y = x + 2.

Solution
Solve simultaneous equations to nd the intersection of the curve and line.
y = x2 (1)
y=x+2 (2)
` x2 = x + 2
x2 x 2 = 0
(x 2 ) (x + 1 ) = 0
x = 2 or 1

Area = # ] x + 2 g dx # x 2 dx
2 2

1 1

= # ] x + 2 x 2 g dx
2

x2 x3 2
=< + 2x F
2 3 1
22 23 (1)2 (1)3
=c + 2 (2) m d + 2 (1) n
2 3 2 3
8 1 1
= c2 + 4 m c 2 + m
3 2 3
1
= 4 units2
2
116 Maths In Focus Mathematics HSC Course

3.8 Exercises
1. Find the area bounded by the line 9. Find the area enclosed between
y = 1 and the curve y = x2. the curve y = x3, the x-axis and
the line y = -3x + 4.
2. Find the area enclosed between
the line y = 2 and the curve 10. Find the area enclosed by the
y = x2 + 1. curves y = (x - 2)2 and y = (x - 4)2.

3. Find the area enclosed by the 11. Find the area enclosed between
curve y = x2 and the line y = x. the curves y = x2 and y = x3.

4. Find the area bounded by the 12. Find the area enclosed by the
curve y = 9 - x2 and the line y = 5. curves y = x2 and x = y2.

5. Find the area enclosed between 13. Find the area bounded by the
the curve y = x2 and the line curve y = x2 + 2x - 8 and the line
y = x + 6. y = 2x + 1.

6. Find the area bounded by the 14. Find the area bounded by the
curve y = x3 and the line y = 4x. curves y = 1 - x2 and y = x2 - 1.

7. Find the area enclosed between 15. Find the exact area enclosed
the curves y = (x - 1)2 and between the curve y = 4 x 2
y = (x + 1)2. and the line x - y + 2 = 0.

8. Find the area enclosed between


the curve y = x2 and the line
y = -6x + 16.

Volumes
The volume of a solid can be found by rotating an area under a curve about
the x-axis or the y-axis.
Chapter 3 Integration 117

Volumes about the x-axis

V = # y 2 dx
b

Proof

Take a disc of the solid with width x.

The disc is approximately a


cylinder with radius y and
height x.

Volume = r2 h
= y 2 x
Taking the sum of an innite number of these discs,
Volume = lim y2 x
x " 0

= # y2 dx
b

= # y2 dx
b

a
118 Maths In Focus Mathematics HSC Course

EXAMPLES

1. Find the volume of the solid of revolution formed when the curve
x2 + y2 = 9 is rotated about the x-axis between x = 1 and x = 3.

Solution

x2 + y2 = 9
` y2 = 9 x2
V = # y2 dx
b

= # ^ 9 x2 h dx
3

1
3
x3
= < 9x F
3 1
1
= c (27 9) (9 ) m
3
28
= units 3
3

2. Find the volume of the solid formed when the line y = 3x - 2 is


rotated about the x-axis from x = 1 to x = 2.

Solution
y = 3x 2
y 2 = ] 3x 2 g 2
#a
b
V= y2 dx

#1
2
= ] 3x 2 g2 dx
2
Use the function of a ] 3x 2 g3
function rule = < F
33 1
43 13 n
= d
9 9
63
=
9
= 7 units3
Chapter 3 Integration 119

PROBLEM

What is wrong with the working out of this volume?


Find the volume of the solid of revolution formed when the curve
y = x 2 + 1 is rotated about the x-axis between x = 0 and x = 2.

Solution
y = x2 + 1
y2 = ^ x2 + 1 h
2
`
V = # y2 dx
b

= # ^ x2 + 1 h dx
2 2

0
3 2
^ x2 + 1 h
== G
3 0

^ 22 + 1 h 3 ^ 02 + 1 h 3
=e o
3 3
53 13 n
=d
3 3
124
= units3
3
What is the correct volume?

Volumes about the y-axis

The formula for rotations about the y-axis is similar to the formula for the
x-axis.

The proof of this result is


similar to that for volumes
about the x-axis.

V = # x2 dy
b

a
120 Maths In Focus Mathematics HSC Course

EXAMPLE

Find the volume of the solid formed when the curve y = x2 - 1 is rotated
about the y-axis from y = -1 to y = 3.

Solution

y = x2 1
` y + 1 = x2
#a
b
V= x2 dy

# 1 ^ y + 1 h dy
3
=
3
y2
== + yG
2 1

9 1
= < c + 3 m c 1 mF
2 2
= 8 units3

3.9 Exercises
1. Find the volume of the solid 3. Find the volume of the solid of
of revolution formed when the revolution that is formed when
curve y = x2 is rotated about the the curve y = x2 + 2 is rotated about
x-axis from x = 0 to x = 3. the x-axis from x = 0 to x = 2.

2. Find the volume of the solid 4. Determine the volume of the


formed when the line y = x + 1 is solid formed when the curve
rotated about the x-axis between y = x3 is rotated about the x-axis
x = 2 and x = 7. from x = 0 to x = 1.
Chapter 3 Integration 121

5. Find the volume of the solid 15. Find the volume of the solid
formed when the curve x = y2 - 5 formed when the curve
is rotated about the x-axis between y = x + 3 is rotated about the
x = 0 and x = 3. x-axis from x = 1 to x = 6.

6. Find the volume of the solid of 16. Find the volume of the solid
revolution formed by rotating the formed when the curve y = x is
line y = 4x - 1 about the x-axis rotated about the y-axis between
from x = 2 to x = 4. y = 1 and y = 4.

7. Find the volume of the 17. Find the volume of the solid
hemisphere formed when the formed when the curve
curve x2 + y2 = 1 is rotated about y = 4 x 2 is rotated about the
the x-axis between x = 0 and y-axis from y = 1 to y = 2.
x = 1.
18. Find the volume of the solid
8. The parabola y = (x + 2) is rotated
2
formed when the line
about the x-axis from x = 0 to x + 3y - 1 = 0 is rotated about the
x = 2. Find the volume of the y-axis from y = 1 to y = 2.
solid formed.
19. Find the volume of the solid
9. Find the volume of the spherical formed when the line
cap formed when the curve x + 3y - 1 = 0 is rotated about the
x2 + y2 = 4 is rotated about the x-axis from x = 0 to x = 8.
y-axis from y = 1 to y = 2.
20. The curve y = x3 is rotated about
10. Find the volume of the the y-axis from y = 0 to y = 1.
paraboloid formed when y = x2 is Find the volume of the solid
rotated about the y-axis from formed.
y = 0 to y = 3.
21. Find the volume of the solid of
11. Find the volume of the solid revolution formed if the area
formed when y = x2 - 2 is rotated enclosed between the curves
about the y-axis from y = 1 to y = x2 and y = (x - 2)2 is rotated
y = 4. about the x-axis.

12. The line y = x + 2 is rotated about 22. The area bounded by the curve
the y-axis from y = -2 to y = 2. y = x2 and the line y = x + 2 is
Find the volume of the solid rotated about the x-axis. Find the
formed. exact volume of the solid formed.

13. Determine the volume of the 23. Show that the volume of a sphere
solid formed when the curve is given by the formula
y = x3 is rotated about the x-axis 4
V = r3 by rotating the
from x = -1 to x = 4. 3
semicircle y = r 2 x 2 about the
14. Find the volume of the solid
x-axis.
formed when the curve y = x is
rotated about the x-axis between
x = 0 and x = 5.
122 Maths In Focus Mathematics HSC Course

Application

The volume of water in a small lake is to be measured by finding the depth of


water at regular intervals across the lake.

What could the volume of this lake be?

The table below shows the results of the measurements, where x stands for the
regular intervals chosen, in metres, and y is the depth in metres.

x (m) 0 50 100 150 200 250 300

y (m) 10.2 39.1 56.9 43.2 28.5 15.7 9.8

Using Simpsons rule to find an approximation for the volume of the lake, correct
to 2 significant figures:

#
b
V= y2 dx
a

= #
300
y 2 dx
0

# h
b
f ] x g dx Z 8 _ y0 + y6 i + 4 _ y1 + y3 + y5 i + 2 _ y2 + y4 i B
a 3
ba
where h=
6
300 0
=
6
= 50
Chapter 3 Integration 123

# 50
300
y 2 dx Z 6 ] 10.22 + 9.82 g + 4 ] 39.12 + 43.22 + 15.72 g + 2 ] 56.92 + 28.52 g @
0 3
= 381 099.3

#
300
` V= y 2 dx
0

= 381 099.3
Z 1197 258.866
= 1 200 000 correct to 2 significant figures
So the volume of the lake is approximately 1 200 000 m3 of water.
124 Maths In Focus Mathematics HSC Course

Test Yourself 3
1. (a) Use the trapezoidal rule with 2 (b)
subintervals to nd an approximation
2 dx
to # .
1 x2

(b) Use integration to nd the exact


2 dx
value of # .
1 x2

2. Find the indenite integral (primitive


function) of
(a) 3x + 1
5x 2 x
(b) x
(c) x
6. Find the area bounded by the curve
(d) ] 2x + 5 g7
y = x 2, the x-axis and the lines x = 1 and
3. In an experiment, the velocity of a x = 2.
particle over time was found and the
results placed in the table below. 7. Use Simpsons rule with 5 function
values to nd the approximate area
1
t 0 1 2 3 4 5 6 enclosed between the curve y = x , the
v 1.3 1.7 2.1 3.5 2.8 2.6 2.2 x-axis and the lines x = 1 and x = 3.

8. Find the area enclosed between the


Use Simpsons rule to nd the
curves y = x 2 and y = 2 x 2 .
approximate value of # f ] t g dt
6

0
9. The line y = 2x 3 is rotated about the
4. Evaluate
x-axis from x = 0 to x = 3. Find the
#0
2
(a) ] x3 1 g dx volume of the solid of revolution.
# 1 x5 dx
1
#1 3x 4 2x 3 + x 2 1
2
(b) 10. Evaluate dx.
x2
#0
1
(c) ] 3x 1 g4 dx 11. Find the area bounded by the curve
5. Draw a primitive function of the y = x 3, the y-axis and the lines y = 0
following curves. and y = 1.
(a) 12. Find the indenite integral (primitive
function) of ] 7x + 3 g11 .

13. Find the area bounded by the curve


y = x 2 x 2, the x-axis and the lines
x = 1 and x = 3.

14. Find the volume of the solid formed if


the curve y = x 2 + 1 is rotated about the
(a) x-axis from x = 0 to x = 2
(b) y-axis from y = 1 to y = 2
Chapter 3 Integration 125

15. (a) Change the subject of y = ] x + 3 g2 to x. 18. Find the volume of the solid formed
(b) Find the area bounded by the curve if the curve y = x 3 is rotated about the
y = ] x + 3 g2, the y-axis and the lines y = 9 y-axis from y = 0 to y = 1.
and y = 16 in the first quadrant.
19. Find
(c) Find the volume of the solid formed
if this area is rotated about the y-axis.
(a) # 5 ] 2x 1 g4 dx
# 34x
5
(b) dx
#0
4
16. Evaluate ] 3t 6t + 5 g dt .
2

17. Find the area bounded by the curve


y = x2 + 2x 15 and the x-axis.

Challenge Exercise 3
1. (a) Find the area enclosed between the 7. Find the area enclosed between the
curves y = x 3 and y = x 2 . 3
curve y = , the y-axis and the lines
(b) Find the volume of the solid formed x2
if this area is rotated about the x-axis. y = 1 and y = 3, using the trapezoidal rule
with 4 subintervals.
2. (a) Show that f ] x g = x 3 + x is an odd
function. 8. Find the exact volume of the solid
1
(b) Hence find the value of # f ] x g dx.
2
formed by rotating the curve y = x about
2
(c) Find the total area between f ] x g, the the x-axis between x = 1 and x = 3.
x-axis and the lines x = 2 and x = 2.
9. Show that the area enclosed between the
3. The curve y = 2 x is rotated about the 1
curve y = x , the x-axis and the lines x = 0
x-axis between x = 1 and x = 2. Use
Simpsons rule with 3 function values to and x = 1 is infinite by using Simpsons
find an approximation of the volume of rule with 3 function values.
the solid formed, correct to 3 significant
10. (a) Sketch the curve y = x ] x 1 g ] x + 2 g .
figures.
(b) Find the total area enclosed between
4. Find the area enclosed between the the curve and the x-axis.
curves y = ] x 1 g2 and y = 5 x 2 .
11. Find the exact area bounded by the
5. (a) Differentiate ] x 4 1 g9 . parabola y = x 2 and the line y = 4 x.
(b) Hence find # x 3 ] x 4 1 g8 dx. 12. Find the volume of the solid formed
2x 2 + 1 when the curve y = ] x + 5 g2 is rotated
6. (a) Differentiate .
3x 2 4 about the y-axis from y = 1 to y = 4.
x
#0
1
(b) Hence evaluate dx.
^ 3x 2 4 h 2
126 Maths In Focus Mathematics HSC Course

13. (a) Find the derivative of x x + 3 . 16. For the shaded region below, find
x+2 (a) the area
(b) Hence find # dx.
x+3 (b) the volume when this area is rotated
about the y-axis.
14. (a) Evaluate # ] x2 x + 1 g dx.
3

1
(b) Use Simpsons rule with 3 function
#1
3
values to evaluate ] x2 x + 1 g dx and
show that this gives the exact value.

15. Find the area enclosed between the


curves y = x and y = x 3 .
4
Exponential
and Logarithmic
Functions
TERMINOLOGY

Exponential equation: Equation where the pronumeral is Logarithm: A logarithm is an index. The logarithm is the
the index or exponent such as 3 x = 9 power or exponent of a number to a certain base
i.e. 2 x = 8 is the same as log2 8 = x
Exponential function: A function in the form y = a x where
the variable x is a power or exponent
Chapter 4 Exponential and Logarithmic Functions 133

INTRODUCTION
THIS CHAPTER INTRODUCES A new irrational number, e, that has
special properties in calculus. You will learn how to differentiate
and integrate the exponential function f (x) = e x.
The denition and laws of logarithms are also introduced in
this chapter, as well as differentiation and integration involving
logarithms.

DID YOU KNOW?

John Napier (15501617), a Scottish theologian and an amateur mathematician, was the first to
invent logarithms. These natural, or Naperian, logarithms were based on e. Napier was also
one of the first mathematicians to use decimals rather than fractions. He invented the notation of
the decimal, using either a comma or a point. The point was used in England, but a few European
countries still use a comma.
Henry Briggs (15611630), an Englishman who was a professor at Oxford, decided that
logarithms would be more useful if they were based on 10 (our decimal system). These are called
common logarithms. Briggs painstakingly produced a table of logarithms correct to 14 decimal
places. He also produced sine tablesto 15 decimal placesand tangent tablesto 10 decimal
places.
The work on logarithms was greatly appreciated by Kepler, Galileo and other astronomers
at the time, since they allowed the computation of very large numbers.

Differentiation of Exponential Functions


When differentiating exponential functions f (x) = a x from rst principles, an
interesting result can be seen. The derivative of any exponential function gives
a constant which is multiplied by the original function.

EXAMPLE

Sketch the derivative (gradient) function of y = 10 x.

Solution

CONTINUED
134 Maths In Focus Mathematics HSC Course

The graph of y = 10 x always has a positive gradient that is becoming steeper.


So the derivative function will always be positive, becoming steeper.

The derivative function of an exponential function will always have a


shape similar to the original function.
We can use differentiation from rst principles to nd how close this
derivative function is to the original function.

EXAMPLE

Differentiate f (x) = 10x from rst principles.

Solution

f (x + h ) f ( x )
f l(x) = lim
h "0 h
10 x + h 10 x
= lim
h "0 h
10 x (10 h 1)
= lim
h "0 h
You can explore 10 h 1
limits using a
= 10 lim
x
h "0 h
graphics package
on a computer or a 10 h 1
graphical calculator.
Using the 10 x key on the calculator, and nding values of when h
h
is small, gives the result:
f l(x) Z 2.3026 10x
or d
(10x ) Z 2.3026 10x
dx

Drawing the graphs of y = 2.3026 10 x and y = 10 x together shows


how close the derivative function is to the original graph.
y
y = 10 x
y = 2.3026 10 x
12
10
8
6
4
2
x
2 1 1 2
Chapter 4 Exponential and Logarithmic Functions 135

Similar results occur for other exponential functions. In general,


d x
(a ) = kax where k is a constant.
dx

Application

dy You will study exponential


If y = a x then = ka x growth and decay in
dx
Chapter 6.
= ky

This means that the rate of change of y is proportional to y itself. That is, if y is
small, its rate of change is small, but if y is large, then it is changing rapidly.

This is called exponential growth (or decay, if k is negative) and has many
applications in areas such as population growth, radioactive decay, the cooling of
objects, the spread of infectious diseases and the growth of technology.

Different exponential functions have different values of k.

EXAMPLES
d x
1. (2 ) Z 0.6931 2 x.
dx
y
y = 0.6931 2x
12 y = 2x
10
8
6
4
2
x
3 2 1 1 2 3

d x
2. (3 ) Z 1.0986 3 x.
dx

12 y = 3x
y = 1.0986 3x
10
8
6
4
2
x
3 2 1 1 2 3
136 Maths In Focus Mathematics HSC Course

Notice that the derivative function of y = 3 x is very close to the original


function.
We can nd a number close to 3 that gives exactly the same graph for the
derivative function. This number is approximately 2.71828, and is called e.

e is an irrational number d x
like . (e ) = e x
dx

DID YOU KNOW?

The number e was linked to logarithms before this useful result in calculus was known. It is
A transcendental
a transcendental (irrational) number. This was proven by a French mathematician, Hermite,
number is a number
in 1873. Leonhard Euler (170783) gave e its symbol, and he gave an approximation of e to
beyond ordinary
23 decimal places. Currently, e is known to about 100 000 decimal places.
numbers. Another
Euler studied mathematics, theology, medicine, astronomy, physics and oriental languages.
transcendental
He did extensive research into mathematics and wrote more than 500 books and papers.
number is .
Euler gave mathematics much of its important notation. He caused to become standard
notation and used i for the square root of 1. He first used small letters to show the sides of
triangles and the corresponding capital letters for their opposite angles. Also, he introduced
the symbol S for sums and f(x) notation.

ex KEY

Use this key to find powers of e.


For example, to find e2:
Press SHIFT e x 2 = e 2 7.389056099
To find e:
Press SHIFT e x 1 = e 1 2.718281828

EXAMPLES

1. Sketch the curve y = e x.

Solution
Use ex on your calculator to draw up a table of values:

x 3 2 1 0 1 2 3
y 0.05 0.1 0.4 1 2.7 7.4 20.1
Chapter 4 Exponential and Logarithmic Functions 137

2. Differentiate 5e x.

Solution
d x
(e ) = e x
dx
d d
` (5e x) = 5 (e x)
dx dx
= 5e x

3. Find the equation of the tangent to the curve y = 3e x at the point (0, 3).

Solution
dy
= 3e x
dx
dy
At (0, 3), = 3e 0
dx
dy
=3 gives the gradient of
dx
` m=3 the tangent.

Equation y y1 = m (x x1)
y 3 = 3 (x 0 )
= 3x
y = 3x + 3

CONTINUED
138 Maths In Focus Mathematics HSC Course

2x + 3
4. Differentiate .
ex
Solution
This is the quotient rule
from Chapter 8 of the dy u lv v lu
Preliminary Course book. =
dx v2
2 . e x e x ( 2 x + 3)
=
( e x) 2
2e x 2xe x 3e x
=
e 2x
e x 2xe x
=
e 2x
e x (1 + 2x)
=
e 2x
(1 + 2x)
=
ex

4.1 Exercises
1. Find, correct to 2 decimal places, (j) x 2 e x
the value of (k) (2x + 1) e x
(a) e 1.5 ex
(b) e 2 (l)
7x 3
(c) 2e 0.3 5x
1 (m)
(d) 3 ex
e
(e) 3e 3.1 4. If f (x) = x 3 + 3x e x, nd f l(1)
and f m(1) in terms of e.
2. Sketch the curve
(a) y = 2e x 5. Find the exact gradient of the
(b) y = e x tangent to the curve y = e x at the
(c) y = e x point (1, e).

6. Find the exact gradient of the


3. Differentiate
normal to the curve y = e x at the
(a) 9e x
point where x = 5.
(b) e x
(c) e x + x 2 7. Find the gradient of the tangent
(d) 2x 3 3x 2 + 5x e x to the curve y = 4e x at the point
(e) (e x + 1) 3 where x = 1.6, correct to 2
(f) (e x + 5)7 decimal places.
(g) (2e x 3) 2
8. Find the equation of the tangent
(h) xe x
to the curve y = e x at the point
ex
(i) x (1, e).
Chapter 4 Exponential and Logarithmic Functions 139

9. Find the equation of the normal 11. Find the rst and second
to the curve y = e x at the point derivatives of y = 7e x. Hence show
where x = 3, in exact form. d 2y
that = y.
10. Find the stationary point on the dx 2
curve y = xe x and determine its d2y
12. If y = 2e x + 1, show that = y 1.
nature. Hence sketch the curve. dx 2

Function of a function rule

Remember that the function of a function rule uses the result


dy dy du You studied this in Chapter 8 of
= . the Preliminary Course book.
dx du dx

EXAMPLE

+ 5x 3
Differentiate e x .
2

Solution
Let u = x 2 + 5x 3
Then y = e u
du dy
= 2x + 5 and = eu
dx du
dy dy du
=
dx du dx
= e u (2x + 5)
= e x + 5x 3 (2x + 5)
2
Can you see a quick way to
do this?
= (2x + 5) e x + 5x 3
2

dy
If y = e f (x) then = f l(x) ef (x)
dx

Proof
Let u = f (x)
Then y = eu
dy du
= e u and = f l(x)
du dx
dy dy du
=
dx du dx
= e u f l(x)
= f l(x) e f (x)
140 Maths In Focus Mathematics HSC Course

EXAMPLES

1. Differentiate e 5x 2

Solution
y l = f l(x) e f (x)
= 5e 5x 2

2. Differentiate x 2 e 3x.

Solution
This is the product rule from
Chapter 8 of the Preliminary dy
Course book. = ulv + vlu
dx
= 2x . e3x + 3e3x . x2
= xe3x (2 + 3x)

d2 y
3. Given y = 2e 3x + 1, show that = 9 (y 1) .
dx 2
Solution
y = 2e 3x + 1
dy
= 6e 3x
dx
d2 y
= 18e 3x
dx 2
= 9 (2e 3x)
= 9 (2e 3x + 1 1)
= 9 (y 1)

4.2 Exercises
1. Differentiate e 3x
(a) e 7x (m)
x2
(b) e x 3 5x
(n) x e
(c) e 6x 2 e 2x + 1
(o)
(d) e x + 1
2

2x + 5
(e) ex + 5x + 7
3

2. Find the second derivative of


(f) e 5x
(e 2x + 1)7.
(g) e 2x
(h) e 10x 3. If f (x) = e 3x 2, nd the exact value
(i) e 2x + x of f l(1) and f m(0) .
(j) x 2 + 2x + e 1 x
4. Find the gradient of the tangent
(k) (x + e 4x)5
to the curve y = e 5x at the point
(l) xe 2x
where x = 0.
Chapter 4 Exponential and Logarithmic Functions 141

5. Find the equation of the tangent d2y dy


to the curve y = e 2x 3x at the 11. Prove 3 + 2y = 0, given
dx 2
dx
point (0, 1).
y = 3e 2x .
6. Find the exact gradient of the
d2 y
normal to the curve y = e 3x at the 12. Show = b 2 y for y = ae bx .
point where x = 1. dx 2
13. Find the value of n if y = e 3x
7. Find the equation of the tangent to satises the equation
the curve y = e x at the point (1, e).
2

d 2y dy
+2 + ny = 0.
8. If f (x) = 4x 3 + 3x 2 e 2x, nd dx 2
dx
f m(1) in terms of e.
14. Sketch the curve y = e x + x 2,
2

9. Find any stationary points on showing any stationary points


the curve y = x 2 e 2x and sketch the and inexions.
curve. x2 + 1
15. Sketch the curve y = ,
ex
10. If y = e 4x + e 4x, show that showing any stationary points
d2y and inexions.
= 16y.
dx 2

Integration of Exponential Functions


d x
Since (e ) = e x, then the reverse must be true.
dx

# e x dx = e x + C

To nd the indefinite integral (primitive function) when the function of


a function rule is involved, look at the derivative rst.

EXAMPLE

Differentiate e 2x + 1.
Hence nd # 2e 2x + 1 dx.
Find # e 2x + 1 dx.
Solution
d 2x + 1
(e ) = 2 e 2x + 1
dx Integration is the inverse of
` # 2e 2x + 1 dx = e 2x + 1 + C differentiation.

# e 2x + 1 dx = 12 # 2e 2x + 1 dx
1 2x + 1
= e +C
2
142 Maths In Focus Mathematics HSC Course

In general

# e ax + b dx = 1a e ax + b + C

Proof
d ax + b
(e ) = ae ax + b
dx
` # ae ax + b dx = e ax + b + C

# e ax + b dx = 1a # ae ax + b dx
1
= a e ax + b + C

EXAMPLES

1. Find # (e 2x e x) dx.
Solution
# (e 2x e x) dx = 12 e 2x (11) e x + C
1 2x
= e + ex + C
2

2. Find the exact area enclosed between the curve y = e 3x, the x-axis and
the lines x = 0 and x = 2.

Solution

Area = # e 3x dx
2

0
2
1
= ; e 3x E
3 0
1 6 1 0
= e e
3 3
1 6
= (e e 0)
3
1
= (e 6 1) units 2
3

3. Find the volume of the solid of revolution formed when the curve
y = e x is rotated about the x-axis from x = 0 to x = 2.

Solution
y = ex
Use index laws to simplify (e x) 2. ` y 2 = ( e x) 2
= e 2x
Chapter 4 Exponential and Logarithmic Functions 143

V = # y2 dx
b

= # e2x dx
2

0
2
1
= ; e2x E
2 0

1 4 1 0
=c e e m
2 2
1 4 1
=c e m
2 2
4
= (e 1) units3
2

4.3 Exercises
1. Find these indenite integrals.
#5
6
(c) (ex + 5 + 2x 3) dx
(a) # e dx 2x

#0
1
(d) (e 3t + 4 t) dt
(b) # e 4x dx
#1
2
(e) (e 4x + e 2x) dx
(c) # e x dx
(d) # e 5x dx 4. Find the exact area enclosed by
the curve y = 2e 2x, the x-axis and
(e) # e 2x dx
the lines x = 1 and x = 2.
(f) # e 4x + 1 dx
5. Find the exact area bounded by
(g) # 3e 5x dx the curve y = e 4x 3, the x-axis and
(h) # e 2t dt the lines x = 0 and x = 1.

(i) # (e 7x 2) dx 6. Find the area enclosed by the


curve y = x + e x, the x-axis and
(j) # (e x 3 + x) dx
the lines x = 0 and x = 2, correct
2. Evaluate in exact form. to 2 decimal places.

#0
1
(a) e 5x dx 7. Find the area bounded by the
curve y = e 5x, the x-axis and the
#0
2
(b) e x dx
lines x = 0 and x = 1, correct to
#1
4
(c) 2e 3x + 4 dx 3 signicant gures.

#2
3
(d) (3x 2 e 2x) dx 8. Find the exact volume of the
solid of revolution formed when
#0
2
(e) (e 2x + 1) dx
the curve y = e x is rotated about
#1
2
(f) (e x x) dx the x-axis from x = 0 to x = 3.

#0
3
(g) (e 2x e x) dx 9. Find the volume of the solid formed
when the curve y = e x + 1 is rotated
3. Evaluate correct to 2 decimal places.
about the x-axis from x = 1 to x = 2,
#1
3
(a) e x dx correct to 1 decimal place.

#0
2
(b) 2e 3y dy
144 Maths In Focus Mathematics HSC Course

10. Use Simpsons rule with 3 12. The curve y = e x + 1 is rotated


function values to nd an about the x-axis from x = 0 to
#1
2
approximation to xe x dx, x = 1. Find the exact volume of
correct to 1 decimal place. the solid formed.

11. (a) Differentiate x 2 e x . 13. Find the exact area enclosed


(b) Hence nd # x(2 + x)e dx. x between the curve y = e 2x and the
lines y = 1 and x = 2.

Application

The exponential function occurs in many fields, such as science and economics.
You will study these formulae in P = P0 e kt is a general formula that describes exponential growth.
Chapter 6. - kt
P = P0 e is a general formula that describes exponential decay.

Logarithms
Logarithm is another name for the index or power of a number. Logarithms
are related to exponential functions, and allow us to solve equations like
2 x = 5. They also allow us to change the subject of exponential equations such
as y = e x to x.

Definition

If y = a x, then x is called the logarithm of y to the base a.

If y = a x, then x = loga y

Logarithm keys

log is used for log 10 x

In is used for log e x


Chapter 4 Exponential and Logarithmic Functions 145

EXAMPLES

1. Find log 10 5.3 correct to 1 decimal place.

Solution
Use the log key.
log 10 5.3 = 0.724275869
= 0.7 correct to 1 decimal place

2. Evaluate log e 80 correct to 3 signicant gures.

Solution
Use the In key.
log e 80 = 4.382026634
= 4.38 correct to 3 significant figures

3. Evaluate log 3 81.

Solution
Let log3 81 = x
Then 3 x = 81 (by definition)
i.e. 3x = 34
` x=4
So log3 81 = 4.

1
4. Find the value of log 2 .
4
Solution
1
Let log2 =x
4
1
Then 2x =
4
1
= 2
2
= 22
` x = 2
1
So log2 = 2.
4
146 Maths In Focus Mathematics HSC Course

Class Investigation

1. Sketch the graph of y = log 2 x.


There is no calculator key for logarithms to the base 2. Use the
denition of a logarithm to change the equation into index form,
and the table of values:
x
y 3 2 1 0 1 2 3

2. On the same set of axes, sketch the curve y = 2 x and the line y = x.
What do you notice?

4.4 Exercises
1. Evaluate 1
(d) log 5
(a) log2 16 25
(b) log4 16 (e) log 7 4 7
(c) log5 125 1
(f) log3 3
(d) log3 3 3
(e) log7 49 1
(g) log4
(f) log7 7 2
(h) log8 2
(g) log5 1
(h) log2 128 (i) log 6 6 6
2
2. Evaluate (j) log2
4
(a) 3 log2 8
4. Evaluate correct to 2 decimal
(b) log5 25 + 1
places.
(c) 3 log3 81
(a) log10 1200
(d) 4 log3 27
(b) log10 875
(e) 2 log10 10 000
(c) loge 25
(f) 1 + log4 64
(d) ln 140
(g) 3 log4 64 + 5
(e) 5 ln 8
(h) 2 + 4 log6 216
log3 9 (f) log10 350 + 4.5
(i) log10 15
2 (g)
log8 64 + 4 2
(j) (h) ln 9.8 + log10 17
log2 8
log10 30
3. Evaluate (i)
loge 30
1
(a) log 2 (j) 4 ln 10 7
2
(b) log 3 3
(c) log4 2
Chapter 4 Exponential and Logarithmic Functions 147

5. Write in logarithmic form. 8. Evaluate y given that log y 125 = 3.


(a) 3 x = y
(b) 5 x = z 9. If log 10 x = 1.65, evaluate x correct
(c) x 2 = y to 1 decimal place.
(d) 2 b = a 10. Evaluate b to 3 signicant gures
(e) b 3 = d if log e b = 0.894.
(f) y = 8 x
(g) y = 6 x 11. Find the value of log2 1. What is
(h) y = e x the value of loga 1?
(i) y = a x
12. Evaluate log5 5. What is the value
(j) Q = e x
of loga a?
6. Write in index form.
13. (a) Evaluate ln e without a
(a) log3 5 = x
calculator.
(b) loga 7 = x
(b) Using a calculator, evaluate
(c) log3 a = b
(i) loge e3
(d) logx y = 9
(ii) loge e2
(e) loga b = y
(iii) loge e5
(f) y = log 2 6
(iv) loge e
(g) y = log 3 x
1
(h) y = log 10 9 (v) loge e
(i) y = ln 4 (vi) eln 2
(j) y = log 7 x (vii) eln 3
(viii) eln 5
7. Solve for x, correct to 1 decimal
(ix) eln 7
place where necessary.
(x) eln 1
(a) log 10 x = 6
(xi) eln e
(b) log 3 x = 5
(c) logx 343 = 3 14. Sketch the graph of y = log e x.
(d) logx 64 = 6 What is its domain and range?
1 15. Sketch y = 10 x, y = log10 x and
(e) log 5 =x
5
y = x on the same number plane.
1
(f) log x 3= What do you notice about the
2
(g) ln x = 3.8 relationship of the curves to
the line?
(h) 3 log 10 x 2 = 10
3 16. Change the subject of y = log e x
(i) log 4 x =
2 to x.
1
(j) log x 4 =
3
148 Maths In Focus Mathematics HSC Course

Class Discussion

1. Investigate these questions on the calculator. Can you see some


patterns?
(a) loge e
(b) loge e2
(c) loge e3
(d) loge e4
(e) loge e5
Can you write a rule for loge ex?

2. Evaluate using a calculator. Can you write a rule to show this pattern?
(a) eln 1
(b) eln 2
(c) eln 3
(d) eln 4
(e) eln 5
Can you write a rule for eln x?

3. Do these rules work if x is negative?

Logarithm laws

Because logarithms are closely related to indices there are logarithm laws that
This corresponds to the correspond to the index laws.
law a m a n = a m + n from
Chapter 1 of the Preliminary
Course book.
loga (xy) = loga x + loga y

Proof
Let x = a m and y = a n
Then m = log a x and n = log a y
xy = a m a n
= am + n
` loga (xy) = m + n (by definition)
= loga x + loga y

This corresponds to the


law a m a n = a m n . x
log a b y l = log a x log a y
Chapter 4 Exponential and Logarithmic Functions 149

Proof
Let x = a m and y = a n
Then m = loga x and n = loga y
x
y =a a
m n

= am n
x
` loga b y l = m n (by definition)
= loga x loga y

This corresponds to the


loga x n = n loga x law (a m) n = a mn .

Proof
Let x = am
Then m = loga x

x n = (a m) n
= a mn
` loga x = mn
n
(by definition)
= n loga x

EXAMPLES

1. Evaluate log6 3 + log6 12.

Solution
log6 3 + log6 12 = log6 (3 12)
= log6 36
log 6 36 = 2, since 6 2 = 36.
=2

2. Given log5 3 = 0.68 and log5 4 = 0.86, nd


(a) log 5 12
(b) log5 0.75
(c) log 5 9
(d) log 5 20

CONTINUED
150 Maths In Focus Mathematics HSC Course

Solution
(a) log5 12 = log5 (3 4)
= log5 3 + log5 4
= 0.68 + 0.86
= 1.54
3
(b) log 5 0.75 = log 5
4
= log 5 3 log 5 4
= 0.68 0.86
= 0.18
(c) log 5 9 = log 5 3 2
= 2 log 5 3
= 2 0.68
= 1.36
(d) log 5 20 = log 5 (5 4)
= log 5 5 + log 5 4
log 5 5 = 1, since 5 1 = 5. = 1 + 0.86
= 1.86

3. Solve log2 12 = log2 3 + log2 x.

Solution
log 2 12 = log 2 3 + log 2 x
= log 2 3x
So 12 = 3x
4=x

4.5 Exercises
1. Use the logarithm laws to 2. Given log7 2 = 0.36 and
simplify log7 5 = 0.83, nd
(a) loga 4 + loga y (a) log7 10
(b) loga 4 + loga 5 (b) log7 0.4
(c) loga 12 loga 3 (c) log7 20
(d) loga b loga 5 (d) log7 25
(e) 3 logx y + logx z (e) log7 8
(f) 2 logk 3 + 3 logk y (f) log7 14
(g) 5 loga x 2 loga y (g) log7 50
(h) loga x + loga y loga z (h) log7 35
(i) log10 a + 4 log10 b + 3 log10 c (i) log7 98
(j) 3 log3 p + log3 q 2 log3 r (j) log7 70
Chapter 4 Exponential and Logarithmic Functions 151

3. Use the logarithm laws to (d) loga x2


evaluate (e) loga xy5
(a) log5 50 log5 2 x2
(f) loga y
(b) log2 16 + log2 4
(g) loga ax
(c) log4 2 + log4 8
a
(d) log5 500 log5 4 (h) loga 2
y
(e) log9 117 log9 13 (i) loga a3y
(f) log8 32 + log8 16 x
(j) loga ay
(g) 3 log2 2 + 2 log2 4
(h) 2 log4 6 (2 log4 3 + log4 2) 6. If loga b = 3.4 and loga c = 4.7,
(i) log6 4 2 log6 12 evaluate
(j) 2 log3 6 + log3 18 3 log3 2 c
(a) loga
b
4. If loga 3 = x and loga 5 = y, nd an (b) loga bc2
expression in terms of x and y for (c) loga (bc)2
(a) loga 15 (d) loga abc
(b) loga 0.6 (e) loga a2c
(c) loga 27 (f) loga b7
(d) loga 25 a
(g) loga c
(e) loga 9 (h) loga a3
(f) loga 75 (i) loga bc4
(g) loga 3a (j) loga b4c2
a
(h) loga
5 7. Solve
(i) loga 9a (a) log4 12 = log4 x + log4 3
125 (b) log3 4 = log3 y log3 7
(j) loga a
(c) loga 6 = loga x 3 loga 2
5. If loga x = p and loga y = q, nd, in (d) log2 81 = 4 log2 x
terms of p and q. (e) logx 54 = logx k + 2 logx 3
(a) loga xy
(b) loga y3
y
(c) loga x

Change of base

Sometimes we need to evaluate logarithms such as log2 7. We use a change of


base formula.

log b x
log a x =
log b a
152 Maths In Focus Mathematics HSC Course

Proof
Let y = log a x
Then x = a y
Take logarithms to the base b of both sides of the equation:
logb x = logb a y
= y logb a
logb x
` =y
logb a
= loga x
You can use the change of base formula to nd the logarithm of any number,
such as log 5 2. You change it to either log 10 x or log e x, and use a calculator.

EXAMPLE

Find the value of log 5 2, correct to 2 decimal places.

Solution
log 2
log 5 2 = (by change of base)
log 5
You can use either log or In
Z 0.430676558
= 0.43

Exponential equations

You can also use the change of base formula to solve exponential equations
such as 5 x = 7.
You studied exponential equations such as 2 x = 8 in the Preliminary
Course. Exponential equations such as 2 x = 9 can be solved by taking
logarithms of both sides, or by using the denition of a logarithm and the
change of base formula.
Chapter 4 Exponential and Logarithmic Functions 153

EXAMPLES

1. Solve 5 x = 7 correct to 1 decimal place.

Solution
5x = 7
Using the denition of a logarithm, this means:
log 5 7 = x
log 7 You can use either
=x (using change of base formula) log or ln.
log 5
1.2 = x

If you do not like to solve the equation this way, you can use the
logarithm laws instead.

Taking logs of both sides:


log 5 x = log 7 n
Use loga x = n loga x
x log 5 = log 7
log 7
` x=
log 5
= 1.2 correct to 1 decimal place

2. Solve 4 y 3 = 9 correct to 2 decimal places.

Solution
4y 3 = 9
Using the logarithm denition and change of base:
log 4 9 = y 3
log 9
=y3
log 4
log 9
+3=y
log 4
4.58 = y
Using the logarithm laws:

Taking logs of both sides:


log 4 y 3 = log 9
(y 3) log 4 = log 9
log 9
y3=
log 4
log 9
y= +3
log 4
= 4.58 correct to 2 decimal place
154 Maths In Focus Mathematics HSC Course

4.6 Exercises
1. Use the change of base formula (d) 7 m = 32
to evaluate to 2 decimal places. (e) 4 k = 50
(a) log4 9 (f) 3t = 4
(b) log6 25 (g) 8 x = 11
(c) log9 200 (h) 2 p = 57
(d) log2 12 (i) 4 x = 81.3
(e) log3 23 (j) 6 n = 102.6
(f) log8 250
(g) log5 9.5 4. Solve, to 1 decimal place.
(h) 2 log4 23.4 (a) 3 x + 1 = 8
(i) 7 log7 108 (b) 53n = 71
(j) 3 log11 340 (c) 2 x 3 = 12
(d) 4 2n 1 = 7
2. By writing each equation as a (e) 7 5x + 2 = 11
logarithm and changing the base, (f) 8 3 n = 5.7
solve the equation correct to (g) 2 x + 2 = 18.3
2 signicant gures. (h) 37xk 3 = 32.9
(a) 4 x = 9 (i) 9 2 = 50
(b) 3 x = 5 (j) 6 2y + 1 = 61.3
(c) 7 x = 14
(d) 2 x = 15 5. Solve each equation correct to
(e) 5 x = 34 3 signicant gures.
(f) 6 x = 60 (a) e x = 200
(g) 2 x = 76 (b) e 3t = 5
(h) 4 x = 50 (c) 2e t = 75
(i) 3 x = 23 (d) 45 = e x
(j) 9 x = 210 (e) 3000 = 100e n
(f) 100 = 20e 3t
3. Solve, correct to 2 decimal places. (g) 2000 = 50e 0.15t
(a) 2 x = 6 (h) 15 000 = 2000e 0.03k
(b) 5 y = 15 (i) 3Q = Qe 0.02t
(c) 3 x = 20 (j) 0.5M = Me 0.016k
Chapter 4 Exponential and Logarithmic Functions 155

Derivative of the Logarithmic Function


Drawing the derivative (gradient) function of a logarithm function gives a
hyperbola.

EXAMPLE

Sketch the derivative function of y = log 2 x.

Solution
The gradient is always positive but is decreasing.

dy 1
If y = log e x then = x
dx

Proof dy 1
= is a special result that
dx dx
dy 1 dy
= can be proved by differentiating
dx dx
from first principles.
dy
Given y = log e x
Then x = e y
dx
= ey
dy
dy 1
` =
dx e y
1
=x
156 Maths In Focus Mathematics HSC Course

Function of a function rule

dy 1 f l(x)
If y = log e f (x), then = f l(x) . =
dx f (x) f (x)

Proof
Let u = f (x)
Then y = log e u
dy 1
` =
du u
du
Also = f l(x)
dx
dy dy du
= .
dx du dx
1
= u . f l(x)
1 .
= f l(x)
f (x)

EXAMPLES

1. Differentiate log e (x 2 3x + 1) .

Solution
d 2x 3
[loge (x 2 3x + 1)] = 2
dx x 3x + 1

x+1
2. Differentiate log e .
3x 4
Solution
x+1
Let y = loge
3x 4
= loge (x + 1) loge (3x 4)
dy 1 3
=
dx x + 1 3x 4
1 (3x 4) 3 (x + 1)
=
(x + 1) (3x 4)
3x 4 3x 3
=
(x + 1) (3x 4)
7
=
(x + 1) (3x 4)
Chapter 4 Exponential and Logarithmic Functions 157

3. Find the gradient of the normal to the curve y = loge (x 3 5) at the


point where x = 2.

Solution
dy
3x 2
=
dx x 3 5
When x = 2,
dy 3 (2) 2
=
dx 2 3 5
m1 = 4
The normal is perpendicular to the tangent This result comes from the
Preliminary Course.
i.e. m1 m2 = 1
4m 2 = 1
1
` m2 =
4

4. Differentiate y = log 2 x.

Solution
y = log 2 x
log e x
=
log e 2
1
= log e x
log e 2
dy 1 1
=
dx log e 2 x
1
=
x log e 2

5. Find the derivative of 2x.

Solution
2 = e ln 2
` 2 x = (e ln 2) x
= e x ln 2
dy
= ln 2e x ln 2
dx
= ln 2 2 x
= 2 x ln 2
158 Maths In Focus Mathematics HSC Course

4.7 Exercises
1. Differentiate 6. Find the gradient of the normal
(a) x + log e x to the curve y = log e (x 4 + x) at the
(b) 1 log e 3x point (1, log e 2).

(c) ln (3x + 1) 7. Find the exact equation of the


(d) loge (x 2 4) normal to the curve y = log e x at
(e) ln (5x 3 + 3x 9) the point where x = 5.
(f) loge (5x + 1) + x 2
8. Find the equation of the tangent
(g) 3x 2 + 5x 5 + ln 4x to the curve y = log e (5x + 4) at
(h) log e (8x 9) + 2 the point where x = 3.
(i) log e (2x + 4) (3x 1)
9. Find the point of inexion on the
4x + 1
(j) log e curve y = x loge x x 2 .
2x 7
(k) (1 + loge x) 5 10. Find the stationary point on the
(l) (ln x x) 9 ln x
curve y = x and determine its
(m) (loge x) 4
nature.
(n) (x 2 + loge x) 6
(o) x log e x 11. Sketch, showing any stationary
log e x points and inexions.
(p) x (a) y = x log e x
(q) (2x + 1) log e x (b) y = (log e x 1) 3
(r) x 3 log e (x + 1) (c) y = x ln x
(s) log e (log e x) 12. Find the derivative of log3 (2x + 5).
ln x
(t)
x2 13. Differentiate
2x
e (a) 3 x
(u) (b) 10 x
loge x
(c) 2 3x 4
(v) e x ln x
(w) 5 (loge x) 2 14. Find the equation of the tangent
to the curve y = 4 x + 1 at the point
2. If f (x) = log e 2 x , nd f l(1).
(0, 4).
3. Find the derivative of log 10 x.
15. Find the equation of the normal
4. Find the equation of the tangent to the curve y = log 3 x at the
to the curve y = log e x at the point where x = 3.
point (2, log e 2).

5. Find the equation of the tangent


to the curve y = log e (x 1) at the
point where x = 2.
Chapter 4 Exponential and Logarithmic Functions 159

Integration and the Logarithmic Function

# dx 1
x = # x dx = log e x + C

f l(x)
# f (x)
dx = log e f (x) + C
Integration is the inverse of
differentiation.

EXAMPLES

1
1. Find the area enclosed between the hyperbola y = x , the x-axis and
the lines x = 1 and x = 2, giving the exact value.

Solution

21
A = # x dx
1

= 7 loge x A 21
= loge 2 loge 1
= loge 2
So area is log e 2 units2.

x2
2. Find # dx.
x3 + 7
Solution
x2 1 3x 2
# dx = # 3
3 x +7
dx
x +7
3

1
= loge (x 3 + 7) + C
3

CONTINUED
160 Maths In Focus Mathematics HSC Course

x+1
3. Find # dx.
x2 + x + 4
Solution
x+1 1 2 (x + 1)
# dx = # 2
2 x + 2x + 4
dx
x + 2x + 4
2

1 2x + 2
= # 2 dx
2 x + 2x + 4
1
= loge (x 2 + 2x + 4) + C
2

4.8 Exercises

1. Find the indenite integral 3. Evaluate correct to 1 decimal


(primitive function) of place.
2 2
#1
3
(a) (a) dx
2x + 5 2x + 5
4x
#2 x dx
5
(b) (b)
2x 2 + 1 +1
2
5x 4
#1 3x dx
7
(c) 5 (c)
x 2 x +2
#0 42 x + 1 dx
3
1 (d)
(d)
2x 2x + x + 1
4 x 1
2
(e) x (e) #3 2 dx
x 2x
5 4. Find the exact area between the
(f)
3x 1
2x 3 curve y = x , the x-axis and the
(g) 2
x 3x lines x = 2 and x = 3.
x
(h) 2 5. Find the exact area bounded by
x +2
1
3x the curve y = , the x-axis and
(i) 2 x1
x +7 the lines x = 4 and x = 7.
x+1
(j) 2
x + 2x 5 6. Find the exact area between the
1
curve y = x , the x-axis and the
2. Find
lines y = x and x = 2 in the rst
(a) # 4x4 1 dx quadrant.

(b) # x dx
+3
7. Find the area bounded by the
x
x2 curve y = 2 , the x-axis and
(c) # dx x +1
2x 7
3
the lines x = 2 and x = 4, correct
x5
(d) # dx to 2 decimal places.
2x 6 + 5
x+3
(e) # dx
x 2 + 6x + 2
Chapter 4 Exponential and Logarithmic Functions 161

8. Find the exact volume of the 12. (a) Show that


solid formed when the curve 3x + 3 1 2 .
= +
1 x2 9 x + 3 x 3
y= is rotated about the x-axis
x 3x + 3
(b) Hence nd # 2 dx.
from x = 1 to x = 3. x 9

9. Find the volume of the solid x6 5


13. (a) Show that =1 .
formed when the curve x1 x1
x6
y=
2
is rotated about the (b) Hence nd # dx.
2x 1 x1
x-axis from x = 1 to x = 5, giving 14. Find the indenite integral
an exact answer. (primitive function) of 3 2x 1.

10. Find the area between the curve 15. Find, correct to 2 decimal places,
y = ln x, the y-axis and the lines the area enclosed by the curve
y = 2 and y = 4, correct to 3 y = log 2 x, the x-axis and the
signicant gures. lines x = 1 and x = 3 by using
Simpsons rule with 3 function
11. Find the exact volume of the values.
solid formed when the curve
y = log e x is rotated about the
y-axis from y = 1 to y = 3.
162 Maths In Focus Mathematics HSC Course

Test Yourself 4
1. Evaluate to 3 signicant gures. 7. Find the volume of the solid formed if
(a) e 2 1 the area bounded by y = e 3x, the x-axis
(b) log 10 95 and the lines x = 1 and x = 2 is rotated
(c) log e 26 about the x-axis.
(d) log 4 7
8. If log 7 2 = 0.36 and log 7 3 = 0.56, nd the
(e) log 4 3
value of
(f) ln 50
(a) log 7 6
(g) e + 3
(b) log 7 8
5e 3
(h) (c) log 7 1.5
ln 4
(i) e ln 6 (d) log 7 14
(j) e ln 2 (e) log 7 3.5

2. Differentiate 9. Find the area enclosed between the curve


(a) e 5x y = ln x, the y-axis and the lines y = 1 and
(b) 2e 1 x y = 3.
(c) log e 4x 10. (a) Use Simpsons rule with 3 function
(d) ln (4x + 5) values to nd the area bounded by the
(e) xe x curve y = ln x, the x-axis and the lines
ln x x = 2 and x = 4.
(f) x
(b) Change the subject of y = ln x to x.
(g) (e x + 1)10
(c) Hence nd the exact area in part (a).
3. Find the indenite integral (primitive
11. Solve
function) of
(a) 3 x = 8
(a) e 4x
(b) 2 3x 4 = 3
x
(b) 2 (c) logx 81 = 4
x 9
(d) log6 x = 2
(c) e x
(e) 12 = 10e 0.01t
1
(d)
x+4 12. Evaluate
#0
1
4. Find the equation of the tangent to the (a) 3e 2x dx
curve y = 2 + e 3x at the point where x = 0. dx
#1
4
(b)
3x 2
5. Find the exact gradient of the normal to
2 2x 3 x 2 + 5x + 3
the curve y = x e x at the point where (c) # x dx
1
x = 2.
13. Find the equation of the tangent to the
6. Find the exact area bounded by the curve curve y = e x at the point (4, e 4).
y = e 2x, the x-axis and the lines x = 2 and
14. Evaluate log 9 8 to 1 decimal place.
x = 5.
Chapter 4 Exponential and Logarithmic Functions 163

15. (a) Find the area bounded by the curve 18. Find the stationary points on the curve
y = e x, the x-axis and the lines x = 1 and y = x 3 e x and determine their nature.
x = 2.
19. Use the trapezoidal rule with 4 strips
(b) This area is rotated about the
to nd the area bounded by the curve
x-axis. Find the volume of the solid of
y = ln (x 2 1), the x-axis and the lines
revolution formed.
x = 3 and x = 5.
16. Simplify
20. Evaluate to 2 signicant gures
(a) 5 loga x + 3 loga y
(a) log 10 4.5
(b) 2 log x k log x 3 + log x p
(b) ln 3.7
17. Find the equation of the normal to the
curve y = ln x at the point (2, ln 2).

Challenge Exercise 4
loge x 1
1. Differentiate . 6. Find the derivative of loge .
e 2x + x 2x 3

7. Use Simpsons rule with 5 function


2. Find the exact gradient of the tangent to
values to nd the volume of the solid
the curve y = e x + log x at the point where
e

formed when the curve y = e x is rotated


x = 1.
about the y-axis from y = 3 to y = 5,
3. If log b 2 = 0.6 and log b 3 = 1.2, nd correct to 2 signicant gures.
(a) log b 6b
8. Differentiate 5 x.
(b) log b 8
(c) log b 1.5b 2 d 2
9. Show that (x loge x) = x (1 + 2 loge x).
dx
4. Differentiate (e 4x + loge x) 9.
#1
3
Hence evaluate 2x (1 + 2 loge x) dx,
5. Find the shaded area, correct to giving an exact answer.
2 decimal places.
10. Find # 3 x dx.
11. (a) Find the point of intersection of the
curves y = log e x and y = log 10 x.
(b) Find the exact equations of the
tangents to the two curves at this point
of intersection.
(c) Find the exact length of the interval
XY where X and Y are the y-intercepts of
the tangents.
164 Maths In Focus Mathematics HSC Course

12. Use Simpsons rule with 3 function d2y dy


values to nd the area enclosed by the 15. Prove 4 5y 10 = 0, given
dx 2
dx
curve y = e 2x, the y-axis and the line y = 3,
y = 3e 2 .
5x

correct to 3 signicant gures.


16. Find the equation of the curve that has
x loge x
13. Find the derivative of . f m(x) = 12e 2x and a stationary point at
ex
( 0 , 3) .
d2y 17. Sketch y = loge (x x 2).
14. If y = e x + e x, show 2
= y.
dx
5
Trigonometric
Functions
TERMINOLOGY
T ERMINOLOGY

Amplitude: Maximum displacement from the mean or Radian: A unit of angular measurement defined as the
centre position in vibrating or oscillating motion length of arc of one unit that an angle subtended at the
centre of a unit circle cuts off
Circular measure: The measurement of an angle as the
length of an arc cut off by an angle at the centre of a Sector: Part of a circle bounded by the centre and arc of a
unit circle. The units are called radians circle cut off by two radii
Period: One complete cycle of a wave or other recurring Segment: Part of a circle bounded by a chord and the
function circumference of a circle
Chapter 5 Trigonometric Functions 167

INTRODUCTION
IN THIS CHAPTER YOU will learn about circular measure, which uses radians
rather than degrees. Circular measure is very useful in solving problems
involving properties of the circle, such as arc length and areas of sectors and
segments, which you will study in this chapter.
You will also study trigonometric graphs in greater detail than in the
Preliminary Course. These graphs have many applications in the movement of
such things as sound, light and waves.

Circular measure is also useful in calculus, and you will learn about
differentiation and integration of trigonometric functions in this chapter.

Circular Measure
Radians

We are used to degrees being used in geometry and trigonometry.


However, they are limited. For example, when we try to draw the graph of
a trigonometric function, it is difcult to mark degrees along the x- or y-axis as
there is no concept of how large a degree is along a straight line. This is one of
the reasons we use radians.
Other reasons for using radians are that they make solving circle problems
simple and they are used in calculus.
A radian is based on the length of an arc in a unit circle (a circle with
radius 1 unit), so we can visualise a radian as a number that can be measured
along a line.

The symbol for 1 radian is 1c


but we usually dont use the
1 unit 1 unitt
symbol as radians are simply
1c distances, or arc lengths.
168 Maths In Focus Mathematics HSC Course

One radian is the angle that an arc of 1 unit


subtends at the centre of a circle of radius 1 unit.

radians = 180

Proof
Circumference of the circle with radius 1 unit is given by:
C = 2 r
= 2 (1)
= 2
The arc length of the whole circle is 2.
` there are 2 radians in a whole circle.
But there are 360 in a whole circle (angle of 2
revolution).
1 unit
So 2 = 360
= 180

EXAMPLES

3
1. Convert into degrees.
2
Solution
Since = 180,
3 3 (180)
=
2 2
= 270

2. Change 60 to radians, leaving your answer in terms in .

Solution
180 = radians

So 1 = radians
180

60 = 60
180
60
=
180

=
3
Chapter 5 Trigonometric Functions 169

3. Convert 50 into radians, correct to 2 decimal places.

Solution
180 = radians

So 1 = radians
180

50 = 50
180
50
=
180
Z 0.87

4. Change 1.145 radians into degrees, to the nearest minute.

Solution
radians = 180
180
` 1 radian =
180
1.145 radians = 1.145
= 65.6
= 65 36l

Press 180 ' 1.145 =


SHIFT , ,,

5. Convert 38 41l into radians, correct to 3 decimal places.

Solution
180 = radians

1 =
180 The calculator may give
the answer in degrees and
38 41l = 38 41l
180 minutes. Simply change this
into a decimal.
= 0.675

6. Evaluate cos 1.145 correct to 2 decimal places.

Solution
For cos 1.145, use radian mode on your calculator.
cos 1.145 = 0.413046135
= 0.41 correct to 2 decimal places
170 Maths In Focus Mathematics HSC Course

180
Notice from the examples that 1 = and 1 radian = . You can use
180
these as conversions rather than starting each time from = 180.

180
To change from radians to degrees: Multiply by .


To change from degrees to radians: Multiply by .
180


Notice that 1 =
180
= 0.017 radians
180
Also 1 radian =
= 57 18l

While we can convert between degrees and radians for any angle, there
are some special angles that we use regularly in this course. It is easier if you
know these without having to convert each time.


= 90
2
= 180
3
= 270
2
2 = 360

= 45
4

= 60
3

= 30
6

5.1 Exercises
1. Change into degrees. 7
(f)
(a) 9
5 4
2 (g)
(b) 3
3 7
5 (h)
(c) 3
4
(i)
7 9
(d)
6 5
(j)
(e) 3 18
Chapter 5 Trigonometric Functions 171

2. Convert into radians in terms 5. Change these radians into


of . degrees and minutes, to the
(a) 135 nearest minute.
(b) 30 (a) 1.09
(c) 150 (b) 0.768
(d) 240 (c) 1.16
(e) 300 (d) 0.99
(f) 63 (e) 0.32
(g) 15 (f) 3.2
(h) 450 (g) 2.7
(i) 225 (h) 4.31
(j) 120 (i) 5.6
(j) 0.11
3. Change into radians, correct to
2 decimal places. 6. Find correct to 2 decimal places.
(a) 56 (a) sin 0.342
(b) 68 (b) cos 1.5
(c) 127 (c) tan 0.056
(d) 289 (d) cos 0.589
(e) 312 (e) tan 2.29 The calculator may give the
(f) sin 2.8 answer in degrees and minutes.
4. Change into radians, to 2 decimal Simply change this into a decimal.
(g) tan 5.3
places.
(h) cos 4.77
(a) 18 34l
(i) cos 3.9
(b) 35 12l
(j) sin 2.98
(c) 101 56l
(d) 88 29l
(e) 50 39l

Trigonometric Results
All the trigonometry that you studied in the Preliminary Course can be done
using radians instead of degrees.

Special triangles

The two triangles that give exact trigonometric ratios can be drawn using
radians rather than degrees.


6

4
2 3
2 1


4 3
1 1
172 Maths In Focus Mathematics HSC Course

Using trigonometric ratios and these special triangles gives the results:

1
sin =
4 2
1
cos =
4 2

tan = 1
4
3
sin =
3 2
1
cos =
3 2

tan = 3
3

1
sin =
6 2
3
cos =
6 2
1
tan =
6 3

EXAMPLE


Find the exact value of 3 cos cosec .
6 4
Solution


3 cos cosec
6 4
1
= 3 cos
6 sin
4
3n 2
= 3d
2 1
3 3
= 2
2
3 3 2 2
=
2 2
3 3 2 2
=
2

Angles of any magnitude

The results for angles of any magnitude can also be looked at using radians.
If we change degrees for radians, the ASTC rule looks like this:
Chapter 5 Trigonometric Functions 173

y

2
2nd quadrant 1st quadrant

S A

0
x
2

T C
+ 2 -

3rd quadrant 4th quadrant


3
2

We can summarise the trigonometric ratios for angles of any magnitude


in radians as follows:

First quadrant:
Angle :
sin is positive
cos is positive
tan is positive

Second quadrant:
Angle :
sin ( ) = sin
cos ( ) = cos
tan ( ) = tan

Third quadrant:
Angle + :
sin ( + ) = sin
cos ( + ) = cos
tan ( + ) = tan

Fourth quadrant:
Angle 2 :
sin (2 ) = sin
cos (2 ) = cos
tan (2 ) = tan
174 Maths In Focus Mathematics HSC Course

EXAMPLES

5
1. Find the exact value of sin .
4
Solution
4
=
4
5 4
So = +
4 4 4

=+
4
In the 3rd quadrant, angles are in the form + , so the angle is in the 3rd
quadrant, and sin is negative in the 3rd quadrant.
5
sin c m = sin c + m 4
4 4
2
= sin 1
4
1
=
2 4
1

11
2. Find the exact value of cos .
6
Solution
12
2 =
6
11 12
So =
6 6 6

= 2
6
In the 4th quadrant, angles are in the form 2 , so the angle is in the
4th quadrant, and cos is positive.
11
cos c m = cos c 2 m
6 6 6

= cos
6
2 3
3
=
2


3
1
Chapter 5 Trigonometric Functions 175

4
3. Find the exact value of tan c m.
3
Solution

3
=
3
4 3
So = c + m
3 3 3

= c + m
3
In the 2nd quadrant, angles are in the form ( + ), so the angle is in the
2nd quadrant, and tan is negative in the 2nd quadrant.

6
4
tan c m = tan ; c + m E
3 3
2 3
= tan
3
= 3
You could change the
3 radians into degrees before
1 finding these ratios.

You could solve all


equations in degrees then
We can use the ASTC rule to solve trigonometric equations. You studied
convert into radians where
these in the Preliminary Course using degrees. necessary.

EXAMPLES

1. Solve cos x = 0.34 for 0 x 2 .

Solution
cos is positive in the 1st and 4th quadrants.
Using a calculator (with radian mode) gives x = 1.224.
i.e. cos 1.224 = 0.34

This is an angle in the 1st quadrant since 1.224 < .
2
In the 4th quadrant, angles are in the form of 2 .
So the angle in the 4th quadrant will be 2 1.224.
So x = 1.224, 2 1.224
= 1.224, 5.06.

1
2. Solve sin = in the domain 0 2.
2
Solution
Here the sin of the angle is negative.
Since sin is positive in the 1st and 2nd quadrants, it is negative in the
3rd and 4th quadrants.
CONTINUED
176 Maths In Focus Mathematics HSC Course

1
sin =
4 2 4
In the 3rd quadrant, angles are + and in the 4th
2
quadrant, 2 . 1


So = + , 2
4 4 4
4 8 1
= + ,
4 4 4 4
5 7
= ,
4 4

5.2 Exercises
1. Copy and complete the table, 3. Find the exact value, with
giving exact values. rational denominator where
relevant.


3 4 6 (a) cos2 + sin2
6 4
sin
(b) sin cos cos sin
cos 3 4 3 4

tan (c) cos cos + sin sin
6 3 6 3
cosec
(d) sin2 + cos2
sec 4 4

cot (e) sec2 + sin2
3 6
2. Find the exact value, with rational
4. Find the exact value with rational
denominator where relevant.
Remember that denominator of
2
tan = (tan )
2
(a) tan 2 (a) sin cos + sin cos
6 3 4 4 3
2
(b) c sin m tan tan
4 4 3
3 (b)
(c) c cos m 1 + tan tan
6 4 3

(d) tan + tan
3 6 5. Show that

(e) sin cos cos cos + sin sin =
4 4 3 4 3 4

(f) tan + cos sin cos + cos sin .
3 3 4 6 4 6
3
(g) sec tan 6. (a) Show that = .
4 3 4 4
(b) Which quadrant is the angle
(h) cos + cot
4 4 3
in?
2 4
(i) c cos tan m 3
4 4 (c) Find the exact value of cos .
4
2
(j) c sin + cos m
6 6
Chapter 5 Trigonometric Functions 177

5 (b) Find the exact value of


7. (a) Show that = .
6 6 9
(i) sin
(b) Which quadrant is the angle 4
5 7
in? (ii) tan
6 3
5
(c) Find the exact value of sin . 11
6 (iii) cos
7 4
8. (a) Show that = 2 . 19
4 4 (iv) tan
(b) Which quadrant is the angle 6
7 10
in? (v) sin
4 3
7
(c) Find the exact value of tan .
4 13. Solve for 0 x 2
4
9. (a) Show that =+ . 1
3 3 (a) cos x =
2
(b) Which quadrant is the angle
1
4 (b) sin x =
in? 2
3
(c) tan x = 1
(c) Find the exact value of
4 (d) tan x = 3
cos . 3
3 (e) cos x =
5 2
10. (a) Show that = 2 .
3 3
(b) Which quadrant is the angle 14. Simplify
5 (a) sin ] g
in?
3 (b) tan (2 x)
5
(c) Find the exact value of sin . (c) cos ] + g
3

(d) cos c x m
11. Find the exact value of each ratio. 2
3
(a) tan (e) tan c m
4 2
11
(b) cos 15. Simplify 1 sin2 c m
6 2
2
(c) tan 3
3 16. If tan x = and < x < , find
5 4 2
(d) sin the value of cos x and sin x.
4
7
(e) tan 17. Solve tan2 x 1 = 0 for 0 x 2
6
13
12. (a) (i) Show that = 2 + .
6 6
(ii) Which quadrant is the
13
angle in?
6
(iii) Find the exact value of
13
cos .
6
178 Maths In Focus Mathematics HSC Course

Circle Results
The area and circumference of a circle are useful in this section.

Area of a circle

A = r 2 where r is the radius of the circle

Circumference of a circle

C = 2 r = d

We use equal ratios to nd the other circle results. The working out is
simpler when using radians instead of degrees.

Length of arc

l = r
( is in radians)

Proof
arc length l angle
=
circumference whole revolution
l
=
Using 360 instead of 2 2 r 2
gives a different formula.
2 r
Can you find it? ` l=
2
= r
Chapter 5 Trigonometric Functions 179

EXAMPLES


1. Find the length of the arc formed if an angle of is subtended at the
4
centre of a circle of radius 5 m.

Solution
l = r

= 5c m
4
5
= m
4

2. The area of a circle is 450 cm2 . Find, in degrees and minutes, the
angle subtended at the centre of the circle by a 2.7 cm arc.

Solution

A = r2
450 = r 2
450
=r
2

450
=r
11.97 = r
Now l = r
2.7 = 11.97
2 .7
=
11.97
0.226 =
radians = 180
180
1 radian =
180
0.226 radians = 0.226
= 12.93
= 12 56l
So = 12 56l.
180 Maths In Focus Mathematics HSC Course

5.3 Exercises
1. Find the exact arc length of a 5. The radius of a circle is 3 cm and
circle if 2
an arc is cm long. Find the
(a) radius is 4 cm and angle 7
angle subtended at the centre of
subtended is
the circle by the arc.
(b) radius is 3 m and angle
6. The circumference of a circle is
subtended is
3
300 mm. Find the length of the
(c) radius is 10 cm and angle
arc that is formed by an angle of
5
subtended is
6 subtended at the centre of the
6
(d) radius is 3 cm and angle circle.
subtended is 30
(e) radius is 7 mm and angle 7. A circle with area 60 cm2 has an
subtended is 45. arc 8 cm long. Find the angle that
is subtended at the centre of the
2. Find the arc length, correct to circle by the arc.
2 decimal places, given
(a) radius is 1.5 m and angle 8. A circle with circumference
subtended is 0.43 124 mm has a chord cut off it
(b) radius is 3.21 cm and angle that subtends an angle of 40 at
subtended is 1.22 the centre. Find the length of the
(c) radius is 7.2 mm and angle arc cut off by the chord.
subtended is 55
9. A circle has a chord of 25 mm
(d) radius is 5.9 cm and angle

subtended is 23 12l with an angle of subtended
6
(e) radius is 2.1 m and angle at the centre. Find, to 1 decimal
subtended is 82 35l. place, the length of the arc cut off
by the chord.
3. The angle subtended at the centre
of a circle of radius 3.4 m is 10. A sector of a circle with radius
29 51l. Find the length of the arc
5 cm and an angle of
cut off by this angle, correct to 3
1 decimal place. subtended at the centre is cut out
of cardboard. It is then curved
4. The arc length when a sector of a around to form a cone. Find its
circle is subtended by an angle of exact surface area and volume.
3
at the centre is m. Find the
5 2
radius of the circle.
Chapter 5 Trigonometric Functions 181

Area of sector

1 2
A= r
2
( is in radians)

Proof
area of sector A angle
=
area of circle whole revolution
A
=
r 2 2
r2
` A=
2
1 2
= r
2

EXAMPLES


1. Find the area of the sector formed if an angle of is subtended at the
4
centre of a circle of radius 5 m.

Solution

1 2
A= r
2
1 2
= ^5h c m
2 4
25 2
= m
8

CONTINUED
182 Maths In Focus Mathematics HSC Course

6
2. The area of the sector of a circle with radius 4 cm is cm2. Find the
5
angle, in degrees, that is subtended at the centre of the circle.

Solution

1 2
A= r
2
6 1 ] g 2
= 4
5 2
= 8
6
=
40
3
=
20
Remember that 3 ] 180 g
= 180 .
=
20
27 =

5.4 Exercises
1. Find the exact area of the sector 3. Find the area, correct to
of a circle if 3 signicant gures, of the sector
(a) radius is 4 cm and angle of a circle with radius 4.3 m and
subtended is an angle of 1.8 subtended at the
(b) radius is 3 m and angle centre.

subtended is 4. The area of a sector of a circle is
3
(c) radius is 10 cm and angle 20 cm2. If the radius of the circle
5 is 3 cm, nd the angle subtended
subtended is
6 at the centre of the circle by the
(d) radius is 3 cm and angle
sector.
subtended is 30
(e) radius is 7 mm and angle 5. The area of the sector of a circle
subtended is 45. that is subtended by an angle of

2. Find the area of the sector, correct at the centre is 6 m2. Find the
3
to 2 decimal places, given radius of the circle.
(a) radius is 1.5 m and angle
subtended is 0.43 6. Find the
(b) radius is 3.21 cm and angle (a) arc length
subtended is 1.22 (b) area of the sector of a circle
(c) radius is 7.2 mm and angle with radius 7 cm if the sector
subtended is 55 is cut off by an angle of 30
(d) radius is 5.9 cm and angle subtended at the centre of
subtended is 23 12l the circle.
(e) radius is 2.1 m and angle
subtended is 82 35l.
Chapter 5 Trigonometric Functions 183

3
7. A circle has a circumference of 10. The area of a sector is cm2
10
185 mm. Find the area of the and the arc length cut off by the

sector cut off by an angle of sector is cm. Find the angle
5 5
subtended at the centre. subtended at the centre of the
8. If the area of a circle is 200 cm2 circle and nd the radius of the
and a sector is cut off by an angle circle.
3
of at the centre, nd the area
4
of the sector.

9. Find the area of the sector of a


circle with radius 5.7 cm if the
length of the arc formed by this
sector is 4.2 cm.

Area of minor segment

1 2
A= r ( sin )
2
( is in radians)

Proof
Area of minor segment = area of sector area of triangle
The area of the triangle is
1 2 1
= r r 2 sin 1
given by A = 2 ab sin C.
2 2
1 2
= r ( sin )
2
184 Maths In Focus Mathematics HSC Course

EXAMPLES


1. Find the area of the minor segment formed if an angle of is
4
subtended at the centre of a circle of radius 5 m.

Solution

1 2]
A= r sin g
2
1 2
= ^ 5 h c sin m
2 4 4
25 1
= e o
2 4 2
25 25
=
8 2 2
25 25 2
=
8 4
25 50 2 2
= m
8

2. An 80 cm piece of wire is bent into a circle shape, and a 10 cm piece


of wire is joined to it to form a chord. Find the area of the minor segment
cut off by the chord, correct to 2 decimal places.

Solution

C = 2 r = 80
80
` r=
2
Dont round off the radius. 40
=
Use the full value in your
calculators display to find . Z 12.7
Chapter 5 Trigonometric Functions 185

a2 + b2 c2
cos C =
2ab
12.72 + 12.72 102
cos =
2 ] 12.7 g ] 12.7 g
= 0.69
` = 0.807
1
A = r 2 ] sin g
2
1 2
= ] 12.7 g ] 0.807 sin 0.807 g
2
Z 6.88 cm2

5.5 Exercises
1. Find the exact area of the minor 4. Find the
segment of a circle if (a) exact arc length
(a) radius is 4 cm and the angle (b) exact area of the sector
subtended is (c) area of the minor segment,
(b) radius is 3 m and the angle to 2 decimal places

subtended is if an angle of is subtended at
3 7
(c) radius is 10 cm and the angle the centre of a circle with radius
5
subtended is 3 cm.
6
(d) radius is 3 cm and the angle
5. The area of the minor segment
subtended is 30 2
(e) radius is 7 mm and the angle cut off by an angle of is
9
2
subtended is 45. 500 cm . Find the radius of the
circle, correct to 1 decimal place.
2. Find the area of the minor
segment correct to 2 decimal 6. Find the
places, given (a) length of the chord,
(a) radius is 1.5 m and the angle to 1 decimal place
subtended is 0.43 (b) length of the arc
(b) radius is 3.21 cm and the (c) area of the minor segment,
angle subtended is 1.22 to 2 decimal places
(c) radius is 7.2 mm and the
angle subtended is 55 cut off by an angle of
6
(d) radius is 5.9 cm and the angle subtended at the centre of a circle
subtended is 23 12l with radius 5 cm.
(e) radius is 2.1 m and the angle
7. A chord 8 mm long is formed by
subtended is 82 35l.
an angle of 45 subtended at the
3. Find the area of the minor centre of a circle. Find
segment formed by an angle of (a) the radius of the circle
40 subtended at the centre of a (b) the area of the minor segment
circle with radius 2.82 cm, correct cut off by the angle, correct
to 2 signicant gures. to 1 decimal place.
186 Maths In Focus Mathematics HSC Course

11.
8. An angle of is subtended at the
8
centre of a circle. If this angle
5
cuts off an arc of cm, nd
4
(a) the exact area of the sector
(b) the area of the minor segment
formed, correct to 1 decimal
place.

9. A triangle OAB is formed where O An arc, centre C, cuts side AC


is the centre of a circle of radius in D.
12 cm, and A and B are endpoints
of a 15 cm chord. Find the exact
(a) length of arc BD
(b) area of ABD
(c) perimeter of BDC

12.

(a) Find the angle ( ) subtended


at the centre of the circle, in
degrees and minutes.
(b) Find the area of OAB, correct
to 1 decimal place.
(c) Find the area of the minor The dartboard above has a radius
segment cut off by the chord, of 23 cm. The shaded area has a
correct to 2 decimal places. height of 7 mm. If a player must
(d) Find the area of the major hit the shaded area to win, nd
segment cut off by the chord, (a) the area of the shaded part, to
correct to 2 decimal places. the nearest square centimetre
(b) the percentage of the
10. The length of an arc is 8.9 cm shaded part in relation to the
and the area of the sector is whole area of the dartboard, to
24.3 cm2 when an angle of 1 decimal place
is subtended at the centre of a (c) the perimeter of the
circle. Find the area of the minor shaded area.
segment cut off by , correct to
1 decimal place.
Chapter 5 Trigonometric Functions 187

13. The hour hand of a clock is (a) the exact perimeter of


12 cm long. Find the exact sector ABC
(a) length of the arc through (b) the approximate ratio of the
which the hand would turn in area of the minor segment to the
5 hours area of the sector.
(b) area through which the hand
15. A wedge is cut so that its
would pass in 2 hours.
cross-sectional area is a sector
14. of a circle, radius 15 cm and

subtending an angle of at the
6
centre. Find
(a) the volume of the wedge
(b) the surface area of the wedge.

Arc BC subtends an angle of 100


at the centre A of a circle with
radius 4 cm. Find

Small Angles
When we use radians of small angles, there are some interesting results.

EXAMPLES

1. Find sin 0.0023.

Solution
Make sure your calculator is in radian mode
sin 0.0023 = 0.002299997972

2. Find tan 0.0023.

Solution
tan 0.0023 = 0.002300004056

3. Find cos 0.0023.

Solution
cos 0.0023 = 0.999997355
188 Maths In Focus Mathematics HSC Course

Use radians for the small Class Investigation


angles. You could use a
computer spreadsheet to find
these trigonometric ratios. Use the calculator to explore other small angles and their trigonometric
ratios. What do you notice?

B C

x
A
O 1

Can you see why? In OAC, OC Z 1 when x is small.


Arc AB = x (definition of a radian)
` AC Z x when x is small
AC
sin x =
OC
x
Z
1
=x
OA
cos x =
OC
1
Z
1
=1
AC
tan x =
OA
x
Z
1
=x

If x (radians) is a small angle then


sin x Z x
tan x Z x
cos x Z 1
and sin x < x < tan x
Chapter 5 Trigonometric Functions 189

Proof
1
Area OAB = ab sin C
2
1
= 1 1 sin x
2
1
= sin x
2
1
Area sector OAB = r2
2
1
= 12 x
2
1
= x
2
1
Area OAC = bh
2
1
= 1 AC
2
1 AC
= 1 tan x c tan x = m
2 1
1
= tan x
2

Area OAB < area sector OAB < area OAC


1 1 1
` sin x < x < tan x
2 2 2
` sin x < x < tan x

Class Investigation

Check that sin x < x < tan x for 0 < x < by using your calculator.
2

Remember that x is in radians. Does this work for x > ?
2

These results give the following result:

sin x tan x
lim
x"0 x =1 Also lim
x "0 x
= 1.

Can you see why?

Proof
As x " 0, sin x " x
sin x
` x "1
190 Maths In Focus Mathematics HSC Course

EXAMPLES

sin 7x
1. Evaluate lim x .
x"0

Solution

sin 7x 7 (sin 7x)


lim
x"0 x = lim
x"0 7x
sin 7x
= 7 lim
x"0 7x
= 7 (1)
=7

2. The planet of Alpha Zeta in a faraway galaxy has a moon with a


diameter of 145 000 km. If the distance between the centre of Alpha Zeta
and the centre of its moon is 12.8 106 km, nd the angle subtended by
the moon at the centre of the planet, in seconds.

Solution

1
r= 145 000
The radius of the moon 2
is 72 500 km. = 72 500
1
=
2
72 500
tan =
12.8 106
= 0.00566
` = 0.00566
= 180
180
` 1=
180
0.00566 = 0.00566
= 0.3245
` = 0.649
= 39l
Chapter 5 Trigonometric Functions 191

5.6 Exercises
1. Evaluate, correct to 3 decimal 5. Given that the wingspan of
places. an aeroplane is 30 m, nd the
(a) sin 0.045 planes altitude to the nearest
(b) tan 0.003 metre if the wingspan subtends
(c) cos 0.042 an angle of 14l when it is directly
(d) sin 0.065 overhead.
(e) tan 0.005
sin x
2. Evaluate lim .
x"0 4x

tan
3
3. Find lim .
"0
4. Find the diameter of the sun
to the nearest kilometre if
its distance from the Earth is
149000000 km and it subtends an
angle of 31l at the Earth.

Trigonometric Graphs
You drew the graphs of trigonometric functions in the Preliminary Course,
using degrees.
You can also draw these graphs using radians.
y = sin x
192 Maths In Focus Mathematics HSC Course

y = cos x

y = tan x

3
y = tan x has asymptotes at x = and since tan x is undened at those
2 2
points. By nding values for x on each side of the asymptotes, we can see
where the curve goes.
We can also sketch the graphs of the reciprocal trigonometric functions, by
nding the reciprocals of sin x, cos x and tan x at important points on the graphs.

Investigation
1 1 1
1. Use cosec x = , sec x = cos x and cot x = to complete the
sin x tan x
table below.

3
x 0 2 2
2
cosec x
sec x
cot x

1
e.g. cosec =
2 sin
2
1
=
1
=1
Chapter 5 Trigonometric Functions 193

2. Find any asymptotes


1
e.g. sec =
2 cos
2
1
= (undefined)
0
Discover what the values are on either side of the asymptotes.

3. Sketch each graph of the reciprocal trigonometric functions.

Here are the graphs of the reciprocal ratios.

y = cosec x
y

1
y = sin x
x
3 2
2 2
-1

y = sec x

1
y = cos x
x
3 2
2 2
-1
194 Maths In Focus Mathematics HSC Course

y = tan x
y

x
3 2
2 2

y = cot x

We have sketched these functions in the domain 0 x 2 which gives


all the angles in one revolution of a circle. However, we could turn around the
circle again and have angles greater than 2 , as well as negative angles.
These graphs all repeat at regular intervals. They are called periodic
functions.

Investigation

Here is a general sine function. Notice that the shape that occurs between
0 and 2 repeats as shown.
y

x
4 2 2 4

1. Draw a general cosine curve. How are the sine and cosine curves
related? Do these curves have symmetry?
2. Draw a general tangent curve. Does it repeat at the same intervals as
the sine and cosine curves?
3. Look at the reciprocal trigonometric curves and see if they repeat in a
similar way.
Chapter 5 Trigonometric Functions 195

4. Use a graphics calculator or a graphing computer program to draw


the graphs of trigonometric functions. Choose different values of k to
sketch these families of trig functions. (Dont forget to look at values
where k is a fraction or negative.)
(a) f (x) = k sin x
(b) f (x) = k cos x
(c) f (x) = k tan x
(d) f (x) = sin kx
(e) f (x) = cos kx
(f) f (x) = tan kx
(g) f (x) = sin x + k
(h) f (x) = cos x + k
(i) f (x) = tan x + k
(j) f (x) = sin (x + k)
(k) f (x) = cos (x + k)
(l) f (x) = tan (x + k)
Can you see patterns in these families?
Could you predict what the graph of f (x) = a sin bx looks like?

The trig functions have period and amplitude.


The period is the distance over which the curve moves along the x-axis
before it repeats.
The amplitude is the maximum distance that the graph stretches out
from the centre of the graph on the y-axis.

y = sin x has amplitude 1 and period 2


y = cos x has amplitude 1 and period 2
y = tan x has no amplitude and period

2
y = a sin bx has amplitude a and period
b
2
y = a cos bx has amplitude a and period
b

y = a tan bx has no amplitude and period
b
196 Maths In Focus Mathematics HSC Course

EXAMPLES

1. Sketch in the domain 0 x 2.


(a) y = 5 sin x for
(b) y = sin 4x for
(c) y = 5 sin 4x for

Solution
(a) The graph of y = 5 sin x has amplitude 5 and period 2.

5
y = 5 sin x

x
3 2
2 2

2
(b) The graph y = sin 4x has amplitude 1 and period or .
4 2

This means that the curve repeats every , so in the domain
2
0 x 2 there will be 4 repetitions.
y

1
y = sin 4x

x
3 5 3 7 2
4 2 4 4 2 4

1
Chapter 5 Trigonometric Functions 197


(c) The graph y = 5 sin 4x has amplitude 5 and period .
2
y

5 y = 5 sin 4x

x
3 5 3 7 2
4 2 4 4 2 4


2. Sketch f (x) = sin b x + l for 0 x 2.
2
Solution
Amplitude = 1
2
Period =
1
= 2
f (x) = sin (x + k) translates the curve k units to the left (see Investigation
on page 1945).

So f (x) = sin c x + m will be moved units to the left. The graph is the
2 2
same as f (x) = sin x but starts in a different position.
If you are not sure where the curve goes, you can draw a table of values.
3
x 0 2
2 2
y 1 0 1 0 1

x
3 2
2 2

1
This is the same as the
graph y = cos x.

CONTINUED
198 Maths In Focus Mathematics HSC Course

x
3. Sketch y = 2 tan for 0 x 2.
2
Solution
There is no amplitude.

Period =
1
2
= 2

x
3 2
2 2

4. (a) Sketch y = 2 cos x and y = cos 2x on the same set of axes for
0 x 2.
(b) Hence or otherwise, sketch y = cos 2x + 2 cos x for 0 x 2.

Solution
(a) y = 2 cos x has amplitude 2 and period 2
2
y = cos 2x has amplitude 1 and period or
2
y

y = 2 cos x

1
y = cos 2x
2

x
3 5 3 7 2
4 2 4 4 2 4

2
Chapter 5 Trigonometric Functions 199

(b) Method 1: Use table

3 5 3 7
x 0 2
4 2 4 4 2 4
cos 2x 1 0 1 0 1 0 1 0 1
2 cos x 2 2 0 2 2 2 0 2 2
cos 2x + 2 cos x 3 1 1 1 3 Notice that the period of
2 2 2 2 this graph is 2 .

Method 2: Add y values together on the graph itself


e.g. when x = 0:
y=1+2
=3

y
3
y = cos 2x
2 + 2 cos x

2
y = 2cos x

1
y = cos 2x
2

x
3 5 3 7 2
4 2 4 4 2 4

2 These graphs can be


sketched more accurately
on a graphics calculator
3 or in a computer program
such as Autograph.

5.7 Exercises
1. Sketch for 0 x 2 2. Sketch for 0 x 2
(a) y = cos x (a) y = cos 2x
(b) f (x) = 2 sin x (b) y = tan 2x
(c) y = 1 + sin x (c) y = sin 3x
(d) y = 2 sin x (d) f (x) = 3 cos 4x
(e) f (x) = 3 cos x (e) y = 6 cos 3x
(f) y = 4 sin x x
(f) y = tan
2
(g) f (x) = cos x + 3 (g) f (x) = 2 tan 3x
(h) y = 5 tan x x
(h) y = 3 cos
(i) f (x) = tan x + 3 2
x
(j) y = 1 2 tan x (i) y = 2 sin
2
x
(j) f (x) = 4 cos
4
200 Maths In Focus Mathematics HSC Course

3. Sketch for x 6. Sketch for 2 x 2


(a) y = sin 2x (a) y = sin x
(b) y = 7 cos 4x (b) y = 3 cos 2 x
(c) f (x) = tan 4x
7. Sketch in the domain 0 x 2
(d) y = 5 sin 4x
(a) y = sin x and y = sin 2x on the
(e) f (x) = 2 cos 2x
same set of axes
x
4. Sketch y = 8 sin in the domain (b) y = sin x + sin 2x
2
0 x 4. 8. Sketch for 0 x 2
(a) y = 2 cos x and y = 3 sin x on
5. Sketch for 0 x 2 the same set of axes
(a) y = sin (x + ) (b) y = 2 cos x + 3 sin x

(b) y = tan b x + l 9. By sketching y = cos x and
2
(c) f (x) = cos (x ) y = cos 2x on the same set of axes
for 0 x 2 , sketch the graph of
(d) y = 3 sin b x l
2
y = cos 2x cos x.
(e) f (x) = 2 cos b x + l
2
10. Sketch the graph of
(f) y = 4 sin b 2x + l
2
(a) y = cos x + sin x
(g) y = cos b x l (b) y = sin 2x sin x
4
(c) y = sin x + 2 cos 2x
(h) y = tan b x + l
4 (d) y = 3 cos x cos 2x
x
(i) f (x) = 2 cos b x + l + 1 (e) y = sin x sin
2 2

(j) y = 2 3 sin b x l
2

Applications

The sine and cosine curves are used in many


applications including the study of waves. There
are many different types of waves, including
water, light and sound waves. Oscilloscopes display
patterns of electrical waves on the screen of
a cathode-ray tube.

Search waves and the oscilloscope on the Internet


for further information.

Simple harmonic motion (such as the movement


of a pendulum) is a wave-like or oscillatory motion
when graphed against time. In 1581, when he
was 17 years old, Gallileo noticed a lamp swinging
backwards and forwards in Pisa cathedral. He
found that the lamp took the same time to swing to and fro, no matter how
much weight it had on it. This led him to discover the pendulum.

Gallileo also invented the telescope. Find out more information about Gallileos
life and his other discoveries.
Chapter 5 Trigonometric Functions 201

Some trigonometric equations are difficult to solve algebraically, but


can be solved graphically by finding the points of intersection between
two graphs.

EXAMPLES

1. Find approximate solutions to the equation sin x = x 1 by sketching


the graphs y = sin x and y = x 1 on a Cartesian plane.

Solution
When sketching the two graphs together, we use 3.14, 2 6.28 and
so on to label the x-axis as shown.
To sketch y = x 1, find the gradient and y-intercept or find the x- and
y-intercepts.
x-intercept (where y = 0) is 1 and y-intercept (where x = 0) is 1.
y

y = x1

x
3 2
1 2 2 3 4 5 6
2

y = sin x
1

The solution to sin x = x 1 is at the point of intersection of the two


graphs. ` x 2.

x
2. How many solutions are there for cos 2x = in the domain 0 x 2 ?
4
Solution
x
Sketch y = cos 2x and y = on the same set of axes.
4
y = cos 2x has amplitude 1 and period
x
y = has x-intercept 0 and y-intercept 0. We can find another point on the line
4
e.g. When x = 4
4
y=
4
=1

CONTINUED
202 Maths In Focus Mathematics HSC Course

y
x
y=
4

3 2
1 2 2 3 4 5 6
2

y = cos 2x
2
1

There are 3 points of intersection of the graphs, so the equation


x
cos 2x = has 3 solutions.
4

We can also look at applications of trigonometric graphs in real life


situations.

EXAMPLE

The table shows the highest average monthly temperatures in Sydney.

Jan Feb Mar Apr May Jun Jul Aug Sep Oct Nov Dec

C 26.1 26.1 25.1 22.8 19.8 17.4 16.8 18.0 20.1 22.2 23.9 25.6

(a) Draw a graph of this data, by hand or on a graphics calculator


or computer.
(b) Is it periodic? Why would you expect it to be periodic?
(c) What is the period and amplitude?

Solution
(a) Temperature
30
25
20
15
10
5
0
ov er
Fe ary

M y
ch

il
ay

A ly
pt st
ne

O er

r
em r
be
D mbe
r

pr

Se ugu

N tob
Ju

b
ua

M
Ju
ar
nu

em
A
br

c
e
Ja

ec

Months
Chapter 5 Trigonometric Functions 203

(b) The graph looks like it is periodic, and we would expect it to be,
since the temperature varies with the seasons. It goes up and down,
and reaches a maximum in summer and a minimum in winter.
(c) This curve is approximately a cosine curve with one full period,
so the period is 12 months.
To find the centre, find the
The maximum temperature is around 26 and the minimum is
average of 18 and 26 which
around 18, so the centre of the graph is 22 with 4 either side. So the 18 + 26
is .
amplitude is 4. 2

5.8 Exercises
1. Show graphically that 7. The graph below show the times
x of sunsets in a city over a period
sin x = has
2 of 2 years.
(a) 2 solutions for 0 x 2 Sunsets
(b) 3 solutions for x . 10
9
8
2. Solve sin x = x for 0 x 2 7
graphically by sketching y = sin x
Time (pm)

6
5
and y = x on the same number 4
plane. 3
2
3. Solve cos x = 2x 3 for 1
0
0 x 2 by nding the points
y
ch

ay

Ja er

ch

ay

r
y
ov r

ov r
be
N be
Se Jul

Se Jul
ar

N be
ar
b

ar
ar

M
em
nu

nu

em
em

em
M

of intersection of the graphs


Ja

pt

pt

y = cos x and y = 2x 3.

4. Solve tan x = x graphically in the


domain 0 x 2 .

5. Solve by graphical means


sin 2x = x for 0 x 2 .

6. Draw the graphs of y = sin x and


y = cos x for 0 x 2 on the
same set of axes. Use your graphs
to solve the equation sin x = cos x
for 0 x 2 .
(a) Find the period and
amplitude of the graph.
(b) At approximately what time
would you expect the sun to set
in July?
204 Maths In Focus Mathematics HSC Course

8. The graph shows the incidence of crimes committed over 24 years in


Gotham City.
1800

1600

1400

1200

1000

800

600

400

200

0
1 2 3 4 5 6 7 8 9 10 11 12 13 14 15 16 17 18 19 20 21 22 23 24

(a) Approximately how many crimes were committed in the 10th year?
(b) What was the
(i) highest and
(ii) lowest number of crimes?
(c) Find the amplitude and the period of the graph.

9. Below is a table showing the average daylight hours over several months.

Month Jan Feb Mar Apr May June July Aug

Daylight hours 15.3 14.7 13.2 13.1 12.7 12.2 12.5 13.8

(a) Draw a graph to show this data.


(b) Is it periodic? If so, what is the period?
(c) Find the amplitude.

10. The table below shows the high and low tides over a three-day period.

Day Friday Saturday Sunday

Time 6.20 11.55 6.15 11.48 6.20 11.55 6.15 11.48 6.20 11.55 6.15 11.48
am am pm pm am am pm pm am am pm pm

Tide (m) 3.2 1.1 3.4 1.3 3.2 1.2 3.5 1.1 3.4 1.2 3.5 1.3

(a) Draw a graph showing the tides.


(b) Find the period and amplitude.
(c) Estimate the height of the tide
at around 8 am on Friday.
Chapter 5 Trigonometric Functions 205

Differentiation of Trigonometric Functions


Derivative of sin x

We can sketch the gradient (tangent) function of y = sin x.

The gradient function is y = cos x.

d
(sin x) = cos x
dx

Proof
Let f ] x g = sin x
Then f (x + h) = sin (x + h)
f (x + h) f (x)
f l(x) = lim
h"0 h
sin (x + h) sin x
= lim This result is a formula in
h"0 h the Extension 1 Course
(sin x cos h + cos x sin h) sin x and you do not need to
= lim know it.
h"0 h
sin x (cos h 1) + cos x sin h
= lim
h"0 h
cos h 1 sin h
= lim sin x c m + cos x c m
h"0 h h
sin h
= lim sin x(0) + cos x(1) c since cos h " 1 as h " 0 and lim = 1m
h"0 h"0 h
= cos x.
sin h
Notice that the result lim = 1 only works for radians. We always use
h"0 h
radians when using calculus.
206 Maths In Focus Mathematics HSC Course

FUNCTION OF A FUNCTION RULE

d
[sin f (x)] = f l(x) cos f (x)
dx

Proof
Let u = f (x)
Then y = sin u
du dy
= f l(x) and = cos u
dx du
dy dy du
= $
dx du dx
= cos u $ f l(x)
= f l(x) cos f (x)

Derivative of cos x

We can sketch the gradient function of y = cos x.

The gradient function is y = sin x.

d
(cos x) = sin x
dx

Proof

Since cos x = sin c xm
2
d d
(cos x) = < sin c x m F
dx dx 2

= 1 cos c x m
2
= sin x
Chapter 5 Trigonometric Functions 207

FUNCTION OF A FUNCTION RULE

d
[cos f (x)] = f l(x) sin f (x) This result can be proven
dx the same way as for sin f(x).

Derivative of tan x

The gradient function is harder to sketch for y = tan x.

The gradient function is y = sec2 x.

sin x
It is easier to see the rule for differentiating y = tan x by using tan x = cos x
and the quotient rule.

d
(tan x) = sec2 x
dx

Proof
sin x
tan x = cos x
d d b sin x l
(tan x) =
dx dx cos x
ulv vlu
=
v2
cos x (cos x) (sin x) sin x
=
cos2 x
cos2 x + sin2 x
=
cos2 x
1
=
cos2 x
= sec2 x
208 Maths In Focus Mathematics HSC Course

FUNCTION OF A FUNCTION RULE

d
This result can be proven [tan f (x)] = f l(x) sec2 f (x) .
the same way as for sin f(x).
dx

EXAMPLES

1. Differentiate sin (5x) .

Solution
d
[sin f (x)] = f l(x) cos f (x)
dx
d
` [sin (5x)] = 5 cos (5x)
dx

2. Differentiate sin x.

Solution
d d x
(sin x) = c sin m
dx dx 180
x
= cos
180 180

= cos x
180

3. Find the exact value of the gradient of the tangent to the curve

y = x2 sin x at the point where x = .
4
Solution
dy
= ulv + vlu
dx
= cos x (x2) + 2x (sin x)
= x2 cos x + 2x sin x

When x = ,
4
dy 2
= c m cos + 2 c m sin
dx 4 4 4 4
2
1 1
= +
16 2 2 2
2

= +
16 2 2 2
2 + 8
=
16 2
2 ( + 8)
=
32
Chapter 5 Trigonometric Functions 209

5.9 Exercises
1. Differentiate 5. Differentiate log e (cos x) .
(a) sin 4x
(b) cos 3x 6. Find the exact gradient of the
(c) tan 5x normal to the curve y = sin 3x at

(d) tan (3x + 1) the point where x = .
18
(e) cos ( x)
7. Differentiate etan x.
(f) 3 sin x
(g) 4 cos (5x 3) 8. Find the equation of the normal
(h) 2 cos (x3) to the curve y = 3 sin 2x at the
(i) 7 tan (x2 + 5)
(j) sin 3x + cos 8x point where x = , in exact form.
8
(k) tan ( + x) + x 2 d2 y
(l) x tan x 9. Show that = 25y if
dx2
(m) sin 2x tan 3x y = 2 cos 5x.
sin x
(n)
2x 10. Given f (x) = 2 sin x, show that
3x + 4 f m(x) = f (x) .
(o)
sin 5x
(p) (2x + tan 7x)9 11. Show that
(q) sin2 x d
[loge (tan x)] = tan x + cot x.
(r) 3 cos3 5x dx
(s) ex cos 2x 12. Find the coordinates of the
(t) sin (1 loge x) stationary points on the curve
(u) sin (ex + x) y = 2 sin x x for 0 x 2 .
(v) loge (sin x)
13. Differentiate
(w) e3x cos 2x
(a) tan x
e2x
(x) (b) 3 cos x
tan 7x
sin x
(c)
2. Find the derivative of cos x sin4 x. 5
14. If y = 2 sin 3x 5 cos 3x, show
3. Find the gradient of the tangent
d2 y
to the curve y = tan 3x at the that = 9 y.
dx2
point where x = .
9 15. Find values of a and b if
4. Find the equation of the tangent d2 y
= ae3x cos 4x + be3x sin 4x,
to the curve y = sin ( x) at the dx2
1 given y = e3x cos 4x.
point c , m , in exact form.
6 2

Integration of Trigonometric Functions


The integrals of the trigonometric functions are their primitive functions.

# cos x dx = sin x + C
210 Maths In Focus Mathematics HSC Course

# sin x dx = cos x + C
Integration is the inverse of
` # sin x dx = cos x + C
differentiation.

# sin x dx = cos x + C

# sec2 x dx = tan x + C

Function of a function rule

# cos (ax + b) dx = 1a sin (ax + b) + C

Proof
d
[sin (ax + b)] = a cos (ax + b)
dx
` # a cos (ax + b) dx = sin (ax + b) + C
# cos (ax + b) dx = 1a # a cos (ax + b) dx
1
= a sin (ax + b) + C

Similarly,

# sin (ax + b) dx = 1a cos (ax + b) + C

# sec2 (ax + b) dx = 1a tan(ax + b) + C

EXAMPLES

1. Find # sin (3x) dx.


Solution

# sin (3x) dx = 13 cos (3x) + C


Chapter 5 Trigonometric Functions 211

#

2
2. Evaluate sin x dx.
0

Solution

#

2
2
sin x dx = ; cos x E
0 0

= cos ( cos 0)
2
= 0 ( 1)
=1

3. Find # cos x dx.

Solution
x
# cos x dx = # cos 180 dx
1 x
= sin 180 + C
180
180
= sin x + C

4. Find the area enclosed between the curve y = cos x, the x-axis and the
3
lines x = and x = .
2 2
Solution

You studied areas in


Chapter 3. The definite
integral is negative as the
area is below the x-axis.

#
3
2
2
cos x dx = ; sin x E

2 2
You could use
3
= sin sin
#
3

2 2 Area =
2
cos x dx
= 1 1
2

= 2 = 2
= 2.
` area is 2 units2.

CONTINUED
212 Maths In Focus Mathematics HSC Course

5. Find the volume of the solid formed if the curve y = sec x is rotated

about the x-axis from x = 0 to x = .
4
Solution

y = sec x
` y2 = sec2 x

#
b
The volume formula comes V= y 2 dx
from Chapter 3. a

#

= 4
sec 2 x dx
0

4
= ; tan x E
0

= c tan tan 0 m
4
= (1 0)
=
So the volume is units 3 .

5.10 Exercises

1. Find the indenite integral x


(n) 4 sin
(primitive function) of 2
(a) cos x x
(o) 3 sec2
(b) sin x 3
(c) sec2 x (p) sin (3 x)
sin x
(d) 2. Evaluate, giving exact answers
4
(e) sin 3x where appropriate.

#

(f) sin 7x 2
(a) cos x dx
(g) sec2 5x 0

#

(h) cos (x + 1) 3
(b) sec 2 x dx
(i) sin (2x 3)
6
(j) cos (2x 1)
#

x
(k) sin ( x) (c) sin dx
2
2
(l) cos (x + )
#

(m) 2 sec2 7x (d) 2
cos 3x dx
0
Chapter 5 Trigonometric Functions 213

by rotating the curve y = sin 2x


#
1
2
(e) sin x dx about the x-axis from x = 0 to
0

x= .
#

(f) 8
sec 2 2x dx 6
0
8. Find the exact area enclosed by
(g) #

12
3 cos 2x dx the curve y = sin x and the line
0
1
y = for 0 x 2 .
(h) #

10
sin 5x dx 2
0 9. Find the exact area bounded by
3. Find the area enclosed between the curves y = sin x and y = cos x
the curve y = sin x and the x-axis in the domain 0 x 2 .
in the domain 0 x 2 .
10. (a) Show that the volume of
4. Find the exact area bounded by the solid formed by rotating
the curve y = cos 3x, the x-axis the curve y = cos x about the

and the lines x = 0 and x = . x axis between x = 0 and x = is
12 2
units3 .
5. Find the area enclosed between
x (b) Use the trapezoidal rule
the curve y = sec2 , the x-axis
4 with 4 subintervals to nd an

and the lines x = and x = , approximation to the volume of
4 2
correct to 2 decimal places. the solid formed by rotating the
curve y = cos x about the x-axis
6. Find the volume, correct to
from x = 0 to x = .
2 decimal places, of the solid 2
formed when the curve y = sec x d 2y
is rotated about the x-axis from 11. A curve has = 18 sin 3x and a
dx2
x = 0 to x = 0.15.
stationary point at c , 2 m . Find
6
7. Find, in exact form, the volume the equation of the curve.
of the solid of revolution formed
214 Maths In Focus Mathematics HSC Course

Test Yourself 5
4
1. A circle with radius 5 cm has an angle of 7. Evaluate

subtended at the centre. Find
#

6 (a) 4
cos x dx
(a) the exact arc length 0

#

(b) the exact area of the sector 3
(b) sec 2 x dx
(c) the area of the minor segment to
6
3 signicant gures.
8. Find the equation of the tangent to the
2. Find the exact value of
1
curve y = sin 3x at the point e , o.
(a) tan 4 2
3
d2 x
(b) cos 9. If x = cos 2t, show that = 4x.
6 dt2
2 10. Find the exact area bounded by the curve
(c) sin
3
y = sin x, the x-axis and the lines x =
3 4
(d) cos
4 and x = .
2
3. Solve for 0 x 2 11. Find the volume of the solid formed if
(a) tan x = 1 the curve y = sec x is rotated about the
(b) 2 sin x = 1
x-axis from x = 0 to x = .
6
4. Sketch for 0 x 2
12. Simplify
(a) y = 3 cos 2x
x sin 5x
(b) y = 7 sin (a) lim x
x"0
2
2 tan
5. Differentiate (b) lim
"0
(a) cos x
13. Find the gradient of the tangent to the
(b) 2 sin x
curve y = 3 cos 2x at the point where
(c) tan x + 1

(d) x sin x x= .
6
tan x
(e) x 14. A circle has a circumference of 8 cm. If
(f) cos 3x
an angle of is subtended at the centre
(g) tan 5x 7
of the circle, nd
6. Find the indenite integral (primitive (a) the exact area of the sector
function) of (b) the area of the minor segment, to
(a) sin 2x 2 decimal places.
(b) 3 cos x 2x
15. (a) Sketch y = cos 2x and y = on the
(c) sec2 5x 3
(d) 1 + sin x same set of axes for 0 x 2 .
2x
(b) Solve cos 2x = for 0 x 2 .
3
Chapter 5 Trigonometric Functions 215

16. Find the exact area with rational dy


denominator bounded by the curve 19. A curve has = 6 sin 2x, and passes
dx

y = sin x, the x-axis and the lines x = through the point c , 3 m . Find the
6 2

and x = . equation of the curve.
4
17. Find the area bounded by the curve 20. (a) Sketch y = 7 sin 3x and y = 2x 1 on
y = cos 2x, the x-axis and the lines x = 0 the same number plane for 0 x 2 .
to x = . (b) Solve 7 sin 3x = 2x 1 for 0 x 2 .

18. Find the equation of the normal to the



curve y = tan x at the point c , 1 m .
4

Challenge Exercise 5
1. Use Simpsons rule with 5 function 7. Find the derivative of tan x.
values to find an approximation to
sec2 x
#

4
tan x dx, correct to 2 decimal places. 8. (a) Show that sec x cosec x = .

tan x
8 (b) Hence, or otherwise, find the exact

#

8
2. Evaluate sec 2 2x dx, in exact form. value of 3
cosec x sec x dx.


12
4
9. Differentiate ex sin 2x.
3. The area of the sector of a circle is
4 units2 and the length of the arc 10. (a) Find the stationary points on the
curve y = sin 2x + 3 over the domain
bounded by this sector is units. Find
8
0 x .
the radius of the circle and the angle that
(b) What is the maximum value of the
is subtended at the centre.
curve?
4. If f (x) = 3 cos x (c) What is the amplitude?
(a) find the period and amplitude of the
11. The area of a sector in a circle of radius
function
8
(b) sketch f (x) for 0 x 4. 4 cm is cm2. Find the area of the
3
d2 y minor segment, in exact form.
5. Given = 9 sin 3x
dx2
(a) find y if there is a stationary point at
12. Find # sin xdx.
13. Find the gradient of the normal to the
c , 1m
2 curve y = x2 + cos x at the point where
d2 y
(b) Show that + 9y = 0 . x = 1.
dx2
6. Sketch y = 5 sin ] x + g for 0 x 2 .
216 Maths In Focus Mathematics HSC Course

14. Use the trapezoidal rule with


17. Sketch y = tan c x m for 0 x 2 .
4 subintervals to find, correct to 4
3 decimal places, an approximation
18. Find all the points of inflexion on the
to the volume of the solid formed
by rotating the curve y = sin x about curve y = 3 cos c 2x + m for 0 x 2 .
4
the x-axis from x = 0.2 to x = 0.6. 19. Find the exact area bounded by the curve
15. Differentiate log e (sin x + cos x) . y = cos x, the x-axis and the lines

x = and x = .
16. Find the exact area of the minor segment 6 4
cut off by a quarter of a circle with radius 20. If f (x) = 2 cos 3x, show that
3 cm. f m(x) = 9 f (x) .
6
Applications of
Calculus to the
Physical World
TERMINOLOGY

Acceleration: The rate of change of velocity with respect Exponential decay: Decay, or decrease in a quantity,
to time where the rate of change of a quantity is in direct
proportion to the quantity itself. The decay becomes less
At rest: Stationary (zero velocity)
rapid over time
Displacement: The movement of an object in relation to
Rate of change: The change of one variable with respect
its original position
to another variable over time
Exponential growth: Growth, or increase in a quantity,
Velocity: The rate of change of displacement of an object
where the rate of change of a quantity is in direct
with respect to time involving speed and direction
proportional to the quantity itself. The growth becomes
more rapid over time
Chapter 6 Applications of Calculus to the Physical World 219

INTRODUCTION
CALCULUS IS USED IN many situations involving rates of change, such as
physics or economics. This chapter looks at rates of change of a particle in
motion, and exponential growth and decay. Both these types of rates of
change involve calculus.

DID YOU KNOW?

Galileo (15641642) was very interested in the behaviour of bodies in motion.


He dropped stones from the Leaning Tower of Pisa to try to prove that they
would fall with equal speed. He rolled balls down slopes to prove that they
move with uniform speed until friction slows them down. He showed that a
body moving through the air follows a curved path at a fairly constant speed.
John Wallis (16161703) continued this study with his publication
Mechanica, sive Tractatus de Motu Geometricus. He applied mathematical
principles to the laws of motion and stimulated interest in the subject of
mechanics.
Soon after Wallis publication, Christiaan Huygens (16291695) wrote
Horologium Oscillatorium sive de Motu Pendulorum, in which he described
various mechanical principles. He invented the pendulum clock, improved the
telescope and investigated circular motion and the descent of heavy bodies.
These three mathematicians provided the foundations of mechanics.
Sir Isaac Newton (16421727) used calculus to increase the understanding of the Galileo
laws of motion. He also used these concepts as a basis for his theories on gravity
and inertia.

Rates of Change

Simple rates of change

The gradient of a straight line measures the rate of change of y with respect to
the change in x.

With a curve, the rate of change of y with respect to x is measured by the


gradient of the tangent to the curve. That is, the derivative measures the rate
of change.
220 Maths In Focus Mathematics HSC Course

dQ
The rate of change of a quantity Q with respect to time t is measured by .
dt

EXAMPLES

1. The number of bacteria in a culture increases according to the formula


B = 2t 4 t 2 + 2000, where t is time in hours. Find
(a) the number of bacteria initially
(b) the number of bacteria after 5 hours
(c) the rate at which the number of bacteria will be increasing after
5 hours.

Solution
(a) B = 2t 4 t 2 + 2000
Initially, t = 0
` B = 2 (0) 4 0 2 + 2000
= 2000
So there are 2000 bacteria initially.
(b) When t = 5,
B = 2 (5)4 52 + 2000
= 3225
So there will be 3225 bacteria after 5 hours.
(c) The rate of change is given by the derivative
dB
= 8t 3 2t
dt
When t = 5,
dB
= 8 (5)3 2 (5)
dt
= 990
So the rate of increase after 5 hours will be 990 bacteria per hour.

2. The volume ow rate of water into a pond is given by R = 4 + 3t 2 litres


per hour. If there is initially no water in the pond, nd the volume of
water after 12 hours.
Chapter 6 Applications of Calculus to the Physical World 221

Solution

R = 4 + 3t 2
dV
i.e. = 4 + 3t 2 Volume ow rate is
dt the rate of change of
` V = # (4 + 3t 2) dt volume over time.

= 4t + t 3 + C
When t = 0, V = 0
` 0 =0+0+C
=C
` V = 4t + t 3
When t = 12
V = 4 (12) + (12) 3
= 1776 L
So there will be 1776 L of water in the pond after 12 hours.

6.1 Exercises
1. Find a formula for the rate of (c) R = 4 r2 is the rate at which
change in each question. the volume V changes with
(a) h = 20t 4t 2 radius r
(b) D = 5t 3 + 2t 2 + 1 (d) R = 7 sin t is the rate at which
(c) A = 16x 2x 2 the distance d of an object
(d) x = 3t 5 x 4 + 2x 3 changes over time t
(e) V = e t + 4 (e) R = 8e 2t 3 is the rate at
(f) S = 3 cos 5 which the speed s changes over
50 time t.
(g) S = 2 r + 2
r
(h) D = x 42 3. If h = t 3 7t + 5, nd the rate of
400 change of h when t = 3.
(i) S = 800r + r
4 4. Given f (t) = sin 2t, nd the rate of
(j) V = r3
3
change when t = .
2. Find an original formula for each 6
question given its rate of change. 5. A particle moves so that its
(a) R = 4t 12t 2 is the rate at distance x is given by x = 2e 3t .
which the height h of an object Find the exact rate of change
changes over time t when t = 4.
(b) R = 8x3 + 1 is the rate at
6. The volume of water owing
which the area A of a gure
through a pipe is given by
changes with side x
V = t2 + 3t. Find the rate at which
the water is owing when t = 5.
222 Maths In Focus Mathematics HSC Course

7. The rate of change of the angle 14. According to Boyles Law, the
sum S of a polygon with n sides is pressure of a gas is given by
a constant 180. If S is 360 when k
the formula P = , where k is a
n = 4, nd S when n = 7. V
constant and V is the volume of
8. A particle moves so that the the gas. If k = 100 for a certain
rate of change of distance D gas, nd the rate of change in the
over time t is given by R = 2e t 1. pressure when V = 20.
If D = 10 when t = 0, nd D when
15. A circular disc expands as it
t = 3.
is heated. The area, in cm2, of
9. For a certain graph, the rate the disc increases according to
of change of y values with the formula A = 4t 2 + t, where
respect to its x values is given t is time in minutes. Find the
by R = 3x 2 2x + 1. If the graph rate of increase in the area after
passes through the point (1, 3), 5 minutes.
nd its equation.
16. A balloon is inated so that
10. The mass in grams of a melting its increase in volume is at a
iceblock is given by the formula constant rate of 10 cm 3 s 1 . If its
M = t 2t 2 + 100, where t is time volume is initially 1 cm3, nd its
in minutes. Show that the rate volume after 3 s.
at which the iceblock is melting
17. The number of people in
is given by R = 1 4t grams per
a certain city is given by
minute and nd the rate at which
P = 200 000e 0.2t , where t is time
it will be melting after 5 minutes.
in years. Find the rate at which
11. The rate of change in velocity the population of the city will be
over time is given by growing after 5 years.
dy
= 4t + t 2 t 3. If the initial 18. A radioactive substance has
dt
velocity is 2 cms 1, nd the a mass that decreases over
velocity after 15 s. time t according to the formula
M = 200e 0.1t . Find
12. The rate of ow of water (a) the mass of the substance
into a dam is given by after 20 years, to the nearest gram
R = 500 + 20t Lh 1 . If there is (b) the rate of its decrease after
15 000 L of water initially in the 20 years.
dam, how much water will there
19. If y = e 4x, show that the rate is
be in the dam after 10 hours?
given by R = 4y.
13. The surface area in cm2 of a
20. Given S = 2e 2t + 3, show that the
balloon being inated is given
rate of change is R = 2 (S 3).
by S = t3 2t2 + 5t + 2, where t is
time in seconds. Find the rate of
increase in the balloons surface
area after 8 s.
Chapter 6 Applications of Calculus to the Physical World 223

Exponential Growth and Decay


Exponential growth or decay are terms that describe a special rate of change
that occurs in many situations. Population growth and growth of bacteria
in a culture are examples of exponential growth. The decay of radioactive
substances, the cooling of a substance and change in pressure are examples of
exponential decay.
When a substance grows or decays exponentially, its rate of change is directly
proportional to the amount of the substance itself. That is, the more of the
substance there is, the faster it grows. For example, in a colony of rabbits, there is
fairly slow growth in the numbers at rst, but the more rabbits there are, the more
rabbits will be born. In other species, such as koalas, the clearing of habitats and
other factors may cause the population to decrease, or decay, exponentially.

The exponential function is a function where the rate of change


(gradient) is increasing as the function increases.

This is why it is called


exponential growth.

In mathematical symbols, exponential growth or decay can be written as


dQ
= kQ , where k is the growth or decay constant.
dt
224 Maths In Focus Mathematics HSC Course

dQ
= kQ is solved by the equation Q = Ae kt
dt

When k is positive there Proof


is exponential growth.
When k is negative there dQ
is exponential decay. = kQ
dt
dt 1
` =
dQ kQ
1
t=# dQ
kQ
1
= log e Q + k 1
k
kt = log e Q + k 2
kt k 2 = log e Q
` Q = e kt k
2

= e kt e k 2

= Ae kt

A is always the initial quantity

Proof
Initial means at rst when
t = 0. Given Q = Ae kt , suppose the initial quantity is Q 0
i.e. Q = Q 0 when t = 0
Substituting into the equation gives
Q 0 = Ae k 0
= Ae 0
=A
A is the initial quantity.
Sometimes the equation is written as Q = Q 0 e kt .

DID YOU KNOW?

Malthus was an economist. Thomas Malthus (17661834), at the beginning of the Industrial Revolution, developed a theory
about population growth that we still use today. His theory states that under ideal conditions,
dN
the birth rate is proportional to the size of the population. That is, = kN (Malthusian Law of
dt
Population Growth).
Malthus was concerned that the growth rate of populations would be higher than the
increase in food supplies, and that people would starve.
Was he right? Is this happening? How could we prove this?
Chapter 6 Applications of Calculus to the Physical World 225

EXAMPLES

1. The population of a colony of rabbits over time t months is given by


P = 1500e 0.046t . Find
(a) the initial population
(b) the population after 2 years
(c) when the population reaches 5000.

Solution
(a) Initially, t = 0
Substituting:
P = 1500e 0.046t
= 1500e 0.046 0
= 1500e 0
= 1500 (e 0 = 1)
So the initial population is 1500 rabbits.
(b) 2 years = 24 months
So t = 24
Substituting:
P = 1500e 0.046t
= 1500e 0.046 24
= 1500e 1.104
= 4524.31
So the population after 2 years is 4524 rabbits.
(c) P = 5000
Substituting:
P = 1500e 0.046t
5000 = 1500e 0.046t
5000
= e 0.046t
1500
5000
ln = ln e 0.046t
1500
= 0.046t ln e
= 0.046t (since ln e = 1)
5000
ln
1500
=t
0.046
26.2 = t
So the population reaches 5000 after 26.2 months.

CONTINUED
226 Maths In Focus Mathematics HSC Course

2. The number of bacteria in a culture is given by N = Ae kt . If 6000


bacteria increase to 9000 after 8 hours, nd
(a) k correct to 3 signicant gures
(b) the number of bacteria after 2 days
(c) the rate at which the bacteria will be increasing after 2 days
(d) when the number of bacteria will reach 1 000 000
(e) the growth rate per hour as a percentage.

Solution
(a) N = Ae kt
When t = 0, N = 6000
A gives the initial number ` 6000 = Ae 0
of bacteria.
=A
So N = 6000e kt
When t = 8, N = 9000
9000 = 6000e 8k
9000
= e 8k
6000
1 .5 = e 8k
log e 1.5 = log e e 8k
= 8k log e e
= 8k
Don't round k off, but put it in
log e 1.5
=k
the calculators memory to use 8
again.
0.0507 Z k
So N = 6000e 0.0507t .
(b) When t = 48, (48 hours in 2 days)
N = 6000e 0.0507 # 48
= 68 344
So there will be 68 344 bacteria after 2 days.
dN
(c) Rate: = 6000 (0.0507e 0.0507t )
dt
= 304.1e 0.0507t
When t = 48,
The rate of growth is the dN
derivative. = 304.1e 0.0507 # 48
dt
= 3464
So after 2 days the rate of growth will be 3464 bacteria per hour.
Another method:
dN
= kN
dt
= 0.0507N
When t = 48, N = 68 344
dN
= 0.0507 68 344 = 3464
dt
Chapter 6 Applications of Calculus to the Physical World 227

(d) When N = 1 000 000


1 000 000 = 6000e 0 .0507t
1 000 000
= e 0.0507t
6000
166.7 = e 0.0507t
log e 166.7 = log e e 0.0507t
= 0.0507t log e e
= 0.0507t
log e 166.7
=t
0.0507
100.9 Z t
So the number of bacteria will be 1 000 000 after 100.9 hours.
(e) dN = kN where k = growth constant
dt
k = 0.0507
= 5.07%
So the growth rate is 5.07% per hour.

3. A 50 g mass of uranium decays to 35 g after 2 years. If the rate of


decay of its mass is proportional to the mass itself, nd the amount of
uranium left after 25 years.

Solution
Q = Ae kt
When t = 0, Q = 50
` 50 = Ae 0
=A
So Q = 50e kt
When t = 2, Q = 35
35 = 50e 2k
35
= e 2k
50
0 .7 = e 2k
log e 0.7 = log e e 2k
= 2k log e e
= 2k Notice that k is negative
for decay. You could use
log e 0.7
=k the formula Q = Ae kt
2 for decay. What would
0.1783 Z k k be?

So Q = 50e 0.1783t
When t = 25
Q = 50e 0.1783 # 25
Z 0.579
So there will be 0.579 grams, or 579 mg, left after 25 years.
228 Maths In Focus Mathematics HSC Course

6.2 Exercises

1. The number of birds in a colony 4. A chemical reaction causes the


is given by N = 80e 0.02t where t is amount of chlorine to be reduced
in days. at a rate proportional to the
(a) How many birds are there in amount of chlorine present at
the colony initially? any one time. If the amount of
(b) How many birds will there be chlorine is given by the formula
after 30 days? A = A 0 e kt , and 100 L reduces to
(c) After how many days will 65 L after 5 minutes, nd
there be 500 birds? (a) the amount of chlorine after
(d) Sketch the curve of the 12 minutes
population over time. (b) how long it will take for the
chlorine to reduce to 10 L.

5. The production output in a


factory increases according to the
equation P = P0 e kt , where t is in
years.
(a) Find P0 if the initial output is
5000 units.
(b) The factory produces 8000
units after 3 years. Find the value
of k, to 3 decimal places.
2. The number of bacteria in a (c) How many units will the
culture is given by N = N 0 e 0.32t , factory produce after 6 years?
where t is time in hours. (d) The factory needs to produce
(a) If there are initially 20 000 20 000 units to make a maximum
bacteria, how many will there be prot. After how many years,
after 5 hours? correct to 1 decimal place, will
(b) How many hours, to the this happen?
nearest hour, would it take for
the number of bacteria to reach 6. The rate of depletion of
200 000? rainforests can be estimated
as proportional to the amount
3. The decay of radium is proportional of rainforests. If 3 million m2
to its mass. If 100 kg of radium of rainforest is reduced to
takes 5 years to decay to 95 kg 2.7 million m2 after 20 years,
(a) show that the mass of radium nd how much rainforest there
is given by M = 100 e 0.01t will be after 50 years.
(b) nd its mass after 10 years
(c) nd its half-life (the time
taken for the radium to halve its
mass).
Chapter 6 Applications of Calculus to the Physical World 229

(b) the rate at which the sugar


will be reducing after 8 hours
(c) how long it will take to
reduce to 50 kg.

10. The mass, in grams, of a


radioactive substance is given
by M = M 0 e kt , where t is time in
years. Find
(a) M 0 and k if a mass of 200 kg
decays to 195 kg after 10 years
(b) mass after 15 years
7. The annual population of a
(c) the rate of decay after 15 years
country is increasing at a rate of
(d) the half-life of the substance
dP
6.9%; that is, = 0.069P. If the (time taken to decay to half its
dt
population is 50 000 in 2015, nd mass).
(a) a formula for the population
11. The number of bacteria in a
growth
culture increases from 15 000 to
(b) the population in the year
25 000 in 7 hours. If the rate of
2020
bacterial growth is proportional
(c) the rate at which the
to the number of bacteria at that
population will be growing in the
time, nd
year 2020
(a) a formula for the number of
(d) in which year the population
bacteria
will reach 300 000.
(b) the number of bacteria after
8. An object is cooling down 12 hours
according to the formula T = T0 e kt , (c) how long it will take for the
where T is temperature in degrees culture to produce 5 000 000
Celsius and t is time in minutes. bacteria.
If the temperature is initially 90C
12. A population in a certain city is
and the object cools down to 81C
growing at a rate proportional
after 10 minutes, nd
to the population itself. After
(a) its temperature after half
3 years the population increases
an hour
by 20%. How long will it take for
(b) how long (in hours and
the population to double?
minutes) it will take to cool down
to 30C.

9. In the process of the inversion of


sugar, the amount of sugar present
is given by the formula S = Ae kt .
If 150 kg of sugar is reduced to
125 kg after 3 hours, nd
(a) the amount of sugar after
8 hours, to the nearest kilogram
230 Maths In Focus Mathematics HSC Course

13. The half-life of radium is


1600 years.
(a) Find the percentage of
radium that will be decayed after
500 years.
(b) Find the number of years that
it will take for 75% of the radium
to decay.

14. The population of a city is P(t) at


any one time. The rate of decline
17. Anthony has a blood alcohol
in population is proportional to
level of 150 mg/dL. The amount
the population P(t), that is,
of alcohol in the bloodstream
dP (t)
= kP (t) . decays exponentially. If it
dt
decreases by 20% in the rst
(a) Show that P(t) = P(t 0) e kt is a
hour, nd
solution of the differential
(a) the amount of alcohol in
dP (t) Anthonys blood after 3 hours
equation = kP (t) .
dt (b) when the blood alcohol level
(b) What percentage decline in
reaches 20 mg/dL.
population will there be after
10 years, given a 10% decline 18. The current C owing in a
in 4 years? conductor dissipates according
(c) What will the percentage rate dC
to the formula = kC. If it
of decline in population be after dt
10 years?
dissipates by 40% in 5 seconds,
(d) When will the population fall
how long will it take to dissipate
by 20%?
to 20% of the original current?
15. The rate of leakage of water out of
19. Pollution levels in a city have
a container is proportional to the
been rising exponentially with a
amount of water in the container
10% increase in pollution levels
at any one time. If the container
in the past two years. At this rate,
is 60% empty after 5 minutes,
how long will it take for pollution
nd how long it will take for the
levels to increase by 50%?
container to be 90% empty.

16. Numbers of sheep in a dQ


20. If = kQ , prove that Q = Ae kt
certain district are dropping dt
exponentially due to drought. satises this equation by
A survey found that numbers had (a) differentiating Q = Ae kt
declined by 15% after 3 years. dQ
If the drought continues, how (b) integrating = kQ .
dt
long would it take to halve the
number of sheep in that district?
Chapter 6 Applications of Calculus to the Physical World 231

Class Investigation

Research some environmental


issues. For example:

1. It is estimated that some


animals, such as pandas and
koalas, will be extinct soon.
How soon will pandas be
extinct? Can we do anything
to stop this extinction?
2. How do the greenhouse effect
and global warming affect the
earth? How soon will they have a noticeable effect on us?
3. How long do radioactive substances such as radium and plutonium
take to decay? What are some of the issues concerning the storage of
radioactive waste?
4. The erosion and salination of Australian soil are problems that may
affect our farming in the future. Find out about this issue, and some
possible solutions.
5. The effect of blue-green algae in some of our rivers is becoming a
major problem. What steps have been taken to remedy this situation?

Look at the mathematical aspects of these issues. For example, what


formulae are used to make predictions? What sorts of time scales are
involved in these issues?

Research other issues, relating to growth and decay, that affect our
environment.

Motion of a Particle in a Straight Line


In this study of the motion of a particle moving along a straight line, we
ignore friction, gravity and other inuences on the motion. We can nd the
particles displacement, velocity and acceleration at any one time. The term
particle, or body, can refer to something quite large (e.g. a car). It describes
any moving object.

DID YOU KNOW?


Calculus was developed in the 17th century as a solution to problems about the study of
motion. Some problems of the time included nding the speed and acceleration of planets,
calculating the lengths of their orbits, nding maximum and minimum values of functions,
nding the direction in which an object is moving at any one time and calculating the areas
and volumes of certain gures.
232 Maths In Focus Mathematics HSC Course

Displacement

Displacement (x) measures the distance of a particle from a xed point


(origin). Unlike distance, displacement can be positive or negative, according
to which side of the origin it is on. Usually, on the right-hand side it will be
positive and on the left it will be negative.
When the particle is at the origin, its displacement is zero. That is, x = 0.

Velocity

Velocity (v) is a measure of the rate of change of displacement with respect


to time. Since the rate of change is the gradient of the graph of displacement
against time,
dx
v= .
dt
.
Velocity can also be written as x.

Velocity (unlike speed) can be positive or negative, according to which


direction the object is travelling in. If the particle is moving to the right,
velocity is positive. If it is moving to the left, velocity is negative.
When the object is not moving, we say that it is at rest. That is, v = 0.

Acceleration

Acceleration is the measure of the rate of change of velocity with respect to


time. This gives the gradient of the graph of velocity against time. That is,

dv d dx d2 x
a= = c m= 2 .
dt dt dt dt
..
Acceleration can also be written as x.
Acceleration can be positive or negative, according to which direction it is
in. If the acceleration is to the right, it is positive. If the acceleration is to the
left, it is negative.
If the acceleration is in the same direction as the velocity, the particle
is speeding up (accelerating). If the acceleration is in the opposite direction
from the velocity, the particle is slowing down (decelerating).
If the object is travelling at a constant velocity, there is no acceleration.
That is, a = 0.

Motion graphs

You can describe the velocity of a particle by looking at the gradient function
of a displacement graph. The acceleration is the gradient function of a velocity
graph.
Chapter 6 Applications of Calculus to the Physical World 233

EXAMPLES

1. The graph below shows the displacement of a particle from the origin
as it moves in a straight line.

(a) When is the particle


(i) at rest?
(ii) at the origin?
(b) When is the particle moving at its greatest velocity?

Solution
dx
(a) (i) When the particle is at rest, velocity is zero. i.e. = 0.
dt
So the particle is at rest at the stationary points t 3 and t 5 .
(ii) The particle is at the origin when x = 0, i.e. on the t-axis. So the
particle is at the origin at t2, t4 and t 6 .
(b) The greatest velocity is at t4 (the curve is at its steepest). Notice that
this is where there is a point of inexion.

2. The graph below shows the displacement x of a particle over time t.


x

t
t1 t2

CONTINUED
234 Maths In Focus Mathematics HSC Course

(a) Draw a sketch of its velocity


(b) Sketch its acceleration
(c) Find values of t for which the particle is
(i) at the origin and
(ii) at rest.

Solution
dx
(a) The velocity is the rate of change of displacement, or . By noting
dt
where the gradient of the tangent is positive, negative and zero, we draw
the velocity graph.

+
+
+
+
+
t
t1 t2
- +
- +
0

t
t1 t2

d2 x
(b) Acceleration is the rate of change of velocity, or . By noting where
dt 2
the gradient of the tangent is positive, negative and zero, we draw the
acceleration graph.
Chapter 6 Applications of Calculus to the Physical World 235

+
t
t1 t2
+

The gradient is a constant, so


acceleration is constant.

t
t1 t2

(c) (i) The particle is at the origin when x = 0.


This is at 0 and t2.
(ii) The particle is at rest when v = 0.
This is at t1 (on the t-axis on the velocity graph or the stationary
point on the displacement graph).

3. The graph below is the velocity of a particle.

t
t1 t2 t3 t4 t5

CONTINUED
236 Maths In Focus Mathematics HSC Course

(a) Draw a sketch showing the displacement of the particle.


(b) Find the times when the particle is
(i) at rest
(ii) at maximum velocity.
(c) Explain why you cant state accurately when the particle is at the
origin.

Solution
(a) For 0 t 1 and between t3 and t5, the velocity is negative (below the
dx
t-axis), so the displacement has < 0.
dt
Between t1 and t3, and all points > t5, the velocity is positive (above
dx
the t-axis), so the displacement has > 0.
dt
dx
At t1, t3 and t5, the velocity is zero, so = 0 and there is a
dt
stationary point.
v

+ +
+

+ +
+

0 0 0
t
t1 t2 t3 t4 t5
- -
-
- -

- -

Notice that at t1 and t5, LHS < 0 and RHS > 0 so they are minimum
turning points.
At t3, LHS > 0 and RHS < 0 so it is a maximum turning point.
Here is one graph that could describe the shape of the displacement.
x

t
t1 t2 t3 t4 t5

(b) (i) At rest, v = 0.


This occurs at t1, t3 and t5.
Chapter 6 Applications of Calculus to the Physical World 237

(ii) Maximum velocity is at t4 since it is furthest from the t-axis.


Notice also that there is a point of inexion at t4 on the
displacement graph.

(c) You cant tell when the particle is at the origin as the graph is not
accurate. For example, the graph in (a) is at the origin at t4 and another
point to the right of t5. However, when sketching displacement from
the velocity (or the original function given the gradient or derivative
function) there are many possible graphs, forming a family of graphs.
x

t
t1 t2 t3 t4 t5

Notice that these cross the t-axis at various places, so we cant tell
where the displacement graph shows the particle at the origin.

6.3 Exercises
1. The graphs at right and overleaf (a) x
show the displacement of an
object. Sketch the graphs for
velocity and acceleration.

t
238 Maths In Focus Mathematics HSC Course

(b) x When is the particle


(a) at the origin?
(b) at rest?
(c) travelling with constant
t
acceleration?
(d) furthest from the origin?

3. The graph below shows a


(c) x particle travelling at a constant
acceleration.

(d)

Sketch the graphs that could


represent
(a) its velocity and
(b) its displacement.

(e) 4. The graph below shows the


velocity of a particle.
x

t
t1 t2 t3 t4 t5 t6

2. The graph below shows the


displacement of a particle as it (a) When is the particle at rest?
moves along a straight line. (b) When is the acceleration
zero?
(c) When is the velocity the
greatest?
(d) Describe the motion of the
particle at (i) t2 and (ii) t3.
Chapter 6 Applications of Calculus to the Physical World 239

5. The graph below shows the (b)


displacement of a pendulum.

(c)

(a) When is the pendulum at


rest?
(b) When is the pendulum in
its equilibrium position (at the
origin)? (d)

6. Describe the motion of


the particle at t 1 for each
displacement graph.

(a)

(e)

Motion and Differentiation


If displacement is given, then the velocity is the rst derivative and the
acceleration is the second derivative.

Displacement x
. dx
Velocity x =
dt
dv d 2 x
Acceleration px = = 2
dt dt
240 Maths In Focus Mathematics HSC Course

EXAMPLES

1. The displacement x cm of a particle over time t seconds is given by


x = 4t t 2 .
(a) Find any times when the particle is at rest.
(b) How far does the particle move in the rst 3 seconds?

Solution
dx
(a) At rest means that the velocity c m is zero.
dt
dx
= 4 2t
dt
dx
When =0
dt
4 2t = 0
4 = 2t
2=t
So the particle is at rest after 2 seconds.
dx
Notice that there is a stationary point at = 0, so the particle turns
dt
around at this time.
(b) Initially, when t = 0:
x = 4(0) 0 2
=0
So the particle is at the origin.
After 3 seconds, t = 3:
x = 4(3) 3 2
=3
So the particle is 3 cm from the origin.
However, after 2 seconds, the particle turns around, so it hasnt
simply moved from the origin to 3 cm away. We need to nd where
it is when it turns around.
When t = 2:
x = 4 (2) 2 2
=4
So the particle is 4 cm from the origin.
The particle moves from the origin (x = 0) to 4 cm away in the rst
2 seconds, so it has travelled 4 cm.
It then turns and goes back to 3 cm from the origin. So in the 3rd
second it travels 1 cm back.
Chapter 6 Applications of Calculus to the Physical World 241

0 1 2 3 4 5

Total distance travelled in the rst 3 seconds is 4 + 1 or 5 cm.

2. The displacement of a particle is given by x = t 2 + 2t + 3 cm, where


t is in seconds.
(a) Find the initial velocity of the particle.
(b) Show that the particle has constant acceleration.
(c) Find when the particle will be at the origin.
(d) Find the particles maximum displacement from the origin.
(e) Sketch the graph showing the particles motion.

Solution

(a) dx
v=
dt
= 2t + 2
Initially, t = 0
` v = 2 (0) + 2
=2
So the initial velocity is 2 cms 1.
d2 x
(b) a = 2
dt
=2
the particle has a constant acceleration of 2 cms 2.
(c) At the origin x = 0
i.e. t2 + 2t + 3 = 0
(t + 1)(t 3) = 0 A time of 1 does
not exist.
` t = 1 or 3
So the particle will be at the origin after 3 s.
(d) For maximum displacement,
dx
=0
dt
2t + 2 = 0
2 = 2t
1=t
(Maximum displacement occurs when t = 1.)
When t = 1,
x = (1) 2 + 2 (1) + 3
=4
So maximum displacement is 4 cm.

CONTINUED
242 Maths In Focus Mathematics HSC Course

(e)

This is the graph of a


parabola. Notice that x can
be positive or negative, but
t is never negative.

3. The displacement of a particle is given by the formula x = 1 + 3 sin 2t,


where x is in metres and t is in seconds.
(a) Sketch the graph of x as a function of t.
(b) Find the times when the particle will be at rest.
(c) Find the position of the particle at those times.

Solution

(a) Graph has period and amplitude 3.

ds
(b) v =
dt
= 6 cos 2t
When the particle is at rest
v=0
i.e. 6 cos 2t = 0
cos 2t = 0
3 5
2t = , , ,...
2 2 2
3 5
` t= , , ,...
4 4 4
Chapter 6 Applications of Calculus to the Physical World 243

(c) x = 1 + 3 sin 2t

When t =
4

x = 1 + 3 sin
2
= 1 + 3 (1)
=4
3
When t =
4
3
x = 1 + 3 sin
2
= 1 + 3 ( 1)
= 2
5 7
Similarly, when t = , , , x moves between 4 m and 2 m from
4 4
the origin.

Investigation

In physics, equations for displacement and velocity are given by


s = ut + 1 at 2 and v = u + at, where u is the initial velocity and a is the
2
acceleration. Differentiate the displacement formula to show the formula
for velocity. What happens when you differentiate again?

6.4 Exercises
1. The displacement of a particle is 2. A particle is moving such that
given by x = t 3 9t cm, where t is its displacement is given by
time in seconds. s = 2t 2 8t + 3, where s is in
(a) Find the velocity of the metres and t is in seconds.
particle after 3 s. (a) Find the initial velocity.
(b) Find the acceleration after 2 s. (b) Show that acceleration is
(c) Show that the particle is constant and nd its value.
initially at the origin, and nd (c) Find the displacement after 5 s.
any other times that the particle (d) Find when the particle will be
will be at the origin. at rest.
(d) Find after what time the (e) What will the particles
acceleration will be 30 cms 2 . displacement be at that time?
(f) Sketch the graph of the
displacement against time.
244 Maths In Focus Mathematics HSC Course

3. A projectile is red into the air (d) Find the times when the
and its height in metres is given pendulum will be at rest.
by h = 40t 5t 2 + 4, where t is in (e) What will the displacement
seconds. be at these times?
(a) Find the initial height. (f) When will there be zero
(b) Find the initial velocity. displacement?
(c) Find the height after 1 s. (g) Show that acceleration
(d) What is the maximum height a = 4x.
of the projectile?
7. An object is travelling along a
(e) Sketch the graph of the
straight line over time t seconds,
height against time.
with displacement according to
4. The displacement in cm after the formula x = t 3 + 6t 2 2t + 1 m.
time t s of a particle moving (a) Find the equations of its
in a straight line is given by velocity and acceleration.
x = 2 t t 2. (b) What will its displacement be
(a) Find the initial displacement. after 5 s?
(b) Find when the particle will be (c) What will its velocity be
at the origin. after 5 s?
(c) Find the displacement (d) Find its acceleration after 5 s.
after 2 s.
8. The displacement, in centimetres,
(d) How far will the particle
of a body is given by x = (4t 3)5 ,
move in the rst 2 seconds?
where t is time in seconds.
(e) Find its velocity after 3 s. .
(a) Find equations for velocity x
..
5. The equation for displacement of and acceleration x .
. ..
a particle is given by x = e 2t + 1 m (b) Find the values of x, x and x
after time t seconds. after 1 s.
(a) Find the initial velocity. (c) Describe the motion of the
(b) Find the exact acceleration body after 1 s.
after 1 s.
9. The displacement of a particle, in
(c) Show that the acceleration is
metres, over time t seconds
always double the velocity.
1
(d) Sketch the graph of velocity is s = ut + gt 2, where u = 5 and
2
over time t. g = 10.
(a) Find the equation of the
6. The displacement of a pendulum
velocity of the particle.
is given by x = cos 2t cm after
(b) Find the velocity after 10 s.
time t seconds.
(c) Show that the acceleration is
(a) Find the equation for the
equal to g.
velocity of the pendulum.
(b) Find the equation for its 10. The displacement in metres after
acceleration. 2t 5
t seconds is given by s = .
(c) Find the initial displacement. 3t + 1
Find the equations for velocity
and acceleration.
Chapter 6 Applications of Calculus to the Physical World 245

11. The displacement of a particle (c) When will the projectile


is given by x = log e (t + 1), where reach the ground?
x is in centimetres and t is in (d) Sketch the graph showing
seconds. the height of the projectile over
(a) Find the initial position of the time t.
particle. (e) How far will the projectile
(b) Find the velocity after 5 s. travel in the rst 4 s?
(c) Find the acceleration after 5 s.
15. A ball is rolled up a slope at a
(d) Describe the motion of the
distance from the base of the
particle at 5 s.
slope, after time t seconds, given
(e) Find the exact time when the
by x = 15t 3t 2 metres.
particle will be 3 cm to the right
(a) How far up the slope will the
of the origin.
ball roll before it starts to roll
12. Displacement of a particle is back down?
given by x = t 3 4t 2 + 3t, where (b) What will its velocity be
s is in metres and t is in seconds. when it reaches the base of the
(a) Find the initial velocity. slope?
(b) Find the times when the (c) How long will the motion of
particle will be at the origin. the ball take altogether?
(c) Find the acceleration after 3 s.
16. The displacement of a particle is
13. The displacement of a particle given by x = 2t 3 3t 2 + 42t.
moving in simple harmonic (a) Show that the particle is
motion is given by x = 3 sin 2t, initially at the origin but never
where x is in centimetres and t is returns to the origin.
in seconds. (b) Show that the particle is
(a) Sketch the graph of the never at rest.
displacement of the particle from
17. A weight on the end of a
t = 0 to t = 2 .
spring has a height given by
(b) Find equations for the
. .. h = 3 sin 2t cm where t is time in
velocity x and acceleration x .
seconds.
(c) Find the exact acceleration
(a) Find its initial height.
after s. (b) Find when the spring is at
3
..
(d) Show that x = 4x. its maximum distance from the
origin.
14. The height of a projectile is
(c) What is the acceleration at
given by h = 7 + 6t t 2, where
these times?
height is in metres and time is in
seconds.
(a) Find the initial height.
(b) Find the maximum height
reached.
246 Maths In Focus Mathematics HSC Course

18. A particle moves so that its 19. A particle is moving in a straight


displacement x cm is given by line so that its displacement x cm
x = e t at t seconds. over time t seconds is given by
2

(a) Find the exact velocity of the x = t 49 t2 .


particle after 4 seconds. (a) For how many seconds does
(b) When is the particle at rest? the particle travel?
(c) Where is the particle at that (b) Find the exact time at which
time? the particle comes to rest.
(c) How far does the particle
move altogether?

Motion and Integration


If the rst derivative of the displacement of a particle gives its velocity, then
Displacement is the area
under the velocitytime the reverse is true. That is, the primitive function (indenite integral) of the
graph. velocity gives the displacement. Similarly, the integral of the acceleration
gives the velocity.

x = # v dt
v = # a dt

EXAMPLES

1. The velocity of a particle is given by v = 3t 2 + 2t + 1. If initially the


particle is 2 cm to the left of the origin, nd the displacement after 5 s.

Solution
x = # v dt
= # (3t 2 + 2t + 1) dt
= t3 + t2 + t + C
Initially means t = 0
When t = 0, x = 2
` 2 = 03 + 02 + 0 + C
=C
` x = t3 + t2 + t 2
When t = 5,
x = 53 + 52 + 5 2
= 125 + 25 + 5 2
= 153
So after 5 s the particle will be 153 cm to the right of the origin.
Chapter 6 Applications of Calculus to the Physical World 247

2
2. The acceleration of a particle is given by a = 6 ms 2. If the
(t + 1) 2

particle is initially at rest 1 m to the right of the origin, nd its exact


displacement after 9 s.

Solution
v= # a dt
= # e 6 (t +21) 2 o dt

= # [6 2 (t + 1) 2 ] dt You must nd C before


going on to nd the
(t + 1) 1
= 6t 2 +C displacement.
1
2
= 6t + +C
t+1
At rest means v = 0.
When t = 0, v = 0
2
` 0 = 6(0) + +C
0+1
=2+C
` C = 2
2
So v = 6t + 2
t+1
x = # v dt
2
= # c 6t + 2 m dt
t +1
= 3t 2 + 2 log e (t + 1) 2t + C
When t = 0, x = 1
` 1 = 3(0)2 + 2 loge (0 + 1) 2(0) + C
=C
` x = 3t2 + 2 loge (t + 1) 2t + 1

When t = 9
x = 3 (9) 2 + 2 loge (9 + 1) 2 (9) + 1
= 243 + 2 loge 10 18 + 1
= 226 + 2 loge 10
= 2 (113 + loge 10)

So after 9 s the displacement will be 2(113 + log e 10) m.


248 Maths In Focus Mathematics HSC Course

6.5 Exercises
1. The velocity of a particle is given a velocity of 4 ms 1, nd the
by v = 3t2 5 cms 1. If the particle equation of the height of the
is at the origin initially, nd its projectile.
displacement after 3 s.
9. A particle is accelerating
2. A particle has velocity given by according to the equation
dx a = (3t + 1)2 ms 2. If the particle
= 2t 3 ms 1. After 3 s the
dt is initially at rest 2 m to the
particle is at the origin. Find its left of 0, nd its displacement
displacement after 7 s. after 4 s.
3. The velocity of a particle is given 10. The velocity of a particle is given
by v = 4 3t cms 1. If the particle by v = e t 1 ms 1. If the particle
is 5 cm from the origin after 2 s, is initially 3 m to the right of
nd the displacement after 7 s. the origin, nd its exact position
4. The velocity of a particle is given after 5 s.
by v = 8t3 3t2 cms 1. If the 11. A particle accelerates according
particle is initially at the origin, to the equation a = e 2t . If the
nd particle has initial velocity
(a) its acceleration after 5 s of zero cms 1 and initial
(b) its displacement after 3 s displacement of 1 cm, nd its
(c) when it will be at the origin displacement after 4 s, to the
again. nearest centimetre.
5. The velocity of a particle is given 12. The acceleration of a particle
by x: = 3e3t cms 1. Given that the is given by a = 9 sin 3t cms 2.
particle is initially 2 cm to the If the initial velocity is 5 cms 1
right of the origin, nd its exact and the particle is 3 cm to the
displacement after 1 s. left of the origin, nd the exact
6. A particle has a constant displacement after s.
acceleration of 12 ms 2. If the 13. The velocity of a particle is given
particle has a velocity of 2 ms 1 t
and is 3 m from the origin after by x: = 2 ms 1. If the particle
t +3
5 s, nd its displacement after 10 s.
is initially at the origin, nd its
7. The acceleration of an object is displacement after 10 s, correct to
given by 6t + 4 cms 2. The particle 2 decimal places.
is initially at rest at the origin. Find
14. The acceleration of an object is
(a) its velocity after 5 s
given by a = e 3t ms 2. If initially
(b) its displacement after 5 s.
the object is at the origin
8. A projectile is accelerating at a with velocity 2 ms 1, nd its
constant rate of 9.8 ms 2. If displacement after 3 seconds,
it is initially 2 m high and has correct to 3 signicant gures.
Chapter 6 Applications of Calculus to the Physical World 249

15. The velocity of a particle is given 17. The acceleration of a particle is


..
by v = 4 cos 2t ms 1. If the particle x = 25 e 5t ms2 and its velocity is
is 3 m to the right of the origin 5 ms1 initially. Its displacement
after s, nd the exact is initially 1 m.
(a) Find its velocity after
(a) displacement after s
6 9 seconds.

(b) acceleration after s. (b) Find its displacement after
6
6 seconds.
16. The acceleration of a particle is (c) Show that the acceleration
2 ..
given by a = 2
ms2 and the x = 25x.
(t + 3)
(d) Find the acceleration when
particle is initially at rest 2 ln 3 m
the particle is 2 m to the right of
to the left of the origin.
the origin.
(a) Evaluate its velocity after
2 seconds.
(b) Find its displacement after
n seconds.
(c) Show that 0 v < 1 for all t.
250 Maths In Focus Mathematics HSC Course

Test Yourself 6
1. A particle moves so that its displacement 8. The acceleration of a particle is
..
after t seconds is x = 4t 2 5t 3 metres. Find x = 6t 12 ms 2 . The particle is initially
(a) its initial displacement, velocity and at rest 3 metres to the left of the origin.
acceleration (a) Find the
(b) when the particle is at the origin (i) acceleration
(c) its maximum displacement. (ii) velocity
(iii) displacement after 5 seconds.
2. The velocity of a particle is given by
(b) Describe the motion of the particle
V = 6t 12t2 ms 1 . Find its displacement
after 5 seconds.
and acceleration after 3 seconds if it is
initially 5 m to the right of the origin. 9. The graph below shows the displacement
of a particle.
3. A city doubles its population in 25 years.
If it is growing exponentially, when will
it triple its population?

4. The displacement of a particle after


t seconds is x = 2e3t cm.
(a) Find its initial velocity.
(b) Find its acceleration after 3 seconds, (a) When is the particle at the origin?
to 1 decimal place. (b) When is it at rest?
.. (c) When is it travelling at its greatest
(c) Show that x = 9x.
speed?
..
5. A particle accelerates at x = 8 cos 2t ms 2. (d) Sketch the graph of its
The particle is initially at rest at the (i) velocity
origin. Find its displacement after (ii) acceleration.

seconds.
6 10. A radioactive substance decays by
10% after 80 years.
6. If a particle has displacement x = 2 sin 3t,
(a) By how much will it decay after
show that its acceleration is given by
.. 500 years?
x = 9x.
(b) When will it decay to a quarter of
7. A particle has displacement its mass?
x = t3 12t2 + 36t 9 cm at time
11. The equation for displacement of a
t seconds.
particle over time t seconds is x = sin t
(a) When is the particle at rest?
metres.
(b) What is the
(a) When is the displacement 0.5 m?
(i) displacement
(b) When is the velocity 0.5 ms 1 ?
(ii) velocity

(iii) acceleration after 1 s? (c) Find the acceleration after s.
4
(c) Describe the motion of the particle
after 1 s.
Chapter 6 Applications of Calculus to the Physical World 251

12. The height of a ball is h = 20t 5t2 metres 14. The graph below shows the velocity of a
after t seconds. particle.
(a) Find the height after 1 s.
(b) What is the maximum height of
the ball?
(c) What is the time of ight of the ball?

13. A bird population of 8500 increases to


12 000 after 5 years. Find
(a) the population after 10 years
(b) the rate at which the population is (a) Sketch a graph that shows
increasing after 10 years (i) displacement
(c) when the population reaches 30 000. (ii) acceleration
(b) When is the particle at rest?

15. A particle is moving with acceleration


a = 10e2t ms 2. If it is initially 4 m to the
right of the origin and has a velocity
of 2 ms 1, nd its displacement after
5 seconds, to the nearest metre.

Challenge Exercise 6
1. (a) Find an equation for the 3. (a) Find an equation for the
displacement of a particle from the displacement of a particle from the
origin if its acceleration is given by origin if its acceleration is given by
dv dv
= 6 cos 3t cms 2 and initially the = 16 cos 4t cms 2 and initially the
dt dt
particle is at rest 2 cm to the right of the particle is at rest 1 cm to the right of the
origin. origin.
(b) Write the acceleration of the particle (b) Find the exact velocity of the particle
in terms of x. when it is 0.5 cm from the origin.

2. The displacement of a particle is given by 4. The velocity, in ms 1, of an object is


x = (t3 + 1) 6, where x is in metres and t is given by v = 5 cos 5t.
in seconds. (a) Show that if the object is initially at
(a) Find its initial displacement and the origin, its acceleration will always be
velocity. 25 times its displacement.
(b) Find its acceleration after 2 s in (b) Find the maximum acceleration of
scientic notation, correct to the object.
3 signicant gures. (c) Find the acceleration when the object
(c) Show that the particle is never at is 0.3 m to the right of the origin.
the origin.
252 Maths In Focus Mathematics HSC Course

5. The population of a ock of birds 6. The Logistic Law of Population Growth,


over t years is given by the formula rst proposed by Verhulst in 1837, is
P = P0 e 0.0151t . dN
given by = kN bN2, where k and b
(a) How long will it take, correct dt
to 1 decimal place, to increase the are constants. Show that the equation
population by 35%? kN0
N= is a solution of this
(b) What will be the percentage increase bN0 + (k bN0) e kt
in population after 10 years, to the differential equation (N0 is a constant).
nearest per cent?
7. A particle moves so that its velocity is
given by x: = tet cms 1.
2

Find its exact acceleration after 1 s.


7
Series

TERMINOLOGY

Annuity:
Terminology Bold Terminology
An annuity is a xed sum of money invested
text... Partial sum: The sum of a certain nite number of terms
every year that accumulates interest over a number of of a sequence
years
Sigma notation: The Greek letter stands for the sum of a
Arithmetic sequence: A set of numbers that form a sequence of a certain number of terms, called a partial
pattern where each successive term is a constant amount sum
(positive or negative) more than the previous one
Sequence: A set of numbers that form a pattern or obey
Common difference: The constant amount in an arithmetic a xed rule
sequence that is added to each term to get the next term
Series: The sum of a sequence of n terms
Common ratio: The constant amount in a geometric
Superannuation: A sum of money that is invested every
sequence that is multiplied to each term to get the next
year (or more frequently) as part of a salary to provide a
term
large amount of money when a person retires from the
Compound interest: Interest is added to the balance of a paid workforce
bank account so that the interest and balance both earn
Term: A term refers to the position of the number in a
interest
sequence. For example the rst term is the rst number
Geometric sequence: A set of numbers that form a in the sequence
pattern where each successive term is multiplied by a
constant amount to get the next term
Limiting sum: (or sum to innity) The sum of innite
terms of a geometric series where the common ratio r
obeys the condition 1 < r < 1
Chapter 7 Series 259

INTRODUCTION
THE INFINITE SUM OF a sequence of numbers (or terms) is called a series. Many
series occur in real lifethink of the way plants grow, or the way money
accumulates as a certain amount earns interest in a bank.
You will look at series in general, arithmetic, geometric series and their
applications.

General Series
A sequence forms a pattern. Some patterns are easy to see and some are
difcult. People are sometimes asked to identify sequences and their patterns
in tests of intelligence.

DID YOU KNOW?

Number patterns and series of numbers have been known since the very beginning of civilisation.
The Rhind Papyrus, an Egyptian book written about 1650 BC, is one of the oldest books
on mathematics. It was discovered and deciphered by Eisenlohr in 1877, and it showed that
the ancient Egyptians had a vast knowledge of mathematics. The Rhind Papyrus describes the
problem of dividing 100 loaves among 5 people in a way that shows the Egyptians were exploring
arithmetic series.
Around 500 BC the Pythagoreans explored different polygonal numbers:

triangular numbers: 1 + 2 + 3 + 4 +
DID YOU KNOW?

Box text...

1 1+ 2 1+ 2 + 3 1+ 2 + 3 + 4

square numbers: 1 + 3 + 5 + 7 +

Could you nd a series for


1 1+ 3 1+ 3 + 5 1+ 3 + 5 + 7 pentagonal or hexagonal
numbers?
260 Maths In Focus Mathematics HSC Course

7.1 Exercises
Find the next 3 terms in each sequence or series of numbers.

1. 5, 8, 11, ... 11. 1, 2, 4, 8,

2. 8, 13, 18, ... 12. 3, 6, 12, 24,

3. 11 + 22 + 33 + 1 1 1
13. , , ,
2 4 8
4. 100, 95, 90, ... 2 4 8
14. , , ,
5 15 45
5. 7+5+3+ 15. 1 + 4 + 9 + 16 + 25 +
6. 99, 95, 91, ... 16. 1, 8, 27, 64, ...
1 1
7. , 1, 1 , 17. 0, 3, 8, 15, 24, ...
2 2
8. 1.3 + 1.9 + 2.5 + f 18. 3 + 6 + 11 + 18 + 27 +
9. 2, 4, 8, ... 19. 2 + 9 + 28 + 65 +
10. 4 + 12 + 36 + 20. 1, 1, 2, 3, 5, 8, 13, ...

DID YOU KNOW?

The numbers 1, 1, 2, 3, 5, 8, are called Fibonacci numbers after Leonardo Fibonacci


(11701250). The Fibonacci numbers occur in natural situations.
For example, when new leaves grow on a plants stem, they spiral around the stem.
The ratio of the number of turns to the number of spaces between successive leaves gives the
1 1 2 3 5 8 13 21 34 55 89
series of fractions , , , , , , , , , , ,
1 2 3 5 8 13 21 34 55 89 144
Chapter 7 Series 261

The Fibonacci ratio is the number of


turns divided by the number of spaces.

Research Fibonacci numbers and nd out where else they appear in nature.

Formula for the nth term of a series

When a series follows a mathematical pattern, its terms can be described by a


formula. We call any general or nth term of a series Tn where n stands for the
number of the term and must be a positive integer.

EXAMPLE

For the series 6 + 13 + 20 + nd


(a) T1
(b) T2
(c) T3
(d) Tn

Solution
(a) The rst term is 6, so when n = 1: T1 = 6
(b) The 2nd term is 13 so T2 = 13
(c) The 3rd term is 20 so T3 = 20
(d) Each term is 7 more than the previous term.
The 1st term is 6
The 2nd term is 13 = 6 + 7
The 3rd term is 20 = 6 + 7 + 7
= 6 + 2 7
CONTINUED
262 Maths In Focus Mathematics HSC Course

Following this pattern:


The 4th term = 6 + 7 + 7 + 7
= 6 + 37
Notice that the product with 7 in each
term involves a number one less than
and so on
the number of the term. Tn = 6 + (n 1) 7
Tn = 6 + 7n 7
= 7n 1

If we are given a formula for the nth term, we can nd out much more
about the series.

EXAMPLES

1. The nth term of a series is given by the formula Tn = 3n + 4. Find the


rst 3 terms and hence write down the series.

Solution
T1 = 3 (1) + 4
=7
T2 = 3 (2) + 4
= 10
T3 = 3 (3) + 4
= 13
So the series is 7 + 10 + 13 +

2. The nth term of a series is given by tn = 5n 1. Which term of the


series is equal to 104?

Solution
tn = 5n 1 = 104
5n = 105
n = 21
So 104 is the 21st term of the series.

3. The nth term of a series is given by u n = 103 3n.


(a) Find the rst 3 terms.
(b) Find the value of n for the rst negative term in the series.
Chapter 7 Series 263

Solution
(a) u1 = 103 3 (1)
= 100
u2 = 103 3 (2)
= 97
u3 = 103 3 (3)
= 94
So the series is 100 + 97 + 94 + . . . .
(b) For the rst negative term, we want
un < 0
i.e. 103 3n < 0
3n < 103
1
n > 34
3
Since n is an integer, n = 35
the 35th term is the rst negative term.

4. The nth term of a series is given by the formula tn = 2n 1. Which


term of the series is equal to 4095?

Solution

tn = 2n 1
We are given that the nth term is 4095 and need to nd n.
4095 = 2n 1
4096 = 2n You could use base e
instead of base 10.
log10 4096 = log10 2n
= n log10 2 You could use trial and
log10 4096 error to guess what
=n power of 2 is equal to
log10 2 4096 if you dont want to
12 = n use logarithms.

So 4096 is the 12th term of the sequence.

7.2 Exercises
1. Find the rst 3 terms of the series (f) un = 3n
with nth term as follows (g) Qn = 2n + 7
(a) Tn = 8n 5 (h) tn = 4n 2n
(b) Tn = 2n + 3 (i) Tn = 8n2 n + 1
(c) un = 6n 1 (j) Tn = n3 + n
(d) Tn = 8 5n
(e) tn = 20 n
264 Maths In Focus Mathematics HSC Course

2. Find the rst 3 terms of each series. 10. Which term of Tn = n2 3 is equal
(a) Tn = 3n 2 to 526?
(b) tn = 4n
(c) Tn = n2 + n 11. For the series Tn = n3, nd
(a) the 12th term
3. Find the 50th term of each series. (b) n if the nth term is 15 625.
(a) Tn = 7n 1
(b) tn = 2n + 5 12. A series is given by the formula
(c) Tn = 5n 2 Tn = 3 + 2n n2.
(d) Tn = 40 3n (a) Find the 25th term.
(e) Un = 8 7n (b) Find n if the nth term is 252.

4. Find the 10th term of each series. 13. Find the value of n that gives the
(a) Tn = 2n + 5 rst term of the series Tn = 3n + 2
(b) tn = 10 3n greater than 100.
(c) Tn = 2n n
14. Find the value of n that gives
(d) un = n2 5n + 3
the rst term of the series Tn = 2n
(e) Tn = n3 + 2
larger than 500.
5. Which of these are terms of the
15. Find the values of n where the
series Tn = 3n + 5?
series Tn = 4n 1 is greater than
(a) 68
350.
(b) 158
(c) 205 16. Find the values of n for which the
(d) 266 series Tn = 400 5n is less than
(e) 300 200.

6. Which of these are terms of the 17. Find the value of n that gives the
series Tn = 5n 1? rst negative term of the series
(a) 3126 Tn = 1000 2n.
(b) 124
18. Find the value of n that gives the
(c) 15 634
rst positive term of the series
(d) 78 124
Tn = 2n 300.
(e) 0
19. Find
7. Which term of the series with nth
(a) the value of n that gives the
term tn = 9n 15 is equal to 129?
rst negative term of the series
8. Is 255 a term of the series with un = 80 6n
nth term Tn = 2n 1? (b) the rst negative term.

9. Which term of the series with nth 20. Find the rst negative term of the
term un = n3 + 5 is 348? series Tn = 50 7n.
Chapter 7 Series 265

Sigma Notation
Series are often written in sigma notation. The Greek letter sigma () is like an
English S. Here it stands for the sum of a series.

Application

The mean of a set of scores is given by

x = fx
f
where f is the sum of frequencies and fx is the sum of the scores frequencies.

EXAMPLES
5
1. Evaluate / r2
r =1

Solution
5
/ r2 means the sum of terms where the formula is r2 with r starting at 1
r =1
and ending at 5.
5
So / r2 = 12 + 22 + 32 + 42 + 52
r =1
= 1 + 4 + 9 + 16 + 25
= 55

7
2. Evaluate / (2n + 5)
3

Solution
7
/ (2n + 5) means the sum of terms where the formula is 2n + 5 with n
3
starting at 3 and ending at 7.
Can you work out how many terms there are in this series?
Notice that there are 5
7
/ (2n + 5) = (2 3 + 5) + (2 4 + 5) + (2 5 + 5) + (2 6 + 5) + (2 7 + 5) terms, since they include
both the 3 and the 7. We
3
= 11 + 13 + 15 + 17 + 19 work out the number of
terms by 7 3 + 1.
= 75

CONTINUED
266 Maths In Focus Mathematics HSC Course

3. Write 7 + 11 + 15 + + (4k + 3) in sigma notation.

Solution

If n is not 1, we could try 2


In this question, the formula is 4n + 3 where the last term is at n = k.
then 3 and so on until we We need to nd the number of terms.
nd the rst term. The rst term is 7 so we guess that n = 1 for this term.
When n = 1
4n + 3 = 4 1 + 3
=7
So the rst term is at n = 1 and the last term is at n = k.
k
We can write the series as / (4n + 3).
1

4. How many terms are there in the series


100
(a) / (3n 7) ?
1
50
(b) / 2n ?
5

Solution
There are 100 1 + 1 (a) The rst term is when n = 1 and the last term is when n = 100.
terms. There are 100 terms.
(b) The rst term is when n = 5 and the last term is when n = 50.
This is harder to work out. Look at example 2 where there are 5 terms.
We can work out the number of terms by 50 5 + 1.
There are 46 terms.

p
The number of terms in the series / f (n) is p q + 1
q

7.3 Exercises
1. Evaluate 5
4 (f) / 1n
(a) / (3 n + 2) 3
n =1 5
5 (g) / (n 2 n)
(b) / n2 n =1
n =2 4
6 (h) / | 3p 2|
(c) / (5n 6) 2
2 6
10 (i) / (2n)
(d) / (3r + 1) n =1
r =1 6
5 (j) / (3 n 2n 5)
(e) / (k 3 1) 3
k =2
Chapter 7 Series 267

2. How many terms are there in 77

each series?
(i) / (2n 3)
3
65
(a) / (7 ) n
(j)
55
/ (7 n + n )
n =1 11
100
(b) / (n 2 + 1 )
n =2
3. Write these series in sigma
80
(c) / (3n + 4) notation.
5 (a) 1 + 3 + 5 + 7 + + 11
200
(d) / (3r2) (b) 7 + 14 + 21 + + 70
r =1 (c) 1 + 8 + 27 + 64 + 125
20
(d) 2 + 8 + 14 + + (6n 4)
(e) / (7n + 1)
10 (e) 9 + 16 + 25 + 36 + + n2
45
(f) 1 2 3 50
(f) / (n 2 + 3n) (g) 3 + 6 + 12 + + 3 2n
7
108 1 1 1
(g) / n3 (h) 1 + + + +
2 4 512
n = 12
(i) a + (a + d ) + (a + 2d ) + +
74
(h) / 4n (a + 5 n 1 ? d )
n =9
(j) a + ar + ar2 + ar3 + + ar n 1

Arithmetic Series
In an arithmetic series each term is a constant amount more than the
previous term. The constant is called the common difference.

EXAMPLES

1. Find the common difference of the series 5 + 9 + 13 + 17 + 21 +

Solution
We can see that the common difference between the terms is 4. To check
this, we notice that 9 5 = 4, 13 9 = 4, 17 13 = 4 and 21 17 = 4.

2. Find the common difference of the series 85 + 80 + 75 + 70 + 65 +

Solution
We can see that the common difference between the terms is 5. To
check this, we notice that 80 85 = 5, 75 80 = 5, 70 75 = 5 and
65 70 = 5.
268 Maths In Focus Mathematics HSC Course

If T1, T2 and T3 are consecutive terms of an arithmetic series then


d = T2 T1 = T3 T2

Generally, if T1, T2, T3, Tn 1, Tn are consecutive terms of an arithmetic


series then d = T2 T1 = T3 T2 = = Tn Tn 1 .

EXAMPLES

1. If 5 + x + 31 + is an arithmetic series, nd x.

Solution
For an arithmetic series,
T2 T1 = T3 T2
5 + 31
i.e. x 5 = 31 x
Notice that x =
2
which is the average of
2x 5 = 31
T1 and T2. We call x the 2x = 36
arithmetic mean.
` x = 18

2.
(a) Evaluate k if (k + 2) + (3k + 2) + (6k 1) + is an arithmetic series.
(b) Write down the rst 3 terms of the series.
(c) Find the common difference d.

Solution
(a) For an arithmetic series, T2 T1 = T3 T2
So (3k + 2) (k + 2) = (6k 1) (3k + 2)
3k + 2 k 2 = 6k 1 3k 2
2k = 3k 3
0 =k3
3=k
(b) The series is (k + 2) + (3k + 2) + (6k 1) +
Substituting k = 3:
T1 = k + 2
=3+2
=5
T2 = 3k + 2
=33+ 2
= 11
T3 = 6k 1
= 6 31
= 17
(c) The sequence is 5 + 11 + 17 + . . .
11 5 = 17 11 = 6
So common difference d = 6.
Chapter 7 Series 269

Terms of an arithmetic series

a + (a + d ) + (a + 2d ) + (a + 3d ) + + [a + (n 1) d ] + is an arithmetic
series with rst term a, common difference d and nth term given by
Tn = a + (n 1) d

Proof
Let the rst term of an arithmetic series be a and the common difference d.
Then rst term is a
second term is a + d
third term is a + 2d
fourth term is a + 3d, and so on.
The nth term is a + (n 1) d.

EXAMPLES

1. Find the 20th term of the series 3 + 10 + 17 + .

Solution
a = 3, d = 7, n = 20
Tn = a + (n 1) d
T20 = 3 + (20 1) 7
= 3 + 19 7
= 136

2. Find an expression for the nth term of the series 2 + 8 + 14 + . . . .

Solution
a = 2, d = 6
Tn = a + (n 1) d
= 2 + (n 1) 6
= 2 + 6n 6
= 6n 4

3. Find the rst positive term of the series 50 47 44 .

Solution
a = 50, d = 3
For the rst positive term,
Tn > 0
CONTINUED
270 Maths In Focus Mathematics HSC Course

i.e. a + (n 1) d >0
50 + (n 1) 3 >0
50 + 3n 3 >0
3n 53 >0
3n > 53
2
n must be a positive integer. n > 17
3
` n = 18 gives the rst positive term
T18 = 50 + (18 1) 3
= 50 + 17 3
=1
So the rst positive term is 1.

4. The 5th term of an arithmetic series is 37 and the 8th term is 55. Find
the common difference and the rst term of the series.

Solution

Tn = a + (n 1) d
T5 = a + (5 1) d = 37
i.e. a + 4d = 37 ( 1)
T8 = a + (8 1) d = 55
i.e. a + 7d = 55 (2)
(2) (1): a + 4d = 37
3d = 18
d=6
Put d = 6 in (1):
a + 4 (6) = 37
a + 24 = 37
a = 13
So d = 6 and a = 13.

7.4 Exercises
1. The following series are arithmetic. (f) 32 + (x 1) + 51 +
Evaluate all pronumerals. (g) 3 + (2k + 3) + 21 +
(a) 5 + 9 + y + (h) x + (x + 3) + (2x + 5) +
(b) 8 + 2 + x + (i) (t 5) + 3t + (3t + 1) +
(c) 45 + x + 99 + (j) (2t 3) + (3t + 1) + (5t + 2) +
(d) 16 + b + 6 +
(e) x + 14 + 21 +
Chapter 7 Series 271

2. Find the 15th term of each series. 10. Which term of the series
(a) 4 + 7 + 10 + 25 + 18 + 11 + is equal to 73?
(b) 8 + 13 + 18 +
11. Is zero a term of the series
(c) 10 + 16 + 22 +
48 + 45 + 42 + ?
(d) 120 + 111 + 102 +
(e) 3 + 2 + 7 + 12. Is 270 a term of the series
3 + 11 + 19 + ?
3. Find the 100th term of each series.
(a) 4 + 2 + 8 + 13. Is 405 a term of the series
(b) 41 + 32 + 23 + 0 + 3 + 6 +?
(c) 18 + 22 + 26 +
(d) 125 + 140 + 155 + 14. Find the rst value of n for
(e) 1 5 9 which the terms of the series
100 + 93 + 86 + become less
4. What is the 25th term of each than 20.
series?
(a) 14 18 22 15. Find the values of n for
(b) 0.4 + 0.9 + 1.4 + which the terms of the series
(c) 1.3 + 0.9 + 0.5 + 86 83 80 are positive.
1
(d) 1 + 2 + 4 + 16. Find the rst negative term of
2
2 3
(e) 1 + 2 + 2 + 54 + 50 + 46 + .
5 5
5. For the series 3 + 5 + 7 + , write 17. Find the rst term that is
an expression for the nth term. greater than 100 in the series
3 + 7 + 11 + .
6. Write an expression for the nth
term of the following series. 18. The rst term of an arithmetic
(a) 9 + 17 + 25 + series is -7 and the common
(b) 100 + 102 + 104 + difference is 8. Find the 100th
(c) 6 + 9 + 12 + term of the series.
(d) 80 + 86 + 92 + 19. The rst term of an arithmetic
(e) 21 17 13 series is 15 and the 3rd term is 31.
(f) 15 + 10 + 5 + (a) Find the common difference
7 1
(g) + 1 + 1 + (b) Find the 10th term of the
8 8
(h) 30 32 34 series.
(i) 3.2 + 4.4 + 5.6 +
20. The rst term of an arithmetic
1 1
(j) + 1 + 2 + series is 3 and the 5th term is 39.
2 4
Find its common difference.
7. Find which term of
3 + 7 + 11 + is equal to 111. 21. The 2nd term of an arithmetic
series is 19 and the 7th term
8. Which term of the series
is 54. Find its rst term and
1 + 5 + 9 + is 213?
common difference.
9. Which term of the series
15 + 24 + 33 + is 276?
272 Maths In Focus Mathematics HSC Course

22. Find the 20th term in an 27. (a) Show that


arithmetic series with 4th term 3 + 12 + 27 + . . . is an
29 and 10th term 83. arithmetic series.
(b) Find the 50th term in
23. The common difference of an
simplest form.
arithmetic series is 6 and the 5th
term is 29. Find the rst term of 28. Find the 25th term of
the series. log2 4 + log2 8 + log2 16 +

24. If the 3rd term of an arithmetic 29. Find the 40th term of
series is 45 and the 9th term is 75, 5b + 8b + 11b +
nd the 50th term of the series.
30. Which term is 213y of the series
25. The 7th term of an arithmetic 28y + 33y + 38y + ?
series is 17 and the 10th term is 53.
Find the 100th term of the series.

26. (a) Show that


log5 x + log5 x2 + log5 x3 + is an
arithmetic series.
(b) Find the 80th term.
(c) If x = 4, evaluate the 10th term
correct to 1 decimal place.

Partial sum of an arithmetic series

The sum of the rst n terms of an arithmetic series (nth partial sum) is given
by the formula:

n
Sn = (a + l) where l = last or nth term
2

Proof
Let the last or nth term be l.
S n = a + ( a + d ) + ( a + 2d ) + + l ( 1)
Sn = l + (l d ) + (l 2d ) + + a (2)
2Sn = (a + l ) + (a + l ) + (a + l ) + + (a + l ) (1) + (2)
= n (a + l )
n
` S n = (a + l )
2
In general,

n
Sn = [ 2 a + ( n 1) d ]
2
Chapter 7 Series 273

Proof
Since l = n th term,
l = a + ( n 1) d
n
` Sn =(a + l)
2
n
= [ a + a + ( n 1 ) d]
2
n
= [2a + (n 1) d]
2
We use this formula when the nth term is unknown.

EXAMPLES

1. Evaluate 9 + 14 + 19 + + 224.

Solution
a = 9, d = 5
First we nd n.
Tn = 224
` Tn = a + (n 1) d
224 = 9 + (n 1) 5
= 9 + 5n 5
= 5n + 4
220 = 5n
44 = n
n
` S n = (a + l )
2
44
S44 = (9 + 224)
2
= 22 233
= 5126

2. For what value of n is the sum of n terms of 2 + 11 + 20 + equal


to 618?

Solution
a = 2, d = 9, Sn = 618
n
Sn = [2a + (n 1) d]
2
n
618 = [2 2 + (n 1) 9]
2
1236 = n (4 + 9n 9)
= n (9n 5)
= 9n2 5n

CONTINUED
274 Maths In Focus Mathematics HSC Course

0 = 9n2 5n 1236
= (n 12) (9n + 103)
n must be a positive integer.
` n = 12 or 11.4
But n cannot be negative, so n = 12.

3. The 6th term of an arithmetic series is 23 and the sum of the rst
10 terms is 210. Find the sum of 20 terms.

Solution
Tn = a + (n 1) d
T6 = a + (6 1) d = 23
a + 5d = 23 (1)
n
S n = [ 2 a + ( n 1 ) d]
2
10
S10 = [2a + (10 1) d] = 210
2
5 (2a + 9d) = 210
2a + 9d = 42 (2 )
(1) (2): 2a + 10d = 46 (3 )
(2) (3): d = 4
d=4
Substitute d = 4 in (1):
a + 5 (4) = 23
a + 20 = 23
a=3
20
S20 = [2 (3) + (20 1) 4]
2
= 10 [6 + 19 (4)]
= 10 82
= 820

50
4. Evaluate / 3r + 2.
r =1

Solution
50
It would take a long time to
add these up!
/ 3r + 2 = (3 1 + 2) + (3 2 + 2) + (3 3 + 2) + + (3 50 + 2)
r =1
= 5 + 8 + 11 + + 152
Arithmetic series with a = 5, d = 3, l = 152, n = 50
n
S n = (a + l)
2
50
S50 = (5 + 152)
2
= 25 157
= 3925
Chapter 7 Series 275

7.5 Exercises
1. Find the sum of 15 terms of the 7. How many terms of the series
series. 45 + 47 + 49 + give a sum of
(a) 4 + 7 + 10 + 1365?
(b) 2 + 7 + 12 +
8. For what value of n is the
(c) 60 + 56 + 52 +
sum of the arithmetic series
2. Find the sum of 30 terms of the 5 + 9 + 13 + equal to 152?
series.
9. How many terms of the series
(a) 1 + 7 + 13 +
80 + 73 + 66 + give a sum
(b) 15 + 24 + 33 +
of 495?
(c) 95 + 89 + 83 +
10. The sum of the rst 5 terms of
3. Find the sum of 25 terms of the
an arithmetic series is 110 and
series.
the sum of the rst 10 terms is
(a) 2 + 5 + 12 +
320. Find the rst term and the
(b) 5 4 13
common difference.
4. Find the sum of 50 terms of the
11. The sum of the rst 5 terms of
series.
an arithmetic series is 35 and
(a) 50 + 44 + 38 +
the sum of the next 5 terms is
(b) 11 + 14 + 17 +
160. Find the rst term and the
5. Evaluate common difference.
(a) 15 + 20 + 25 + + 535
12. Find S25, given an arithmetic
(b) 9 + 17 + 25 + + 225
series whose 8th term is 16 and
(c) 5 + 2 1 91
whose 13th term is 81.
(d) 81 + 92 + 103 + + 378
(e) 229 + 225 + 221 + + 25 13. The sum of 12 terms of an
(f) 2 + 6 + 14 + + 94 arithmetic series is 186 and the
(g) 0 9 18 216 20th term is 83. Find the sum
(h) 79 + 81 + 83 + + 229 of 40 terms.
(i) 14 + 11 + 8 + 43
1 3 1 14. How many terms of the series
(j) 1 + 1 + 2 + + 25
2 4 4 20 + 18 + 16 + give a sum of
6. Evaluate 104?
20
(a) / 4n 7 15. The sum of the rst 4 terms of
n =1
15 an arithmetic series is 42 and
(b) / 5 3r the sum of the 3rd and 7th term
r =1
20 is 46. Find the sum of the rst
(c) / 4 6r 20 terms.
r =3
50
(d) / 5n + 3 16. (a) Show that
n =1 (x + 1) + (2x + 4) + (3x + 7) + is
40
an arithmetic series.
(e) / 4 3n
5 (b) Find the sum of the rst 50
terms of the series.
276 Maths In Focus Mathematics HSC Course

17. The 20th term of an arithmetic 19. Prove that Tn = Sn Sn 1 for any
series is 131 and the sum of the series.
6th to 10th terms inclusive is 235.
Find the sum of the rst 20 terms. 20. Find the sum of all integers
between 1 and 100 that are not
18. The sum of 50 terms of an multiples of 6.
arithmetic series is 249 and the
sum of 49 terms of the series is
233. Find the 50th term of the
series.

Class Investigation

Look at the working out of question 14 in the previous set of exercises.


Why are there two values for n?

DID YOU KNOW?

Here is an interesting series:


1 1 1 1 1
= + + .
4 1 3 5 7 9

Gottfried Wilhelm Leibniz (16461716) discovered this result. It is interesting that while
is an irrational number, it can be written as the sum of rational numbers.

Geometric Series
In a geometric series each term is formed by multiplying the preceding term
by a constant. The constant is called the common ratio.

EXAMPLE

Find the common ratio of the series 3 + 6 + 12 +

Solution
By looking at this sequence, each term is multiplied by 2 to get the next
term.
If you cant see this, we divide the terms as follows:
6 12
= =2
3 6
Chapter 7 Series 277

If T1 + T2 + T3 + is a geometric series then


T2 T3
r= =
T1 T2

In general, if T1 + T2 + T3 + T4 + + Tn 1 + Tn is a geometric series then


T2 T3 T4 Tn
r= = = == .
T1 T2 T3 Tn 1

EXAMPLES

1. Find x if 5 + x + 45 + is a geometric series.

Solution
T2 T3
For a geometric series =
T1 T2
x 45
= x
5 x is called the
x2 = 225 geometric mean.
x = 225
= 15 When r is negative, the
If x = 15 the series is 5 + 15 + 45 + (r = 3) . signs alternate.

If x = 15, the series is 5 15 + 45 (r = 3) .

1 1 1
2. Is + + + a geometric series?
4 6 18
Solution
T2 1 1
=
T1 6 4
1 4
=
6 1
2
=
3
T3 1 1
=
T2 18 6
1 6
=
18 1
1
=
3
T2

T1
the series is not geometric.
278 Maths In Focus Mathematics HSC Course

Terms of a geometric series

a + ar + ar2 + ar3 + + arn 1 + is a geometric series with rst


term a, common ratio r and nth term given by
Tn = ar n 1

Proof
Let the rst term of a geometric series be a and the common ratio be r.
Then rst term is a
second term is ar
third term is ar 2
fourth term is ar 3, and so on
nth term is ar n 1

EXAMPLES

1.
(a) Find the 10th term of the series 3 + 6 + 12 +
(b) Write an expression for the nth term of the series.

Solution
(a) This is a geometric series with a = 3 and r = 2.
We want the 10th term, so n = 10.
Tn = ar n 1
T10 = 3 (2)10 1
= 3 29
= 1536

(b) Tn = ar n 1
= 3 ( 2) n 1

2 4 8
2. Find the common ratio of + + + and hence nd the 8th
3 15 75
term in index form.

Solution
4 2 8 4
r= c= m
15 3 75 15
4 3
=
15 2
2
=
5
Chapter 7 Series 279

Tn = ar n 1
2 2 81
T8 = c m
3 5
2 2 7
= c m
3 5
28 Terms can become very
= large in geometric series.
3 57

3. Find the 10th term of the series 5 + 10 20 +

Solution
a = 5, r = 2, n = 10
Tn = ar n 1
T10 = 5 ( 2)10 1
= 5 ( 2)9
= 5 ( 512)
= 2560

4. Which term of the series 4 + 12 + 36 + is equal to 78 732?

Solution
This is a geometric series with a = 4 and r = 3.
The nth term is 78 732.
Tn = ar n 1
78 732 = 4 (3)n 1
19 683 = 3n 1
log10 19 683 = log10 3n 1
= (n 1) log 3
log10 19 683
= n 1
log10 3
9 = n 1
You could use logarithms to
10 = n the base e or ln instead of
logarithms to the base 10.
So the 10th term is 78 732.

CONTINUED
280 Maths In Focus Mathematics HSC Course

5. The third term of a geometric series is 18 and the 7th term is 1458.
Find the first term and the common ratio.

Solution
Tn = ar n 1
T3 = ar 3 1 = 18
ar 2 = 18 (1)
71
T7 = ar = 1458
Always divide the equations in
ar = 1458
6
(2)
geometric series to eliminate a.

(2) (1): r4 = 81
r = 4 81
= 3
Substitute r = 3 into (1):
a (3)2 = 18
9a = 18
a=2

1
6. Find the first value of n for which the terms of the series + 1 + 5 +
5
exceed 3000.

Solution
1
a= ,r = 5
5
For terms to exceed 3000,
Tn > 3000
ar n 1 > 3000
1 n1
(5 ) > 3000
5
5n 1 > 15 000
log 5n 1 > log 15 000
(n 1) log 5 > log 15 000
log 15 000
n 1 >
log 5
log 15 000
n> +1
log 5
n must be an integer. n > 6.97
` n=7
So the 7th term is the first term to exceed 3000.
Chapter 7 Series 281

7.6 Exercises
1. Are the following series (h) 1000 100 + 10
geometric? If they are, nd the (i) 3 + 9 27 +
common ratio. 1 2 4
(j) + + +
(a) 5 + 20 + 60 + 3 15 75
1
(b) 4 + 3 2 + 4. Find the 6th term of each series.
4
3 3 3 (a) 8 + 24 + 72 +
(c) + + +
4 14 49 (b) 9 + 36 + 144 +
5 1 (c) 8 32 + 128 +
(d) 7 + 5 + 3 +
6 3 (d) 1 + 5 25 +
(e) 14 + 42 168 + 2 4 8
1 8 8 (e) + + +
(f) 1 + + + 3 9 27
3 9 27
(g) 5.7 + 1.71 + 0.513 + 5. What is the 9th term of each
1 7 81
(h) 2 1 + + series?
4 20 100
7 (a) 1 + 2 + 4 +
(i) 63 + 9 + 1 + (b) 4 + 12 + 36 +
8
7 (c) 1 + 1.04 + 1.0816 +
(j) 1 + 15 120 +
8 (d) 3 + 6 12 +
3 3 3
2. Evaluate all pronumerals in these (e) +
4 8 16
geometric series.
(a) 4 + 28 + x + 6. Find the 8th term of each series.
(b) 3 + 12 + y + (a) 3 + 15 + 75 +
(c) 2 + a + 72 + (b) 2.1 + 4.2 + 8.4 +
(d) y + 2 + 6 + (c) 5 20 + 80
(e) x + 8 + 32 + 1 3 9
(d) + +
(f) 5 + p + 20 + 2 10 50
47 10 5
(g) 7 + y + 63 + (e) 1 +2 + 3 +
81 27 9
(h) 3 + m 12 +
(i) 3 + (x 4) + 15 + 7. Find the 20th term of each series,
(j) 3 + (k 1) + 21 + leaving the answer in index form.
1 1 (a) 3 + 6 + 12 +
(k) + t + + ,
4 9
(b) 1 + 7 + 14 +
1 4
(l) + t + +, (c) 1.04 + 1.042 + 1.043 +
3 3
1 1 1
(d) + + +
3. Write an expression for the nth 4 8 16
3 9 27
term of the following series. (e) + + +
4 16 64
(a) 1 + 5 + 25 +
(b) 1 + 1.02 + 1.0404 +
(c) 1 + 9 + 81 +
(d) 2 + 10 + 50 +
(e) 6 + 18 + 54 +
(f) 8 + 16 + 32 +
1
(g) + 1 + 4 +
4
282 Maths In Focus Mathematics HSC Course

8. Find the 50th term of 16. The rst term of a geometric


1 + 11 + 121 + in index form. series is 7 and the 6th term is
1701. Find the common ratio.
9. Which term of the series
4 + 20 + 100 + is equal 17. The 4th term of a geometric series
to 12 500? is 648 and the 5th term is 3888.
(a) Find the common ratio.
10. Which term of 6 + 36 + 216 +
(b) Find the 2nd term.
is equal to 7776?
18. The 3rd term of a geometric series
11. Is 1200 a term of the series 2 3
is and the 5th term is 1 . Find
2 + 16 + 128 + ? 5 5
its rst term and common ratio.
12. Which term of 3 + 21 + 147 +
is equal to 352 947? 19. Find the value of n for
the rst term of the series
13. Which term of the series 5000 + 1000 + 200 + that is
1
8 4 + 2 is ? less than 1.
128
14. Which term of 54 + 18 + 6 + 20. Find the rst term of the series
2 2 6 4
is ? + + 2 + that is greater
243 7 7 7
15. Find the value of n if the nth than 100.
term of the series
1 1 81
2 + 1 1 + is .
2 8 128

Partial sum of a geometric series

The sum of the rst n terms of a geometric series (nth partial sum) is given by
the formula:

a ( r n 1)
Sn = for |r | > 1
You studied absolute values r1
in the Preliminary Course
| r | < 1 means 1 < r < 1.
a (1 r n )
Sn = for |r | < 1
What does | r | > 1 mean? 1r

Proof
The sum of a geometric series can be written
Sn = a + ar + ar 2 + . . . + ar n 1 (1)
rSn = ar + ar 2 + ar 3 + . . . + ar n (2)
(1) (2): Sn (1 r) = a ar n
= a (1 r n )
a (1 r n )
Sn =
1r

(2) (1) gives the formula


The two formulae are the same, a (r n 1 )
just rearranged. Sn =
r1
Chapter 7 Series 283

EXAMPLES

1. Find the sum of the rst 10 terms of the series 3 + 12 + 48 +

Solution
This is a geometric series with a = 3, r = 4 and we want n = 10.
Since r > 1 we use the rst formula.
a (r n 1 )
Sn =
r1
3 (410 1)
=
41
3 (410 1)
=
3
=4 1
10

= 1 048 575

11
2. Evaluate / 2n .
3

Solution
11
/ 2n = 23 + 24 + 25 + . . . + 211
3
= 8 + 16 + 32 + . . . + 2048 Can you see why n = 9?
Count the terms carefully.
Geometric series with a = 8, r = 2, n = 9
a (r n 1 )
Sn =
r1
8 (2 9 1 )
S9 =
21
8 (512 1)
=
1
= 8 511
= 4088

2 20
3. Evaluate 60 + 20 + 6 ++ .
3 81
Solution
1 20
a = 60, r = ,T = First nd n.
3 n 81
20
Tn = ar n 1 =
81

CONTINUED
284 Maths In Focus Mathematics HSC Course

1 n 1 20
60 c m =
3 81
1 n1 1
c m =
3 243
1 1
n1
=
3 243
1 1
=
3n 1 35
`n1 =5
n=6
a (1 r n )
Sn =
1r
1 6
60 = 1 c m G
3
S6 =
1
1
3
1
60 c 1 m
729
=
2
3
728 3
= 60 c m
729 2
71
= 89
81

4. The sum of n terms of 1 + 4 + 16 + is 21 845. Find the value of n.

Solution

a = 1, r = 4, Sn = 21 845
a (r n 1 )
Sn =
r1
1 (4 n 1 )
21 845 =
41
4n 1
=
3
65 535 = 4 1
n

65 536 = 4n
48 = 4n
` n=8
So 8 terms gives a sum of 21 845.
Chapter 7 Series 285

7.7 Exercises
1. Find the sum of 10 terms of the 8. Find the sum of 25 terms of
series. the series 1 + 1.12 + 1.122 + f ,
(a) 6 + 24 + 96 + correct to 2 decimal places.
(b) 3 + 15 + 75 +
9. Find the value of n if the
2. Find the sum of 8 terms of the sum of n terms of the series
series. 11 + 33 + 99 + is equal to
(a) 1 + 7 49 + 108 251.
(b) 8 + 24 + 72 +
10. How many terms of the series
3. Find the sum of 15 terms of the 1 1 1
+ + + give a sum of
series. 2 4 8
(a) 4 + 8 + 16 + 1023
?
3 3 3 1024
(b) +
4 8 16 11. The common ratio of a geometric
4. Evaluate series is 4 and the sum of the rst
(a) 2 + 10 + 50 + + 6250 5 terms is 3069. Find the rst
1 9 term.
(b) 18 + 9 + 4 + +
2 64
(c) 3 + 21 + 147 + + 7203 12. Find the number of terms needed
3 1 3 1 to be added for the sum to
(d) + 2 + 6 + + 182
4 4 4 4 exceed 1 000 000 in the series
(e) 3 + 6 12 + + 384 4 + 16 + 64 +

5. Evaluate 13. Lucia currently earns $25 000.


8
(a) /2 n1 Her wage increases by 5% each
n =1 year. Find
6
(b) / 31n (a) her wage after 6 years
n =1 (b) her total earnings (before tax)
10
in 6 years.
(c) / 5n 3
n =2
8 14. Write down an expression for the
1
(d) / series 2 10 + 50 + 2 ( 5k 1 )
r =1 2r 1
7
(a) in sigma notation
(e) / 4k + 1 (b) as a sum of n terms.
k =1

15. Find the sum of the rst


6. Find the 10 terms of the series
(a) 9th term 3 + 7 + 13 + + [2n + (2n 1)] +
(b) sum of 9 terms of the
series 7 + 14 + 28 +

7. Find the sum of 30 terms of the


series 1.09 + 1.092 + 1.093 + f ,
correct to 2 decimal places.
286 Maths In Focus Mathematics HSC Course

PUZZLES

1. A poor girl saved a rich king from drowning one day. The king offered the
girl a reward of sums of money in 30 daily payments. He gave the girl a
choice of payments:
Choice 1: $1 the rst day, $2 the second day, $3 the third day and so on.
Choice 2: 1 cent the rst day, 2 cents the second day, 4 cents the third day
and so on, the payment doubling each day.
How much money would the girl receive for each choice? Which plan
would give the girl more money?
This is a hard one! 2. Can you solve Fibonaccis Problem?
A man entered an orchard through 7 guarded gates and gathered a certain
number of apples. As he left the orchard he gave the guard at the rst gate
half the apples he had and 1 apple more. He repeated this process for each
of the remaining 6 guards and eventually left the orchard with 1 apple.
How many apples did he gather? (He did not give away any half apples.)

Limiting sum (sum to innity)

In some geometric series the sum becomes very large as n increases.


For example,

2 + 4 + 8 + 16 + 32 + 64 + 128 +

We say that this series diverges, or it has an innite sum.


In some geometric series, the sum does not increase greatly after a few
terms. We say this series converges and it has a limiting sum (the sum is
limited to a nite number).

EXAMPLE

1 1 1 1
For the series 2 + 1 + + + + + , notice that after a while the
2 4 8 16
Can you estimate the sum of terms are becoming closer and closer to zero and so will not add much to
this series? the sum of the whole series.
(a) Evaluate, correct to 4 decimal places, the sum of
(i) 10 terms
(ii) 20 terms.
(b) Estimate its limiting sum.
Chapter 7 Series 287

Solution
1
(a) a = 2, r =
2
(i) n = 10
a (1 r n )
Sn =
1r
1 10
2e1 c m o
2
=
1
1
2
1
2 c 1 10 m
2
=
1
2
= 3.9961
(ii) n = 20
a (1 r n )
Sn =
1r
1 20
2e1 c m o
2
=
1
1
2
1
2 c 1 20 m
2
=
1
2
= 4.0000
(b) The limiting sum is 4.

Can you see why the series 2 + 4 + 8 + 16 + diverges and the series
1 1
2 + 1 + + + converges?
2 4
The difference is in the common ratio. Only geometric series with
common ratios | r | < 1 will converge and have a limiting sum.

a
S =
1r
| r | < 1 is the necessary condition for the limiting sum to exist.
288 Maths In Focus Mathematics HSC Course

Proof
a (1 r n )
Sn =
1r
For | r | < 1, as n increases, r n decreases and approaches zero
1
e.g. When r = :
2
1 10 1 1 20 1
c m = and c m =
2 1024 2 1 048 576
a (1 0)
` S =
1r
a
=
1r

EXAMPLES

1 1
1. Find the limiting sum of 2 + 1 + + +
2 4
Solution
In the previous example, we guessed that the limiting sum was 4. Here we
will use the formula to nd the limiting sum.
1
a = 2, r =
2
a
S =
1r
2
=
1
1
2
2
=
1
2
2
=2
1
= 4

2
2. Find the sum of the series 6 + 2 + +
3
Solution
2
2 3 1
a = 6, r = = =
6 2 3
Chapter 7 Series 289

a
S =
1r
6
=
1
1
3
6
=
2
3
3
=6
2
=9

3. Which of the following series have a limiting sum?


3 15 11
(a) + +1 +
4 16 64
(b) 100 + 50 + 25 +
1 11
(c) 3 + 2 +1 +
4 16
Solution
15 11
1
16 64
(a) r = =
3 15
4 16
1
=1
4
>1
So this series does not have a limiting sum.
50 25
(b) r = =
100 50
1
=
2
<1
So this series does have a limiting sum.
1 11
2 1
4 16
(c) r= =
3 1
2
4
3
=
4
<1
So this series does have a limiting sum.

CONTINUED
290 Maths In Focus Mathematics HSC Course

n
4. Evaluate / 2 c 23 m
n =2

Solution
n
2 2 2 3 2 4
/ 2 c 23 m= 2c m + 2c m + 2c m +
3 3 3
n =2
4 8 16
= 2c m + 2c m + 2c m +
9 27 81
8 16 32
= + + +
9 27 81
8 2
a= ,r=
9 3
a
S =
1r
8
9
=
2
1
3
8
9
=
1
3
8 3
=
9 1
2
=2
3

7.8 Exercises
1. Which of the following series 2. Find the limiting sum of each
have a limiting sum? Find its series.
limiting sum where it exists. (a) 40 + 20 + 10 +
(a) 9 + 3 + 1 + (b) 320 + 80 + 20 +
1 1 (c) 100 50 + 25
(b) + + 1 +
4 2 1
(d) 6 + 3 + 1 +
(c) 16 4 + 1 2
2 7 49 2 6 18
(d) + + + (e) + + +
3 9 54 5 35 245
2 4 (f) 72 24 + 8
(e) 1 + + + 1
3 9 (g) 12 + 2 +
5 1 1 3
(f) + + + 3 1 1
8 8 40 (h) +
(g) 6 + 36 216 + 4 2 3
3
1 7 27 (i) 12 + 9 + 6 +
(h) 2 + 1 1 + 4
4 8 48
2 5 25
1 1 1 (j) + +
(i) + + + 3 12 96
9 6 4
4 8
(j) 2 +
5 25
Chapter 7 Series 291

3. Find the difference between the 8. A geometric series has limiting


limiting sum and the sum of sum 40 and its rst term is 5.
6 terms of Find the common ratio of the
(a) 56 28 + 14 series.
(b) 72 + 24 + 8 +
1 1 9. A geometric series has limiting
(c) 1 + + + 2
5 25 sum 6 and a rst term of 8.
1 1 1 5
(d) + + + Find its common ratio.
2 4 8
1 15 45
(e) 1 + + + 10. The limiting sum of a geometric
4 16 64
3
4. Evaluate series is and its rst term
10
r1 1
(a) / c 15 m is . Find the common ratio of
2
r =1
n
the series.
(b) / c 27 m
n =1
11. The second term of a geometric

1 k
(c) / c m series is 2 and its limiting sum
3 4 is 9. Find the values of rst term
n1
(d) / c 35 m a and common ratio r.
n =1
p2 12. A geometric series has 3rd term
(e) / c 23 m 12 and 4th term3. Find a, r and
p =2
n the limiting sum.
(f) / 5 c 12 m
n =1

13. A geometric series has 2nd term
1 n
(g) / 2 c m 2
and 4th term
8
. Find a, r and
1 6 3 27
n1 its limiting sum.
(h) / 4 c 23 m
n =3
n2 14. The 3rd term of a geometric series
(i) / 2 c 19 m is 54 and the 6th term is 11
83
.
2
125

1 2 n Evaluate a, r and the limiting
(j) / c m
2 5
n =0 sum.
5. A geometric series has limiting
1 15. The 2nd term of a geometric
sum 6 and common ratio . 4
3 series is and the 5th term is
Evaluate the rst term of the 15
32
series. . Find the values of a and r
405
6. A geometric series has a limiting and its limiting sum.
sum of 5 and rst term 3. Find 16. The limiting sum of a geometric
the common ratio. series is 5 and the 2nd term is
1
7. The limiting sum of a geometric 1 . Find the rst term and the
5
1 common ratio.
series is 9 and the common
3
x x
2 17. The series x + + + has
ratio is . Find the rst term of 4 16
5
7
the series. a limiting sum of . Evaluate x.
8
292 Maths In Focus Mathematics HSC Course

18. (a) For what values of k does the (c) Evaluate p if the limiting sum
limiting sum exist for the series 7
of the series is .
k + k2 + k3 + ? 8

(b) Find the limiting sum of the 20. Show that in any geometric
2 series the difference between
series when k = .
3 the limiting sum and the sum of
(c) Evaluate k if the limiting sum ar n
n terms is .
of the series is 3. 1r

19. (a) For what values of p will the


limiting sum exist for the series
1 2p + 4p2 ?
(b) Find the limiting sum when
1
p= .
5

Applications of Series
General applications

Arithmetic and geometric series are useful in solving real life problems
involving patterns. It is important to choose the correct type of series and
know whether you are asked to nd a term or a sum of terms.

EXAMPLES

1. A stack of cans on a display at a supermarket has 5 cans on the top


row. The next row down has 2 more cans and the next one has 2 more
cans and so on.
(a) Calculate the number of cans in the 11th row down.
(b) If there are 320 cans in the display altogether, how many rows are
there?

Solution
The rst row has 5 cans, the 2nd row has 7 cans, the 3rd row 9 cans and
so on. This forms an arithmetic series with a = 5 and d = 2.
(a) For the 11th row, we want n = 11.
Tn = a + (n 1) d
T11 = 5 + (11 1) 2
= 5 + 10 2
= 25
So there are 25 cans in the 11th row.
Chapter 7 Series 293

(b) If there are 320 cans altogether, this is the sum of cans in all rows.
So Sn = 320
n
Sn = [2a + (n 1) d ]
2
n
320 = [2 5 + (n 1) 2]
2
n
= [10 + 2n 2]
2
n
= [2 n + 8 ]
2
= n 2 + 4n
0 = n2 + 4n 320
= (n 16) (n + 20)
n 16 = 0, n + 20 = 0
n = 16, n = 20
Since n must be a positive integer, then n = 16.
There are 16 rows of cans.

2. A layer of tinting for a car window lets in 95% of light.


(a) What percentage of light is let in by
(i) 2 layers
(ii) 3 layers
(iii) 10 layers
of tinting?
(b) How many layers will let in 40% of light?

Solution
(a) (i) 1 layer lets in 95% of light.
So 2 layers let in 95% 95% of light.
95% 95% = 0.95 0.95 To convert a percentage to
= 0.9025 a decimal, divide by 100
and to convert a decimal
= 90.25% to a percentage, multiply
by 100.
So 2 layers let in 90.25% of light.
(ii) 1 layer lets in 95% or 0.95 of light.
2 layers let in 0.95 0.95 or 0.952 of light.
3 layers let in 0.952 0.95 or 0.953 of light.
0.953 = 0.857
= 85.7%
So 3 layers let in 85.7% of light.
CONTINUED
294 Maths In Focus Mathematics HSC Course

(iii) The number of layers forms the geometric series


0.95 + 0.952 + 0.953 + with a = 0.95, r = 0.95.
For 10 layers, n = 10
Tn = ar n 1
T10 = 0.95 (0.95) 101
= 0.95 (0.95) 9
= 0.9510
= 0.5987
= 59.87%
So 10 layers let in 59.87% of light.
(b) We want to nd n when the nth term is 40% or 0.4.
Tn = ar n 1
0.4 = 0.95 (0.95)n 1
= 0.95n
log 0.4 = log 0.95n
= n log 0.95
log 0.4
=n
log 0.95
17.9 = n
So around 18 layers of tinting will let in 40% of light.

The limiting sum can also be used to solve various problems.

EXAMPLES

0.5 is called a recurring

decimal.
1. Write 0.5 as a fraction.

Solution

0. 5 = 0.5555555
5 5 5
= + + +
10 100 1000
5 1 1
This is a geometric series with a = = and r = .
a 10 2 10
S =
1r
1
2
=
1
1
10
1
2
=
9
10
Chapter 7 Series 295

1 10
=
2 9
5
=
9

1
2. A ball is dropped from a height of 1 metre and bounces up to of
1 3
its height. It continues bouncing, rising of its height on each bounce
3
until it eventually reaches the ground. What is the total distance through
which it travels?

Solution

1m

1 1
m m
3 3 1 1
m m
9 9

Notice that there is a series for the ball coming downwards and another
series upwards.
There is more than one way of calculating the total distance. Here is one Can you nd the total
way of solving it. distance a different way?

1 1 1 1 1 1
Total distance = 1 + + + + + + +f
3 3 9 9 27 27
1 1 1
= 1 + 2c + + +f m
3 9 27
1 1 1 1 1
+ + + is a geometric series with a = and r =
3 9 27 3 3
a
S =
1r
1
3
=
1
1
3
1
3
=
2
3
1 3
=
3 2
1
=
2
1
Total distance = 1 + 2 c m
2
=1+1
=2
So the ball travels 2 metres altogether.
296 Maths In Focus Mathematics HSC Course

Investigation

1. In the second example above, in theory will the ball ever stop?

2. Kim owes $1000 on her credit card. If she pays back 10% of the amount
owing each month, she will never nish paying it off. Is this true or false?

7.9 Exercises
1. A market gardener plants daffodil 3. A set of books is stacked in
bulbs in rows, starting with a row layers, where each layer contains
of 45 bulbs. Each successive row three books fewer than the layer
has 5 more bulbs. below. There are 6 books in the
(a) Calculate the number of bulbs top layer, 9 in the next layer
in the 34th row. and so on. There are n layers
(b) Which row would be the altogether.
rst to have more than 100 bulbs (a) Write down the number of
in it? books in the bottom layer.
(c) The market gardener plants 3
(b) Show that there are n (n + 3)
10 545 bulbs. How many rows are 2
there? books in the stack altogether.

4. A painting appreciates (increases


its value) by 16% p.a. It is
currently worth $20 000.
(a) How much will it be worth in
(i) 1 year?
(ii) 2 years?
(iii) 3 years?
(b) How much will it be worth in
11 years?
(c) How long will it take for it to
be worth $50 000?

5. Water evaporates from a pond at


2. A stack of logs has 1 on the top, an average rate of 7% each week.
then 3 on the next row down, (a) What percentage of water is
and each successive row has left in the pond after
2 more logs than the one on top (i) 1 week?
of it. (ii) 2 weeks?
(a) How many logs are in the (iii) 3 weeks?
20th row? (b) What percentage is left after
(b) Which row has 57 logs? 15 weeks?
(c) If there are 1024 logs (c) If there was no rain,
altogether, how many rows are in approximately how long would
the stack? it take for the pond to only have
25% of its water left?
Chapter 7 Series 297

6. A timber fence is to be built on (a) How far does the child run to
sloping land, with the shortest get to the kth apple?
piece of timber 1.2 m and the (b) How far does the child run
longest 1.8 m. There are 61 pieces to fetch all k apples, including
of timber in the fence. return trips to P?
(c) The child runs 270 m to fetch
all apples and return them to the
bucket. How many apples are
there?

9. The price of shares in a particular


(a) What is the difference in
company is falling by an average
height between each piece of
of 2% each day.
timber?
(a) What percentage of their
(b) What length of timber is
value are they after 2 days?
needed for the fence altogether?
(b) How many days will it take
7. A logo is made with vertical for the shares to halve in value?
lines equally spaced as shown. (c) After how many days will
The shortest line is 25 mm, the the shares be worth 25% of their
longest is 217 mm and the sum value?
of the lengths of all the lines is
10. A southern brown bandicoot
5929 mm.
population in WA is decreasing
by 5% each year.
(a) What percentage of the
217 mm population is left after 5 years?
(b) After how many years will the
25 mm population be only 50%?
(c) How many years will it take
(a) How many lines are in the for the population to decrease by
logo? 80%?
(b) Find the difference in length
between adjacent lines.

8. In a game, a child starts at point


P and runs and picks up an
apple 3 m away. She then runs
back to P and puts the apple in
a bucket. The child then runs to
get the next apple 6 m away, and
runs back to P to place it in the
bucket. This continues until she
has all the apples in the bucket.
298 Maths In Focus Mathematics HSC Course

11. Write each recurring decimal as a 15. An object rolls 0.5 m in the rst
fraction. second. Then each second

(a) 0.4 5

after, it rolls by of its previous
(b) 0.7 6
roll. Find how far it will roll
(c) 1.2
altogether.
(d) 0.25

(e) 2.8 1 2
16. A lamb grows by of its previous
(f) 0.23
5
growth each month. If a lamb is
(g) 1.47
45 cm tall,
(h) 1.015
(a) how tall will it be after
(i) 0.132
6 months?
(j) 2.36 1
(b) what will its nal height be?
12. A frog jumps 0.5 metres. It
then jumps 0.1 m and on each
subsequent jump, travels 0.2 m
of the previous distance. Find the
total distance through which the
frog jumps.

13. A tree 3 m high grows by another


2
2 m after 5 years, then by
5
23
another 1 m after 5 more years
25
4 2
and so on, each year growing 17. A 100 m cliff erodes by of its
5 7
more than the previous years height each year.
growth. Find the ultimate height (a) What will the height of the
of the tree. cliff be after 10 years?
(b) After how many years will the
cliff be less than 50 m high?

18. An elastic string drops down


60 cm and then bounces back
2
to of its initial height. It keeps
3
bouncing, each time rising back
2
to of its previous height. What
3
is the total distance through
which the string travels?

19. Mary bounces a ball, dropping


it from 1.5 m on its rst bounce.
4 2
14. An 8 cm seedling grows by 4 It then rises up to of its
5 5
cm in the rst week, and then height on each bounce. Find the
3
keeps growing by of its previous distance through which the ball
5
weeks growth. How tall will it travels.
grow?
Chapter 7 Series 299

20. A roadside wall has a zig zag pattern on it as shown.

The two longest lines are each 2 m long, then the next two lines are
3 7
1 m long and each subsequent pair of lines are of the length of the
4 8
previous lines. Find the total distance of the lines.

21. Kate receives an email chain (b) how many people would
letter that she is asked to send receive the email when it is
to 8 friends. If Kate forwards this sent for the 9th time?
email on to 8 friends and each of (c) how many people would
them sends it on to 8 friends, and have received the email
so on, altogether if it is sent 9 times?
(a) describe the number of
people receiving the email as a
sequence (including Kates email)

Compound interest

Compound interest refers to when the interest earned on an investment is


then added to the amount of the investment. This means that each amount of
interest earned is calculated on the previous interest as well as the investment.
Money that earns compound interest grows according to a geometric
series.

EXAMPLE
You may remember that
p.a. means per annum or
An amount of $2000 is invested at the rate of 6% p.a. Find the amount in annually.

the bank at the end of n years.

Solution
After 1 year: The amount in the bank is the original amount plus the
interest of 6% earned on the amount.
Amount 1 = $2000 + 6% of $2000
= $2000 (1 + 6%)
= $2000 (1 + 0.06)
= $2000 (1.06)

CONTINUED
300 Maths In Focus Mathematics HSC Course

After 2 years: The amount in the bank is the previous amount plus the
interest of 6% earned on this amount.

Factorise using a common


Amount 2 = Amount 1 + 6% of amount 1
factor of $2000(1.06). = $2000 (1.06) + 6% of $2000 (1.06)
= $2000 (1.06) (1 + 6%)
= $2000 (1.06) (1 + 0.06)
= $2000 (1.06) (1.06)
= $2000 (1.06)2
Similarly after 3 years the amount is $2000(1.06)3 and so on.
The amount after n years will be $2000 (1.06)n .

In general the compound interest formula is

A = P (1 + r)n where
P = principal (initial amount)
r = interest rate as a decimal
n = number of time periods

Proof
Let $P be invested at the rate of r p.a. compound interest for n years where r is
a decimal.
After 1 year:
Amount = P + r of P
= P ( 1 + r)
Factorise using a common
After 2 years:
factor of P (1 + r). Amount = P (1 + r) + r of P (1 + r)
= P ( 1 + r ) ( 1 + r)
= P ( 1 + r) 2
After 3 years:
Amount = P (1 + r)2 + r of P (1 + r)2
= P (1 + r) 2 ( 1 + r)
= P (1 + r) 3
Following this pattern, the amount after n years will be P (1 + r)n .
Chapter 7 Series 301

EXAMPLES

1. Find the amount that will be in the bank after 6 years if $2000 is
invested at 12% p.a. with interest paid
(a) yearly
(b) quarterly
(c) monthly.

Solution
P = 2000
(a) r = 12%
= 0.12
n=6
A = P (1 + r)n
= 2000 (1 + 0.12)6
= 2000 (1.12)6
= 3947.65 Always round money off
to 2 decimal places (to the
So the amount is $3947.65. nearest cent).

(b) For quarterly interest, it is divided into 4 amounts each year.


r = 0.12 p.a.
= 0.12 4 quarterly
= 0.03
Also the interest is paid in 4 times a year.
n=64
= 24 quarters
A = P (1 + r) n
= 2000 (1 + 0.03)24
= 2000 (1.03)24
= 4065.59
So the amount is $4065.59.

(c) For monthly interest, it is divided into 12 amounts each year.


r = 0.12 p.a.
= 0.12 12 monthly
= 0.01
Also the interest is paid in 12 times a year.
n = 6 12
= 72 months
A = P (1 + r) n
= 2000 (1 + 0.01)72
= 2000 (1.01)72
= 4094.20
So the amount is $4049.20.

CONTINUED
302 Maths In Focus Mathematics HSC Course

2. Geoff wants to invest enough money to pay for a $10 000 holiday in
5 years time. If interest is 8% p.a., how much does Geoff need to invest
now?

Solution
A = 10 000, r = 8% = 0.08 and n = 5
We want to nd the principal P.
A = P (1 + r)n
10 000 = P (1 + 0.08)5
= P (1.08)5
10 000
=P
1.085
6805.83 = P
So Geoff will need to invest $6805.83 now.

3. An amount of $1800 was invested at 6% p.a. and is now worth


$2722.66. For how many years was the money invested if interest is paid
twice a year?

Solution
P = 1800, A = 2722.66
Interest is paid twice a year.
r = 6% = 0.06 p.a.
= 0.06 2
= 0.03 twice a year
We want to nd n
A = P (1 + r) n
2722.66 = 1800 (1 + 0.03)n
= 1800 (1.03)n
2722.66
= 1.03n
1800
1.5126 = 1.03n
log 1.5126 = log 1.03n
= n log 1.03
log 1.5126
=n
log 1.03
14 = n
Since interest is paid in twice a year, the number of years will be 14 2.
So the money was invested for 7 years.
Chapter 7 Series 303

7.10 Exercises
1. Find the amount of money in the 8. Find the amount of money there
bank after 10 years if will be after 15 years if $6000
(a) $500 is invested at 4% p.a. earns 9% p.a. interest, paid
(b) $7500 is invested at 7% p.a. quarterly.
(c) $8000 is invested at 8% p.a.
9. How much money will be in
(d) $5000 is invested at 6.5% p.a.
a bank account after 5 years if
(e) $2500 is invested at 7.8% p.a.
$500 earns 6.5% p.a. with interest
2. Sam banks $1500 where it earns paid monthly?
interest at the rate of 6% p.a.
10. Find the amount of interest
Find the amount after 5 years if
earned over 4 years if $1400 earns
interest is paid
6% p.a. paid quarterly.
(a) annually
(b) twice a year 11. How much money will be in a
(c) quarterly. credit union account after 8 years
if $8000 earns 7.5% p.a. interest
3. Chad banks $3000 in an account
paid monthly?
that earns 5% p.a. Find the
amount in the bank after 10 years 12. Elva wins a lottery and invests
if interest is paid $500 000 in an account that
(a) quarterly earns 8% p.a. with interest paid
(b) monthly. monthly. How much will be in
the account after 12 years?
4. I put $350 in the bank where it
earns interest of 8% p.a. with 13. Calculate the amount deposited
interest paid annually. Find the 4 years ago at 5% p.a. if the
amount there will be in the current amount in the account is
account after 2 years. (a) $5000
(b) $675
5. How much money will there be
(c) $12 000
in an investment account after
(d) $289.50
3 years if interest of 4.5% p.a. is
(e) $12 800.
paid twice a year on $850?
14. How much was banked 3 years
6. Find the amount of money there
ago if the amount in the bank
will be in a bank after 8 years if
now is $5400 and interest is
$1000 earns interest of 7% p.a.
5.8% p.a. paid quarterly?
with interest paid twice a year.
15. How many years ago was an
7. Abdul left $2500 in a building
investment made if $5000 was
society account for 4 years, with
invested at 6% p.a. paid monthly
interest of 5.5% p.a. paid yearly.
and is now worth $6352.45?
How much money did he have
in the account at the end of that
time?
304 Maths In Focus Mathematics HSC Course

16. Find the number of years that 19. Kate has $4000 in a bank account
$10 000 was invested at 8% p.a. that pays 5% p.a. with interest
with interest paid twice a year if paid annually and Rachel has
there is now $18 729.81 in the $4000 in a different account
bank. paying 4% quarterly. Which
person will receive the most
17. Jude invested $4500 ve years
interest over 5 years and by how
ago at x% p.a. Evaluate x if Jude
much?
now has an amount of
(a) $6311.48 20. A bank offers investment account
(b) $5743.27 A at 8% p.a. with interest
(c) $6611.98 paid twice a year and account
(d) $6165.39 B with interest paid at 6% p.a.
(e) $6766.46 in the bank. at monthly intervals. If Georgie
invests $5000 over 6 years, which
18. Hamish is given the choice of a
account pays the most interest?
bank account in which interest
How much more does it pay?
is paid annually or quarterly.
If he deposits $1200, nd the
difference in the amount of
interest paid over 3 years if
interest is 7% p.a.

Annuities

An annuity is a fund where a certain amount of money is invested regularly


(often annually, which is where the name comes from) for a number of years.
Scholarship and trust funds, certain types of insurance and superannuation are
examples of annuities.
While superannuation and other investments are affected by changes in
the economy, due to their long-term nature, the amount of interest they earn
balances out over the years. In this course, we average out these changes, and
use a constant amount of interest annually.

EXAMPLES

1. A sum of $1500 is invested at the beginning of each year in a


superannuation fund. If interest is paid at 6% p.a., how much money will
be available at the end of 25 years?

Solution
It is easier to keep track of each annual amount separately.
The rst amount earns interest for 25 years, the 2nd amount earns
interest for 24 years, the 3rd amount for 23 years and so on.
Chapter 7 Series 305

The 25th amount earns interest for 1 year.


P = 1500 and r = 6% = 0.06
A = P ( 1 + r) n
= 1500 (1 + 0.06)n
= 1500 (1.06)n
A1 = 1500 (1.06)25
A2 = 1500 (1.06)24
A3 = 1500 (1.06)23
.
Since the last amount is
.
deposited at the beginning
. of the year, it earns interest
for 1 year.
A25 = 1500 (1.06)1
Total superannuation amount
T = A1 + A2 + A3 + + A25
= 1500 (1.06)25 + 1500 (1.06)24 + 1500 (1.06)23 + + 1500 (1.06)1
= 1500 (1.0625 + 1.0624 + 1.0623 + + 1.061)
= 1500 (1.06 + 1.062 + 1.063 + + 1.0625)

1.06 + 1.062 + 1.063 + . . . + 1.0625 is a geometric series with


a = 1.06, r = 1.06 and n = 25
a (r n 1 )
Sn =
r1
1.06 (1.0625 1)
S25 =
1.06 1
= 58.16
T = 1500 58.16
= 87 234.57
So the total amount of superannuation after 25 years is $87 234.57.

2. An amount of $50 is put into an investment account at the end of


each month. If interest is paid at 12% p.a. paid monthly, how much is in
the account at the end of 10 years?

Solution
P = 50
r = 12%
= 0.12
The monthly interest rate is
0.12 12 = 0.01.
Monthly interest over 10 years gives
n = 10 12
= 120
CONTINUED
306 Maths In Focus Mathematics HSC Course

The rst amount earns interest for 119 months, since it is deposited at the
end of the month.
The 2nd amount earns interest for 118 months, the 3rd amount for
117 months and so on.
The 120th amount earns no interest as it goes in at the end of the last
month.

A = P (1 + r)n
= 50 (1 + 0.01)n
= 50 (1.01)n
A1 = 50 (1.01)119
A2 = 50 (1.01)118
A3 = 50 (1.01)117
.
.
.
A120 = 50 (1.01) 0
Total amount
T = A1 + A2 + A3 + + A120
= 50 (1.01)119 + 50 (1.01)118 + 50 (1.01)117 + + 50 (1.01)0
= 50 (1.01119 + 1.01118 + 1.01117 + + 1.010 )
= 50 (1.010 + 1.011 + 1.012 + + 1.01119 )
= 50 (1 + 1.011 + 1.012 + + 1.01119 )
1 + 1.011 + 1.012 + . . . + 1.01119 is a geometric series with a = 1, r = 1.01 and
n = 120
a (r n 1 )
Sn =
r1
1 (1.01120 1)
S120 =
1.01 1
= 230.04
T = 50 230.04
= 11 501.93
So the total amount after 10 years is $11 501.93.

7.11 Exercises
1. Find the amount of 2. Michael works for a company
superannuation available at the that puts aside $1200 at the
end of 20 years if $500 is invested beginning of each year for his
at the beginning of each year and superannuation. If the money
earns 9% p.a. earns interest at the rate of
5% p.a., nd the amount of
superannuation that Michael will
have at the end of 30 years.
Chapter 7 Series 307

3. How much superannuation will 11. Lliam wants to save up $15 000
there be at the end of 20 years if for a car in 5 years time. He
$800 is invested at 10% p.a. at invests $2000 at the end of each
the beginning of each year? year in an account that pays
7.5% p.a. interest. How much
4. Rachel starts working for a
more will Lliam have to pay at
business at the beginning of
the end of 5 years to make up the
2005. If she retires at the end of
$15 000?
2034, how much superannuation
will she have if $1000 is invested 12. A school invests $5000 at the end
at the beginning of each year at of each year at 6% p.a. towards a
9.5% p.a.? new library. How much will the
school have after 10 years?
5. A sum of $1500 is invested at
the end of each year for 15 years 13. Jacques puts aside $500 at the
at 8% p.a. Find the amount of end of each year for 5 years. If the
superannuation available at the money is invested at 6.5% p.a.,
end of the 15 years. how much will Jacques have at
the end of the 5 years?
6. How much superannuation will
there be at the end of 18 years if 14. An employee contributes $200
$690 is invested at 8.5% p.a. at into a superannuation fund at the
the beginning of each year? end of each year. If the interest
rate on this fund is 11.5% p.a.,
7. If Phan pays $750 into a
how much will the employee
superannuation fund at the
have at the end of 20 years?
beginning of each year, how much
will she have at the end of 29 years 15. Mohammeds mother invests
if the interest is 6.8% p.a.? $200 for him each birthday up to
and including his 18th birthday.
8. A sum of $1000 is invested at
The money earns 6% p.a. How
the end of each year for 22 years,
much money will Mohammed
at 9% p.a. Find the amount of
have on his 18th birthday?
superannuation available at the
end of the 22 years. 16. Xuan is saving up for a holiday.
She invests $800 at the end of
9. Matthew starts work at the
each year at 7.5% p.a. How much
beginning of 2010. If he retires
will she have for her holiday after
at the end of 2037, how much
5 years time?
superannuation will he have if he
invests $700 at the beginning of 17. A couple saves $3000 at the end
each year at 12.5% p.a.? of each year towards a deposit
on a house. If the interest rate
10. An apprentice starts work for
is 5% p.a., how much will the
a small business. If she invests
couple have saved after 6 years?
$400 at the beginning of each
year, how much superannuation
will she have at the end of
25 years if the money earns
15.5% p.a.?
308 Maths In Focus Mathematics HSC Course

18. Lucia saves up $2000 each year have enough? If so, how much
and at the end of the year she over will it be? If she doesnt earn
invests it at 6% p.a. enough, how much will she need
(a) She does this for 10 years. to add to this money to make it
What is her investment worth? up to the $10 000?
(b) Lucia continues investing
23. Farmer Brown puts aside part
$2000 a year for 5 more years.
of the farms earnings at the
What is the future value of her
beginning of each month to buy
investment?
a new truck in 10 years time
19. Jodie starts work in 2012 and puts when the old one wears out.
$1000 in a superannuation fund He invests $400 each month at
at the end of the year. She keeps 9% p.a. and estimates the cost
putting in this same amount of a new truck at $80 000. Will
at the end of every year until the investment earn enough to
she retires at the end of 2029. buy the new truck? What is the
If interest is paid at 10% p.a., difference?
calculate how much Jodie will
24. Marcus and Rachel want to save
have when she retires.
up $25 000 for a deposit on
20. Michelle invests $1000 at the end an apartment in 6 years time.
of each year. The interest rate is They aim to pay around half the
8% p.a. deposit each. Marcus invests an
(a) How much will her inheritance of $9000 in a bank
investment be worth after account where it earns 8% p.a.
6 years? Rachel invests $100 at the
(b) How much more would beginning of each month where
Michelles investment be worth it earns interest of 9% p.a.
after 6 years if she had invested (a) What is the future value
$1200 each year? of Marcuss investment after
6 years?
21. Jack cannot decide whether to
(b) How much will Rachels
invest $1000 at the end of each
investment be worth after
year for 15 years or $500 for
6 years?
30 years in a superannuation
(c) Will they be able to pay the
fund. If the interest rate is
deposit in 6 years time?
5% p.a., which would be the
better investment for Jack? 25. Jenny puts aside $20 at the end
of each month for 3 years. How
22. Kate is saving up to go overseas in
much will she have then if the
8 years time. She invests $1000
investment earns 8.2% p.a., paid
at the end of each year at 7% p.a.
monthly?
and estimates that the trip will
cost her around $10 000. Will she
Chapter 7 Series 309

Loan repayments

The formula for compound interest and the geometric series can help with
working out regular loan repayments.

EXAMPLES

1. A sum of $20 000 is borrowed at 12% p.a. and paid back at regular
monthly intervals over 4 years. Find the amount of each payment.

Solution
Let M stand for the monthly repayment.
The number of payments will be 4 12 or 48.
Monthly interest will be 12% 12 or 1% (0.01).
Each month, interest for that month is added to the loan and the
repayment amount is taken off.
The interest added to the rst month will be 20 000 (1 + 0.01)1
or 20 000 (1.01) 1.
The interest added to other months will be A (1 + 0.01)1 or A (1.01)1. The 20 000(1.01)1 is
The amount owing for the rst month: the interest added for the
month and the M is the
A1 = 20 000 (1 + 0.01)1 M repayment subtracted from
= 20 000 (1.01)1 M the balance.

The amount owing for the 2nd month is what was owing from the 1st
month, together with that months interest, minus the repayment.
A2 = A1 (1 + 0.01)1 M
= A1 (1.01)1 M
= 7 20 000 (1.01)1 M A (1.01)1 M (substituting in A1)
= 20 000 (1.01)2 M (1.01)1 M
= 20 000 (1.01)2 M (1.011 + 1)
Similarly,
A3 = A2 (1 + 0.01)1 M
= 7 20 000 (1.01)2 M (1.011 + 1) A (1.01)1 M
= 20 000 (1.01)3 M (1.011 + 1) (1.011 ) M
= 20 000 (1.01)3 M (1.012 + 1.011 ) M
= 20 000 (1.01) 3 M (1.012 + 1.011 + 1)

CONTINUED
310 Maths In Focus Mathematics HSC Course

Continuing this pattern, after 4 years (48 months) the amount owing will be
A48 = 20 000 (1.01)48 M (1.0147 + 1.0146 + f + 1.012 + 1.011 + 1).
But the loan is paid out after 48 months.
So A 48 = 0

0 = 20 000 (1.01)48 M (1.0147 + 1.0146 + f + 1.012 + 1.011 + 1)

M (1.0147 + 1.0146 + f + 1.012 + 1.011 + 1) = 20 000 (1.01) 48


20 000 (1.01) 48
M=
1.0147 + 1.0146 + f + 1.012 + 1.011 + 1
1.0147 + 1.0146 + f + 1.012 + 1.011 + 1
= 1 + 1.011 + 1.012 + f + 1.0146 + 1.0147
This is a geometric series with a = 1, r = 1.01 and n = 48.
a (r n 1 )
Sn =
r1
1 (1.0148 1)
S48 =
1.01 1
= 61.223
20 000 (1.01) 48
` M=
61.223
= 526.68
So the monthly repayment is $526.68.

2. A store charges 10% p.a. for loans and repayments do not have to be
made until the 4th month. Ivan buys $8000 worth of furniture and pays
it off over 3 years.
(a) How much does Ivan owe after 3 months?
(b) What are his monthly repayments?
(c) How much does Ivan pay altogether?

Solution
Let P stand for the payments each month.
Number of payments = 3 12 3 (3 months of no repayments)
= 33 months
Monthly interest rate
r = 10% 12
= 0.1 12

= 0.0083
(a) After 3 months, the amount owing is
A = P (1 + r ) n

= 8000 (1 + 0.0083) 3

= 8000 (1.0083) 3
= 8201.67
So the amount owing after 3 months is $8201.67.
Chapter 7 Series 311

(b) The rst repayment is in the 4th month.


1
A1 = 8000 (1.0083)
2
A2 = 8000 (1.0083)
3
A3 = 8000 (1.0083) The rst repayment
4 is in the 4th month.
A4 = 8000 (1.0083) P
4 1
A5 = 8 8000 (1.0083) P B (1.0083) P
5
= 8000 (1.0083) P (1.0083)1 P
5
= 8000 (1.0083) P (1.00831 + 1)
5 1
A6 = 9 8000 (1.0083) P (1.00831 + 1) C (1.0083) P
6
= 8000 (1.0083) P (1.00831 + 1) (1.0083)1 P
6
= 8000 (1.0083) P (1.00832 + 1.00831) P
6
= 8000 (1.0083) P (1.00832 + 1.00831 + 1)
Continuing this pattern,
36
A36 = 8000 (1.0083) P (1.008332 + 1.008331 + . . . + 1.00831 + 1)
The loan is paid out after 36 months.
So A 36 = 0

0 = 8000 (1.0083)36 P (1.008332 + 1.008331 + . . . + 1.00831 + 1)

P (1.008 332 + 1.008 331 + . . . + 1.008 31 + 1) = 8000 (1.008 3)36

8000 (1.0083) 36
P=
1.008332 + 1.008331 + . . . + 1.00832 + 1.00831 + 1

1.008332 + 1.008331 + . . . + 1.00831 + 1

= 1 + 1.00831 + . . . + 1.008331 + 1.008332

This is a geometric series with a = 1, r = 1.0083 and n = 33.
a (r n 1)
Sn =
r1

1 (1.008333 1)
S33 =
1.0083 1
= 37.804

8000 (1.0083) 36
` P=
37.804
= 285.30
So the monthly repayment is $285.30. If you round off the
repayment, your answer will
(c) Ivan pays $285.30 33 or $9414.94 altogether. be slightly different.
312 Maths In Focus Mathematics HSC Course

7.12 Exercises
1. An amount of $3000 is borrowed 7. A $2000 loan is offered at
at 22% p.a. and repayments made 18% p.a. with interest charged
yearly for 5 years. How much will monthly, over 3 years.
each repayment be? (a) If no repayment need be paid
for the rst 2 months, nd the
2. The sum of $20 000 is borrowed
amount of each repayment.
at 18% p.a. interest over 8 years.
(b) How much will be paid back
How much will the repayments
altogether?
be if they are made monthly?
8. Bill thinks he can afford a
3. David borrows $5000 from the
mortgage payment of $800
bank and pays back the loan
each month. How much can he
in monthly instalments over
borrow, to the nearest $100, over
4 years. If the loan incurs interest
25 years at 11.5% p.a.?
of 15% p.a., nd the amount of
each instalment. 9. Get Rich Bank offers a mortgage
1
4. Mr and Mrs Nguyen mortgage at 7 % p.a. over 10 years and
2
their house for $150 000. Capital Bank offers a mortgage at
(a) Find the amount of the 1
5 % p.a. over 25 years.
monthly repayments they will 2
have to make if the mortgage (a) Find the amount of the
is over 25 years with interest at monthly repayments for each
6% p.a. bank on a loan of $80 000.
(b) If the Nguyens want to pay (b) Find the difference in the
their mortgage out after 15 years, total amount paid on each
what monthly repayments would mortgage.
they need to make?
10. Majed buys a $35 000 car. He
5. A loan of $6000 is paid back puts down a 5% deposit and pays
in equal annual instalments the balance back in monthly
over 3 years. If the interest is instalments over 4 years at
12.5% p.a., nd the amount of 12% p.a. Find the total amount
each annual instalment. that Majed pays for the car.
6. The Smith family buys a car for 11. Amy borrowed money over
$38 000, paying a 10% deposit 7 years at 15.5% p.a. and she pays
and taking out a loan for the $1200 a month. How much did
balance. If the loan is over 5 years she borrow?
with interest of 1.5% monthly,
nd
(a) the amount of each monthly
loan repayment
(b) the total amount that the
Smith family paid for the car.
Chapter 7 Series 313

12. NSW Bank offers loans at 9% p.a. (a) How much does Ali owe after
with an interest-free period of 6 months?
3 months, while Sydney Bank (b) What are the monthly
offers loans at 7% p.a. Compare repayments?
these loans on an amount of (c) How much does Ali pay for
$5000 over 3 years and state the furniture altogether?
which bank offers the best loan
15. A loan of $6000 over 5 years
and why.
at 15% p.a. interest, charged
13. Danny buys a plasma TV for monthly, is paid back in 5 annual
$10 000. He pays a $1500 deposit instalments.
and borrows the balance at (a) What is the amount of each
18% p.a. over 4 years. instalment?
(a) Find the amount of each (b) How much is paid back
monthly repayment. altogether?
(b) How much did Danny pay
altogether?

14. A store offers furniture on


hire purchase at 20% p.a. over
5 years, with no repayments for
6 months. Ali buys furniture
worth $12 000.
314 Maths In Focus Mathematics HSC Course

Test Yourself 7
1. Find a formula for the nth term of each
sequence. 3 mm

(a) 9, 13, 17,


(b) 7, 0, 7,
(c) 2, 6, 18,
1
(d) 200, 50, 12 ,
2
(e) 2, 4, 8,

2. For the series 156 + 145 + 134 +


(a) Find the 15th term.
(b) Find the sum of 15 terms.
(c) Find the sum of 14 terms.
(d) Write a relationship between
T15, S15 and S14.
(e) Find the value of n for the rst
negative term.
5. Find the amount invested in a bank
3. Evaluate account at 9.5% p.a. if the balance in the
50
account is $5860.91 after 6 years.
(a) / (9r 7)
r =1
7 6. State whether each series is
1
(b) / b n l as a fraction (i) arithmetic
3
12
(ii) geometric
(c) / 5 (3n) (iii) neither.
2
100
(a) 97 + 93 + 89 +
(d) / (7n 6) 2 1 3
(b) + + +
1 3 2 8
n2
(c) 5 + 20 + 45 +
(e) / 2 c 15 m
n =3 (d) 1.6 0.4 + 0.6
4. A bamboo blind has 30 slats. When the (e) 3.4 + 7.5 + 11.6 +
blind is down, each gap between slats, (f) 48 + 24 + 12 +
and between the top and bottom of the 1
(g) + 1 5 +
5
window, is 3 mm. When the blind is up,
(h) 105 + 100 + 95 +
the slats have no gaps between them.
1 1
(a) Show that when the blind is up, the (i) 1 + 1 + 1 + ...
2 4
bottom slat rises 90 mm. (j) log10 x + log10 x2 + log10 x3 +
(b) How far does the next slat rise?
(c) Explain briey why the distances the 7. The nth term of the series 8 + 13 + 18 +
slats rise when the blind is up form an is 543. Evaluate n.
arithmetic series.
8. Amanda earned $20 000 in one year.
(d) Find the distance the 17th slat from
At the beginning of the next year she
the bottom rises.
received a salary increase of $450.
(e) What is the sum of the distances that
all slats rise?
Chapter 7 Series 315

She now receives the same increase each 18. (a) For what values of x does the
year. geometric series 1 + x + x2 + have a
(a) What will her salary be after 10 years? limiting sum?
3
(b) How much will Amanda earn (b) Find the limiting sum when x = .
5
altogether over the 10 years? (c) Evaluate x when the limiting sum
1
9. The 11th term of an arithmetic series is is 1 .
2
97 and the 6th term is 32. Find the rst
19. The rst term of an arithmetic series is
term and common difference.
4 and the sum of 10 terms is 265. Find
10. A series has nth term given by Tn = n 3 5. the common difference.
Find
20. Every week during a typing course, Tony
(a) the 4th term
improves his typing speed by 3 words
(b) the sum of 4 terms
per minute until he reaches 60 words per
(c) which term is 5827.
minute by the end of the course.
11. A series has terms 5 + x + 45 + Evaluate (a) If he can type 18 words per minute in
x if the series is the rst week of the course, how many
(a) arithmetic words per minute can he type by week 8?
(b) geometric. (b) How many weeks does the course
run for?
12. Convert each recurring decimal to a
fraction. 21. A farmer borrows $50 000 for farm

(a) 0.4 machinery at 18% p.a. over 5 years and

(b) 0.72 makes equal yearly repayments on the

(c) 1.57 loan at the end of each year.
(a) How much does he owe at the end
13. If x, 2x + 3 and 5x are the rst 3 terms of of the rst year, just before he makes the
an arithmetic series, calculate the value rst repayment?
of x. (b) How much is each yearly repayment?
14. Find the 20th term of 22. A ball drops from a height of 1.2 metres
(a) 3, 10, 17, 3
then bounces back to of this height.
(b) 101, 98, 95, 5
3
(c) 0.3, 0.6, 0.9, On the next bounce, it bounces up to
5
15. Find the limiting sum (sum to innity) of of this height and so on. Through what
81 + 27 + 9 + distance will the ball travel?

16. Karl puts $300 aside as an annuity for 23. If x + 2, 7x 2 and 15x + 6 are
his son at the beginning of each year. If consecutive terms in a geometric series,
interest is 7% p.a., how much will his evaluate x.
son receive at the end of 15 years? 24. (a) If $2000 is invested at 4.5% p.a., how
17. For each series, write an expression for much will it be worth after 4 years?
the sum of n terms. (b) If interest is paid quarterly, how
(a) 5 + 9 + 13 + much would the investment be worth
(b) 1 + 1.07 + 1.072 + after 4 years?
316 Maths In Focus Mathematics HSC Course

25. Evaluate 8 + 14 + 20 + + 122. (a) how much is each monthly


repayment?
26. (a) Calculate the sum of all the multiples (b) how much does Scott pay altogether?
of 7 between 1 and 100.
(b) Calculate the sum of all numbers 28. The sum of n terms of the series
between 1 and 100 that are not 214 + 206 + 198 + is 2760. Evaluate n.
multiples of 7.
29. Evaluate n if the nth term of the series
27. Scott borrows $200 000 to buy a house. 4 + 12 + 36 + is 354 292.
If the interest is 6% p.a. and the loan is
over 20 years,

Challenge Exercise 7
1. The nth term of a sequence is given by 6. Find the sum of all integers between
n 2 1 and 200 that are not multiples of 9.
Tn = .
n+1
7. Find the least number of terms for which
(a) What is the 9th term of the sequence? 4
1 the sum of the series 20 + 4 + + is
(b) Which term is equal to 18 ? 5
20 greater than 24.99.
3 5
2. For the sequence , , . . . evaluate 8. Find the values of n for which the
4 4
sequence given by Tn = n2 4n is
(a) the common difference
negative.
(b) the 7th term
(c) the sum of 6 terms. 9. The sum of the rst 5 terms of a
geometric sequence is 77 and the sum of
Hint: 3. Evaluate the sum of the rst 20 terms of
the next 5 terms is 2464. Find the 4th
3 = 2 + 1, 5 = 4 + 1 the series
and so on. term of the sequence.
(a) 3 + 5 + 9 + 17 + 33 + 65 +
(b) 5 2 + 10 8 + 15 32 + 10. Janes mother puts $300 into an account
at the beginning of each year to pay for
4. A factory sells shoes at $60 a pair. For
Janes education in 5 years time. If 6%
10 pairs of shoes there is a discount,
p.a. interest is paid quarterly, how much
whereby each pair costs $58. For 20 pairs,
money will Janes mother have at the
the cost is $56 a pair and so on. Find
end of the 5 years?
(a) the price of each pair of shoes on an
order of 100 pairs of shoes. 11. The geometric series 2x + 4x2 + 8x3 +
(b) the total price on an order of 60 pairs has a limiting sum of 3. Evaluate x.
of shoes.
12. Find the amount of money in a bank
5. Which term of the sequence account if $5000 earns 8.5% p.a. for
7 14 28 224 4 years, then 6.5% p.a. for 3 years, with
, , , . . . is equal to ?
9 45 225 28125 interest paid monthly for all 7 years.
Chapter 7 Series 317

13. Find the value of k for which 81 920 is (c) the amount of each instalment
the 8th term of the geometric sequence (d) the total amount of interest Kim pays.
5 + k + 80 +
15. (a) Find the limiting sum of the series
14. Kim borrows $10 000 over 3 years at a 1 + cos2 x + cos4 x + in simplest form.
rate of 1% interest compounded each (b) Why does this series have a limiting
month. If she pays off the loan in three sum?
equal annual instalments, nd
16. A superannuation fund paid 6% p.a. for
(a) the amount Kim owes after one
the rst 10 years and then 10% p.a. after
month
that. If Thanh put $5000 into this fund
(b) the amount she owes after the
at the end of each year, how much would
rst year, just before she pays the rst
she have at the end of 25 years?
instalment
8
Probability

TERMINOLOGY

Complement: The complement of an event E is when the branch with the sample space listed at the right of each
event E does not occur branch
Equally likely outcomes: Each outcome has the same Random experiments: Experiments that are made with no
chance of occurring pattern or order where each outcome is equally likely to
occur
Independent events: Events are independent if the result
of one event does not inuence the outcome of another Sample space: The set of all possible outcomes in an event
event. There is no overlap between the events or series of events
Multi-stage events: A series of successive independent Tree diagram: A diagram that uses branches to show
events multi-stage events where the probabilities on each
branch are equal
Mutually exclusive results: Two events with the same
sample space that cannot both occur at the same time Venn diagram: A special diagram to show the sample
space for non-mutually exclusive events using circles for
Non-mutually exclusive results: Two events with the same
each event drawn inside a rectangle which represents the
sample space that can occur at the same time i.e. there is
sample space
some overlap
Probability tree: A diagram that uses branches to show
multi-stage events and sets out the probability on each
Chapter 8 Probability 319

INTRODUCTION
PROBABILITY IS THE STUDY of how likely it is that something will happen.
It is used to make predictions in different areas, ranging from gambling to
determining the rate of insurance premiums. For example an actuary looking
at death-rate statistics can estimate the probable age to which someone will
live, and set life insurance premiums accordingly.
Another example of where probability is useful is in biology.
The probability of certain diseases or genetic defects can be
calculated in high-risk families.
Probability is also closely related to statistics and data analysis,
as well as games of chance such as card games, tossing coins,
backgammon and so on. It is also relevant to buying raffle and
lottery tickets.
In this chapter, you will revise simple probability that you
have learnt in previous years and extend this to more complex
probability involving multi-stage events.

DID YOU KNOW?

Girolamo Cardano (150176) was a doctor and mathematician who developed the first theory of
probability. He was a great gambler, and he wrote De Ludo Aleae (On Games of Chance). This
work was largely ignored, and it is said that the first book on probability was written by Christiaan
Huygens (162995).
The main study of probability was done by Blaise Pascal (162362), whom you have already
heard about, and Pierre de Fermat (160165).

Simple Probability
Mutually exclusive events

Mutually exclusive events means that if one event occurs, the other cannot.
For example, when rolling a die, a 6 cannot occur at the same time as a 2.
We can measure probability in theory as long as the events are random.
However, even then, probability only gives us an approximate idea of the
likelihood of certain events happening.
For example, in Lotto draws, there is a machine that draws out the balls
at random and a panel of supervisors checks that this happens properly. Each
ball is independent of the others and is equally likely to be drawn out.
320 Maths In Focus Mathematics HSC Course

In a horse race, it is difficult to measure probability as the horses are not


all equally likely to win. Other factors such as ability, training, experience and
weight of the jockey all affect it. The likelihood of any one horse winning is
not random.

Class Discussion

Discuss these statements:

1. The probability of one particular football team winning a


1
competition is as there are 16 teams.
16
2. The probability of Tiger Woods winning the US Open golf
1
tournament is if there are 78 players in the tournament.
78
3. A coin came up tails 8 times in a row. So there is a greater chance
that the next time it will come up heads.

The probability of an event E happening, P(E), is given by the number


of ways the event can occur, n(E), compared with the total number of
outcomes possible n(S) (the size of the sample space)

n (E )
P (E ) =
n (S)
If P(E) = 0 the event is impossible

If P(E) = 1 the event is certain (it has to happen)

0 P(E) 1

The sum of all (mutually exclusive) probabilities is 1

EXAMPLES

1. A container holds 5 blue, 3 white and 7 yellow marbles. If one marble


is selected at random, find the probability of getting
(a) a white marble
(b) a white or blue marble
(c) a yellow, white or blue marble
(d) a red marble.
Chapter 8 Probability 321

Solution
The size of the sample space, or total number of marbles is 5 + 3 + 7 or 15.
3
(a) P (W) =
15
1
=
5
3+5
(b) P (W or B) =
15
8
=
15
7+3+5
(c) P (Y or W or B) =
15
15
=
15
=1
0
(d) P (R) =
15 Getting a red marble is
=0 impossible!

2. The probability that a traffic light will turn green as a car approaches
5
it is estimated to be . A taxi goes through 192 intersections where there
12
are traffic lights. How many of these would be expected to turn green as
the taxi approached?

Solution
5
It is expected that of the traffic lights would turn green.
12
5
192 = 80
12
So it would be expected that 80 traffic lights would turn green as the taxi
approached.

8.1 Exercises
1. Peter is in a class of 30 students. 2. A pack of cards contains
If one student is chosen at 52 different cards, one of which
random to make a speech, find is the ace of diamonds. If one
the probability that the student card is chosen at random, find
chosen will be Peter. the probability that it will be the
ace of diamonds.
322 Maths In Focus Mathematics HSC Course

3. There are 6 different newspapers 10. A die is thrown. Calculate the


Die is singular for sold at the local newsagent probability of throwing
dice. each day. Kim sends her little (a) a 6
brother to buy her a newspaper (b) an even number
one morning but forgets to tell (c) a number less than 3.
him which one. What is the
11. A book has 124 pages. If the book
probability that he will buy the
is opened at any page at random,
correct newspaper?
nd the probability of the page
4. A rafe is held in which 200 number being
tickets are sold. If I buy 5 tickets, (a) either 80 or 90
what is the probability of my (b) a multiple of 10
winning the prize in the rafe? (c) an odd number
(d) less than 100.
5. In a lottery, 200 000 tickets are
sold. If Lucia buys 10 tickets, 12. In the game of pool, there are
what is the probability of her 15 balls, each with the number
winning rst prize? 1 to 15 on it. In Kelly Pool,
each person chooses a number
6. A bag contains 6 red balls and
at random to determine which
8 white balls. If one ball is drawn
ball to sink. If Tracey chooses a
out of the bag at random, nd
number, nd the probability that
the probability that it will be
her ball will be
(a) white
(a) an odd number
(b) red.
(b) a number less than 8
7. A shoe shop orders in 20 pairs (c) the 8 ball.
of black, 14 pairs of navy and
13. At a school dance, each girl is
3 pairs of brown school shoes.
given a ticket with a boys name
If the boxes are all mixed up,
on it. The girl then must dance
nd the probability that one box
with that boy for the next dance.
selected at random will contain
If the tickets are given out at
brown shoes.
random and there are 50 boys at
8. Abdul has 12 CDs in his car the dance, what is the probability
glovebox. The labels have that Jill will get to dance with her
become mixed up. If he chooses boyfriend?
one of the CDs at random, nd
14. Find the probability of a coin
the probability that it is his
coming up heads when tossed. If
favourite CD.
the coin is double-headed, nd
9. A bag contains 5 black marbles, the probability of tossing a head.
4 yellow marbles and 11 green
marbles. Find the probability of
drawing 1 marble out at random
and getting
(a) a green marble
(b) a yellow or a green marble.
Chapter 8 Probability 323

15. In a bag of caramels, there are 21. A biased coin is weighted so that
21 with red wrappers and 23 with heads comes up twice as often
blue wrappers. If Leila chooses a as tails. Find the probability of
caramel at random from the bag, tossing a tail.
nd the probability that she will
22. A die has the centre dot painted
choose one with a blue wrapper.
white on the 5 so that it appears
16. A student is chosen at random as a 4. Find the probability of
to write about his/her favourite throwing
sport. If 12 students like tennis (a) a 2
best, 7 prefer soccer, 3 prefer (b) a 4
squash, 5 prefer basketball and (c) a number less than 5.
4 prefer swimming, nd the
23. Discuss these statements.
probability that the student
(a) The probability of one
chosen will write about
particular horse winning the
(a) soccer
1
(b) squash or swimming Melbourne cup is if there are
20
(c) tennis. 20 horses in the race.
(b) The probability of Greg
17. A school has 875 students. If
Norman winning a masters golf
5 students are chosen at random
1
to help show some visitors tournament is if there are
15
around, nd the probability 15 players in the tournament.
that a particular student will be (c) A coin came up tails 8 times
chosen. in a row. So the next toss must be
a head.
18. A box containing a light globe
1 (d) A family has three sons.
has a probability of holding There is more chance of getting a
20
a defective globe. If 160 boxes daughter next time.
are checked, how many would be (e) The probability of a Holden
expected to be defective? winning the car race at Bathurst
6
this year is as there are
19. There are 29 red, 17 blue, 47
21 yellow and 19 green chocolate 6 Holdens in the race and 47 cars
beans in a packet. If Kate altogether.
chooses one at random, nd the
probability that it will be red or
yellow.

20. The probability of breeding a


2
white budgerigar is . If Mr Seed
9
breeds 153 budgerigars over
the year, how many would be
expected to be white?
324 Maths In Focus Mathematics HSC Course

Complementary events

When we nd the probabilities of events, the total of all the possible events
will always add up to 1.

EXAMPLE

A ball is chosen at random from a bag containing 5 blue, 3 red and


7 yellow balls. The probabilities are as follows:
5
P (blue) =
15
3
P (red) =
15
7
P (yellow) =
15
5 3 7
Total probability = + +
15 15 15
15
=
15
=1

The complement of an event happening is the event not happening. That


is, the complement of P(E) is P(not E). We can write this as P ^ L
Eh

EXAMPLE

A die is thrown. Find the probability of


(a) throwing a 6
(b) not throwing a 6.

Solution
1
(a) P ] 6 g =
6
(b) P (not 6) = P (1, 2, 3, 4, or 5)
5
=
6

In general,
P (E) + P (L
E) = 1
or P (E) = 1 P (L
E)
Chapter 8 Probability 325

Proof
Let e be the number of ways E can happen out of a total of n events. Then the
number of ways E will not happen is n e.
e
Then P (E) = n
ne
P (L
E) = n
n e
=n n
e
= 1 n
= 1 P ( E)

EXAMPLES

1
1. The probability of a win in a rafe is . What is the probability of
350
losing?

Solution
P (lose) = 1 P ] win g
1
= 1
350
349
=
350

2. The probability of a tree surviving a re is 72%. Find the probability


of the tree failing to survive a re.

Solution
P ^ failing to survive h = 1 P ^ surviving h
= 100% 72%
= 28%

8.2 Exercises
1. The probability of a bus arriving 3. If a baby has a 0.2% chance of
18 being born with a disability, nd
on time is estimated at . What
33 the probability of the baby being
is the probability that the bus will
born without any disabilities.
not arrive on time?
4. The probability of selecting a card
2. The probability of a seed
with the number 5 on it is 0.27.
7
producing a pink ower is . What is the probability of not
9
Find the probability of the ower selecting this card?
producing a different colour.
326 Maths In Focus Mathematics HSC Course

5. There is a 62% chance of a 12. The probabilities of a certain


student being chosen as a prefect. number of seeds germinating
What is the probability of a when 4 seeds are planted are as
student not being selected as a follows:
prefect? 4
P ] 4 seeds g =
49
6. A machine has a 1.5% chance of
8
breaking down at any given time. P ] 3 seeds g =
49
What is the probability of the 16
P ] 2 seeds g =
machine not breaking down? 49
18
7. The life of a certain type of P ] 1 seed g =
49
computer is about 7 years. If the 3
P (0 seeds) =
probability of its needing repairs 49
1
in that time is , nd the Find the probability of at least
23
probability that it will not need 1 seed germinating.
repairs. 13. The probabilities of 4 friends
8. A certain trafc light has a being chosen for a soccer team
13 are as follows:
probability of of being green.
18 1
Find the probability of the light P ] 4 chosen g =
15
not being green when a car 4
P ] 3 chosen g =
comes to the trafc light. 15
6
9. A certain organism in a river has P ] 2 chosen g =
15
a probability of 0.79 of surviving 2
P ] 1 chosen g =
a ood. What is the probability of 15
its not surviving? Find the probability of
(a) none of the friends being
10. A city has an 8.1% chance of
chosen
being hit by an earthquake. What
(b) at least 1 of the friends being
is its chance of not having an
chosen.
earthquake?

11. The probabilities when 3 coins


are tossed are as follows:
1
P (3 heads) =
8
3
P (2 heads and 1 tail) =
8
3
P (1 head and 2 tails) =
8
1
P (3 tails) =
8
Find the probability of tossing at
least one head.
Chapter 8 Probability 327

14. A dog breeder tries to produce a 15. The probabilities of 3 new cars
dog with a curly tail. If 2 puppies passing a quality control check
are born, the probabilities are as are as follows:
follows: 1
P ^ 3 passing h =
4 16
P ^ no curly tails h = 7
11 P ^ 2 passing h =
5 16
P ^ 1 curly tail h = 3
11 P ^ 1 passing h =
2 16
P ^ 2 curly tails h = 5
11 P ^ 0 passing h =
16
Find the probability that at least
1 puppy will have a curly tail. Find the probability that at least
1 car will fail the check.

Non-mutually exclusive events

All the examples of probability given so far in this chapter are mutually
exclusive. This means that if one event occurs, then another one cannot.
For example, if a die is thrown, a 6 cannot occur at the same time as a 2.
Sometimes, there is an overlap where more than one event can occur at
the same time. We call these non-mutually exclusive events.
It is important to count the possible outcomes carefully when this
happens. If there are not too many outcomes, we can simply list them, but if
this is difcult, we can use a Venn diagram to help.

EXAMPLES

1. One card is drawn from a set of cards numbered 1 to 10. Find the
probability of drawing out an odd number or a multiple of 3.

Solution
The odd cards are 1, 3, 5, 7 and 9.
The trick with this question
The multiples of 3 are 3, 6 and 9. is not to count the 3
The numbers 3 and 9 are both odd and multiples of 3. and the 9 twice.

So there are 6 numbers that are odd or multiples of 3: 1, 3, 5, 6, 7 and 9


6
P (odd or multiple of 3) =
10
3
=
5

CONTINUED
328 Maths In Focus Mathematics HSC Course

Using a Venn diagram:

Count all the numbers inside the


two circles.

There are 6 numbers From the Venn diagram:


inside the circles and 6 3
10 numbers altogether. Probability is = .
10 5

2. In year 7 at Mt Random High School, every student must do art or


music. In a group of 100 students surveyed, 47 do music and 59 do art.
If one student is chosen at random from year 7, nd the probability that
this student does
(a) both art and music
(b) only art
(c) only music.

Solution
47 + 59 = 106
But there are only 100 students!
This means 6 students have been counted twice.
i.e. 6 students do both art and music.
Students doing music only: 47 6 = 41
Students doing art only: 59 6 = 53
A Venn diagram shows this information.

6 3
(a) P ] both g = =
100 50
53
(b) P ^ art only h =
100
41
(c) P ^ music only h =
100

DID YOU KNOW?

Venn diagrams are named after John Venn (18341923), an English probabilist
and logician.
Chapter 8 Probability 329

There is a formula that can be used for non-mutually exclusive events.

P ] A or B g = P ] A g + P ] B g P ] A and B g

Notice that P ] A g + P ] B g counts P ] A and B g twice, since it occurs in both


P(A) and P(B). This can be adjusted by subtracting P(A and B).

EXAMPLE

From 100 cards, numbered from 1 to 100, one is selected at random. Find
the probability that the card selected is even or less than 20.

Solution
Some cards are both even and less than 20 (i.e. 2, 4, 6, 8, 10, 12, 14, 16, 18).
9
P (even and < 20) =
100
50
P (even) =
100
19
P (< 20) =
100
P (even or < 20) = P (even) + P (< 20) P (even and < 20)
50 19 9
= +
100 100 100
60
=
100
3
=
5

8.3 Exercises
1. A number is chosen at random 2. A set of 50 cards is labelled from
from the numbers 1 to 20. Find 1 to 50. One card is drawn out at
the probability that the number random. Find the probability that
chosen will be the card will be
(a) divisible by 3 (a) a multiple of 5
(b) less than 10 or divisible by 3 (b) an odd number
(c) a composite number (c) a multiple of 5 or an odd
(d) a composite number or a number
number greater than 12. (d) a number greater than 40 or
an even number
(e) less than 20.
330 Maths In Focus Mathematics HSC Course

3. A set of 26 cards, each with a 7. A survey of 80 people with dark


different letter of the alphabet on hair or brown eyes showed that
it, is placed in a box and one is 63 had dark hair and 59 had
drawn out at random. Find the brown eyes. If one of the people
probability that the letter on the surveyed is chosen at random,
card drawn will be nd the probability that the
(a) a vowel person will
(b) a vowel or one of the letters (a) have dark hair but not brown
in the word random eyes
(c) a consonant or one of the (b) have brown eyes but not dark
letters in the word movies. hair
(c) have both brown eyes and
4. A set of discs is numbered 1 to
dark hair.
100 and one is chosen at random.
Find the probability that the
number on the disc will be
(a) less than 30
(b) an odd number or a number
greater than 70
(c) divisible by 5 or less than 20.

5. In Lotto, a machine holds 8. A list is made up of people with


45 balls, each with a number experience of either computers
between 1 and 45 on it. The or digital cameras. On the list
machine draws out one ball of 20 people, 13 have computer
at a time at random. Find the experience while 9 have
probability that the rst ball experience with a digital camera.
If one name is chosen at random
drawn out will be
from the list, nd the probability
(a) less than 10 or an even
that the person will have
number.
experience with
(b) between 1 and 15 inclusive,
(a) both computers and digital
or divisible by 6
cameras
(c) greater than 30 or an odd
(b) computers only
number.
(c) digital cameras only.
6. A class of 28 students puts on
a concert with all class members
performing. If 15 dance and
19 sing in the performance, nd
the probability that any one
student chosen at random from
the class will
(a) both sing and dance
(b) only sing
(c) only dance.
Chapter 8 Probability 331

9. In a group of 75 students, all 10. In a group of 20 dogs at


do either History or Geography. obedience school, 14 dogs will
Altogether 54 do History and walk to heel and 12 will stay
31 do Geography. If I select one when told. If one dog is chosen at
student at random, find the random, find the probability that
probability that he/she will do the dog will
(a) only Geography (a) both walk to heel and stay
(b) both History and Geography (b) walk to heel but not stay
(c) History but not Geography. (c) stay but not walk to heel.

Multi-Stage Events
Product rule of probability

Class Discussion
Break up into pairs and try these experiments with one doing the activity
and one recording the results.

1. Toss two coins as many times as you can in a 5 minute period and
record the results in the table:

Result Two heads One head and one tail Two tails
Tally

Compare your results with others in the class. What do you notice? Is this
surprising?

2. Roll two dice as many times as you can in a 5 minute period, find the
total of the two uppermost numbers on the dice and record the results
in the table:

Total 2 3 4 5 6 7 8 9 10 11 12
Tally

Compare your results with others in the class. What do you notice? Is this
surprising?
Why dont these results appear to be equally likely?

You learned about tree


The counting of all possible outcomes (the size of the sample space) is diagrams in your earlier
important. This is why we use tables and tree diagrams. studies.
332 Maths In Focus Mathematics HSC Course

EXAMPLES
Compare these probabilities
with your results in the Find the size of the sample space and the probability of each outcome for
experiments. each question by using a table or tree diagram.

1. Tossing two coins

Solution
Using a table gives

H T
H HH HT
T TH TT
Using a tree diagram gives
H
H
T

H
T
T

Since there are four separate outcomes (HH, HT, TH, TT) each outcome
1
has a probability of .
4
1
Remember that each outcome when tossing 1 coin is .
2
1 1 1
Notice that = .
2 2 4

2. Rolling 2 dice and recording the sum of the uppermost numbers.

Solution
Can you see why a tree
diagram is too difficult here? A tree diagram would be too difficult to draw for this question.
Using a table:

1 2 3 4 5 6
1 2 3 4 5 6 7
2 3 4 5 6 7 8
3 4 5 6 7 8 9
4 5 6 7 8 9 10
5 6 7 8 9 10 11
6 7 8 9 10 11 12
1
Since there are 36 outcomes, each has a probability of
36
1
Remember that each outcome when rolling 1 die is .
6
1 1 1
Notice that =
6 6 36
Chapter 8 Probability 333

If A and B are independent events, then the probability of both occurring


is the product of their probabilities.

P ] AB g = P ] A g $ P ] B g

EXAMPLES

1
1. The probability of getting a 6 when rolling a die is . Find the
6
probability of getting a double 6 when rolling two dice.

Solution
1 1
P ] double 6 g =
6 6
1
=
36

7
2. The probability that a certain missile will hit a target is . Find the
8
probability that the missile will
(a) hit two targets
(b) miss two targets.

Solution
7 7
(a) P ] 2 hits g =
8 8
49
=
64
7
(b) P ] miss g = 1
8
1
=
8
1 1
P ] 2 misses g = Using these answers, could
8 8
you calculate the probability
1
= that the missile hits one
64 target and not the other?

Sometimes the outcomes change when looking at more than one event.
334 Maths In Focus Mathematics HSC Course

EXAMPLES

1. Maryam buys 5 tickets in a rafe in which 95 tickets are sold


altogether. There are two prizes in the rafe. What is the probability of her
(a) winning both rst and second prizes?
(b) winning neither prize?
(c) winning at least one of the prizes?

Solution
5
(a) Probability of winning rst prize =
95
After winning rst prize, Maryams winning ticket is taken out of
the draw. She then has 4 tickets left in the rafe out of a total of
94 tickets left.
4
Probability of winning second prize =
94
5 4
P ] WW g =
95 94
2
=
893
5
(b) Probability of not winning rst prize = 1
95
90
=
95
After not winning rst prize, Maryams 5 tickets are all left in the
draw, but the winning ticket is taken out, leaving 94 tickets in the
rafe.
5
Probability of winning second prize =
94
5
Probability of not winning second prize = 1
94
89
=
94
90 89
P ] LL g =
95 94
801
=
893
(c) P ] at least one W g = 1 P ] LL g
801
=1
893
92
=
893

2. I choose 3 balls at random from a bag containing 7 blue and 5 red


balls.
(a) Find the probability of getting 3 blue balls if
(i) I replace each ball before choosing the next one
(ii) I dont replace each ball before choosing the next one.
(b) Find the probability of getting at least one red ball (without
replacement).
Chapter 8 Probability 335

Solution
7
(a) (i) P ]Bg =
12
So P ] BBB g = 7 7 7
12 12 12
343
=
1728
7
(ii) P ] B g =
12
After the rst blue ball has been chosen, the bag now contains 6 blue and
5 red balls.

6
P ] 2nd B g =
11
After the second blue ball has been chosen, the bag contains 5 blue and
5 red balls.

5
P ] 3rd B g =
10
7 6 5
So P ] BBB g =
12 11 10
7
=
44
(b) P ] at least one R g = 1 ] no R g
= 1 P ] BBB g
7
=1
44
37
=
44

8.4 Exercises
1. If 2 dice are thrown, nd the 5. A box contains 2 black balls, 5 red
probability of throwing two 6s. balls and 4 green balls. If I draw
out 2 balls at random, replacing
2. Find the probability of
the rst before drawing out the
getting 2 heads if a coin is
second, nd the probability that
tossed twice.
they will both be red.
3. A coin is tossed 3 times. Find the
6. The probability of a conveyor belt
probability of tossing 3 tails.
in a factory breaking down at any
4. A card has a picture on one side one time is 0.21. If the factory
and is blank on the other. If the has 2 conveyor belts, nd the
card is thrown into the air twice, probability that at any one time
nd the probability that it will (a) both machines will break down
land with the picture side up (b) neither machine will break
both times. down.
336 Maths In Focus Mathematics HSC Course

7. The probability of a certain plant 14. A photocopier has a paper jam


owering is 93%. If a nursery on average around once every
has 3 of these plants, nd the 2400 sheets of paper.
probability that they will all ower. (a) What is the probability that a
particular sheet of paper will jam?
8. An archery student has a 69%
(b) What is the probability that
chance of hitting a target. If she
two particular sheets of paper
res 3 arrows at a target, nd the
will jam?
probability that she will hit the
(c) What is the probability that
target each time.
two particular sheets of paper will
9. A bag contains 8 yellow and not jam?
6 green lollies. If I choose
2 lollies at random, nd the
probability that they will both
be green
(a) if I replace the rst lolly
before selecting the second
(b) if I dont replace the rst lolly.

10. I buy 10 tickets in a rafe in


which 250 tickets are sold. Find
the probability of winning both 15. In Yahtzee, 5 dice are rolled. Find
rst and second prizes. the probability of rolling
11. Two cards are drawn from a (a) ve 6s
deck of 20 red and 25 blue cards (b) no 6s
(without replacement). Find the (c) at least one 6.
probability that they will both 16. The probability of a faulty
be red. computer part being
12. Find the probability of winning manufactured at Omega
the rst 3 prizes in a rafe if Peter 3
Computer Factory is .
5000
buys 5 tickets and 100 tickets are
If 2 computer parts are examined,
sold altogether.
nd the probability that
13. The probability of a pair of small (a) both are faulty
parrots breeding an albino bird (b) neither are faulty
2 (c) at least one is faulty.
is . If they lay three eggs, nd
33
the probability of the pair 17. A set of 10 cards is numbered
(a) not breeding any albinos 1 to 10 and two are drawn out at
(b) having all three albinos random with replacement. Find
(c) breeding at least one albino. the probability of drawing
(a) two odd numbers
(b) two numbers that are
divisible by 3
(c) two numbers less than 4.
Chapter 8 Probability 337

18. A bag contains 5 white, 4 black (a) all arrows hitting the target
and 3 red marbles. If 2 marbles (b) no arrows hitting the target
are selected from the bag at (c) at least one arrow hitting the
random without replacement, target.
find the probability of selecting
20. A coin is tossed n times. Find
(a) two red marbles
the probability in terms of n of
(b) two black marbles
tossing
(c) no white marbles
(a) all heads
(d) at least one white marble.
(b) no tails
19. The probability of an arrow hitting (c) at least one tail.
a target is 85%. If 3 arrows are shot,
find the probability as a percentage,
correct to 2 decimal places, of

Tree diagrams and probability trees

When using the product rule to find the probability of successive events
occurring, sometimes there is more than one possible result. For example,
when tossing two coins, there are two ways of getting a head and a tail
(HT and TH). We add these results together.

This is called the


P ] A or B g = P ] A g + P ] B g addition rule of
probability.

We use tree diagrams or probability trees to combine the product and


addition rules.
We use the product rule by multiplying along the branches and the
addition rule by adding up the probabilities from different branches.

EXAMPLES

1. If 2 coins are tossed, find the probability of tossing a head and a tail.

Solution

This is a probability
tree as it has the
probabilities listed.

CONTINUED
338 Maths In Focus Mathematics HSC Course

P ] head and tail g = P (HT ) + P (TH )


1 1 1 1
= c m+c m
2 2 2 2
1 1
= +
4 4
1
=
2

2. A person has probability of 0.2 of winning a prize in a


competition. If he enters 3 competitions, nd the probability of his
winning
(a) 2 competitions
(b) at least 1 competition.

Solution

Probability of losing is
1 0.2. (a) Probability of losing is 0.8
P (2W) = P (WWL) + P (WLW) + P (LWW )
= (0.2 0.2 0.8) + (0.2 0.8 0.2) + (0.8 0.2 0.2)
= 0.032 + 0.032 + 0.032
= 0.096
(b) P (at least one W ) = 1 P (LLL)
= 1 (0.8 0.8 0.8)
= 1 0.512
= 0.488
Chapter 8 Probability 339

3. A bag contains 3 red, 4 white and 7 blue marbles. Two marbles are
drawn at random from the bag
(a) replacing the rst before the second is drawn
(b) without replacement
Find the probability of drawing out a red and a white marble in these cases.

Solution
(a)

P (R and W ) = P (RW ) + P (WR)


3 4 4 3
=c m+c m
14 14 14 14
12 12
= +
196 196
6
=
49
(b)

The probabilities in each


set of branches must add
up to 1.

CONTINUED
340 Maths In Focus Mathematics HSC Course

P (R and W ) = P (RW ) + P (WR)


3 4 4 3
=c m+c m
14 13 14 13
12 12
= +
182 182
12
=
91

8.5 Exercises
1. Two coins are tossed. Find the replaced before the second is drawn
probability of getting out, nd the probability of getting
(a) 2 heads (a) 2 red marbles
(b) a head followed by a tail (b) a red and a blue marble.
(c) a head and a tail.
6. A certain breed of cat has a 35%
2. Three coins are tossed. Find the probability of producing a white
probability of getting kitten. If a cat has 3 kittens, nd
(a) 3 tails the probability that she will
(b) 2 heads and 1 tail produce
(c) at least 1 head. (a) no white kittens
(b) 2 white kittens
3. In a set of 30 cards, each one
(c) at least 1 white kitten.
has a number on it from 1 to
30. If 1 card is drawn out, then 7. The probability of a certain type of
replaced and another drawn out, photocopier in a school needing a
nd the probability of getting service on any one day is 0.3. Find
(a) two 8s the probability that a school with
(b) a 3 on the rst card and an 2 of these photocopiers will need
18 on the second card to service, on a particular day,
(c) a 3 on one card and an 18 on (a) 1 machine
the other card. (b) both machines
(c) neither of them
4. Five cards are labelled A, B, C, D
and E. If 2 are selected at random, 8. The probability of rain in May
with replacement, nd the 3
each year is given by .A
probability that they will be 10
school holds a fete in May for
(a) both As
three years running. Find the
(b) an A and a D.
probability that it will rain at
5. A bag contains 5 red marbles and (a) 2 of the fetes
8 blue marbles. If 2 marbles are (b) 1 fete
chosen at random, with the rst (c) at least 1 fete
Chapter 8 Probability 341

9. A certain type of plant has a 13. Mary buys 20 tickets in a lottery


probability of 0.85 of producing that has 5000 tickets altogether.
a variegated leaf. If I grow 3 of Find the probability that Mary
these plants, nd the probability will win
of getting a variegated leaf in (a) rst and second prize
(a) 2 of the plants (b) second prize only
(b) none of the plants (c) neither rst nor
(c) at least 1 plant. second prize.

14. Two musicians are selected at


random to lead their band. One
person is chosen from Band A,
which has 8 females and 7 males,
and the other is chosen from
Band B, which has 6 females and
9 males. Find the probability of
choosing
(a) 2 females
(b) 1 female and 1 male.
10. A bag contains 6 white balls and
15. The two machines in a workshop
5 green balls. If 2 balls are chosen
1
at random, nd the probability of each have a probability of
45
getting a white and a green ball of breaking down. Find the
(a) with replacement probability that at any one time
(b) without replacement. (a) neither machine will be
broken down
11. A bag contains 3 yellow balls,
(b) 1 machine will be broken
4 pink balls and 2 black balls. If
down.
2 balls are chosen at random, nd
the probability of getting a yellow 16. Two tennis players are said to
and a black ball 2
have a probability of and
(a) with replacement 5
3
(b) without replacement. respectively of winning a
4
tournament. Find the probability
12. Alan buys 4 tickets in a rafe that
in which 100 tickets are sold (a) 1 of them will win
altogether. There are two prizes (b) neither one will win.
in the rafe. Find the probability
that Alan will win
(a) rst prize 17. If 4 dice are thrown, nd the
(b) both prizes probability that the dice will
(c) 1 prize have
(d) no prizes (a) four 6s
(e) at least 1 prize. (b) only one 6
(c) at least one 6.
342 Maths In Focus Mathematics HSC Course

18. In a batch of 100 cars, past 22. In a group of people, 32 are


experience would suggest that Australian born, 12 were born
3 could be faulty. If 3 cars are in Asia and 7 were born in
selected at random, find the Europe. If 2 of the people are
probability that selected at random, find the
(a) 1 is faulty probability that
(b) none are faulty (a) they were both born in Asia
(c) all 3 cars are faulty. (b) at least 1 of them will be
Australian born
19. In a certain poll, 46% of people
(c) both were born in Europe.
surveyed liked the current
government, 42% liked the 23. There are 34 men and 32 women
opposition and 12% had no at a party. Of these, 13 men
preference. If 2 people from the and 19 women are married. If
survey are selected at random, 2 people are chosen at random,
find the probability that find the probability that
(a) both will prefer the (a) both will be men
opposition (b) 1 will be a married woman
(b) 1 will prefer the government and the other an unmarried man
and the other will have no (c) both will be married.
preference
24. In a certain city, the
(c) both will prefer the
probability that the pollution
government.
level will be high is 0.27. If
20. A manufacturer of X brand of soft the pollution is monitored
drink surveyed a city and found for 4 successive days, find the
that 31 people liked X drinks probability that the pollution
best, 19 liked another brand levels will be
better and 5 did not drink soft (a) high on 2 days
drink. If any 2 people are selected (b) high on 1 day
at random from that city, find the (c) low on at least 1 day.
probability that
(a) one person would like the X
brand of soft drink
(b) both people would not drink
soft drink.

21. A bag contains 5 red, 6 blue,


2 white and 7 green balls. If 2
are selected at random (without
replacement), find the probability
of getting
(a) 2 red balls
(b) a blue and a white ball
(c) 2 green balls
(d) at least 1 red ball.
Chapter 8 Probability 343

25. At City Heights School it was 28. The ratio of girls to boys at
found that 75% of students in a school is four to ve. Two
year 12 study 13 units, 21% study students are surveyed at random
12 units and 4% study 11 units. If from the school. Find the
2 students are selected at random probability that the students are
from year 12, nd the probability (a) both boys
that (b) a girl and a boy
(a) 1 student will study 12 units (c) at least one girl.
(b) at least 1 student will study
29. The number of cats to dogs at a
13 units.
pet hotel is in the ratio of 4 to 7.
26. Three dice are rolled. Find the If 3 pets are chosen at random,
probability of rolling nd the probability that
(a) 3 sixes (a) they are all dogs
(b) 2 sixes (b) just one is a dog
(c) at least 1 six. (c) at least one is a cat.

27. A set of 5 cards, each labelled 30. A set of 20 cards is numbered


with the letters A, B, C, D and 1 to 20 and three are selected at
E, is placed in a hat and two random with replacement. Find
selected at random without the probability of selecting
replacement. Find the probability (a) all three 10s
of getting (b) no 10s
(a) D and E (c) at least one 10.
(b) Neither D nor E on either card
(c) At least one D.
344 Maths In Focus Mathematics HSC Course

Test Yourself 8
1. The probability that a certain type 6. A set of 100 cards numbered 1 to 100 is
of seed will germinate is 93%. If 3 of placed in a box and one drawn at
this type of seeds are planted, nd the random. Find the probability that the
probability that card chosen will be
(a) odd
(a) all will germinate
(b) less than 30
(b) just 1 will germinate
(c) a multiple of 5
(c) at least 1 will germinate.
(d) less than 30 or a multiple of 5
2. A game is played where the differences of
(e) odd or less than 30.
the numbers on 2 dice are taken.
3
(a) Draw a table showing the sample 7. One game has a probability of of
5
space (all possibilities). winning and a second game has a
(b) Find the probability of getting a 2
probability of of winning. If Jenny
difference of 3
(i) 3 plays one of each game, nd the
(ii) 0 probability that she wins
(iii) 1 or 2. (a) both games
3. Mark buys 5 tickets in a rafe in which (b) one game
200 are sold altogether. (c) neither game.
(a) What is the probability that he will
8. A bag contains 5 black and 7 white
(i) win
marbles. Two are chosen at random from
(ii) not win the rafe?
the bag (a) with replacement (b) without
(b) If the rafe has 2 prizes, nd the
replacement. Find the probability of
probability that Mark will win just
getting a black and a white marble.
1 prize.
9. There are 7 different colours and 8
4. In a class of 30 students, 17 study
different sizes of leather jackets in a
history, 11 study geography and 5 study
shop. If Jean selects a jacket at random,
neither. One of these students is chosen
nd the probability that she will select
at random. Find the probability that this
one the same size and colour as her
student will
friend does.
(a) study geography but not history
(b) study both history and geography. 10. Each of a certain type of machine in
a factory has a probability of 4.5%
5. In the casino, when tossing 2 coins,
of breaking down at any time. If the
2 tails came up 10 times in a row. So there
factory has 3 of these machines, nd the
is less chance that 2 tails will come up
probability that at any one time
next time. Is this statement true? Why?
(a) all will be broken down
(b) at least one will be broken down.
Chapter 8 Probability 345

11. A bag contains 4 yellow, 3 red and 6 blue die is rolled, nd the probability that it
balls. Two are chosen at random. Find comes up
the probability of choosing (a) 2
(a) 2 yellow balls (b) even.
(b) a red and a blue ball
16. Amie buys 3 rafe tickets. If 150 tickets
(c) 2 blue balls.
are sold altogether, nd the probability
12. In a group of 12 friends, 8 have seen the that Amie wins
movie Star Wars 20 and 9 have seen the (a) 1st prize
movie Mission Impossible 6. Everyone in (b) only 2nd prize
the group has seen at least one of these (c) 1st and 2nd prizes
movies. If one of the friends is chosen (d) neither prize.
at random, nd the probability that this
17. A bag contains 6 white, 8 red and 5 blue
person has seen
balls. If two balls are selected at random,
(a) both movies
nd the probability of choosing a red
(b) only Mission Impossible 6.
and a blue ball
2
13. A game of chance has a probability of (a) with replacement
5
3 (b) without replacement.
a win or a probability of a draw.
8
18. A group of 9 friends go to the movies. If
(a) If I play one of these games, nd the
5 buy popcorn and 7 buy ice creams, nd
probability of losing.
the probability that one friend chosen at
(b) If I play 2 of these games, nd the
random will have
probability of
(a) popcorn but not ice cream
(i) a win and a draw
(b) both popcorn and ice cream.
(ii) a loss and a draw
(iii) 2 wins. 19. The probability that an arrow will hit a
8
14. A card is chosen at random from a set target is . If 3 arrows are red, nd the
9
of 10 cards numbered 1 to 10. A second
probability that
card is chosen from a set of 20 cards
(a) 2 hit the target
numbered 1 to 20. Find the probability
(b) at least 1 hits the target.
that the combination number these cards
make is 3
20. The probability of winning Game A is
5
(a) 911
and the probability of winning Game B is
(b) less than 100 7
(c) between 300 and 500. . Find the probability of winning
10
2 (a) both games
15. A loaded die has a probability of
3 (b) neither game
coming up 6. The other numbers have (c) one game.
an equal probability of coming up. If the
346 Maths In Focus Mathematics HSC Course

Test Yourself
Challenge 4
Exercise 8
1. In a group of 35 students, 25 go to the (a) both cards are As
movies and 15 go to the football. If all (b) one card is an A and the other is a D
the students like at least one of these (c) neither card is an A or D.
activities, nd the probability that a
7. In the game of Yahtzee, 5 dice are rolled.
student chosen at random will
Find the probability of rolling
(a) go to both the movies and the
(a) all 6s
football
(b) all the same number
(b) only go to the movies.
8. Out of a class of 30 students, 19 play a
2. A certain soccer team has a probability of
musical instrument and 7 play both a
0.5 of winning a match and a probability
musical instrument and a sport. Two
of 0.2 of drawing. If the team plays 2
students play neither.
matches, nd the probability that it will
(a) One student is selected from the class
(a) draw both matches
at random. Find the probability that this
(b) win at least 1 match
person plays a sport but not a musical
(c) not win either match.
instrument.
3. A game of poker uses a deck of 52 cards (b) Two people are selected at random
with 4 suits (hearts, diamonds, spades from the class. Find the probability that
and clubs). Each suit has 13 cards, both these people only play a sport.
consisting of an ace, cards numbered
9. A game involves tossing 2 coins and rolling
from 2 to 10, a jack, queen and king.
2 dice. The scoring is shown in the table.
If a person is dealt 5 cards nd the
probability of getting
Result Score (points)
(a) four aces
(b) a ush (all cards the same suit). 2 heads and 5
double 6
4. If a card is drawn out at random from a
2 heads and 3
set of playing cards nd the probability
double (not 6)
that it will be
(a) an ace or a heart 2 tails and double 6 4
(b) a diamond or an odd number 2 tails and double 2
(c) a jack or a spade. (not 6)
5. Bill does not select the numbers 1, 2,
(a) Find the probability of getting
3, 4, 5 and 6 for Lotto as he says this
2 heads and a double 6.
combination would never win. Is he
(b) Find the probability of getting 2 tails
correct?
and a double that is not 6.
6. In a set of 5 cards, each has one of the (c) What is the probability that Andre will
letters A, B, C, D and E on it. If two score 13 in three moves?
cards are selected at random without (d) What is the probability that Justin
replacement, nd the probability that will beat Andres score in three moves?
PRACTICE ASSESSMENT TASK 1 127

Practice Assessment Task


SET 1
1. Prove that the opposite sides of a
#1
3
12. Evaluate (6x2 + 4x) dx.
parallelogram are equal.
13. ABC is an isosceles triangle with D
2. Find all values of x for which the curve
the midpoint of BC. Show that AD is
y = ^ 2x 1 h2 is decreasing.
perpendicular to BC.
3. Find # (3x2 2x + 1) dx.
4. Find the maximum value of the curve
y = x2 + 3x 4 in the domain 1 x 4.

5. The area of a rectangle is 4 m2. Find its


minimum perimeter.

6. Show that ABDE is a parallelogram,


given that ACDF is a parallelogram and
BC = FE. 14. Find the volume of the solid of
revolution formed, correct to 3
signicant gures, if y = x2 + 2 is rotated
about
(a) the x-axis from x = 0 to x = 2
(b) the y-axis from y = 2 to y = 3.

15. Find the domain over which the curve


7. Find the primitive function of 3x8 + 4x. y = 3x3 + 7x2 3x 1 is concave upwards.

8. Sketch the curve y = x3 3x2 9x + 2, 16. If f (x) = 2x4 x3 7x + 9, nd


showing all stationary points and f (1), f l(1) , f m(1) . What is the geometrical
inexions. signicance of these results? Illustrate by
a sketch of y = f (x) at x = 1.
9. Find the volume of the solid of
revolution formed when the curve 17. A piece of wire of length 4 m is cut
y = x3 + 1 is rotated about into 2 parts. One part is bent to form
(a) the x-axis from x = 0 to x = 2 a rectangle with sides x and 3x, and
(b) the y-axis from y = 1 to y = 9. the other part is bent to form a square
with sides y. Prove that the total area
10. Find the area enclosed between the curve
of the rectangle and square is given by
y = x2 1 and the x-axis.
A = 7x2 4x + 1, and nd the dimensions
11. If f (x) = x3 2x2 + 5x 9, nd f l(3) and of the rectangle and square when the
f m( 2) . area has the least value.
128 Maths In Focus Mathematics HSC Course

18. The gradient function of a curve is given (b) This area is rotated about the y-axis.
by f l(x) = 4x 3. If f (2) = 3, nd f (1) . Find the exact volume of the solid formed.
9
#0 #
3
19. Evaluate (2x + 1) dx. 29. Evaluate y dy.
4

20. The following table gives values for 30. Find the area enclosed between the curve
1 y = (x 1) 2 and the line y = 4.
f (x) = 2 .
x
31. Prove ABC and CBD are similar.
x 1 2 3 4 5
f(x) 1 1 1 1 1
4 9 16 25

Use the table together with Simpsons


dx
#1
5
rule to evaluate correct to 3
x2
signicant gures.
32. Show that the curve y = 2x4 has a
21. Show that ABC is right angled at +B. minimum turning point at (0, 0) .

33. Find the maximum volume of a


rectangular prism with dimensions x, 2x
and y and whose surface area is 12 m2.

34. If a function has a stationary point at


22. Find # (3x + 5) 7
dx. ( 1, 2) and f m(x) = 2x - 4, nd f (2) .

#0
2
23. Evaluate (x2 + x 5) dx. 35. Find the area enclosed between the
curves y = x2 and y = x2 + 2x + 4.
24. Find the point of inexion on the curve
f (x) = x3 3x2 + 4x 1. 36. Find the minimum surface area of a
dx closed cylinder with volume 100 m3,
#1
3
25. Find an approximation to x by using correct to 1 decimal place.
(a) Simpsons rule with 3 function values
(b) the trapezoidal rule with 37. A curve has a stationary point at (3, 2) .
2 subintervals. If f m(x) = 6x 8, nd the equation of the
function.
26. Find the stationary points on the curve
f (x) = x4 2x2 + 3 and distinguish 38. Sketch the function
between them. f (x) = x3 3x2 9x + 5, showing any
2
stationary points and inexions.
27. Evaluate #1 5x 1 dx as a fraction.
39. Prove that the area of a rhombus is
28. (a) Find the area enclosed between the 1
A = xy where x and y are the lengths of
2
curve y = x2 1 and the y-axis between
the diagonals.
y = 1 and y = 2 in the rst quadrant, to 3
signicant gures.
PRACTICE ASSESSMENT TASK 1 129

40. (a) y

x
Given BG < CD and GF < DE, prove
AB AF
= .
AC AE

41. Given f l(x) = x2 2x 1, evaluate f (2) if


f (0) = 5.

42. (a) Complete the statement: The y


(b)
primitive function of xn = . . .
(b) Explain why there is a constant C in
the primitive function.

43. The graph below is the derivative of a


function.
y x

x
(c) y

x
Which graph could represent the
function? There may be more than one
answer.
130 Maths In Focus Mathematics HSC Course

y
(d) (c) y

x x

(d) y
dy
44. A function has > 0. To which of the
dx
following graphs does this apply?

(a) y

45. The area of a rectangle with sides x and y


is 45. The perimeter is given by:
(a) P = x + 45x2
45
(b) P = x + x
(b) y
90
(c) P = 2x + x
45
(d) P = 2x + x
46. The area enclosed between the curve
y = x3 1, the y-axis and the lines y = 1
and y = 2 is given by

#1 (x3 1) dy
2
(a)
x
#1 ( y + 1) dy
2
(b)

#1 (3
2
(c) y + 1) dy

#1 (3
2
(d) y + 1 ) dy
PRACTICE ASSESSMENT TASK 1 131

47. C The curve has


dy d2 y
(a) > 0, 2 > 0
dx dx
D dy d2 y
O (b) > 0, <0
dx dx2
A
dy d2 y
(c) < 0, >0
dx dx2
dy d2 y
B (d) < 0, <0
dx dx2
Given O is the centre of the circle, 50. A cone with base radius r and height h
triangles OAB and OCD can be proven to has a volume of 300 cm3. Its slant height
be congruent using the test is given by
(a) SSS
h3 + 900
(b) SAS (a) l =
h
(c) AAS
(d) RHS. h2 + 900
(b) l =
h
48. The area bounded by the curve y = x3, the h2 + 810 000
x-axis and the lines x = 2 and x = 2 is (c) l =
h
(a) 8 units2
h3 + 900
(b) 2 units2 (d) l =
h
(c) 0 units2
(d) 4 units2.

49. y

x
PRACTICE ASSESSMENT TASK 2 253

Practice Assessment Task


SET 2
1. The population of a city over t years is 11. Find the volume, to 1 decimal place, of
given by the formula P = 100 000e0.71t. the solid formed by rotating y = ex about
After how many years, to 1 decimal the x-axis from x = 1 to x = 3.
place, will the population become
12. Find the derivative of loge (4x + 3) 3.
1 million?
2x + 1
2. Evaluate #1
3 dx 13. Find # dx.
x correct to 3 decimal places. 3x 2 + 3x 2
14. The volume in litres of a rectangular
3. The acceleration of a particle is given by
container that is leaking over time t
a = 6t ms 2 and the particle is initially at
minutes is given by V = t2 + 4t + 100.
rest at the origin. Find where it will be
Find
after 3 seconds.
(a) the initial volume
1 (b) the volume after 10 minutes
4. Find log5 .
25
(c) the rate of change in volume after
5. Write loge x as an equation with x in 10 minutes
terms of y. Hence nd the value of x, to (d) how long it will take, to 1 decimal
3 signicant gures, when y = 1.23. place, until the container is empty.
6. Differentiate x3 + e2x . 15. Find # (6x2 2x + 4x 1) dx.
1 3
7. Solve logx = 4. 16. Find the exact value of #1
7
dx.
16 x+5
8. If 100 g of a substance decays to 80 g 17. The velocity v cms-1 of a particle over
after 3 years, nd, to 1 decimal place, 2
time t is given by v = . If the particle
(a) its mass after 10 years t+1
(b) the rate of decay after 10 years is initially 3 cm from the origin, nd
(c) when it will decay to 50 g. (a) its displacement after 10 s,
to 2 signicant gures
9. A particle has displacement x in (b) its acceleration after 5 s,
centimetres over time t in seconds to 1 signicant gure.
according to the formula x = 2 sin 3t.

(a) Find its velocity after s.
18. Find # (e4x + 1) dx.
2
(b) Show that the acceleration is given x
19. Differentiate .
by px = 9x. e2x
20. The rate at which an epidemic of
10. Water is owing out of a pool at the rate
measles in a certain city is spreading is
given by R = 20t litres per minute. If the
proportional to the number of people
volume of water in the pool is initially
dP
8000 L, nd with measles. That is, = kP. If 40
dt
(a) the volume after 5 minutes people initially have measles and after
(b) how long it will take to empty the 10 days, 110 have measles, nd
pool.
254 Maths In Focus Mathematics HSC Course

(a) the value of k to 3 signicant gures 29. The length of an arc in a circle of radius
(b) how many people will have measles 2 cm is 1.6 cm. Find the area, correct to
after 6 weeks 2 decimal places, of the
(c) after how long, to the nearest day, (a) sector
300 people will have measles (b) minor segment cut off by this arc.
(d) the rate at which the disease will be
30. A certain chemical treatment of blue-
spreading after
green algae in a river causes it to decrease
(i) 10 days
at a rate proportional to the amount of
(ii) 6 weeks.
blue-green algae in the river. If 250 kg of
21. (a) Find the value of log3 7 by changing blue-green algae reduces to 150 kg after
the base to e. 3 months, nd how long it will take, to
(b) Differentiate log3 x by changing the the nearest month, to reduce the blue-
base to e. green algae to 20 kg.

22. Find the exact volume of the solid of 31. An angle of 30 is subtended at the
revolution formed when the curve y = ex centre of a circle with radius 5 cm. Find
is rotated about the x-axis from x = 1 to the exact
x = 3. (a) arc length
(b) area of the sector.
23. A particle is moving such that its
x
displacement after t seconds is given by 32. Sketch y = 2 cos for 0 x 2 .
2
x = 3 sin 2t metres.

(a) Find the initial velocity and 33. Find # 0


2
sin (2x) dx.
acceleration.
34. Differentiate loge (sin x).
(b) Find the maximum displacement.
(c) Find the times when the particle will 35. Find the derivative of tan (e5x + 1).
be at rest.
(d) Prove that the acceleration is given 36. Evaluate # 3
sec 2 x dx, giving the exact
by a = 4x. 0
value.
24. Find the area enclosed between the curve 37. (a) Sketch y = sin 2x for 0 x 2 .
y = loge x, the y-axis and the lines y = 1 (b) Find the area bounded by the curve
and y = 2, correct to 3 signicant gures. y = sin 2x, the x-axis and the lines x = 0
and x = .
25. The acceleration of a particle is given by
6t 12 ms 2 . If the particle is initially at 38. Evaluate #0
4
tan x dx correct to 3 decimal
rest 2 m to the left of the origin, nd its
places by using Simpsons rule with three
displacement after 5 seconds.
ordinates.
26. Solve 72x = 3.
39. Differentiate
27. Find the equation of the tangent to (a) ex sin x
y = ex + 1 at the point where x = 1. (b) tan3 x

(c) 2 cos b 3x l
2
28. Find the stationary point on the curve
y = xe2x and determine its nature.
PRACTICE ASSESSMENT TASK 2 255

40. A particle moves so that its displacement 51. Differentiate tan (loge x + 1).
after time t seconds is given by x = 5e2t m. 2x + 5
2

(a) Find its velocity after 2 s, to 52. Find # 3x dx.


x2
3 signicant gures. 53. Find the volume of the solid formed,
(b) Show that its acceleration is given by correct to 2 decimal places, when the
a = 4x. curve y = loge x is rotated about the
(c) Find the initial acceleration. x-axis from x = 1 to x = 4, by using the
dx
#0
5
41. Evaluate , giving the exact value. trapezoidal rule with 3 subintervals.
x+3
42. A gardener wishes to make a rectangular 54. Find # (e5x sin x) dx.
garden bed using a wall as one of the 55. Solve graphically sin x = x 1 for
sides. She has 24 m of edging strip to place 0 x 2 .
around the garden. What dimensions will
give the maximum garden area? 56. Find the exact area enclosed between the
curve y = ex, the x-axis, the y-axis and the
43. Find the volume of the solid formed line x = 2.
when the curve y = cos x is rotated
57. Evaluate # cos b
x
+ l dx.
about the x-axis from x = 0 to x = .
3 3
2
44. Sketch the curve y = 3x4 4x3 12x2 + 1,
showing any stationary points. 58. y
5
45. Find the equation of the tangent to the 4

curve y = tan 3x at the point where x = . 3
4
2
46. Find the exact area enclosed between the 1
1
curve y = sin x and the line y = for the
x
2 -1
3 2
domain 0 x 2 . 2 2
-2
3 x
47. Differentiate sin (e ) . The graph has the equation
48. Find the exact area bounded by the curve (a) y = 4 2 cos x
y = loge (x + 4), the y-axis and the lines (b) y = 4 + 2 cos x
y = 0 and y = 1. (c) y = 2 2 cos x
(d) y = 2 4 cos x.
49. Find the exact value of
7 59. A radioactive substance decays by 60%
(a) cos
4 after 200 years. This information can be
shown by the equation of its mass
(b) sin b 2 l.
3
(a) 0.6M = Me200k
50. The length of an arc in a circle of radius (b) 0.6M = Me 200k
6 cm is 7 cm. Find the area of the (c) 0.4M = Me200k
(a) sector (d) 0.4M = Me 200k
(b) minor segment cut off by this arc.
256 Maths In Focus Mathematics HSC Course

3x
60. The value of sin b l is
3
62. Evaluate # dx
3 2x 2 5
(a) (a) 3 ln (2x2 5) + C
2
3 3 ln (2x2 5)
(b) (b) +C
2 4
1 ln (2x2 5)
(c) (c) +C
2 12
1 4 ln (2x 5)
2
(d) . (d) +C
2 3
61. A particle has a displacement of
63. The rate at which a waterfall is owing
x = 4 cos 7t. Its acceleration can be
over a cliff is R = 4t + 3t2 m3 s 1. Find the
written as
amount of water owing after a minute
(a) px = 49x
if the amount of water is 10 970 m3 after
(b) px = 196x
20 seconds.
(c) px = 196x
(a) 223 220 m3
(d) px = 49x.
(b) 8800 m3
(c) 225 370 m3
(d) 226 250 m3
PRACTICE ASSESSMENT TASK 3 347

Practice Assessment Task


SET 3
2 11 9. Two dice are thrown and the total of
1. Find the 5th term of 4 + 2 +1 + ...
5 25 the dice is noted. Find the probability of
throwing
2. The probability that a seed will produce a (a) double ls
1
plant with a white flower is . If 4 seeds (b) any double
7
are planted, find the probability that (c) at least one 3
(a) no plants will have white flowers (d) a total of 6
(b) exactly 1 plant will have white flowers (e) a total of at least 8.
(c) at least 1 plant will have white flowers.
10. Find the sum of the first 9 terms of
a a
3. By writing the recurring decimal a+ + + ...
3 9
0.444 . . . as an infinite geometric series
11. A plant grows so that it increases its
or otherwise, express it as a rational
height each month by 0.2 of its previous
number.
months height. If it grows to 3 m, find
4. Find in index form the 10th term of its height in the first month.
3 3
3+ + + ... 12. The population of an insect colony is
4 16
given by the formula P = t3 t + 3 000 000,
5. George deposits $400 at the beginning of where t is time in days, Find
each year in an investment account that (a) the initial population.
pays 13% interest per annum. How much (b) the population after a week.
will there be in the account after 5 years? (c) the rate at which the population will
6. The second term of a geometric series is be increasing after a week.
52 and the 4th term is 13. Find two series 13. Find the stationary point on the curve
that satisfy these requirements. y = (x 2)3 and determine its nature.
7. Find which term -370 is in the series Hence sketch the curve.
17 + 8 1 . . . 14. Each card in a set of 100 cards has a
8. A man promises his son a sum of money different number on it, ranging from 1 to
on his 18th birthday, made up of $10 for 100. If one card is chosen at random,
his first year of life, $15 for his second find the probability of getting
year, $20 for his third year and so on, (a) an even number less than 30
up to 18 years. How much will his son (b) an odd number or a number divisible
receive? by 9.

pat_3.indd 347 7/16/09 5:28:04 PM


348 Maths In Focus Mathematics HSC Course

15. Two families travelling on holidays drive (a) 2 white marbles


along roads that intersect at right angles. (b) 1 white and 1 red marble.
One family is initially 230 km from
23. I put $2000 in the bank, where it earns
the intersection and drives towards the
interest at the rate of 12% p.a., paid
intersection at an average of 65 kmh1.
quarterly. How much will there be in the
The other family is initially 125 km from
account after 3 years?
the intersection and travels towards it at
an average of 80 kmh1. 24. The displacement of a pendulum is given
(a) Show that their distance by x = 3 cos 4t cm after time t seconds.
apart after t hours is given by (a) Find the equation for the velocity of
d2 = 10 625t2 49 000t + 68 525. the pendulum.
(b) Hence find how long it will take them (b) Find the equation for its acceleration.
to reach their minimum distance apart. (c) Find the initial displacement.
(c) Find their minimum distance apart. (d) Find the times when the pendulum
will be at rest.
16. Find the 50th term of 3 + 7 + 11 + . . . and
(e) What is the displacement at these
calculate the sum of 50 terms.
times?
17. If I buy 10 tickets, find the probability of (f) When will there be zero
my winning both first and second prizes displacement?
in a raffle in which 100 tickets are sold. (g) Show that acceleration a = 16x.

18. The nth term of a series is given by o as a rational number.


25. Express 0.1 7
7n 3
26. A coin is tossed and a die thrown. Find
(a) Find the first 3 terms and the 12th
the probability of getting
term. 20
(a) a head and a six
(b) Evaluate / (7n 3).
n =1 (b) a tail and an odd number.
(c) Which term is 200?
27. I borrow $5000 at 18% interest p.a. and

19. Find the exact value of sin b + l make equal monthly payments over
4
3 years, at the end of which the loan is
20. Sketch the graph of
fully paid out. Find the amount of each
(a) y = x2 5x + 6
monthly payment.
(b) y = 2 sin 4x for 0 x
(c) y = e x 28. Prove that 3ABC is right angled.

21. The 4th term of an arithmetic series is 18


and the 8th term is 62. Find the series.

22. A bag contains 7 red and 5 white


marbles. Two are drawn out at random.
Find the probability of drawing out

pat_3.indd 348 7/16/09 5:28:07 PM


PRACTICE ASSESSMENT TASK 3 349

29. The geometric series x + x2 + x3 + . . . has a 39. A farmer wants to fence a rectangular
sum to infinity of 5. Find the value of x. paddock and a square paddock with a
combined area of 5000 m2. If the length
30. Prove that if one pair of adjacent sides
of the rectangular paddock is to be
is equal in a rhombus, then all sides are
3 times the size of its breadth
equal.
(a) find the dimensions of the paddocks
31. Josie buys 5 lottery tickets in which that give the largest perimeter
200 000 tickets are sold. What is the (b) calculate the cost of fencing the
probability that she wins 1st prize? paddocks, at $19.95 per metre.

32. The mass (in grams) of a radioactive 40. Two dice are rolled, and the numbers of
substance is given by M = M0e kt , where the dice are totalled. Find the probability
t is time in years. Find of rolling
(a) k if its mass is halved after 28 years (a) a total of 8
(b) when the mass will be 35% of the (b) a total less than 7
original amount (c) a total greater than 9
(c) the time taken to reduce the mass (d) a total of 4 or 5
by 40%. (e) an odd total.

33. Evaluate 7 + 12 + 17 + . . . + 872. 41. Sketch the function f (x) = 3x4 + 4x3 12x2
showing all stationary points.
34. A rectangle is cut from a circular disc of
radius 15 cm. Find the area of the largest 42. The area of a circle is 5 and an arc 3 cm
rectangle that can be produced. long cuts off a sector with an angle of
subtended at the centre. Find
35. The equation for displacement of a (a) in degrees and minutes
particle is given by x = 3e 4t + 2 metres (b) the area of the minor segment cut off
after time t seconds. by the arc, correct to 2 decimal places.
(a) Find the initial velocity.
(b) Find the exact acceleration after 1 s. 43. Ian pays $50 into a superannuation fund
(c) Show that the acceleration is always at the beginning of each month, where it
4 times greater than the velocity. earns interest of 4% p.a. How much will
(d) Sketch the graph of velocity over time t. be in the fund at the end of 20 years?

36. Find the first value of n for which the 44. (a) Show that log 3 + log 9 + log 27 + . . .
4 is an arithmetic series.
sum of the series 20 + 4 + + . . . is
5 (b) Find the exact sum of 20 terms of
greater than 24.85. the series.
d2 y
37. If y = sin 7x, show that = 49y. 45. Sketch the curve y = 2x3 + 3x2 36x + 1,
dx2

2 showing any stationary points and
38. Evaluate / r . points of inflexion.
r =2 5

pat_3.indd 349 7/16/09 5:28:09 PM


350 Maths In Focus Mathematics HSC Course

46. A chemical reaction causes the amount 51. Find the limiting sum of the series
of hydrogen to be reduced at a rate 2 1 3
+ + + ...
proportional to the amount of hydrogen 3 2 8
present at any one time. If the amount (a) 2
1
of hydrogen is given by the formula (b) 1
3
A = A0e kt, and 800 mL reduces to 650 mL 2
(c) 2
after 3 minutes, find 3
(a) the amount of hydrogen after 1 hour 8
(d)
(b) how long it will take for the 9
hydrogen to reduce to 100 mL. 52. Which statement is the same as 3x = 7?
There may be more than one answer.
47. Find the domain over which the curve 7
y = x3 + 3x2 24x + 7 is concave (a) x = log
3
downwards. (b) log3 x = 7
(a) x < 1 (c) log3 7 = x
(b) x > 1 log 7
(c) 4 < x < 2 (d) x =
log 3
(d) x < 4, x > 2
53. The nth term of the series
48. I borrow $10 000 over 5 years at 1.85% 7 + 49 + 343 + is
monthly interest. How much do I need (a) 7n
to pay each month? (b) 7n 1
(c) 7n 1
49. Find the probability of drawing out a blue (d) 7n
and a white ball from a bag containing
7 blue and 5 white balls if the first ball 54. An amount of $6000 is invested at
is not replaced before taking out the 3.5% p.a. with interest paid quarterly.
second. Find the balance after 10 years.
70 (a) $8501.45
(a)
121 (b) $8463.59
70
(b) (c) $6546.16
144
1225 (d) $6899.96
(c)
17 424 3 2 4
70 55. The limiting sum of + + + . . . is
(d) 5 5 15
132 1
(a)
5
50. The limiting sum of a geometric series
9
exists when (b)
10
(a) | r | > 1 4
(c) 1
(b) | r | < 1 5
(c) | r | 1 (d)
6
(d) | r | 1 15

pat_3.indd 350 7/16/09 5:28:11 PM


PRACTICE ASSESSMENT TASK 3 351

56. In a group of 25 students, 19 catch a 57. The formula for the sum
train to school and 21 catch a bus. If one 1 + 1.03 + 1.032 + . . . + 1.03n 1 is
of these students is chosen at random, 1.03(1.03n 1 1)
(a) S =
find the probability that the student only 1.03 1
catches a bus to school. 1.03(1.03n 1)
(b) S =
(a)
6 1.03 1
25 1.03n 1 1
(c) S =
(b)
21 1.03 1
25 1.03n 1
(d) S =
(c)
3 1.03 1
5
3
(d)
20

pat_3.indd 351 7/16/09 5:28:13 PM


352 Maths In Focus Mathematics HSC Course

Sample Examination Papers

This chapter contains two sample (e) Two cities are 3295 km apart. Write
Mathematics HSC papers. They are designed this number correct to 2 significant
to give you some practice in working through figures.
an examination. These papers contain work (f) Solve | x + 3| < 7.
from the Year 11 Preliminary Course, as up
to 20% of questions are allowed to contain QUESTION 2
this work. You may need to revise this work (a)
before you try these papers.
If you can, set yourself a time limit and
work under examination conditions. Give
yourself 3 hours to do each Mathematics
paper. Try not to look up any notes while
working through these papers.
(i) Copy the diagram into your
examination booklet.
MATHEMATICSPAPER 1 (ii) Show that AC = CE.
(iii) Find the length of DE.
Time allowedThree hours
(b)
(Plus 5 minutes reading time)

DIRECTIONS TO CANDIDATES
All questions may be attempted.
All questions are of equal value.
All necessary working should be shown
in every question, as marks are awarded
(i) Copy the diagram into your
for this. Badly arranged or careless work
examination booklet.
may not receive marks.
(ii) Use the sine rule to calculate MO
QUESTION 1 correct to 1 decimal place.
(a) Find, correct to 2 decimal places, the (iii) Find MP to the nearest metre.
28.3
value of . QUESTION 3
15.7 2.4
(b) Factorise 3x2 - 11x + 6. (a) Differentiate
x x1 (i) x + 5x 3 + 1
(c) Solve the equation = 5.
2 3 1
(ii) 3 ln x + x
(d) The volume of a cone is given by
1 (iii) (2x + 3)5
V = r 2. If a cone has volume 12 m3,
3 (b) Find
find its radius correct to 2 decimal places.
(i) # (x e x ) dx
#0 (sin + 1) d

(ii)
SAMPLE EXAMINATION PAPERS 353

(c) (i) Rationalise the denominator of QUESTION 6


5 2
(a) A plant has a probability of of
. 7
21
producing white flowers. If 3 plants are
(ii) Find integers a and b such that
chosen at random, find the probability that
5
= a + b. (i) 2 plants will produce white flowers
21
(ii) at least 1 plant will produce white
QUESTION 4 flowers.
(a) (i) Plot points A(3, 2) and B(-1, 5) 3
(b) Solve sin x = for 0 x 360.
on a number plane. 2
(c) A particle P moves so that it is
(ii) Show that line AB has equation
initially at rest at the origin, and its
3x + 4y - 17 = 0.
acceleration is given by a = 6t + 4 cms-2.
(iii) Find the perpendicular distance
(i) Find the velocity of P when t = 5.
from the origin to line AB.
(ii) Find the displacement of P when
(iv) Find the area of triangle OAB
t = 2.
where O is the origin.
(b)
QUESTION 7
(a)

Find the length of side BC using the


ABCD is a parallelogram with AD = 1,
cosine rule and give your answer correct
AB = 2 and +ADX = 60. AX is drawn so
to the nearest centimetre.
that it bisects DC.
(i) Copy the diagram into your
QUESTION 5
examination booklet.
(a) Kate earns $24 600 p.a. In the
(ii) Show that ADX is an equilateral
following year she receives a pay increase
triangle.
of $800. Each year after that, she receives
(iii) Show that AXB is a right-angled
a pay increase of $800.
triangle.
(i) What percentage increase did
(iv) Find the exact length of side BX.
Kate receive in the first year?
(b) On a number plane, shade in the
(ii) What will be Kates salary after
region given by the two conditions y x2
12 years?
and x + y 3.
(iii) What will Kates total earnings
(c) The number of people P with
be over the 12 years?
chickenpox is increasing but the rate at
(b) A function is given by
which the disease is spreading is slowing
y = x3 - 3x2 - 9x + 2.
down over t weeks.
(i) Find the coordinates of the
(i) Sketch a graph showing this
stationary points and determine their
information.
nature. dP d2 P
(ii) Find the point of inflexion. (ii) Describe and 2 for this
dt dt
(iii) Draw a sketch of the function. information.
354 Maths In Focus Mathematics HSC Course

QUESTION 8 (b) Kim invests $500 at the beginning of


(a) Consider the function given by each year in a superannuation fund. The
y = log10 x. money earns 12% interest per annum. If
(i) Copy and complete the table, to she starts the fund at the beginning of
3 decimal places, in your examination 1996, what will the fund be worth at the
booklet. end of 2025?

x 1 2 3 4 5
y 0 QUESTION 10
(a)
(ii) Apply the trapezoidal rule
with 4 subintervals to find an
approximation, to 2 decimal
places, of # log10 x dx.
5

1
(b) A population of mice at time t in
weeks is given by P = P0ekt , where k is a
constant and P0 is the population when
t = 0.
(i) Given that 20 mice increase to
100 after 6 weeks, calculate the value (i) Find the exact point of
of k, to 3 decimal places. intersection of the curve y = ex and
(ii) How many mice will there be the line y = 4.
after 10 weeks? (ii) Find the exact shaded area
(iii) After how many weeks will there enclosed between the curve and
be 500 mice? the line.
(b) (i) The quadratic equation
QUESTION 9
x2 + (k - 1)x + k = 0 has real and equal
(a)
roots. Find the exact values of k in
simplest surd form.
(ii) If k = 5, show that
x2 + (k - 1)x + k > 0 for all x.
(c) A parabola has equation y = x2 + 2px + q.
(i) Show that the coordinates of its
vertex are (-p, q - p2).
The surface area of a cylinder is given by (ii) Find the coordinates of its focus.
the formula S = 2 r (r + h) . (iii) Find the distance between
A cylinder is to have a surface area of the point (m, 3m2 + q) and point P
160 cm2. vertically below it on the parabola
(i) Show that the volume is given x2 = 8y when q > 0.
by V = 80r r3.
(ii) Find the value of r, to 2 decimal
places, that gives the maximum
volume.
(iii) Find the maximum volume, to
1 decimal place.
SAMPLE EXAMINATION PAPERS 355

y QUESTION 3
x2 = 8y
(a) Differentiate
(m, 3m2 + q)
(i) x cos x
(ii) e5x
(iii) loge (2x2 1)
(b) Find the indefinite integral (primitive
P
function) of
(i) (3x - 2)4
x
(ii) 3 sin 2x
#0
3
y = x2 + 2px + q (c) Evaluate (ex e x ) dx.
d2y
(iv) Find the minimum distance (d) For a certain curve, = 18x 6. If
dx 2
between these two points when there is a stationary point at (2, 1), find
m + q = 5. the equation of the curve.

QUESTION 4
MATHEMATICSPAPER 2 (a) Mark plays a game of chance that has
Time allowedThree hours 2
a probability of of winning the game.
(Plus 5 minutes reading time) 5
Hong plays a different game in which
3
there is a probability of of winning.
QUESTION 1 8
(a) Solve | x 3 | = 5. Find the probability that
(b) If f (x) = 5 - x2, find x when f (x) = -4. (i) both Mark and Hong win their
5 games
(c) Find the exact value of sin .
6 (ii) one of them wins the game
(d) Factorise fully a - 2a - 4a + 8.
3 2
(iii) at least one of them wins the
(e) Simplify 2 24 150 .
game.
(f) Evaluate loga 50 if loga 5 = 1.3 and
(b) (i) On a number plane, shade the
loga 2 = 0.43.
region where y 0, x 3 and y x + 2.
(g) Find the midpoint of (-3, 4) and
(ii) Find the area of this shaded
(0, -2).
region.
(iii) This area is rotated about the
QUESTION 2 x-axis. Find the volume of the solid
1 1
Plot points A e 5, 1 o , B e 1, 1 o and formed.
2 2
(c) (i) Sketch the graph of y = 2x 1
C(-4, -1) on a number plane.
on a number plane.
(a) Show that the equation of line AB is
(ii) Hence solve 2x 1 < 3.
given by 3x + 4y - 9 = 0.
(b) Find the equation of the straight line
QUESTION 5
l through C that is perpendicular to AB.
(a) For the curve y = 2x3 - 9x2 + 12x - 7
(c) Find the point of intersection P of the
(i) find any stationary points on the
two lines, AB and l.
curve and determine their nature
(d) Find the area of triangle ABC.
(ii) find any points of inflexion
(e) Find the coordinates of D such that
(iii) sketch the curve in the domain
ADCB is a rectangle.
-3 x 3.
356 Maths In Focus Mathematics HSC Course

(b) A plane leaves Bankstown airport and (ii) Differentiate ln (cos x).

flies for 850 km on a bearing of 100.
It then turns and flies for 1200 km on a (iii) Hence evaluate #
0
4
tan x dx to
2 decimal places.
bearing of 040.
(b) A bridge is 40 metres long, held by
(i) Draw a diagram showing this
wires at A and C with angles of elevation
information.
of 47 and 23 as shown.
(ii) How far from the airport is the
plane, to the nearest km?
(c) Simplify
(cosec + cot )(cosec cot ).

QUESTION 6
(a) (i) Find the equation of the normal
(i) Find the length of AD to
to the curve y = x2 at point P(-2, 4).
3 significant figures.
(ii) This normal cuts the parabola
(ii) Find the height of the bridge BD
again at point Q. Find the coordinates
to 1 decimal place.
of Q.
(c) Triangle BEC is isosceles with
(iii) Find the shaded area enclosed
BC = CE. Also +BEC = 50, +ABE = 130,
between the parabola and the normal,
and +ADC = 80.
to 3 significant figures.
(b) The graph below shows the
displacement of a particle over time t.

Prove that ABCD is a parallelogram.

QUESTION 8
(i) When is the particle at the origin? (a) (i) Sketch the curve
(ii) When is the particle at rest? y = 3 sin 2x for 0 x 2 .
(c) The temperature T of a metal is (ii) On the same set of axes, sketch
x
cooling exponentially over t minutes. It y= .
2
cools down from 97C to 84C after (iii) How many roots does the equation
5 minutes. Find x
3 sin 2x = have in this domain?
(i) the temperature after 15 minutes 2
(ii) when it cools down to 20C. (b) Solve 2 sin x - 1 = 0 for 0 x 360.
(c) If logx 3 = p and logx 2 = q, write in
QUESTION 7 terms of p and q
(a) (i) Find the area bounded by the (i) logx 12
curve y = tan x, the x-axis and the lines (ii) logx 2x
(d) A stack of oranges has 1 orange at
x = 0 and x = , by using Simpsons rule
4 the top, 3 in the next row down, then
with 5 function values (to 2 decimal
each row has 2 more oranges than the
places).
previous one.
SAMPLE EXAMINATION PAPERS 357

(i) How many oranges are in the (iii) Find the value of k if the tangent
12th row? has an x-intercept of 2.
(ii) If there are 289 oranges stacked (e) Find the area of the sector to
altogether, how many rows are there? 1 decimal place.

QUESTION 9
(a) The diagram below shows the graph
of a function y = f (x).

QUESTION 10
(a) The graph of y = (x + 2)2 is drawn
below. Copy the graph into your writing
booklet.

(i) Copy this diagram into your


writing booklet.
(ii) On the same set of axes, draw a
sketch of the derivative f l(x) of the
function.
(b) A bag contains yellow, blue and
white marbles. Therefore if I choose one
(i) Shade the region bounded by the
marble at random from the bag, the
1 curve, the x-axis and the line x = 1.
probability that it is blue is . (ii) This area is rotated about the
3
Is this statement true or false? Explain x-axis. Find the volume of the solid of
why in no more than one sentence. revolution formed.
(c) Solve the equation 2 ln x = ln (2x + 3). (b) The accountant at Acme Business
Solutions calculated that the hourly cost
(d)
of running a business car is s2 + 7500
cents where s is the average speed of
the car. The car travels on a 3000 km
journey.
(i) Show that the cost of the journey
7500
is given by C = 3000 b s + s l.
The diagram shows the graph of y = ex (ii) Find the speed that minimises
and the tangent to the curve at x = k. the cost of the journey.
(i) Find the gradient of the tangent (iii) Find the cost of the trip to the
at x = k. nearest dollar.
(ii) Find the equation of the tangent
at x = k.
358 Maths In Focus Mathematics HSC Course

Answers
Chapter 1: Geometry 2 8. +OCA = +OCB = 90 (given)
OA = OB (equal radii)
Exercises 1.1 OC is common
1. +ABE = 180 +ABD (straight angle 180 ) ` by RHS, OAC / OBC
+CBE = 180 +CBD (straight angle) ` AC = BC
= 180 +ABD (+ABD =+CBD given) (corresponding sides in congruent s)
= +ABE ` OC bisects AB

2. +DFB = 180 (180 x) (+AFB is a straight angle) 9. +CDB = +BEC = 90 (altitudes given)
=x +ACB = +ABC (base angles of isosceles )
` +AFC = x (vertically opposite angles) CB is common
+CFE = 180 (x + 180 2x) ` by AAS, CDB / BEC
(+AFB is a straight angle)
` CE = BD
=x (corresponding sides in congruent s)
` +AFC = +CFE
` CD bisects +AFE 10. AB = AD (given)
BC = DC (given)
3. +WBC + +BCY = 2x + 115 + 65 2x
AC is common
= 180
` by SSS, ABC / ADC
These are supplementary cointerior angles.
` +DAC = +BAC
`VW < XY
(corresponding angles in congruent s)
4. x + y = 180 (given) So AC bisects +DAB
` +A + +D = 180 Also +BCA = +DCA
These are supplementary cointerior angles. (corresponding angles in congruent s)
` AB < DC ` AC bisects +DCB
Also +A = +B (similarly)
These are supplementary cointerior angles. 11. (a) +NMO = +MOP (alternate angles, MN < PO)
` AD < BC +PMO = +MON (alternate angles, PM < ON)
` ABCD is a parallelogram. MO is common
` by AAS,
5. +ADB = +CDB = 110 (given)
MNO / MPO
+ABD = +CBD (BD bisects +ABC)
(b) +PMO = +MON (alternate angles, PM < ON)
BD is common
MN = NO (given)
` by AAS,
+MON = +NMO (base angles of isosceles )
ABD / CBD
` +PMO = +NMO
i.e. +PMQ = +NMQ
6. (a) AB = AE ( given)
+B = +E ( base angles of isosceles ) (c) MN = NO (given)
BC = DE ( given) PM = NO
(corresponding sides in congruent s)
` by SAS, ABC / AED
` PM = MN
(b) +BCA = +EDA
(corresponding angles in congruent s) +PMQ = +NMQ (from (b))
MQ is common
+ACD = 180 +BCA (BCD is a straight angle)
` by SAS, PMQ / NMQ
= 180 +EDA
= +ADC (d) +MQN = +MQP
` since base angles are equal, ACD is isosceles (corresponding angles in congruent s)
But +MQN ++MQP = 180 (+PQN straight angle)
7. DC = BC ( given)
` +MQN = +MQP = 90
+B = +D = 90 ( given)
1 12.
DM = AD ( given)
2
1
and BN = AB
2
` DM = BN
` by SAS, MDC / NBC
` MC = NC
(corresponding sides in congruent s)
ANSWERS 359

Let ABCD be a parallelogram with diagonal AC. 19.


+DAC = +ACB (alternate angles, AD < BC)
+BAC = +ACD (alternate angles, AB < DC)
AC is common
` by AAS, ADC / ABC

13.
Let ABCD be a rectangle
AD = BC (opposite sides in < gram)
+D = +C = 90
DC is common
` by SAS, ADC / BCD
ADC / ABC (see question 12) ` AC = DB
` +ADC = +ABC (corresponding angles in congruent s) (corresponding sides in congruent s)
Similarly, by using diagonal BD we can
prove +A = +C 20.
` opposite angles are equal

14. AB = DC (opposite sides in < gram)


BM = DN (given)
` AB BM = DC DN
i.e. AM = NC Let ABCD be a rectangle with +D = 90
Also AM < NC (ABCD is a < gram)
` +C = 180 90
Since one pair of sides is both parallel and (+D and +C cointerior angles, AD < BC)
equal, AMCN is a parallelogram.
= 90
+B = 180 90
15. AD = BC (opposite sides in < gram) (+B and +C cointerior angles, AB < DC)
BC = FE (similarly) = 90
` AD = FE +A = 180 90
Also AD < BC (ABCD is a < gram) (+A and +B cointerior angles, AD < BC)
and BC < FE (BCEF is a < gram) = 90
` AD < FE ` all angles are right angles
Since one pair of sides is both parallel and equal, AFED
is a parallelogram. 21. AD = CD (given)
AD = BC (opposite sides of < gram)
16. +DEC = +DCE (base angles of isosceles )
Also AB = CD (similarly)
Also, +DEC = +ECB (alternate angles, AD < BC)
` AB = AD = BC = CD
` +DCE = +ECB
` all sides of the rhombus are equal
` CE bisects +BCD

22.
17. AB = CD (given)
+BAC = +DCA (given)
AC is common
` by SAS, ABC / ADC
` AD = BC
(corresponding sides in congruent s)
Since two pairs of opposite sides are equal, Construct BE < AD
ABCD is a parallelogram. Then AD = BE (opposite sides of < gram)
But AD = BC (given)
Then BE = BC
18. (a) AE = EC ` +BCD = +BEC (base angles of isosceles )
(diagonals bisect each other in < gram)
Also, +ADC = +BEC (corresponding angles, AD < BE)
+AEB = +CEB = 90 (given)
` +ADC = +BCD
EB is common
` by SAS, ABE / CBE 23. AD = AB (given)
` AB = BC DC = BC (given)
(corresponding sides in congruent s) AC is common
` by SSS, ADC / C
(b) +ABE = +CBE
(corresponding angles in congruent s)
` +ADC = +ABC
(corresponding angles in congruent s)
360 Maths In Focus Mathematics HSC Course

24. AD = BC (opposite sides of < gram) Exercises 1.3


+D = +C = 90 (given)
1. Show m AB = mCD = 4 and
DE = EC (E is the midpointgiven)
4
` by SAS, ADE / BCE m AD = m BC =
7
` AE = BE
(corresponding sides in congruent s) 7 4
2. Show m AC m BC = = 1;
4 7
right-angled triangle with +C = 90
25. (a) AD = BC (opposite sides of < gram)
AB = DC (similarly) 3. (a) AB = AC = 73 , BC = 6 (b) 8 units (c) 24 units2
DB is common
1
` by SSS, ADB / BCD 4. Show m XY = mYZ =
5
(b) +ABE = +CBE
(corresponding angles in congruent s) 5. (a) Show AB = AD = 26 ,
(c) AB = BC (adjacent sides in rhombus) BC = CD = 80
+ABE = +CBE (found) (b) Show m AC m BD = 1
BE is common (c) E = (1, 2), CE = 72 = 6 2,
` by SAS, ABE / CBE AE = 18 = 3 2
(d) +AEB = +BEC
(corresponding angles in congruent s) 6. r =1
But +AEB + +BEC = 180 (AEC is a straight angle)
7. (a) 2x 3y + 13 = 0
` +AEB = +BEC = 90
(b) Substitute (7, 9) into the equation
(c) Isosceles

Exercises 1.2 8. +AOB = +COD = 90


1. (a) 14 452 mm2 (b) 67 200 mm3 2. 3
90 m OD 4
= =2
OB 2
3. V = 2x3 + 3x2 2x OC 14
= =2
OA 7
4. V = r2 h OD OC
` =
250 = r2 h OB OA
250 Since 2 pairs of sides are in proportion and their
= r2
h included angles are equal, OAB<;OCD.
250
=r
h 9. (a) OB is common
OA = BC = 5
3b 2
5. V = 5 r3 6. A = 7. V = ] x + 2 g3 AB = OC = 20
2
= x3 + 6x2 + 12x + 8 ` by SSS OAB / OCB
8. S = 24 h2 1
(b) Show mOA = m BC = 1 and m AB = mOC = 2
3
9. V = x ] 3 2x g2
= 4x3 12x2 + 9x 10. +ABC = 90 and AB = BC = 2
So ABC is isosceles
10. 262 cm3 11. V = 3h2 + 2h ` +CAB = +ACB
But +CAB + +ACB = 90 (angle sum of triangle)
12. V = 2h2 + 5h
` +CAB = +ACB = 45
1 Similarly, other angles are 45.
13. V = (6x3 5x2 34x 15)
3
11. PR = QS = 145
14. (a) V = 18x3 12x2 + 2x (b) S = 54x2 30x + 4
Since diagonals are equal, PQRS is a rectangle.
15. l = h2 + r2
12. (a) X = (2, 2), Y = ^ 1, 0 h
x2 y 400 750 r2 (b) m XY = m BC = 2
16. V = 17. h = 18. h =
4 r 2 r So XY < BC
(c) XY = 5 , BC = 20 = 2 5
850 r2
19. l = So BC = 2XY
r

20. y = 810 000 x2


ANSWERS 361

13. m AC m BD = 1 1 = 1 50 r2
4. h=
So AC and BD are perpendicular. r
a a
Midpoint AC = midpoint BD = d , n 5. (a) +ADE = 45 (corresponding+s BC < AD)
2 2
So AC and BD bisect each other. +EAD = 90 45 (+ sumof )
So AC and BD are perpendicular bisectors. = 45
So ADE is isosceles.
14. (a) Distance from X = distance from Y = 1 unit (b) AE = DE =y (isoceles )
1 CD = y (CD = DE, given)
(b) Z = d , 0 n
4 AB = y (opposite sides in < gram)
1 AB < CE (given)
(c) 1 units2
4
So ABCE is a trapezium.
1 1
15. Midpoint AB: W = d 2, 1 n A = h (a + b)
2 2
Midpoint BC: X = ^ 2, 3 h 1
= y (y + 2y)
1 1 2
Midpoint CD: Y = d 4 , n 1
2 2 = y 3y
1 2
Midpoint AD: Z = d , 2 n
2 3y 2
3 =
mWX = mZY = 2
8
So WX < ZY CB CG
6. (a) = (equal ratio of intercepts)
BA GF
7
m XY = mWZ = CG CD
5 = (similarly)
GF DE
So XY < WZ CB CD
WXYZ is a parallelogram. ` =
BA DE
(b) 20.4 cm
Test yourself 1
4 4
1. (a) AB = AC (given)
7. m BC = =0
2 5
So BD = EC (midpoints) 4 3
mCD = =7
+DBC = +ECB (base +s in isoceles T) 56
BC is common 33
m AD = =0
`BEC / BDC (SAS) 1 6
(b) ` BE = DC (corresponding sides in /Ts) 4 3
m AB = =7
2 1
2. BC < AD, CD < AB
So ABCD is a parallelogram.
1y
2 8. DC2 + BC2 = 122 + 52
1x
2 = 144 + 25
= 169
c2 = a2 + b2 = 132
2
x 2 eyo = DB2
=d n +
2 2 ` +C = 90 (Pythagoras)

x2 y
2 So ABCD is a rectangle.
= +
4 4 9.18 5.13
x2 + y2 9. = = 2.7
= 3.4 1.9
4 +Y = +P = 39 (given)
x2 + y2 ` PQR <; WXY
c=
4 (two pairs of sides in proportion, with included+s equal)
x2 + y2
= 10. Let +BAC =x
2 Then +DAC =x (givenAC bisects +BAD)
+DCA =x (base+s of isosceles DAC)
3. AB = (7 4)2 + (5 1)2
` +BAC = +DCA
=5
BC = (7 1)2 + ( 5 3)2
= 10
AC = (4 1)2 + (1 3)2
=5
Since AB = AC BC, ABC is isosceles.
362 Maths In Focus Mathematics HSC Course

These are equal alternate angles. Challenge exercise 1


` AB < EC
1. BD is common
+EDA = 2x (exterior+of DAC)
+DEA = +DAE (base+s of isosceles AED)
+ADB = +CDB = 90 (given)

180 2x AD = DC (BD bisects ACgiven)


+DAE = (+ sum of isosceles ) ` by SAS, ABD / CBD
2
= 90 x ` AB = BC (corresponding sides in congruent s)
+EAC = 90 x + x So ABC is isosceles.
= 90 1
2. (a) AD = AB
` +EAC = +ACB 2
These are equal alternate angles. AD 1
` =
` AE < CB AB 2
So ABCE is a parallelogram. 1
AE = AC
2
11. ^ 1, 0 h AE 1
` =
AC 2
12. (a) 4x 3y 1 = 0 AD AE
(b) 2.4 units ` =
AB AC
(c) 12 units2 +A is common
Since two pairs of sides are in proportion and their
13. (a) AB = AD (given)
included angles are equal,
BC = DC (given)
ADE <; ABC
AC is common
` +ADE = +ABC
` ABC / ADC (SSS)
These are equal corresponding angles
(b) AB = AD (given)
` DE < BC
+BAE = +DAE (corresponding +s in / s)
AE is common (b) Since ADE <; ABC,
` ABE / ADE (SAS) AD DE 1
= =
AB BC 2
(c) BE = DE (corresponding sides in / s)
DE 1
` AC bisects BD ` =
BC 2
+BEA = +DEA (coresponding+s in / s)
1
But +BEA + +DEA = 180 (straight +) ` DE = BC
2
` +BEA = +DEA = 90
So AC is perpendicular to BD.
3.

1
14. (a) AB: m1 = , BC: m2 = 2
2
1
m1 m2 = 2 = 1
2
So AB and BC are perpendicular.
(b) (3, 2) Let ABCD be a rhombus with AD = DC
1
(c) (3, ) To prove:
2
+ADE = +CDE
(d) 5 units
Proof
15. (a) 500 = 4x2 + 6xh AD = DC (given)
500 4x2 = 6xh ` ADC is isosceles
250 2x2 = 3xh ` +DAE = +DCE
250 2x2 AE = EC (diagonals bisect each other)
=h
3x ` by SAS, ADE / CDE
(b) V = 2x2 h ` +ADE = +CDE
250 2x2 (corresponding angles in congruent s)
= 2x 2 e o
3x
250 2x2 (Note: We can prove other pairs of angles equal
= 2x e o
3 similarly.)
500x 4x 3
= 4. 1189 mm 841 mm
3
4000
5. S = 2x 2 +
x
ANSWERS 363

(n 2) 180 AE is common
6. Each angle = ` by SAS, ADE / ABE
n
(180n 360) ` AB = AD (corresponding sides in congruent s)
=
n
360 CP CR 1
= c 180 m 10. = = (P and R are midpoints)
n PA RB 1
` PR < AB (equal ratios on < lines)
7.
Similarly PQ < CB and QR < AC
+QPR = +PRC (alternate +s, PQ < CB)
+CPR = +PRQ (alternate +s, AC < RQ)

PR is common
` by AAS, PQR / CPR

11. 188 mm
+EAF = +ACD (alternate angles, AB < DC)
DP DS 1
+ADC = +ABC (opposite angles in < gram) 12. (a) = = (P, S are midpoints)
DA DC 2
+ABC = +AEF (corresponding angles, BC < EF)
+D is common
` +ADC = +AEF
Since 2 pairs of sides are in proportion and the included
Since 2 pairs of angles equal, third is equal by angle sum angles are equal,
of DPS <; DAC.
` AEF <; ADC DP DS 1
(b) = =
PA SC 1
8.
` PS < AC (equal rations on < lines)
BQ BR
Similarly, =
QA RC
` QR < AC
` PS < QR
AP AQ 1
(c) = = (P, Q are midpoints)
PD QB 1
` PQ < DB (equal ratios on < lines)

Similarly SR < DB
a + a b + b
(a) Midpoint of BD: d , n = (a, 0) `PQ < SR
2 2 Since PS < QR, PQ < SR.
0 + 2a 0 + 0 PQRS is a parallelogram.
Midpoint of AC: d , n = (a, 0)
2 2
` diagonals BD and AC bisect each other at E(a, 0) 13. 70 cm
BD is vertical and AC is horizontal
`the diagonals are perpendicular
Chapter 2: Geometrical applications
(b) AB = (a 0) 2 + (b 0)2 of calculus
= a2 + b2
BC = (2a a)2 + (0 b) 2 Problem
= a2 + b2
^ 0.25, 1.125 h
CD = (2a a)2 + (0 + b)2
= a2 + b2
Exercises 2.1
AD = (a 0)2 + ( b 0)2
= a2 + b2 1. (a)
` all sides are equal

(c) BC = CD (from (a))


BE = ED (from (a))
CE is common
` by SSS CBE / CDE
` +BCE = +DCE (corresponding +s in congruent s)
So AC bisects +BCD

9. DE = BE (digonals bisect in < gram)


+AEB = +AED = 90 (given)
364 Maths In Focus Mathematics HSC Course

(b) 17. y

x
2

(c)

18. y

(d)
x
4

19. y

1
2. x< 3. x < 0
4
4. (a) x < 1.5 (b) x > 1.5 (c) x = 1.5
x
5. f l(x) = 2 < 0 for all x 1

6. yl= 3x2 > 0 for all x 0

7. (0, 0) 8. x = 3, 2 9. (a) (1, 4) (b) (0, 9)


(c) (1, 1) and (2, 0) (d) (0, 1), (1, 0) and (1, 0) 20. y

10. (2, 0) 11. 1 < x < 1 12. x < 5, x > 3

13. (a) x = 2, 5 (b) 2 < x < 5 (c) x < 2, x > 5

14. p = 12
x
1 -2 5
15. a = 1 , b = 6
2

dy
16. (a) = 3x2 6x + 27
dx
(b) The quadratic function has a > 0
b2 4ac = 288 < 0 21. y
So 3x2 6x + 27 > 0 for all x
The function is monotonic increasing for all x.

x
3
ANSWERS 365

22. y Exercises 2.3


1. 7x6 10x4 + 4x3 1; 42x5 40x3 + 12x2;
210x4 120x2 + 24x; 840x3 240x + 24

x 2. f m(x) = 72x7 3. f l(x) = 10x4 3x2, f m(x) = 40x3 6x


-2
-1 4. f l(1) = 11, f m( 2) = 168

5. 7x6 12x5 + 16x3; 42x5 60x4 + 48x2; 210x4 240x3 + 96x

dy d2 y
6. = 4x 3, =4
2 13 dx dx2
23. (2, 0) and c , 3 m
3 81
7. f l(1) = 16, f m(2) = 40 8. 4x 5, 20x 6
x 2 2 3
3x + 2
24. + x+1 = ; f , p 1 d2 h
2 x+1 2 x+1 3 9 9. g m(4) = 10. = 26 when t = 1
32 dt2
25. a = 1.75
7 1
11. x = 12. x >
1 3 18 3
26. !0 27. !0
2 x x4
13. 20 (4x 3)4 ; 320 (4x 3)3

1
Exercises 2.2 14. f l(x) = ;
2 2x
1. (0, 1); yl< 0 on LHS, yl> 0 on RHS 1
f m(x) =
4 (2 x) 3
2. (0, 0) minimum 3. (0, 2) inflexion
16 96
15. f l^ x h = ; f m(x) =
4. ( 2, 11); show f l(x) > 0 on LHS and f l(x) < 0 on RHS. (3x 1) 2 (3x 1) 3

5. (1, 2) minimum 6. (4, 0) minimum d2 v 2 3 3 2


16. = 24t + 16 17. b = 18. f m(2) = =
dt2 3 4 2 8
7. (0, 5) maximum, (4, 27) minimum
19. f m(1) = 196 20. b = 2.7
8. f l(0) = 0, f l(x) > 0 on LHS and RHS
Exercises 2.4
9. (0, 5) maximum, (2, 1) minimum
1
10. (0, 3) maximum, (1, 4) minimum, (1, 4) minimum 1. x> 2. x < 3 3. y m = 8 < 0 4. y m = 2 > 0
3
11. (1, 0) minimum, (1, 4) maximum 1
5. x <2 6. (1, 9) 7. (1, 17) and (1, 41)
3
1
12. m = 6 8. (0, 2); y m < 0 on LHS, y m > 0 on RHS 9. 2 < x < 1
12

13. x = 3 minimum 14. x = 0 minimum, x = 1 maximum 10. (a) Nominimum at (0, 0)


(b) Yesinflexion at (0, 0)
15. x = 1 inflexion, x = 2 minimum (c) Yesinflexion at (0, 0)
(d) Yesinflexion at (0, 0)
dP 50 (e) Nominimum at (0, 0)
16. (a) =2 2
dx x
(b) (5, 20) minimum, ^ 5, 20 h maximum 11. y

1
17. d 1, n minimum
2
18. (2.06, 54.94) maximum, ^ 20.6, 54.94 h minimum

19. (4.37, 54.92) minimum, ^ 4.37, 54.92 h maximum


dA x2 x
20. (a) = 3600 x2
dx 3600 x2
3600 2x2
=
3600 x2
(b) (42.4, 1800) maximum, ^ 42.4, 1800 h minimum
366 Maths In Focus Mathematics HSC Course

12. y 3.

x
1

4. (a)

13. None: (2, 31) is not an inflexion since concavity does


not change.
12
14. f m(x) =
x4
x4 > 0 for all x ! 0
12
So > 0 for all x ! 0 (b)
x4
So the function is concave upward for all x ! 0.

15. (a) (0, 7), (1, 0) and ^ 1, 14 h (b) (0, 7)


d2 y
16. (a) = 12x2 + 24
dx2
x2 0 for all x
So 12x2 0 for all x
12x2 + 24 24
(c)
So 12x2 + 24 0 and there are no points of inflexion.
(b) The curve is always concave upwards.

17. a = 2 18. p = 4 19. a = 3, b = 3

20. (a) ^ 0, 8 h, ^ 2, 2 h
dy
(b) = 6x5 15x4 + 21
dx
dy
At ^ 0, 8 h: = 6 ] 0 g5 15 ] 0 g4 + 21
dx (d)
= 21
0
dy
At ^ 2, 2 h : = 6 ] 2 g5 15 ] 2 g4 + 21
dx
= 27
0
So these points are not horizontal points of inflexion.

Exercises 2.5
5.
dy d2 y dy d2 y
1. (a) > 0, > 0 (b) < 0, <0
dx dx2 dx dx2
dy d y
2
dy d2 y
(c) > 0, < 0 (d) < 0, >0
dx dx2 dx dx2
dy d2 y
(e) > 0, >0
dx dx2

dP d2 P
2. (a) > 0, < 0 (b) The rate is decreasing.
dt dt2
ANSWERS 367

6. Exercises 2.7
1.

dM d2 M
7. < 0, >0
dt dt2

8. (a) The number of fish is decreasing.


(b) The population rate is increasing.
(c)
2.

9. The level of education is increasing, but the rate is


slowing down.

10. The population is decreasing, and the population rate is


decreasing.

Exercises 2.6
3.
1. (1, 0) minimum 2. (0,1) minimum

3. (2, 5), y m = 6 > 0 4. (0.5, 0.25), y m < 0 so maximum

5. (0, 5); f m(x) = 0 at (0, 5), f m(x) < 0 on LHS, f m(x) > 0 RHS

6. Yesinflexion at (0, 3)

7. ^ 2, 78 h minimum, ^ 3, 77 h maximum

8. (0, 1) maximum, ^ 1, 4 h minimum, 4.


^ 2, 31 h minimum

9. (0, 1) maximum, (0.5, 0) minimum,


^ 0.5, 0 h minimum

10. (a) (4, 176) maximum, (5, 175) minimum


(b) (4.5, 175.5)

11. (3.67, 0.38) maximum

12. ^ 0, 1 h minimum, ^ 2, 15 h maximum,


^ 4, 1 h minimum
2
13. (a) a = (b) maximum, as y m < 0
3

1
14. m = 5 15. a = 3, b = 9
2
368 Maths In Focus Mathematics HSC Course

5. 9.

6.

10.

7.

11.

8. (a) ^ 0, 7 h minimum, ^ 4, 25 h maximum


(b) ^ 2, 9 h
(c)
12.
ANSWERS 369

13. 2. Maximum value is 9, minimum value is 7.

dy 2
14. = 0
dx (1 + x) 2
3. Maximum value is 25.

4. Maximum value is 86, minimum value is 39.

5. Maximum value is 2.
15. 1
6. Maximum value is 5, minimum value is 16 .
3

7. Absolute maximum 29, relative maximum 3, absolute


Exercises 2.8
minimum 35, relative minimum 35, 8

1. Maximum value is 4. 8. Minimum 25, maximum 29


370 Maths In Focus Mathematics HSC Course

9. Maximum 3, minimum 1 4. V = r2h


400 = r2 h
400
=h
r2
S = 2 r2 + 2 rh
400
= 2 r 2 + 2 r e o
r2
800
= 2 r 2 +
r

5. (a) x + y = 30
` y = 30 x
(b) The perimeter of one square is x, so its side is
10. Maximum , minimum 1 1
x. The other square has side y.
4 4
1 2 1 2
A =d xn + d yn
4 4
x2 y2
= +
16 16
x + y2
2

=
16
x2 + (30 x)2
=
16
x2 + 900 60x + x2
=
16
2x2 60x + 900
=
16
2 (x2 30x + 450)
=
Problem 16
x2 30x + 450
30 =
The disc has radius cm. (This result uses Stewarts 8
7
theoremcheck this by research.)
6. (a) x2 + y2 = 2802 (b) A = xy
= 78 400 = x 78 400 x2
Exercises 2.9 y2 = 78 400 x2
y= 78 400 x2
1. A = xy
50 = xy 7.
50
` =y
x
P = 2 x + 2y
50
= 2x + 2
x
100 V = x (10 2x) (7 2x)
= 2x +
x = x (70 20x 14x + 4x2)
= x (70 34x + 4x2)
2. 2x + 2y = 120 = 70x 34x2 + 4x3
2y = 120 2x
y = 60 x 8. Profit per person = Cost Expenses
A = xy = (900 100x) (200 + 400x)
= x (60 x) = 900 100x 200 400x
= 60x x2 = 700 500x
For x people, P = x (700 500x)
3. xy = 20
= 700x 500x2
20
y=
x
S =x+y
20
=x+
x
ANSWERS 371

9. (b) 63.2 m by 63.2 m


(c) $12 332.89

5. 4 m by 4 m 6. 14 and 14 7. -2.5 and 2.5

8. x = 1.25 m, y = 1.25 m

9. (a) V = x (30 2x) (80 2x)


= x (2400 220x + 4x2)
= 2400x 220x2 + 4x3
2
(b) x = 6 cm
3

After t hours, Joel has travelled 75t km. He is (c) 7407.4 cm3
700 75t km from the town.
10. V = r2h = 54
After t hours, Nick has travelled 80t km. He is
54
680 80t km from the town. h=
r 2
d= (700 75t)2 + (680 80t)2 54
= 2
= 490 000 105 000t + 5625t2 + 462 400 r
108 800t + 6400t2 S = 2r (r + h)
= 952 400 213 800t + 12 025t2 54
= 2 r d r + 2 n
r
10. The river is 500 m, or 0.5 km, wide Distance AB: 108
= 2 r +
2
r
d=
x2 + 0.52
=
x2 + 0.25 Radius is 3 m.
distance
Speed = 17 200
time 11. (a) S = 2 r2 + (b) 2323.7 m2
r
distance
` Time =
speed 12. 72 cm2
x2 + 0.25
t= 13. (a) xy = 400
5
400
` y=
Distance BC: x
d =7x A = (y 10) (x 10)
distance = xy 10y 10x + 100
Time = 400 400
speed = xc m 10 c m 10x + 100
x x
7x
t= 4000
4 = 400 10x + 100
x
So total time taken is:
4000
x2 + 0.25 7 x = 500 10x
t= + x
5 4
(b) 100 cm2

Exercises 2.10 14. 20 cm by 20 cm by 20 cm 15. 1.12 m2

1. 2 s, 16 m 2. 7.5 km 16. (a) 7.5 m by 7.5 m (b) 2.4 m

3. 2x + 2y = 60 1
17. 301 cm2 18. 160 cm2 19. 1.68 cm, 1.32 cm
2y = 60 2x 6
y = 30 x (1) 20. d2 = (200 80t) 2 + (120 60t) 2
A = xy = 40 000 32 000t + 6400t2
= x (30 x) from (1) + 14 400 14 400t + 3600t2
= 30x x2 = 10 000t2 46 400t + 54 400
Max. area 225 m2 24 km
1
4. (a) A = xy = 4000 21. (a) d = ] x2 2x + 5 g ] 4x x2 g (b) unit
2
4000 = x 2 x + 5 4x + x
2 2

` y= (1) = 2x 6x + 5
2
x
P = 2x + 2 y
4000
= 2x + 2 c m from (1)
x
8000
= 2x +
x
372 Maths In Focus Mathematics HSC Course

1 y (b) 95 km/h
22. (a) Perimeter = 2x + y + (2r) where r =
2 2 (c) $2846
1 y
1200 = 2x + y + e 2 o
2 2
y Exercises 2.11
= 2x + y +
2 x3
1. (a) x2 3x + C (b) + 4x 2 + x + C
y 3
1200 y = 2x
2 x6 x3
(c) x4 + C (d) x2 + x + C
y y 6 3
600 =x
2 4 (e) 6x + C
2400 2y y
=x
4 x2 x5
2. (a) f (x) = 2x3 +C (b) f (x) = x 3 + 7x + C
1 2 2 5
(b) A = xy + r x2 x3
2 (c) f (x) = 2x + C (d) f (x) = x 2 3x + C
2400 2y y y2 2 3
=e oy + 1 e o 3
4 2 2 2x 2
2400y 2y2 y2 y2 (e) f (x) = +C
= + 3
4 8
4800y 4y 2y
2 2
y 2 x 3
3. (a) y = x5 9x + C (b) y = + 2x 1 + C
= + 3
8 8
x4 x3 2
4800y 4y2 y2 (c) y = +C (d) y = + C
= 20 3 x
8
x4 x2
(e) y = +x+C
(c) x = 168 m, y = 336 m 4 3

23. (a) Equation AB: 2 x3 x 2


y = mx + b 4. (a) +C (b) +C
3 2
b
= x+b 1 1 1
a (c) +C (d) 2x 2 + 6x 3 + C
7x 7
Substitute ^ 1, 2 h x 6
(e) + 2x 1 + C
b 6
2 = (1) + b
a
b x4 1
= +b 5. y= x 3 + 5x 6. f (x) = 2x2 7x + 11
a 4 4
2a = b + ab
1
= b ] 1 + a g 7. f (1) = 8 8. y = 2x 3x2 + 19 9. x = 16
3
= b ]a 1 g
2a 10. y = 4x2 8x + 7 11. y = 2x3 + 3x2 + x 2
=b
a1
12. f (x) = x3 x2 x + 5 13. f (2) = 20.5
(b) a = 2, b = 4
x3 x2 1 4x 3 1
14. y = + 12x + 24 15. y = 15x 14
24. 26 m 3 2 2 3 3

d x3 2
25. (a) s= 16. y = 2 x 2 + 3x 4 17. f (x) = x4 x3 + 2x2 + 4x 2
t 3 3
d
So t =
s 18. y = 3x2 + 8x + 8 19. f ] 2 g = 77 20. y = 0
1500
=
s
Cost of trip taking t hours:
Test yourself 2
C = ] s2 + 9000 g t 1. ( 3, 11) maximum, (1, 15) minimum
1500
= ] s2 + 9000 g
s 1
2. x >1 3. y = 2x3 + 6x2 5x 33
9000 1500 6
= 1500s +
s
9000 4. (a) -8 (b) 26 (c) -90 5. 50 m
c
= 1500 s + m
s
6. (0, 0) minimum 7. x > 1
ANSWERS 373

8. (a) (0, 1) maximum, ( 4, 511) minimum, 14. x < 3 15. y = x3 + 3x2 + 3 16. 150 products
(2, 79) minimum
dy
(b) 17. For decreasing curve, <0
dx
dy
= 3x 2
dx
<0 (since x2 > 0 for all x 0)
` monotonic decreasing function

18. (a)

1 5x 3
9. c , 1 m 10. f (x) = + 6x2 49x + 59
2 2

11. (a) V = r 2 h
375 = r 2 h (b)
375
=h
r 2
S = 2r2 + 2rh
375
= 2 r 2 + 2 r d n
r 2
750
= 2 r 2 +
r
(b) 3.9 cm
(c)
12. (a) (0, 0) and (1, 1)
(b) (0, 0) minimum, (1, 1) point of inflexion
(c)

19. (a) x2 + y2 = 52 = 25
y2 = 25 x2
y= 25 x2
13. (a)
1
A= xy
2
1
= x 25 x2
2
(b) 6.25 m2
S = 2x2 + 4xh
250 = 2x2 + 4xh 20. (a) (4, 171) minimum, ( 6, 329) maximum
250 2x2 = 4xh (b) (1, 79)
250 2x2 (c)
=h
4x
2 (125 x ) 2

=h
4x
125 x 2
=h
2x
V =x h2

125 x2
= x2 d n
2x
x (125 x ) 2

=
2 21. x < 1
125x x3
=
2 22. f ] x g = 2x3 3x2 31x + 68

(b) 6.45 cm 6.45 cm 6.45 cm 23. (0, 1) and ^ 3, 74 h


374 Maths In Focus Mathematics HSC Course

24. 179 10. r = 3.17 cm, h = 6.34 cm


x3
25. y 11. y = x2 15x 1
3
12. 110 km/h

13. y = x2 + 2x + 3 (There may be other solution.)


x 4 2x 3 3x 2
14. (a) f (x) =
4 3 2
1 7
(b) (0, 0), c 3, 11 m, c 1, m
4 12
x
2 (c)

Challenge exercise 2
20x2 120x 1
1. ; 15. 4 m 4 m 4 m
(4x2 + 1)4
8(60x3 420x2 9x + 15) 1
16. f (3) = 22 17. (a) -2 (b) -1
(4x2 + 1)5 6
18. yl= 0 at (0, 0);
2.
(a) y m > 0 on LHS and RHS
(b) y m < 0 on LHS, y m > 0 on RHS
1
19. 21 cm3 20. (a) (0, 1) (b) k = 2, 4, 6, 8, . . .
3
1
21. minimum 1; maximum
5
22. 87 kmh1

Chapter 3: Integration

Exercises 3.1
1 1. 2.5 2. 10 3. 2.4 4. 0.225 5. (a) 28 (b) 22
3. x < ,x >4
2 6. 0.39 7. 0.41 8. 1.08 9. 0.75 10. 0.65
11. 0.94 12. 0.92 13. 75.1 14. 16.5 15. 650.2
4. 16 m2

5. 27; -20.25
Exercises 3.2
6. f l(0.6) = f m(0.6) = 0 and concavity changes 1. 48.7 2. 30.7 3. 1.1 4. 0.41 5. (a) 3.4475
(b) 3.4477 6. 2.75 7. 0.693 8. 1.93 9. 72
7. Show sum of areas is least when r = s = 12.5 10. 5.25 11. 0.558 12. 0.347 13. 3.63 14. 7.87 15. 175.8
5
8. 25
6 Exercises 3.3
dy 1
9. (a) = 0 1
dx 2 x1 1. 8 2. 10 3. 125 4. -1 5. 10 6. 54 7. 3
3
(b) Domain: x 1; range: y 0 2 2 1
(c) 8. 16 9. 50 10. 52 11. 12. 21 13. 0
3 3 4
2 1 1 1
14. 4 15. 1 16. 4 17. 0 18. 2 19. 0
3 4 3 3
2 1 3 2 1
20. 6 21. 101 22. 12 23. 22 24. 2
9 4 4 3 3
25. 0.0126
ANSWERS 375

1 1 2 1 1
Exercises 3.4 2. (a) 288.2 (b) 1 (c) (d) 60 (e) (f)
4 8 3 6 7
x3 x6 2x 5 m2 2 1 1 3
1. +C 2. +C 3. +C 4. +m+C (g) 4 (h) (i) 1 (j)
3 2 5 2 3 8 5 5

t3 h8 y2
5. 7t + C 6. + 5h + C 7. 3y + C
3 8 2 Exercises 3.6
1 2
b3 b2 a4 a2 1. 1 units2 2. 36 units2 3. 4.5 units2 4. 10 units2
8. x 2 + 4x + C 9. + +C 10. a+C 3 3
3 2 4 2
x3 1
11. + x 2 + 5x + C 12. x4 x3 + 4x2 x + C 5. units2 6. 14.3 units2 7. 4 units2 8. 0.4 units2
3 6

x5 x4 x 8 3x 7 1
13. x6 + + +C 14. 9x + C 9. 8 units2 10. 24.25 units2 11. 2 units2 12. 9 units2
5 2 8 7 3
2 1 2 1
x4 x3 x2 x6 x4 13. 11 units2 14. units2 15. units2 16. units2
15. + 2x + C 16. + + 4x + C 3 6 3 3
2 3 2 6 4
1
17. 5 units2 18. 18 units2 19. = 3.14 units2
4x 3 5x 2 3x 5 x 4 x 2 3
17. 8x + C 18. + +C
3 2 5 2 2
a4
20. units2
3x 4 5x 3 x 2 2
19. + 4x + C 20. x 3 2x 1 + C
2 3 2
4

1 3x 3 x4
21. 7 + C 22. +C 23. x3 + x2 + C Exercises 3.7
7x 4 4
1 2
4x 3 x 3 3x 2 3 1. 21 units2 2. 20 units2 3. 4 units2
24. x 2x2 + +C 25. + 10x + C 26. +C 3 3
3 3 2 x
1 1
1 4 3 7 4. 1.5 units2 5. 1 units2 6. 2 units2
27. +C 28. x+ 2 4 +C 4 3
2x 2 x 2x 4x
2 1 7
y3 y 6 7. 10 units2 8. units2 9. 3 units2
t4 t3 3 6 9
29. + + 5y + C 30. 2t2 + 4t + C
3 6 4 3
1
10. 2 units2 11. 11 units2 12. 60 units2
2 x3 1 3 3 x4 4
31. +C 32. 4 + C 33. +C
3 2t 4 1
13. 4.5 units2 14. 1 units2 15. 1.9 units2
2 x 5
2 x 3 3
34. +C 35. +x+C
5 3

Exercises 3.8
Exercises 3.5
1 1 1
(3x 4)3 1. 1 units2 2. 1 units2 3. units2
1. (a) (i) 3x3 12x2 + 16x + C (ii) +C 3 3 6
9
(x + 1)5 (5x 1)10 2 5
4. 10 units2 5. 20 units2 6. 8 units2
(b) +C (c) +C 3 6
5 50
(3y 2)8 (4 + 3x)5 2 2
7. units2 8. 166 units2 9. 0.42 units2
(d) +C (e) +C 3 3
24 15
2 1 1
(7x + 8)13 (1 x)7 10. units2 11. units2 12. units2
(f) +C (g) +C 3 12 3
91 7
2
(2x 5)3 2 (3x + 1) 3 13. 36 units2 14. 2 units2 15. ( 2) units2
(h) +C (i) +C 3
3 9
1
(j) 3 (x + 7) 1 + C (k) +C
16 (4x 5) 2 Problem
3 3
(4x + 3) 4 1
(l) +C (m) 2 (2 x) + C 2 206
16 units3
15
2 (t + 3) 2 (5x + 2)
5 7

(n) +C (o) +C
5 35
376 Maths In Focus Mathematics HSC Course

Exercises 3.9 6. 3 units2 7. 1.1 units2


2 3 1
243 485 8. 2 units2 9. 9 units3 10. 4
units2 11.
1. units3 2. units3 3 4 2
5 3 (7x + 3)12 206
12. + C 13. 3 units2 14. (a) units3
376 39 758 84 15
3. units3 4. units3 5. units3 6. units3
15 7 2 3 2
(b) units3 15. (a) x = y 3 (b) 3 units2
2 3
2 992 5 9
7. units3 8. units3 9. units3 10. units3 5
3 5 3 2 (c) units3
2
27 64 16 385
11. units3 12. units3 13. units3 1 3
2 3 7 16. 36 17. 85 units2 18. units3
3 5
25 65 1023
14. units3 15. units3 16. units3 (2x 1)5 x6
2 2 5 19. (a) +C (b) +C
2 8
5 344
17. units3 18. 13 units3 19. units3
3 27 Challenge exercise 3
3 2 72
20. units3 21. units3 22. units3
5 5 5 1 2
1. (a) units2 (b) units3
12 35
23. y = r2 x2
` y2 = r2 x2 2. (a) Show f ( x) = f (x) (b) 0 (c) 12 units2

V = # y2 dx
b

a
3. 27.2 units3 4. 9 units2 5. (a) 36x3 (x4 1)8

= # ] r2 x2 g dx
r
(x4 1)9
r (b) +C
x3 r 36
= < r2 x F
3 r 22x 1 2
( r)3 6. (a) (b) 7. 7.35 units2 8. units3
r3 (3x2 4)2 8 3
= > d r3 n e r3 oH
3 3
1
2r 3 2r 3 9. f (0) = =
=d n 0
3 3
4 r 3
10. (a)
= units3
3

Test yourself 3

1. (a) 0.535 (b) 0.5


3x 2 5x 2 2 x3
2. (a) + x + C (b) x+C (c) +C
2 2 3
(2x + 5)8 (b) 3.08 units2
(d) +C
16
17 17 215
1 11. units2 12. units3
3. 14.83 4. (a) 2 (b) 0 (c) 2 6 6
5
3x + 6 3 (x + 2) 2x x + 3
13. (a) = (b) +C
5. (a) 2 x+3 2 x+3 3

2 2
14. (a) 6 (b) 6
3 3
5
15. units2
12

8a 2
16. (a) units2
3
(b) (b) 2a3 units3
ANSWERS 377

Practice assessment task set 1 13. AB = AC (given)


BD = CD (given)
1. AD is common.
` by SSS ABD / ACD
` +ADB = +ADC
(corresponding +s in congruent s)
But +ADB + +ADC = 180
(+BDC is a straight +)
` +ADB = +ADC = 90
Let ABCD be a parallelogram with diagonal AC. ` AD = BC
+ACD = +BAC (alternate +s, AB < DC) 7
14. (a) 78.7 units3 (b) 1.57 units3 15. x >
+DAC = +BCA (alternate +s, AD < BC) 9
AC is common 16. f (1) = 3, f l(1) = 2, f m(1) = 18; curve is decreasing and
` by AAS ACD / ACB concave upwards at (1, 3)
` AB = DC and AD = BC
(corresponding sides in congruent s)
` opposite sides are equal
1
2. x< 3. x3 x2 + x + C 4. 24 5. 8 m
2

6. AC = FD (opposite sides of < gram equal)


BC = FE (given)
` AB = AC BC
= FD FE 17. P = 8x + 4y = 4
= ED 4y = 4 8x
y = 1 2x
Also AB < ED (since ACDF is < gram)
` since AB = ED and AB < ED, A = 3x 2
+ y2
ABDE is a parallelogram + ] 1 2x g2
= 3x 2
= 3x 2
+ 1 4x + 4x 2
x9 = 7x 2
4x + 1
7. + 2x 2 + C
3 2 6 3
Rectangle m m, square with sides m
7 7 7
8.
18. f (1) = 0 19. 12 20. 0.837

21. AB2 = 242


= 576
BC2 = 322
= 1024
AC2 = 402
= 1600
AB2 + BC2 = 576 + 1024
= 1600
= AC2

` ABC is right angled at +B (Pythagoras theorem)

(3x + 5)8 1
22. +C 23. 5 24. (1, 1)
24 3
198 96
9. (a) units3 (b) units3
7 5
25. (a) 1.11 (b) 1.17
1
10. 1 units2 11. f l(3) = 20, f m( 2) = 16 12. 68 26. ^ 0, 3 h maximum, (1, 2) minimum, (1, 2) minimum
3
8 5
27. 2 28. (a) 1.58 units2 (b) units3
15 2
378 Maths In Focus Mathematics HSC Course

2 2 43. (c), (d) 44. (a), (b) 45. (c)


29. 12 30. 10 units2
3 3
46. (d) 47. (b) 48. (a) 49. (b) 50. (d)
31. +B is common
+BDC = +ACB = 90 (given)
` ABC <; CBD (AAA) Chapter 4: Exponential and
logarithmic functions
32. Show f l(x) = f m(x) = 0 and f m(x) > 0 on both LHS and
RHS of (0, 0) Exercises 4.1
2 3 1. (a) 4.48 (b) 0.14 (c) 2.70 (d) 0.05 (e) -0.14
33. 2 m 34. f (2) = 16 35. 9 units2
3
2. (a)
36. 119.3 m2 37. f (x) = x3 4x2 3x + 20

38.

(b)

39.

(c)
Let ABCD be a rhombus with AC = x and BD = y.
+AEB = 90
(diagonals perpendicular in rhombus)
1
DE = BE = y
2
(diagonals bisect each other)
1 1 1
ACB has area x y = xy
2 2 4
1 1 1
ADC has area x y = xy
2 2 4
1 1 1
` ABCD has area xy + xy = xy
4 4 2
AB AG 3. (a) 9ex (b) ex (c) ex + 2x (d) 6x2 6x + 5 ex
40. = (equal ratios of intercepts, BG < CD)
AC AD
(e) 3ex ] ex + 1 g 2 (f) 7 ex (ex + 5)6 (g) 4ex ] 2ex 3 g
AG AF
= (equal ratios of intercepts, GF < DE)
AD AE ex (x 1)
(h) ex ] x + 1 g (i) (j) xex ] x + 2 g
AB AF x2
` =
AC AE ex (7x 10)
(k) (2x + 1)ex + 2ex = ex (2x + 3) (l)
(7x 3) 2
2
41. f (2) = 1 5ex 5xex 5 (1 x)
3 (m) =
e 2x ex
xn + 1
42. (a) +C 1
n+1 4. f l(1) = 6 e; f m(1) = 6 e 5. e 6. e 5 =
e5
d
(b) Since (C) = 0, the primitive function 7. 19.81 8. ex + y = 0 9. x + e3 y 3 e6 = 0
dx
could include C.
ANSWERS 379

1 11. y = 3e 2 x
10. c 1, m min
e dy
= 6e 2 x
dx
d2 y
= 12e2x
dx2
d2 y dy
LHS = 3 + 2y
dx 2
dx
= 12e 3 (6e2x) + 2 (3e2x)
2x

= 12e2x 18e2x + 6e2x


=0
= RHS
d2 y dy
` 3 + 2y = 0
dx2 dx

dy d2 y 12. y = aebx
11. = 7ex; = 7e x = y dy
dx dx 2
= baebx
dx
dy d2 y d2 y
12. = 2ex; = 2e x = b2aebx
dx dx2 dx2
y = 2e + 1
x
= b2 y
` y 1 = 2e x
d2 y 13. n = 15
` =y1
dx2
14.

Exercises 4.2
1. (a) 7e7x (b) e x (c) 6e6x 2 (d) 2xex +1
2

(e) (3x + 5) e
2 x 3 + 5x + 7
(f) 5e 5x
(g) 2e 2x
(h) 10e10x

(i) 2e2x + 1 (j) 2x + 2 e1 x (k) 5 (1 + 4e4x) (x + e4x)4


e3x (3x 2)
(l) e2x (2x + 1) (m) (n) x2 e5x (5x + 3)
x3
4e2x + 1 (x + 2) 15.
(o)
(2x + 5)2

2. 28e2x (e2x + 1)5 (7e2x + 1)

3. f (1) = 3e; f m(0) = 9e 2 4. 5


1
5. x + y 1 = 0 6.
3e 3
7. y = 2ex e 8. f m(1) = 18 4e2

9. (0, 0) min; (1, e 2) max

Exercises 4.3
1 2x 1 4x 1 5x
1. (a) e +C (b) e +C (c) e x + C (d) e +C
2 4 5
1 2x 1 4x + 1 3 5x
(e) e +C (f) e +C (g) e +C
2 4 5
1 2t 1 7x x2
(h) e +C (i) e 2x + C (j) ex 3 + +C
2 7 2
dy
10. = 4e 4 x 4e 4 x 1 5 1 2
dx 2. (a) (e 1) (b) e 2 1 = 2 1 (c) e7 (e9 1)
d2 y 5 e 3
= 16e4x + 16e 4x 1 4 2 1 4 1 1
dx2 (d) 19 e (e 1) (e) e + 1 (f) e2 e 1
2 2 2 2
= 16 (e4x + e 4x)
1 6 1
= 16y (g) e + e 1
3
2 2
380 Maths In Focus Mathematics HSC Course

3. (a) 0.32 (b) 268.29 (c) 37 855.68 (d) 346.85 15. Curves are symmetrical about the line y = x.
(e) 755.19
1
4. e4 e2 = e2 (e2 1) units2 5. (e e 3) units2
4

6. 2.86 units 7. 29.5 units 8. (e6 1) units3
2 2
2
9. 4.8 units3 10. 7.4 11. (a) x (2 + x) ex (b) x2 ex + C

12. e units3
1 4
13. (e 5) units2
2

Exercises 4.4
1. (a) 4 (b) 2 (c) 3 (d) 1 (e) 2 (f ) 1 (g) 0 (h) 7
16. x = ey
2. (a) 9 (b) 3 (c) 1 (d) 12 (e) 8 (f ) 4 (g) 14 (h) 14
(i) 1 (j) 2 Exercises 4.5
1 1 1 1 1 b
3. (a) 1 (b) (c) (d) 2 (e) (f ) (g) 1. (a) loga 4y (b) loga 20 (c) loga 4 (d) loga
2 2 4 3 2 5
1 1 1 xy
(h) (i) 1 (j) 1 x5
3 2 2 (e) logx y3 z (f ) logk 9y3 (g) loga (h) loga
y2 z
4. (a) 3.08 (b) 2.94 (c) 3.22 (d) 4.94 (e) 10.40 p3 q
(f ) 7.04 (g) 0.59 (h) 3.51 (i) 0.43 (j) 2.21 (i) log10 ab4 c3 (j) log3
r2

5. (a) log3 y = x (b) log5 z = x (c) logx y = 2 2. (a) 1.19 (b) -0.47 (c) 1.55 (d) 1.66 (e) 1.08
(d) log2 a = b (e) logb d = 3 (f ) log8 y = x (f ) 1.36 (g) 2.02 (h) 1.83 (i) 2.36 (j) 2.19
(g) log6 y = x (h) loge y = x (i) loga y = x
(j) loge Q = x 3. (a) 2 (b) 6 (c) 2 (d) 3 (e) 1 (f ) 3 (g) 7
1
(h) (i) -2 (j) 4
6. (a) 3x = 5 (b) ax = 7 (c) 3b = a (d) x9 = y (e) ay = b 2
(f ) 2y = 6 (g) 3y = x (h) 10y = 9 (i) ey = 4 (j) 7y = x
4. (a) x + y (b) x y (c) 3x (d) 2y (e) 2x (f ) x + 2y
7. (a) x = 1 000 000 (b) x = 243 (c) x = 7 (d) x = 2 (g) x + 1 (h) 1 y (i) 2x + 1 (j) 3y 1
(e) x = 1 (f ) x = 3 (g) x = 44.7 (h) x = 10 000
(i) x = 8 (j) x = 64 5. (a) p + q (b) 3q (c) q p (d) 2p (e) p + 5q (f ) 2p q
(g) p + 1 (h) 1 2q (i) 3 + q (j) p 1 q
8. y = 5 9. 44.7 10. 2.44 11. 0 12. 1
6. (a) 1.3 (b) 12.8 (c) 16.2 (d) 9.1 (e) 6.7 (f ) 23.8
1
13. (a) 1 (b) (i) 3 (ii) 2 (iii) 5 (iv) (v) -1 (vi) 2 (g) -3.7 (h) 3 (i) 22.2 (j) 23
2
(vii) 3 (viii) 5 (ix) 7 (x) 1 (xi) e 7. (a) x = 4 (b) y = 28 (c) x = 48 (d) x = 3 (e) k = 6

14. Domain: x > 0; range: all real y


Exercises 4.6
1. (a) 1.58 (b) 1.80 (c) 2.41 (d) 3.58 (e) 2.85 (f ) 2.66
(g) 1.40 (h) 4.55 (i) 4.59 (j) 7.29

2. (a) x = 1.6 (b) x = 1.5 (c) x = 1.4 (d) x = 3.9


(e) x = 2.2 (f ) x = 2.3 (g) x = 6.2 (h) x = 2.8
(i) x = 2.9 (j) x = 2.4

3. (a) x = 2.58 (b) y = 1.68 (c) x = 2.73 (d) m = 1.78


(e) k = 2.82 (f ) t = 1.26 (g) x = 1.15 (h) p = 5.83
(i) x = 3.17 (j) n = 2.58

4. (a) x = 0.9 (b) n = 0.9 (c) x = 6.6 (d) n = 1.2


(e) x = 0.2 (f ) n = 2.2 (g) x = 2.2 (h) k = 0.9
(i) x = 3.6 (j) y = 0.6

5. (a) x = 5.30 (b) t = 0.536 (c) t = 3.62 (d) x = 3.81


(e) n = 3.40 (f ) t = 0.536 (g) t = 24.6 (h) k = 67.2
(i) t = 54.9 (j) k = 43.3
ANSWERS 381

Exercises 4.7 (c)

1 1 3 2x
1. (a) 1 + (b) (c) (d)
x x 3x + 1 x2 4
15x2 + 3 5 10x2 + 2x + 5
(e) (f) + 2x =
5 x + 3x 9
3
5x + 1 5x + 1
1 8 6x + 5
(g) 6x + 5 + (h) (i)
x 8x 9 (x + 2) (3x 1)
4 2 30
(j) =
4x + 1 2x 7 (4x + 1) (2x 7)
5 1 2
(k) (1 + loge x)4 (l) 9 c 1 m (ln x x)8 12. 13. (a) 3x ln 3 (b) 10x ln 10
x x (2x + 5) loge 3
4 1
(m) (loge x) (n) 6 c 2x + m (x2 + loge x)5
3
(c) 3 ln 2 23x 4
x x
1 loge x
(o) 1 + loge x (p) 14. 4 ln 4 $ x y + 4 = 0 15. 3 loge 3 $ x + y 1 9 loge 3 = 0
x2
2x + 1 x3
(q) + 2 loge x (r) + 3x2 loge (x + 1) Exercises 4.8
x x+1
1 x 2 x loge x 1. (a) loge (2x + 5) + C (b) loge (2x2 + 1) + C
(s) (t)
x loge x x (x 2) 2 1 1
(c) ln (x5 2) + C (d) loge x + C or loge 2x + C
2 2
e2x (2x loge x 1)
(u) 5
x (loge x) 2 (e) 2 ln x + C (f) loge x + C (g) loge (x2 3x) + C
3
1 1 3
(v) ex c + loge x m (h) ln (x2 + 2) + C (i) loge (x2 + 7) + C
x 2 2
1
10 loge x (j) loge (x + 2x 5) + C
2

(w) 2
x
2. (a) ln (4x 1) + C (b) loge (x + 3) + C
1 1
2. f l(1) = 3. 4. x 2y 2 + 2 loge 2 = 0 1 1
2 x loge 10 (c) ln (2x3 7) + C (d) loge (2x6 + 5) + C
6 12
2
5. y = x 2 6. 7. 5x + y loge 5 25 = 0 1
5 (e) loge (x2 + 6x + 2) + C
1 1 1 1 2
8. 5x 19y + 19 loge 19 15 = 0 9. d , loge n
2 2 2 4
3. (a) 0.5 (b) 0.7 (c) 1.6 (d) 3. 1 (e) 0.5
1
10. c e, m maximum
e
4. loge 3 loge 2 = loge 1.5 units2 5. loge 2 units2
11. (a) 6. (0.5 + loge 2) units2 7. 0.61 units2

8. loge 3 units3 9. 2 loge 9 units3


2 4
10. 47.2 units2 11. e (e 1) units3
2

1 2
12. (a) RHS = +
x+3 x3
1 (x 3)
=
(x + 3) (x 3)
2 (x + 3)
(b) +
(x + 3) (x 3)
x 3 + 2x + 6
=
x2 9
3x + 3
= 2
x 9
= LHS
3x + 3 1 2
` 2 = +
x 9 x+3 x3
(b) loge (x + 3) + 2 loge (x 3) + C
382 Maths In Focus Mathematics HSC Course

5 17. 2x + y ln 2 4 = 0
13. (a) RHS = 1
x1
18. (0, 0) point of inflexion, ( 3, 27e 3) minimum
x1 5
=
x1 x1 19. 5.36 units2
x6
= 20. (a) 0.65
x1
(b) 1.3
= LHS

x6 5 Challenge exercise 4
` =1
x1 x1
1
(b) x 5 loge (x 1) + C (e2x + x) (2e2x + 1) loge x
x
1. 2. 2e
(e2x + x) 2
2x 1
3
14. +C 15. 1.86 units2 3. (a) 2.8 (b) 1.8 (c) 2.6
2 loge 3
1 m 4x
4. 9 c 4e 4 x + (e + loge x) 8 5. 0.42 units2
Test yourself 4 x
2
1. (a) 6.39 (b) 1.98 (c) 3.26 (d) 1.40 (e) 0.792 6. 7. 12 units3 8. 5x loge 5
2x 3
(f) 3.91 (g) 5.72 (h) 72.4 (i) 6 (j) 2
1 4 d 2 3x
2. (a) 5e5x (b) 2e1 x (c) (d) (e) ex (x + 1) 9. (x loge x) = x (1 + 2 loge x); 18 loge 3 10. +C
x 4x + 5 dx loge 3
1 ln x
(f) (g) 10ex (ex + 1)9 11. (a) (1, 0) (b) x y 1 = 0; x loge 10 $ y 1 = 0
x2
1 1 1
3. (a) e4x + C (b) ln (x2 9) + C (c) e x + C (c) f 1 p units 12. 0.645 units2
4 2 loge 10

(d) ln (x + 4) + C ex (1 + loge x) xex loge x


e2 1 13.
4. 3x y + 3 = 0 5. 2 6. e4 (e6 1) units2 e 2x
e +1 2 1 + loge x x loge x
6 6 =
7. e (e 1) units 3
ex
6
8. (a) 0.92 (b) 1.08 (c) 0.2 (d) 1.36 (e) 0.64 14. y = ex + e x
dy
9. e (e2 1) units2 = ex e x
dx
d2 y
10. (a) 2.16 units2 (b) x = ey (c) 2.16 units2 = ex ( e x)
dx2
11. (a) x = 1.9 = ex + e x
(b) x = 1.9 =y
(c) x = 3
15. y = 3e 5 x 2
(d) x = 36
dy
(e) t = 18.2 = 15e5x
dx
3 2 d2 y
12. (a) (e 1) = 75e5x
2 dx2
1 d2 y dy
(b) ln 10 LHS = 4 5y 10
3 dx2 dx
1 = 75e 4 (15e5x) 5 (3e5x 2) 10
5x
(c) 8 + 3 ln 2
6 = 75e5x 60e5x 15e5x + 10 10
=0
13. e4 x y 3e4 = 0 = RHS

14. 0.9 16. f (x) = 3e2x 6x

15. (a) e (e 1) units2 17.



(b) e2 (e2 1) units3
2

16. (a) loga x5 y3


k2 p
(b) logx
3
ANSWERS 383

Chapter 5: Trigonometric functions


5. LHS = cos cos + sin sin
3 4 3 4
Exercises 5.1
1 1 3 1
= +
1. (a) 36 (b) 120 (c) 225 (d) 210 (e) 540 (f) 140 2 2 2 2
(g) 240 (h) 420 (i) 20 (j) 50 1 3
= +
3 5 4 5 7 2 2 2 2
2. (a) (b) (c) (d) (e) (f) (g)
4 6 6 3 3 20 12
RHS = sin cos + cos sin
5 5 2 4 6 4 6
(h) (i) (j) 3
2 4 3 1 1 1
= +
2 2 2 2
3. (a) 0.98 (b) 1.19 (c) 1.78 (d) 1.54 (e) 0.88 3 1
= +
2 2 2 2
4. (a) 0.32 (b) 0.61 (c) 1.78 (d) 1.54 (e) 0.88
LHS = RHS

5. (a) 62 27 (b) 44 0 (c) 66 28 (d) 56 43 So cos cos + sin sin = sin cos +
3 4 3 4 4 6
(e) 18 20 (f) 183 21 (g) 154 42 (h) 246 57
(i) 320 51 (j) 6 18 cos sin
4 6

6. (a) 0.34 (b) 0.07 (c) 0.06 (d) 0.83 (e) 1.14 3 4 1
(f) 0.33 (g) 1.50 (h) 0.06 (i) 0.73 (j) 0.16 6. (a) = (b) 2nd (c)
4 4 4 2

=
4
Exercises 5.2
5 6 1
7. (a) = (b) 2nd (c)
1. 6 6 6 2

=
6
3 4 6

1 1 7 8
3 8. (a) = (b) 4th (c) 1
sin 4 4 4
2 2 2

= 2
1 1 3 4
cos
2 2 2
4 3 1
1 9. (a) = + (b) 3rd (c)
3 3 3 2
tan 3 1
3
=+
3
cosec 2 2 2
5 6 3
3 10. (a) = (b) 4th (c)
3 3 3 2
2
sec 2 2 = 2
3 3

1 3 1 1
cot 1 3 11. (a) 1 (b) (c) 3 (d) (e)
3 2 2 3

13 12 3
1 1 3 3 4 3 2 3+1 12. (a) (i) = + (ii) 1st (iii)
2. (a) (b) (c) (d) (e) 0 (f) 6 6 6 2
3 2 8 3 2
= 2 +
2+2 32 2 2+ 3 6
(g) 2 3 (h) (i) (j)
2 2 2 1 1 1 3
(b) (i) (ii) 3 (iii) (iv) (v)
2 2 3 2
1 6 2 3 1
3. (a) 1 (b) (c) (d) 1 (e) 4
4 4 2 4 5 5 7 5 4
13. (a) , (b) , (c) , (d) ,
3 3 4 4 4 4 3 3
6+ 2
4. (a) (b) 32
4 5 7
(e) ,
6 6

14. (a) sin (b) tan x (c) cos (d) sin x (e) cot
384 Maths In Focus Mathematics HSC Course

15. sin2 10. 9.4 cm2

4 3 11 121 396 121


16. cos x = ; sin x = 11. (a) cm (b) 22 = cm2
5 5 9 18 18
3 5 7 11 11 (18 + )
17. x = , , , (c) 22 + = cm
4 4 4 4 9 9

Exercises 5.3 12. (a) 5 cm2 (b) 0.3% (c) 15.6 cm

25 7 13. (a) 10 cm (b) 24 cm2


1. (a) 4 cm (b) m (c) cm (d) cm (e) mm
3 2 4
2. (a) 0.65 m (b) 3.92 cm (c) 6.91 mm (d) 2.39 cm 20 4 (18 + 5 )
14. (a) 8 + = cm (b) 3:7
(e) 3.03 m 9 9
2 225 105 15 (7 + 24)
3. 1.8 m 4. 7.5 m 5. 6. 25 mm 7. 1.83 15. (a) cm3 (b) + 180 = cm2
21 2 2 2
7 175
8. 13 mm 9. 25.3 mm 10. SA = cm2,
9 36
Exercises 5.6
125 35
V= cm3
648 1. (a) 0.045 (b) 0.003 (c) 0.999 (d) 0.065 (e) 0.005

1 1
Exercises 5.4 2. 3.
4 3
3 2 125 3
1. (a) 8 cm2 (b) m (c) cm2 (d) cm2
2 3 4 4. 1 343 622 km 5. 7367 m
49
(e) mm2
8
Exercises 5.7
2. (a) 0.48 m2 (b) 6.29 cm2 (c) 24.88 mm2 (d) 7.05 cm2 1. (a) y
(e) 3.18 m2

4 7 49
3. 16.6 m2 4. = 4 5. 6 m 6. (a) cm (b) cm2
9 6 12
1
6845
7. mm2 8. 75 cm2 9. 11.97 cm2
8

10. = , r = 3 cm x
15
3 2
2 2
Exercises 5.5
-1
6 9 3 2 125 75
1. (a) 8 cm2 (b) m (c) cm2
4 3
3 ( 3) 49 ( 2 2 )
(d) cm2 (e) mm2 (b) y
4 8

2. (a) 0.01 m2 (b) 1.45 cm2 (c) 3.65 mm2 (d) 0.19 cm2
(e) 0.99 m2
2
3 9
3. 0.22 cm 2
4. (a) cm (b) cm2 (c) 0.07 cm 2
7 14
5 x
5. 134.4 cm 6. (a) 2.6 cm (b) cm (c) 0.29 cm2
6 3 2
2 2
7. (a) 10.5 mm (b) 4.3 mm2 -2

25
8. (a) cm2 (b) 0.5 cm2
4

9. (a) 77 22l (b) 70.3 cm2 (c) 26.96 cm2 (d) 425.43 cm2
ANSWERS 385

(c) y (g) y

2
4
3
2
1 1
x
3 2
2 2
x
3 2 (h) y
2 2
-1

y 5
(d)
x
3 2
2 2
3

2 (i) y

x
3 2 3
2 2 x
3 2
(e) y 2 2

(j) y
3

x
3 2
2 2 1
x
3 2
-3
2 2

(f) y

2. (a) y
4

x
3 2 1
2 2
-4

x
3 5 3 7 2
4 2 4 4 2 4

-1
386 Maths In Focus Mathematics HSC Course

(b) y (g) y

x
x 2 4 5 2
3 5 3 7 2 3 3 3 3
4 2 4 4 2 4

(h) y
(c) y

3
1

x
2 4 5 x
2
3 3 3 3 3 2
2 2
1

-3
(d) y

(i) y

x
3 5 3 7 2
4 2 4 4 2 4 x
3 2
2 2
3

2
(e) y

6 (j) y

x
2 4 5 2
3 3 3 3
4
-6

(f) y
x
3 2
2 2

-4
x
2
ANSWERS 387

3. (a) y 4. y

8
1

x x
- 3 2 3 4
- 3 - 2 -
4 4 2
4 4
-1
-8
(b) y

7 5. (a) y

1
x
- 3 - -
3
- 2 4 4
4 2 4
-7 x
3 2
(c) y
2 2

-1

(b) y

x
- 3 - 3
- -
4 2 4 4 2 4

x
3 2
(d) y 2 2

5
(c) y

x 1
- 3 3
- - -
4 2 4 4 2 4
-5
x
(e) y 3 2
2 2

-1
2
(d) y

x 3
- 3 - 3
- -
4 2 4 4 2 4

-2
x
3 2
2 2

-3
388 Maths In Focus Mathematics HSC Course

(e) y (j) y

5
2
4

3
x
3 2 2
2 2
1
-2
x
3 2
(f) y
-1 2 2

4
6. (a) y

x
3 5 3 7 2
1
4 2 4 4 2 4

-4 x
-2 -1 1 2
(g) y
-1
1
(b) y

x 3
3 5 3 7 2
4 2 4 4 2 4
x
-1 -2 -1 1 2

(h) y -3

7. (a) y

1
1 y = sin x
x
3 5 3 7 2
4 2 4 4 2 4 x
3 2
-1 2 2
y = sin 2x

(i) y
(b) y

3
2
2
1
1
x
x 3 2
3 2
-1 2 2
-1 2 2

-2
ANSWERS 389

8. (a) y 10. (a) y

5
4 2
3
y = cos x + sin x
2 1
y = 2 cos x
1
x
3 2 x
1 3 2
2 2 2 2
2 y = 3 sin x
-1
3
4
-2
5

(b) y (b) y

5
4 2
3
y = sin 2x sin x
2
1
1
x
3 2
1 2 x
2 3 2
2 2 2
3 1
4 y = 2 cos x + 3 sin x
5
2

9. y (c) y

2 y = cos 2x - cos x
2 y = sin x + 2 cos 2x

1
1
x
3 2 x
2 2 3 2
-1 2 2
1
-2
2

(d) y

3
2 y =3 cos x cos 2x

x
3 2
1 2 2
2
3
4
390 Maths In Focus Mathematics HSC Course

(e) y 6. x = 0.8, 4

y
3

1 1
x y = cos x
3 2
-1 2 2
x x
-2 y = sin x - sin 3 2
2
2 2
-3
y = sin x
-1
-4

7. (a) Period 12 months, amplitude 1.5 (b) 5.30 p.m.


Exercises 5.8 8. (a) 1300 (b) (i) 1600 (ii) 1010
1. (a) y (c) Amplitude 300, period 10 years

y= x 9. (a) 18
2 16
14
1 12
10
8
6
x 4
1 2 3 4 3 5 6 2
2
2 2
0
January February March April May June July August
y = sin x
-1 (b) It may be periodic - hard to tell from this data.
Period would be about 10 months.
(c) Amplitude is 1.5
There are 2 points of intersection, so there are
2 solutions to the equation.
10. (a) 4
(b) y
3.5
x
y= 3
2
2.5
1
2

y = sin x 1.5

1
x

- -3 -2 - -1 1 2 3 0.5
2 2
0
am

am

m
am

m
pm

am
pm

pm
5p

5p

0a

5p
5a
0

0
.55

.55
.48

.48

.48
6.2

6.2

6.2
.5
6.1

6.1

6.1

-1
11

11

11
11

11

11

(b) Period 24 hours, amplitude 1.25 (c) 2.5 m


There are 3 points of intersection, so there are
3 solutions to the equation. Exercises 5.9

2. x =0 3. x = 1.5 4. x = 0, 4.5 5. x = 0, 1 1. (a) 4 cos 4x (b) 3 sin 3x (c) 5 sec2 5x


(d) 3 sec2 (3x + 1) (e) sin ( x) (f) 3 cos x
(g) 20 sin (5x 3) (h) 6x2 sin (x3)
(i) 14x sec2 (x2 + 5) (j) 3 cos 3x 8 sin 8x
ANSWERS 391

(k) sec2 ( + x) + 2x (l) x sec2 x + tan x


13. (a) sec2 x (b) sin x (c) cos x
(m) 3 sin 2x sec2 3x + 2 tan 3x cos 2x 180 60 900
2x cos x 2 sin x x cos x sin x 14. y = 2 sin 3x 5 cos 3x
(n) =
4x 2 2x 2 dy
3 sin 5x 5 (3x + 4) cos 5x = 6 cos 3x + 15 sin 3x
(o) dx
sin2 5x d2 y
= 18 sin 3x + 45 cos 3x
(p) 9 (2 + 7 sec2 7x) (2x + tan 7x)8 dx2
(q) 2 sin x cos x (r) 45 sin 5x cos2 5x = 9 (2 sin 3x 5 cos 3x)
= 9y
1
(s) ex + 2 sin 2x (t) cos (1 loge x)
x
15. a = 7, b = 24
cos x
(u) (ex + 1) cos (ex + x) (v) = cot x
sin x
(w) 2e3x sin 2x + 3e3x cos 2x Exercises 5.10
= e3x (3 cos 2x 2 sin 2x)
1. (a) sin x + C (b) cos x + C (c) tan x + C
2e2x tan 7x 7e2x sec2 7x 45 1 1
(x) (d) cos x + C (e) cos 3x + C (f) cos 7x + C
tan2 7x 3 7
e2x (2 tan 7x 7 sec2 7x) 1
= (g) tan 5x + C (h) sin (x + 1) + C
tan2 7x 5
1 1
(i) cos (2x 3) + C (j) sin (2x 1) + C
2. 4 cos2 x sin3 x sin5 x 2 2
= sin3 x (4 cos2 x sin2 x) (k) cos ( x) + C (l) sin (x + ) + C
2 x
3. 12 4. 6 3 x 12y + 6 3 = 0 (m) tan 7x + C (n) 8 cos +C
7 2
sin x 2 2 3 x
5. = tan x 6. = (o) 9 tan + C (p) cos (3 x) + C
cos x 9 3
3 3
1 2 3 2 1
2. (a) 1 (b) 3 = (c) = 2 (d)
7. sec2 xetanx 8. 8 2 x + 48y 72 2 2 = 0 3 3 2 3
1 1 3 1
(e) (f) (g) (h)
9. y = 2 cos 5x 2 4 5
dy
= 10 sin 5x
dx 1 2
3. 4 units2 4. = units2 5. 0.86 units2
d y
2
3 2 6
= 50 cos 5x
dx2
= 25 (2 cos 5x)
6. 0.51 units3 7. units3
= 25y 4

10. f (x) = 2 sin x 3 3


8. 3 = units2 9. 2 2 units2
f l(x) = 2 cos x 3 3
f m(x) = 2 sin x
10. (a)
#
b
= f (x) V = y 2 dx
a

d
11. LHS =
dx
[loge (tan x)]
= # 0
2
cos x dx
sec2 x
=
tan x 2
= ; sin x E
tan2 x + 1 0
=
tan x = (sin sin 0)
tan2 x 1 2
= +
tan x tan x = (1 0)
= tan x + cot x = units 3
= RHS
(b) 3.1 units3
d
` 7 loge (tan x) A = tan x + cot x 11. y = 2 sin 3x
dx


12. d , 3 n maximum,
3 3
5 5
d , 3 n minimum
3 3
392 Maths In Focus Mathematics HSC Course

Test yourself 5 15. (a)

5 25
1. (a) cm (b) cm2 (c) 0.295 cm2
6 12

3 3 1
2. (a) 3 (b) (c) (d)
2 2 2

3 7 5
3. (a) x = , (b) x = ,
4 4 6 6

4. (a) (b) x = 0.6

3 2
16. units2
2

17. 2 units2 18. 4x + 8y 8 = 0 19. y = 3 cos 2x

20. (a)

(b)

(b) x = 0.9, 2.3, 3

Challenge exercise 5
5. (a) sin x (b) 2 cos x (c) sec2 x (d) x cos x + sin x
1 1 3 3
x sec2 x tan x 1. 0.27 2. e1 o= 3. r = 64 units, =
(e) (f) 3 sin 3x (g) 5 sec2 5x 2 3 6 512
x2
4. (a) Period = 2, amplitude = 3
1 1
6. (a) cos 2x + C (b) 3 sin x + C (c) tan 5x + C
2 5 (b)
(d) x cos x + C

1 2 3
7. (a) (b)
2 3

3
8. 3x + 2 y 1 =0
4

9. x = cos 2t
dx 5. (a) y = sin 3x
= 2 sin 2t
dt d2 y
(b) LHS = + 9y
d x
2
dx2
= 4 cos 2t
dt2 = 9 sin 3x + 9 ( sin 3x)
= 4x = 9 sin 3x 9 sin 3x
=0
1 = RHS
10. units2 11. units3 12. (a) 5 (b) 2
2 3

8
13. 3 3 14. (a) cm2 (b) 0.12 cm2
7
ANSWERS 393

6. 5 9 13
18. d , 0 n, d , 0 n, d , 0 n, d , 0n
8 8 8 8

1 1 21
19. = units2
2 2 2

20. f (x) = 2 cos 3x


f l(x) = 6 sin 3x
f m(x) = 18 cos 3x
= 9(2 cos 3x)
= 9f (x)

7. sec2 x
180
sec2 x
8. (a) RHS = Chapter 6: Applications of calculus to
tan x
1 the physical world
cos2 x
= Exercises 6.1
sin x
cos x 1. (a) R = 20 8t (b) R = 15t2 + 4t (c) R = 16 4x
1 cos x
= (d) R = 15t 4t + 2 (e) R = e (f) R = 15 sin 5
4 3 t
cos2 x sin x
1 1 100 x 400
= (g) R = 2 3 (h) R = (i) R = 800 2
cos x sin x r x2 4 r
= sec x cosec x (j) R = 4 r 2

= LHS
2. (a) h = 2t2 4t3 + C (b) A = 2x4 + x + C
sec x 2 4
` sec x cosec x = (c) V = r3 + C (d) d = 7 cos t + C
tan x 3
1 (e) s = 4e2t 3t + C
(b) loge 3 = loge 3
2
3. 20 4. 1 5. 6e12 6. 13 7. 900 8. 2e3 + 5
3
9. (2x cos 2x + sin 2x) e x sin 2x
10. (a) d , 4 n and d , 2n dM
4 4 9. y = x3 x2 + x + 6 10. R == 1 4t; R = 19
dt
(b) Maximum = 4 (c) Amplitude = 1
[i.e. melting at the rate of 19 g per minute (g min-1)]

4 (2 3 3 ) 11. 11 079.25 cm per second (cms-1) 12. 21 000 L


3
11. 8 e o cm2 = cm2
3 2 3
13. 165 cm2 per second (cm2s-1) 14. 0.25

180 1 15. 41 cm2 per minute (cm2 min-1) 16. 31 cm3


12. cos x + C 13.
2
cos x sin x 17. 108 731 people per year 18. (a) 27 g
14. 0.204 units3 15. (b) 2.7 (i.e. decaying at a rate of 2.7 g per year)
sin x + cos x
19. y = e 4x
9 9 ( 2)
16. d 1n = cm2 dy
2 2 4 = 4e 4 x
dx
= 4y
17.
20. S = 2e 2 t + 3
dS
= 2 (2e2t )
dt
= 2 (2e2t + 3 3)
= 2 (S 3)
394 Maths In Focus Mathematics HSC Course

Exercises 6.2 20. (a) Q = Aekt


dQ
1. (a) 80 (b) 146 (c) 92 days = kAekt
(d) dt
= kQ
dQ
(b) = kQ
dt
dt 1
So =
dQ kQ
1
t =# dQ
kQ
1 1
= # dQ
k Q
1
= ln Q + C
k
kt = ln Q + C1
kt C1 = ln Q
ekt C = Q
1

2. (a) 99 061 (b) 7 hours


ekt e C = Q
1

3. (a) When t = 0, M = 100 Aekt = Q


` M = 100e kt
When t = 5, M = 95
` 95 = 100e 5k Exercises 6.3
0.95 = e 5k
ln 0.95 = 5k 1. (a) v a
0.01 =k
So M = 100e 0.01t
(b) 90.25 kg
t t
(c) 67.6 years

4. (a) 35.6 L (b) 26.7 minutes

5. (a) P0 = 5000 (b) k = 0.157 (c) 12 800 units


(d) 8.8 years (b) v a

6. 2.3 million m2 7. (a) P = 50 000e0.069t (b) 70 599


(c) 4871 people per year (d) 2040
t t
8. (a) 65.61 C (b) 1 hour 44 minutes

9. (a) 92 kg (b) Reducing at the rate of 5.6 kg per hour


(c) 18 hours

10. (a) M0 = 200; k = 0.00253 (b) 192.5 g (c) v a


(c) Reducing by 0.49 g per year (d) 273.8 years

11. (a) B = 15 000e0.073t (b) 36 008 (c) 79.6 hours

12. 11.4 years 13. (a) 19% (b) 3200 years t t

14. (a) P (t) = P (t0) e kt


dP (t)
= kP (t0) e kt
dt
= kP (t)
(b) 23% (c) 2% decline per year (d) 8.5 years

15. 12.6 minutes 16. 12.8 years

17. (a) 76.8 mg/dL (b) 9 hours 18. 15.8 s 19. 8.5 years
ANSWERS 395

(d) Exercises 6.4


1. (a) 18 cms-1 (b) 12 cms-2 (c) When t = 0, x = 0; after 3 s
(d) After 5 s

2. (a) 8 ms 1 (b) a = 4; constant acceleration of 4 ms-2


(c) 13 m (d) after 2 s (e) 5 m
(f)

(e)

2. (a) t2, t4, t6


(b) 0 to t1, t3, t5
(c) 0 to t1
(d) t5 3. (a) 4 m (b) 40 ms-1 (c) 39 m (d) 84 m
(e)
3. (a)

(b)

4. (a) 2 cm (b) After 1 s (c) -4 cm (d) 6 cm


(e) -7 cms-1

5. (a) 2 ms-1 (b) 4e2 ms 2 (c) a = 4e2t = 2 (2e2t ) = 2v


(d)

4. (a) O, t2, t4, t6 (b) t1, t3, t5 (c) t5


(d) (i) At rest, accelerating to the left.
(ii) Moving to the left with zero acceleration.

3 5 3
5. (a) , , , (b) 0, , , ,
4 4 4 2 2

6. (a) At the origin, with positive velocity and positive


constant acceleration (moving to the right and speeding 6. (a) v = 2 sin 2t (b) a = 4 cos 2t (c) 1 cm
up).
3 3 5
(b) To the right of the origin, at rest with negative (d) 0, , , ,...s (e) 1 cm (f) , , ...s
2 2 4 4 4
constant acceleration.
(g) a = 4 cos 2t = 4x
(c) To the left of the origin, with negative velocity and
positive acceleration (moving to the left and slowing down).
7. (a) v = 3t2 + 12t 2; a = 6t + 12 (b) 266 m (c) 133 ms-1
(d) To the right of the origin, with negative velocity and
(d) 42 ms-1
acceleration (moving to the left and speeding up).
(e) To the left of the origin, at rest with positive
acceleration.
396 Maths In Focus Mathematics HSC Course

8.

(a) x = 20 (4t 3)4 , x = 320 (4t 3)3 dx
(b) = 6t2 6t + 42
(b) x = 1, cm, ox = 20 cms 1, px = 320 cms 2 dt
(c) The particle is on the RHS of the origin, travelling to dx
At rest: =0
the right and accelerating. dt
6t2 6t + 42 =0
9. (a) v = 5 10t (b) 95 ms 1 (c) a = 10 = g t2 t + 7 =0
b2 4ac = (1) 2 4 (1) (7)
17 102
10. v = ,a= = 27
(3t + 1) 2 (3t + 1)3
<0
1 1
11. (a) At the origin (b)cms 1 (c) cms 2 So the quadratic equation has no real roots.
6 36
So the particle is never at rest.
(d) The particle is moving to the right but decelerating
(e) (e3 1) s 3 5
17. (a) 0 cm (at the origin) (b) , , ,...s (c) 12 cm
4 4 4
12. (a) 3 ms-1 (b) When t = 0 s, 1 s, 3 s (c) 10 ms-2
18. (a) 8e16 cms-1 (b) 0 s (initially) (c) 1 cm
13. (a)
7 7 2
19. (a) 7 s (b) or s (c) 49 cm
2 2

Exercises 6.5
1. 12 cm 2. 28 m 3. -42.5 cm

4. (a) 570 cms-2 (b) 135 cm (c) After 0.5 s

5. (e3 + 1) cm 6. 163 m 7. (a) 95 cms-1 (b) 175 cm

8. h = 4.9t2 + 4t + 2 9. 262 m 10. (e5 3) m

11. -744 cm 12. (2 3) cm 13. 1.77 m 14. 893 m


(b) ox = 6 cos 2t, px = 12 sin 2t
(c) 6 3 cms-2 15. (a) ( 3 + 3) m (b) 4 3 ms 2
(d) px = 12 sin 2t = 4 (3 sin 2t) = 4x
4 2n
16. (a) ms-1 (b) x = 2ln (n + 3) m
14. (a) 7 m (b) 16 m (c) After 7 s 15 3
(d)
2 2
(c) v =
3 t+3
2 (t + 3) 6
=
3 (t + 3)
2t + 6 6
=
3 (t + 3)
2t
=
3t + 9
t 0
When t = 0: v = 0
When t > 0: v > 0
Also 2t > 0 and 3t + 9 > 0 when t > 0
(e) 10 m So 2t < 3t + 9
2t
<1
15. (a) 18.75 m (b) -15 ms-1 (c) 5 s 3t + 9
`0 v <1
16. (a) At the origin: x = 0
2t3 3t2 + 42t = 0 17. (a) 5e45 ms1 (b) e30 m (c) px = 25e5t (d) 50 ms-2
t (2t2 3t + 42) = 0 = 25x
t = 0, 2t2 3t + 42 = 0
Since t = 0, the particle is initially at the origin. Test yourself 6
2t2 3t + 42 = 0
1. (a) 0 m, 0 ms-1, 8 ms-2 (b) 0, 0.8 s (c) 0.38 m
b2 4ac = (3) 2 4 (2) (42)
= 327 2. -76 m, -66 ms-2 3. 39.6 years
<0
So the quadratic equation has no real roots.
So the particle is never again at the origin.
ANSWERS 397

4. (a) 6 cms 1 (b) 145 855.5 cms-2 13. (a) 16 941 (b) 1168 birds/year
(c) x = 2e3t (c) 18.3 years
ox = 6e3t
px = 18e3t 14. (a) (i)
= 9 (2e3t )
= 9x

5. 1m

6. x = 2 sin 3t
ox = 6 cos 3t
px = 18 sin 3t
= 9 (2 sin 3t) (ii)
= 9x

7. (a) 2, 6 s
(b) (i) 16 cm
(ii) 15 cms 1
(iii) 18 cms 2
(c) Particle is 16 cm to the right of the origin, travelling
at 16 cms 1 to the right. Acceleration is 18 cms 2 (to the
left), so the particle is slowing down. (b) t1, t3

8. (a) (i) 18 ms 2 15. 55 033 m


(ii) 15 ms 1
(iii) -28 m
Challenge exercise 6
(b) Particle is 28 m to the left of the origin, travelling at
15 ms 1 to the right, with 18 ms 2 acceleration (to the 2 8 2 (4 cos 3t)
right), so the particle is speeding up. 1. (a) x = cos 3t + =
3 3 3
8
9. (a) t1, t3, t5 (b) t2, t4 (c) t3 and after t5 (b) px = 9 c x m
3
(d) (i)
2. (a) 1 m, 0 ms-1 (b) 3.26 107 ms-2
(c) Show (t3 + 1) 6 = 0 has no solution for t 0

3. (a) x = cos 4t (b) 2 3 cms-1

4. (a) v = 5 cos 5t
x = # (5 cos 5t) dt
= sin 5t + C
(ii)
When t = 0, x = 0
0 = sin 0 + C
=C
` x = sin 5t
d
a= (5 cos 5t)
dt
= 25 sin 5t
= 25x

10. (a) 48.2% (b) 1052.6 years (b) 25 ms 2 (c) 7.5ms2


5 13 17 5 7 11
11. (a) , , , ,...s (b) , , , ,...s 5. (a) 19.9 years (b) 16%
6 6 6 6 3 3 3 3
1
(c) ms 2
2
12. (a) 15 m (b) 20 m (c) 4 s
398 Maths In Focus Mathematics HSC Course

kN0 23. (a) v = 6 ms 1, a = 0 ms 2 (b) 3 m


6. N= 3 5
bN0 + (k bN0) e kt (c) , , , . . . seconds
4 4 4
dN kN0 7 k (k bN0) e A
kt

= (d) a = 12 sin 2t
dt 7 bN0 + (k bN0) e kt A
2
= 4 (3 sin 2t)
k2 N0 (k bN0) e kt = 4x
=
7 bN0 + (k bN0) e kt A
2

24. 4.67 units2 25. 27 m 26. x = 0.28


k2 N0 7 bN0 + (k bN0) e kt bN0 A
= 1 1
7 bN0 + (k bN0) e kt A
2
27. x y + 2 = 0 28. c , m minimum
2 2e
k2 N0 7 bN0 + (k bN0) e kt A
= 29. (a) 1.60 cm2 (b) 0.17 cm2
7 bN0 + (k bN0) e kt A
2

bk2 N20 30. 15 months



7 bN0 + (k bN0) e kt A
2

k2 N0 5 25
31. (a) cm (b) cm2
= 6 12
bN0 + (k bN0) e kt
2
kN0 32.
bf p
bN0 + (k bN0) e kt

= kN bN2

7. 3e cms 2

Practice assessment task set 2

1. 3.2 years 2. 1.099 3. 27 m 4. 2


1
5. x = ey, x = 3.42 6. 3x 2 + 2e 2 x 7. x =
2
8. (a) 47.5 g (b) 3.5 g/year (c) After 9.3 years

9. (a) 0 cms 1 (b) a = 18 sin 3t = 9x


33. 1 34. cot x 35. 5e5x sec2 (e5x + 1) 36. 3
10. (a) 7750 L (b) 28 minutes
12 37. (a)
11. 622.1 units3 12.
4x + 3
1
13. loge (3x2 + 3x 2) + C
3
14. (a) 100 L (b) 40 L
(c) 16 L per minute, i.e. leaking at the rate of 16 L per
minute
(d) 12.2 minutes

15. 2x3 x2 + 4 loge x + C 16. 3 loge 2

17. (a) 7.8 cm (b) 0.06 cms 2 (b) 2 units2

1 4x 1 2x 38. 0.348
18. e +x+C 19.
4 e 2x
39. (a) ex (sin x + cos x) (b) 3 tan2 x sec2 x
20. (a) k = 0.101 (b) 2801 (c) 20 days
(c) 6 sin c 3x m
(d) (i) 11 people per day 2
(ii) 283 people per day
40. (a) 546 ms 1 (b) a = 20e2t (c) 20 ms-2
1 = 4 (5e )
2t
21. (a) 1.77 (b)
x loge 3 = 4x
8
41. ln 8 ln 3 = ln 42. 6 m 12 m
e2 4 3
22. (e 1) units3 3
2 43. units3
2
ANSWERS 399

44. 1
7. 2, 2 , 3 8. 3.1, 3.7, 4.3 9. 16, 32, 64
2
10. 108, 324, 972 11. 16, 32, 64 12. 48, 96, 192

1 1 1 16 32 64
13. , , 14. , ,
16 32 64 135 405 1215

15. 36, 49, 64 16. 125, 216, 343 17. 35, 48, 63

18. 38, 51, 66 19. 126, 217, 344 20. 21, 34, 55

Exercises 7.2
1. (a) T1 = 3, T2 = 11, T3 = 19 (b) T1 = 5, T2 = 7, T3 = 9
3 (c) u1 = 5, u2 = 11, u3 = 17 (d) T1 = 3, T2 = 2, T3 = 7
45. 6x y 1 =0
2 (e) t1 = 19, t2 = 18, T3 = 17 (f) u1 = 3, u2 = 9, u3 = 27
(g) Q1 = 9, Q2 = 11, Q3 = 15 (h) t1 = 2, t2 = 12, t3 = 58
3 3 (i) T1 = 8, T2 = 31, T3 = 70 (j) T1 = 2, T2 = 10, T3 = 30
46. 3 = units2
3 3
2. (a) T1 = 1, T2 = 4, T3 = 7 (b) t1 = 4, t2 = 16, t3 = 64
47. 3ex sin2 (ex) cos (ex) (c) T1 = 2, T2 = 6, T3 = 12

48. (5 e) units2 3. (a) 349 (b) 105


(c) 248 (d) -110 (e) -342
1 3
49. (a) (b)
2 2 4. (a) 1029 (b) -59 039 (c) 1014
(d) 53 (e) 1002
50. (a) 21 cm2 (b) (21 + 9) cm2
5. (a), (b), (d) 6. (b), (d) 7. 16th term 8. Yes
sec2 (loge x + 1)
51. 9. 7th term 10. 23rd term 11. (a) 1728 (b) 25th term
x
12. (a) -572 (b) 17th term
5
52. 3x 2 ln x +C
x
13. n = 33 14. n = 9 15. n = 88, 89, 90, . . .
1 1
53. 8.32 units 3
54. e5x + cos x + C
5 16. n = 41, 42, 43 17. n = 501 18. n = 151
55. x Z 1.8
19. (a) n = 14 (b) -4 20. -6

Exercises 7.3
1. (a) 128 (b) 54 (c) 70 (d) 175 (e) 220
47
(f) (g) 40 (h) 21 (i) 126 (j) 1024
60

2. (a) 65 (b) 99 (c) 76 (d) 200 (e) 11


(f) 39 (g) 97 (h) 66 (i) 75 (j) 45

6 10 5 n
56. (e2 1) units2 57. 1 3. (a) / 2n 1 (b) / 7n (c) / n3 (d) / 6k 4
n =1 n =1 n =1 k =1

n 50 n 9
58. (c) 59. (d) 60. (a) 1
(e) / k2 (f) / (n) (g) / 3.2k (h) /
k =3 n =1 k =0 n =0 2n
61. (d) 62. (b) 63. (c) n n
(i) / a + (k 1) d (j) / ar k 1
k =1 k =1

Chapter 7: Series
Exercises 7.4
Exercises 7.1
1. (a) y = 13 (b) x = 4 (c) x = 72 (d) b = 11 (e) x = 7
1. 14, 17, 20 2. 23, 28, 33 3. 44, 55, 66
1 1
(f ) x = 42 (g) k = 4 (h) x = 1 (i) t = 2 (j) t = 3
4. 85, 80, 75 5. 1 , 1, 3 6. 87, 83, 79 2 2
400 Maths In Focus Mathematics HSC Course

2. (a) 46 (b) 78 (c) 94 (d) -6 (e) 67 4. (a) 4850 (b) 4225

3. (a) 590 (b) -850 (c) 414 (d) 1610 (e) -397 5. (a) 28 875 (b) 3276 (c) 1419 (d) 6426
4 (e) 6604 (f) 598 (g) 2700
4. (a) -110 (b) 12.4 (c) -8.3 (d) 37 (e) 15 (h) 11 704 (i) 290 (j) 1284
5
5. Tn = 2n + 1 6. (a) 700 (b) 285 (c) 1170 ] 18 terms g
(d) 6525 (e) 2286
6. (a) Tn = 8n + 1 (b) Tn = 2n + 98 (c) Tn = 3n + 3
(d) Tn = 6n + 74 (e) Tn = 4n 25 (f) Tn = 20 5n 7. 21 8. 8 9. 11 10. a = 14, d = 4
n+6
(g) Tn = (h) Tn = 2n 28 (i) Tn = 1.2n + 2
8 11. a = 3, d = 5 12. 2025 13. 3420
3n 1
(j) Tn = 14. 8 and 13 terms 15. 1010
4
7. 28th term 8. 54th term 9. 30th term 16. (a) (2x + 4) (x + 1)
= (3x + 7) (2x + 4)
10. 15th term 11. Yes 12. No =x+3
13. Yes 14. n = 13 15. n = 30, 31, 32 . . . (b) 25 (51x + 149)

16. -2 17. 103 18. 785 17. 1290 18. 16

19. (a) d = 8 (b) 87 19. Sn = Sn 1 + Tn


` Sn Sn 1 = Tn
20. d = 9 21. a = 12, d = 7
20. 4234
22. 173 23. a = 5

24. 280 25. 1133 Exercises 7.6


3 2
1. (a) No (b) Yes, r = (c) Yes, r =
26. (a) T2 T1 = log5 x2 log5 x 4 7
= 2 log5 x log5 x (d) No (e) No (f ) No (g) Yes, r = 0.3
= log5 x 3
(h) Yes, r = (i) No (j) Yes, r = 8
T3 T2 = log5 x3 log5 x2 5
= 3 log5 x 2 log5 x
2. (a) x = 196 (b) y = 48 (c) a = 12
= log5 x
2
(d) y = (e) x = 2 (f ) p = 10
Since T2 T1 = T3 T2 it is an arithmetic series with 3
d = log5 x. (g) y = 21 (h) m = 6 (i) x = 4 ! 3 5
(b) 80 log5 x or log5 x80 1 2
(j) k = 1 3 7 (k) t = (l) t =
(c) 8.6 6 3

27. (a) T2 T1 = 12 3 3. (a) Tn = 5n 1 (b) Tn = 1.02n 1 (c) Tn = 9n 1


= 4 3 3 (d) T = 2 . 5 n 1 (e) Tn = 6 . 3 n 1
(f) Tn = 8 . 2n 1
n
=2 3 3 = 2n + 2
= 3 1
(g) Tn = 4 n1
(h) Tn = 1000 ] 10 g n 1

T3 T2 = 27 12 4
=4 n2 = ] 10 gn + 2
= 9 3 4 3 1 2 n 1
=3 3 2 3 (i) Tn = 3 ] 3 gn 1 (j) Tn = d n
3 5
= 3 = ] 3 gn

Since T2 T1 = T3 T2 it is an arithmetic series 4. (a) 1944 (b) 9216 (c) -8192


with d = 3. 64
(d) 3125 (e)
729
(b) 50 3
5. (a) 256 (b) 26 244 (c) 1.369
28. 26 29. 122b 30. 38th term
3
(d) -768 (e)
1024
Exercises 7.5
6. (a) 234 375 (b) 268.8 (c) -81 920
1. (a) 375 (b) 555 (c) 480 2187
(d) (e) 27
156 250
2. (a) 2640 (b) 4365 (c) 240
7. (a) 3 219 (b) 719 (c) 1.0420
3. (a) 2050 (b) 2575
ANSWERS 401

1 1 19 1 3 20 1 4 5
(d) d n = 21 (e) d n (h) 5 (i) 1 (j)
4 2 2 4 3 5 6
2 3 7 1
5. a = 4 6. r = 7. a = 5 8. r = 9. r =
8. 1149 9. 6th term 5 5 8 4
2 2 1
10. r= 11. a = 3, r = and a = 6, r =
10. 5th term 11. No 3 3 3
1 3
12. a = 192, r = , LS = 153
12. 7th term 13. 11th term 4 5
2 2 3
13. a = 1, r = , LS = 3, a = 1, r = , LS =
14. 9th term 15. n = 5 16. r = 3 3 3 4
3
14. a = 150, r = , LS = 375
17. (a) r = 6 5
(b) 18 2 2 1 2 3
15. a = , r = , LS = 1 16. a = 3, r = and a = 2, r =
5 3 5 5 5
1 2 21 2 3
18. a = , r = 2 19. n = 7 20. 208 17. x= 18. (a) 1 1 k 1 1 (b) (c) k =
10 7 32 5 4
1 1 5 1
19. (a) 1 p 1 (b) (c) p =
Exercises 7.7 2 2 7 14
a a ^ 1 r nh
1. (a) 2 097 150 (b) 7 324 218 20. LS Sn =
1r 1r
2. (a) 720 600 (b) 26 240 a a ^ 1 r nh
=
32 769 1r
3. (a) 131 068 (b)
65 536 a a + ar n
=
1r
55
4. (a) 7812 (b) 35 ar n
64 =
1r
(c) 8403 (d) 273 (e) 255
364
5. (a) 255 (b) (c) 97 656.2
729 Exercises 7.9
127
(d) 1 (e) 87 376 1. (a) 210 (b) 13th (c) 57
128
2. (a) 39 (b) 29th (c) 32
6. (a) 1792 (b) 3577
3. (a) 3n + 3
7. 148.58 8. 133.33
n
(b) Sn = [2a + (n 1) d ]
9. n=9 10. 10 terms 2
1
= n [2 6 + (n 1) 3]
11. a = 9 12. 10 terms 2
1
= n (12 + 3n 3)
13. (a) $33 502.39 (b) $178 550.21 2
n ( 5) n 1 1 ( 5) n 1
14. (a) / 2 ( 5) k 1 (b) Sn = = = n (3n + 9)
2
k =1 3 3
3
15. 2146 = n (n + 3)
2

Puzzles 4. (a) (i) $23 200 (ii) $26 912 (iii) $31 217.92
(b) $102 345.29 (c) 6.2 years
1. Choice 1 gives $465.00. Choice 2 gives
$10 737 418.23! 5. (a) (i) 93% (ii) 86.49% (iii) 80.44%
2. 382 apples (b) 33.67% (c) 19 weeks
Exercises 7.8
6. (a) 0.01 m (b) 91.5 m
1 4
1. (a) Yes LS = 13 (b) No (c) Yes LS = 12 (d) No 7. (a) 49 (b) 4 mm
2 5
25
(e) Yes LS = 3 (f) Yes LS = (g) No 8. (a) 3k m (b) k (3k + 3) m (c) 9
32
5 3
(h) Yes LS = 1 (i) No (j) Yes LS = 1 9. (a) 96.04% (b) 34 (c) 68.6
22 7
2 2 7
2. (a) 80 (b) 426 (c) 66 (d) 12 (e) (f) 54 10. (a) 77.4% (b) 13.5 (c) 31.4
3 3 10
2 9 16
(g) 10 (h) (i) 48 (j) 4 7 2
7 20 39 11. (a) (b) (c) 1
7 4 1 1 3645 9 9 9
3. (a) (b) (c) (d) (e) 25 9 7
12 27 12500 64 4096 (d) (e) 2 (f)
1 2 1 1 2 99 11 30
4. (a) 1 (b) (c) (d) 2 (e) 3 (f) 5 (g)
4 5 48 2 5
402 Maths In Focus Mathematics HSC Course

43 7 131 361 23. No, shortfall of $2013.75


(g) 1 (h) 1 (i) (j) 2
90 450 990 999
24. (a) $14 281.87 (b) $9571.96
12. 0.625 m 13. 15 m 14. 20 cm 15. 3 m (c) No, they will only have $23 853.83.

16. (a) 74.7 cm (b) 75 m 25. $813.16

17. (a) 4.84 m (b) After 3 years


Exercises 7.12
18. 300 cm 19. 3.5 m 20. 32 m 1. $1047.62 2. $394.46 3. $139.15

21. (a) 1, 8, 64, (b) 16 777 216 people 4. (a) $966.45 (b) $1265.79
(c) 19 173 961 people
5. $2519.59
Exercises 7.10
1. (a) $740.12 (b) $14 753.64 (c) $17 271.40 6. (a) $592.00 (b) $39 319.89
(d) $9385.69 (e) $5298.19
7. (a) $77.81 (b) $2645.42
2. (a) $2007.34 (b) $2015.87 (c) $2020.28
8. $78 700
3. (a) $4930.86 (b) $4941.03
9. (a) Get Rich $949.61, Capital Bank $491.27
4. $408.24 5. $971.40 (b) $33 427.80 more through Capital Bank

6. $1733.99 7. $3097.06 10. $43 778.80 11. $61 292.20

8. $22 800.81 9. $691.41 12. NSW Bank $175.49 a month ($5791.25 altogether)
Sydney Bank $154.39 a month ($5557.88 altogether)
10. $1776.58 11. $14 549.76 12. $1 301 694.62 Sydney Bank is better

13. (a) $4113.51 (b) $555.32 13. (a) $249.69


(c) $9872.43 (d) $238.17 (e) $10 530.59 (b) $13 485.12

14. $4543.28 15. 4 years 16. 8 years 14. (a) $13 251.13 (b) $374.07 (c) $20 199.77

17. (a) x = 7 (b) x = 5 (c) x = 8 15. (a) $1835.68 (b) $9178.41


(d) x = 6.5 (e) x = 8.5

18. $7.68 19. Kate $224.37 Test yourself 7


1. (a) Tn = 4n + 5 (b) Tn = 14 7n
20. Account A $844.94 n 1 1 n 1
(c) Tn = 2:3 (d) Tn = 200 d n
4
(e) Tn = ] 2 gn
Exercises 7.11
1. $27 882.27 2. $83 712.95 2. (a) 2 (b) 1185 (c) 1183
(d) T15 = S15 S14
3. $50 402.00 4. $163 907.81 S15 = S14 + T15
5. $40 728.17 6. $29 439.16 (e) n = 16
13
7. $67 596.72 8. $62 873.34 3. (a) 11 125 (b) 1
140
9. $164 155.56 (28 years) 10. $106 379.70 (c) 3 985 785 (d) 34 750
1
(e)
11. $3383.22 12. $65 903.97 2

13. $2846.82 14. $13 601.02 4. (a) Each slat rises 3 mm so the bottom one rises up
30 3 mm or 90 mm.
15. $6181.13 16. $4646.71 17. $20 405.74 (b) 87 mm
(c) 90, 87, 84, . . . which is an arithmetic sequence
18. (a) $26 361.59 (b) $46 551.94
with a = 90, d = 3
19. $45 599.17 (d) 42 mm (e) 1395 mm

5. $3400.01
20. (a) $7335.93 (b) $1467.18
6. (a) (i) (b) (ii) (c) (i) (d) (iii) (e) (i) (f) (ii)
21. $500 for 30 years 22. Yes, $259.80 over (g) (ii) (h) (i) (i) (i) (j) (i)
ANSWERS 403

7. n = 108 15. (a) cosec2 x


(b) 1 cos x 1
8. (a) $24 050 (b) $220 250 So 0 cos2 x 1
| cos2 x | 1
9. a = 33, d = 13 So the limiting sum exists.
10. (a) 59 (b) 80 (c) 18th term 16. $240 652.62.
11. (a) x = 25 (b) x = 15
4 13 19 Chapter 8: Probability
12. (a) (b) (c) 1
9 18 33
Exercises 8.1
13. x = 3 1 1 1 1 1
1. 2. 3. 4. 5.
30 52 6 40 20 000
14. (a) 136 (b) 44 (c) 6
4 3 3 1 11 3
1 6. (a) (b) 7. 8. 9. (a) (b)
15. 121 16. $8066.42 7 7 37 12 20 4
2
17. (a) Tn = 4n + 1 (b) Tn = 1.07n 1 1 1 1
10. (a) (b) (c)
1 1 6 2 3
18. (a) 1 < x < 1 (b) 2 (c) x =
2 3 1 3 1 99
11. (a) (b) (c) (d)
62 31 2 124
19. d = 5
8 7 1 1 1 23
20. (a) 39 words/min (b) 15 weeks 12. (a) (b) (c) 13. 14. , 1 15.
15 15 15 50 2 44
21. (a) $59 000 (b) $15 988.89
7 7 12 1 25
2 16. (a) (b) (c) 17. 18. 8 19.
22. 4.8 m 23. x = ,2 31 31 31 175 43
17
1 1 1 5
20. 34 21. 22. (a) (b) (c)
24. (a) $2385.04 (b) $2392.03 3 6 3 6

25. 1300 23. (a) False: outcomes are not equally likely. Each horse
and rider has different skills.
26. (a) 735 (b) 4315 (b) False: outcomes are not equally likely. Each golfer
has different skills.
27. (a) $1432.86 (b) $343 886.91 (c) False: outcomes are not dependent on the one before.
Each time the coin is tossed, the probability is the same.
28. n = 20
(d) False: outcomes are not dependent on the one
29. n = 11 before. Each birth has the same probability of producing
a girl or boy.
(e) False: outcomes are not equally likely. Each car and
Challenge exercise 7
driver has different skills.
1. (a) 8.1 (b) 19th term
Exercises 8.2
9 33
2. (a) (b) (c) 5 2
4 4 4 1. 2. 3. 99.8% 4. 0.73 5. 38%
11 9
3. (a) 2 097 170 (b) -698 775
22 5
6. 98.5% 7. 8. 9. 0.21 10. 91.9%
4. (a) $40 (b) $2880 23 18
7 46 2 13 7 15
5. 6th term 6. 17 823 11. 12. 13. (a) (b) 14. 15.
8 49 15 15 11 16

7. 5 terms 8. n = 1, 2 , 3
Exercises 8.3
9. 56 10. $1799.79 3 3 11 7
1. (a) (b) (c) (d)
10 5 20 10
3
11. x = 12. $8522.53 13. k = 20
8
1 1 3 3 19
2. (a) (b) (c) (d) (e)
14. (a) $10 100 (b) $11 268.25 5 2 5 5 50
(c) $4212.41 (d) $2637.23
404 Maths In Focus Mathematics HSC Course

5 9 12 29 13 9 1 1 64 152 13
3. (a) (b) (c) 4. (a) (b) (c) 12. (a) (b) (c) (d) (e)
26 26 13 100 20 25 25 825 825 165 165

27 4 2 3 13 9 19 498 1 239 771


5. (a) (b) (c) 6. (a) (b) (c) 13. (a) (b) (c)
45 9 3 14 28 28 1 249 750 124 975 1 249 750

16 38 1936 88
21 17 21 1 11 7 14. (a) (b) 15. (a) (b)
7. (a) (b) (c) 8. (a) (b) (c) 75 75 2025 2025
80 80 40 10 20 20
11 3 1 125 671
7 2 44 3 2 3 16. (a) (b) 17. (a) (b) (c)
9. (a) (b) (c) 10. (a) (b) (c) 20 20 1296 324 1296
25 15 75 10 5 10
84 681 912 673 27
18. (a) (b) (c)
1 000 000 1 000 000 1 000 000

19. (a) 17.6% (b) 11% (c) 21.2%


Exercises 8.4
1488 1 1 6 21 17
1 1 1 1 25 20. (a) (b) 21. (a) (b) (c) (d)
1. 2. 3. 4. 5. 3025 121 19 95 190 38
36 4 8 4 121
22 368 7 17 133 496
22. (a) (b) (c) 23. (a) (b) (c)
6. (a) 0.0441 (b) 0.6241 7. 80.4% 8. 32.9% 425 425 425 65 715 2145

9 15 3 19 1 24. (a) 0.23 (b) 0.42 (c) 0.995 25. (a) 33% (b) 94%
9. (a) (b) 10. 11. 12.
49 91 2075 99 16 170
1 5 91 1 3 2
26. (a) (b) (c) 27. (a) (b) (c)
29 791 8 35 929 216 72 216 10 10 5
13. (a) (b) (c)
35 937 35 937 35 937
25 40 56
28. (a) (b) (c)
1 1 5 755 201 81 81 81
14. (a) (b) (c)
2400 5 760 000 5 760 000
343 336 988
29. (a) (b) (c)
1 3125 4651 1331 1331 1331
15. (a) (b) (c)
7776 7776 7776
1 6859 1141
30. (a) (b) (c)
9 24 970 009 29 991 8000 8000 8000
16. (a) (b) (c)
25 000 000 25 000 000 25 000 000
Test yourself 8
1 9 9
17. (a) (b) (c)
4 100 100 l. (a) 80.4% (b) 1.4% (c) 99.97%

1 1 7 15 2. (a) 1 2 3 4 5 6
18. (a) (b) (c) (d)
22 11 22 22
1 0 1 2 3 4 5
19. (a) 61.41% (b) 0.34% (c) 99.66% 2 1 0 1 2 3 4

1 1 1 2 1 n 3 2 1 0 1 2 3
20. (a) (b) (c) 1 =
2n 2n 2n 2n 4 3 2 1 0 1 2
5 4 3 2 1 0 1
Exercises 8.5 6 5 4 3 2 1 0

1 1 1 1 3 7
1. (a) (b) (c) 2. (a) (b) (c) 1 1 1 1 39
4 4 2 8 8 8 (b) (i) (ii) (iii) 3. (a) (i) (ii)
6 6 2 40 40
1 1 1 1 2 39 4 1
3. (a) (b) (c) 4. (a) (b) (b) 4. (a) (b)
900 900 450 25 25 796 15 10

25 80 5. False: the events are independent and there is the


5. (a) (b) 6. (a) 27.5% (b) 23.9% (c) 72.5%
169 169 1
same chance next time d n
189 441 4
7. (a) 0.42 (b) 0.09 (c) 0.49 8. (a) (b)
1000 1000
657 1 29 1 11 16
(c) 9. (a) 0.325 (b) 0.0034 (c) 0.997 6. (a) (b) (c) (d) (e)
1000 2 100 5 25 25

2 7 35 2 35 1
60 6 4 1 7. (a) (b) (c) (b) 9.8. (a)
10. (a) (b) 11. (a) (b) 5 15 72 15 66 56
121 11 27 6
1 3 5
10. (a) 0.009% (b) 12.9% 11. (a) (b) (c)
13 13 26
ANSWERS 405

5 1 15. (a)
12. (a) (b)
12 3
9 3 27 4
13. (a) (b) (i) (ii) (iii)
40 10 160 25
1 81 11 1 4
14. (a) (b) (c) 15. (a) (b)
200 200 100 15 5

1 147 1 3577
16. (a) (b) (c) (d)
50 7450 3725 3725
d2 = (230 65t) 2 + (125 80t) 2 (Pythagoras)
80 40 2 1
17. (a) (b) 18. (a) (b) = 52 900 29 900t + 4225t2 + 15 625 20 000t + 6400t2
361 171 9 3
= 10 625t2 49 900t + 68 525
64 728 21 3 23 (b) 2.3 h (c) 109.7 km
19. (a) (b) 20. (a) (b) (c)
243 729 50 25 50
1
16. 199; 5050 17.
110
18. (a) T1 = 4, T2 = 11, T3 = 18, T12 = 81
Challenge exercise 8
(b) 1410 (c) 29th term
1 4
1. (a) (b) 2. (a) 0.04 (b) 0.75 (c) 0.25
7 7
1
19.
1 33 4 25 4 2
3. (a) (b) 4. (a) (b) (c)
54 145 173 264 13 52 13

5. Noany combination of numbers is equally likely to win. 20. (a)

1 3 1 1
6. (a) 0 (b) (c) 7. (a) (b)
10 10 7776 1296

3 12 1 5 7 3
8. (a) (b) 9. (a) (b) (c) (d)
10 145 144 144 144 144

Practice assessment task set 3

324 1296 864 1105 (b)


1. 2. (a) (b) (c)
625 2401 2401 2401
4 3
3. 4. 5. $2929.08 6. 104 + 52 26 +
9 49
7. 44th term 8. $945
1 1 11 5 5
9. (a) (b) (c) (d) (e)
36 6 36 36 12
9841a
10. 11. 2.4 m
6561
(c)
12. (a) 3 000 000 (b) 3 000 336 (c) 146 insects per day

13. (2, 0), inflexion

5 35
21. 15 4 + 7 + . . . 22. (a) (b) 23. $2851.52
33 66
24. (a) v = 12 sin 4t (b) a = 48 cos 4t (c) 3 cm
3 5
7 11 (d) t = 0, , , . . . s (e) 3 cm (f) t = , , ,... s
14. (a) (b) 4 2 8 8 8
50 20
(g) a = 48 cos 4t
= 16 (3 cos 4t)
= 16x
406 Maths In Focus Mathematics HSC Course

8 1 1 39. (a) Square 46.3 m 46.3 m, rectangle 30.9 m 92.7 m


25. 26. (a) (b) 27. $180.76
45 12 4 (b) $8626.38
5 5 1 7 1
28. AC2 = 162 = 256 40. (a) (b) (c) (d) (e)
36 12 6 36 2
AB + BC2 = 9.62 + 12.82
2

= 256 41.
Since AC2 = AB2 + BC2,
ABC is right angled at +B.

5
29. x =
6

30.

42. (a) = 76 52l (b) 0.92 cm2 43. $18 399.86

44. (a) log 3 + log 9 + log 27 + . . .


= log 3 + log 32 + log 33 + . . .
Let ABCD be a rhombus with AB = BC. = log 3 + 2 log 3 + 3 log 3 + . . .
AB = DC and AD = BC Arithmetic series, since
(opposite sides in a parallelogram) 2 log 3 log 3 = 3 log 3 2 log 3
` AB = BC = DC = AD = log 3
` all sides are equal (b) 210 log 3
1
31. 45.
40 000

32. (a) k Z 0.025 (b) after 42.4 years (c) 20.6 years

33. 76 473 34. 450 cm2

35. (a) 12 ms 1 (b) 48e4 ms 2


(c) x = 3e4t + 2
.
x = 12e4t
..
x = 48e4t
= 4 (12e4t )
. 46. (a) 12.6 mL (b) 30 minutes 47. (a)
= 4x
(d) 48. $277.33 49. (d) 50. (b) 51. (c) 52. (c), (d)

53. (d) 54. (a) 55. (c) 56. (a) 57. (d)

Sample examination papers

MathematicsPaper 1
1. (a) 0.75
(b) (3x 2) (x 3)
6 x 6 x 1
36. n = 4 (c) d n d n = 6 (5)
1 2 1 3
3x 2(x 1) = 30
37. y = sin 7x 3x 2x + 2 = 30
dy x + 2 = 30
= 7 cos 7x
dx x = 28
d2 y 1 2
= 7 (7 sin 7x) (d) 12 = r
dx2 3
= 49 sin 7x 36 = r2
= 49y 36
= r2

1 36
38. =r
10
3.39 = r
ANSWERS 407

x2 1 x x2
(e) 3300 (b) (i) e +C = + ex + C
2 1 2
(f) x+3<7

x <4 (ii) ; cos + E = ] cos + g ] cos 0 + 0 g
or ] x + 3 g < 7 0

x + 3 > 7 = ] 1 g + + 1
=2+
x > 10
` 10 < x < 4 5 2+1 5 2+5
(c) (i) =
21 2+1 ( 2 ) 2 12
2. (a) (ii) Since AB = BD, AB: AD = 1:2 5 2+5
=
+ABC = +ADE (corresponding +s, BC < DE) 1
+ACB = +AED (similarly) =5 2 +5
+A is common (ii) 5 2 + 5 = 5 + 5 2
` ABC <; ADE = 5 + 25 2
AB AC 1 = 5 + 50
` = =
AD AE 2 ` a = 5, b = 50
` AE = 2AC 4. (a) (i)
AC + CE = 2AC
` CE = AC
AB BC 1
(iii) = =
AD DE 2
3.4 1
` =
DE 2
DE = 2 3.4
= 6.8 cm

(b) (ii) +NOM = 105 (straight angle)


+NMO = 32 (angle sum of T)
a b (ii) Substitute A(3, 2) into 3x + 4y 17 = 0
=
sin A sin B 3(3) + 4(2) 17 = 0
MO 5 9 + 8 17 = 0
=
sin 43 sin 32 0 = 0 (true)
5 sin 43
MO = ` A lies on the line
sin 32
Substitute B(1, 5) into 3x + 4y 17 = 0
Z 6.4 m
3(1) + 4(5) 17 = 0
a b
(iii) = 3 + 20 17 = 0
sin A sin B
0 = 0 (true)
MP 6.4
= ` B lies on the line
sin 75 sin 53
6.4 sin 75 Since both A and B lie on the line 3x + 4y 17 = 0,
MP = this is the equation of AB
sin 53
Z 8m | ax1 + by1 + c |
1
(iii) d =
3. (a) (i) x + 5x3 + 1 = x + 5x3 + 1
2 a2 + b2
dy 1 1 | 3 (0) + 4 (0) 17 |
1 =
= x 2 + 15x2 = + 15x2 32 + 42
dx 2 2 x
| 17 |
1 =
(ii) 3 ln x + = 3 ln x + x 1
x 9 + 16
dy 1 17
= 3 x 2 =
dx x 25
3 1 17
= 2 =
x x 5
3x 1 = 3.4 units
=
x2 (iv) Length AB: d = (x2 x1)2 + (y2 y1)2
dy
(iii) = 5] 2x + 3 g4 2 = 6 3 (1) @ 2 + (2 5)2
dx = 16 + 9
= 10] 2x + 3 g4 = 25
=5
408 Maths In Focus Mathematics HSC Course

1 6. (a)
Area OAB: A = bh
2
1
= 5 3.4
2
= 8.5 units2
(b) c2 = a2 + b2 2ab cos C
BC2 = 62 + 42 2 ] 6 g ] 4 g cos 87
Z 49.49
BC = 49.49
Z 7 cm
800
5. (a) (i) 100% = 3.25%
24 600
(ii) Arithmetic series with a = 24 600, d = 800 and
n = 12
Tn = a + ] n 1 g d
T12 = 24 600 + ] 12 1 g 800
= 33 400
So Kate earns $33 400 in the 12th year.
n
(iii) Sn = [2a + (n 1)d ] (i) P (2W, 1N) = P(WWN) + P(WNW)
2
12 + P(NWW)
= [2 24 600 + (12 1)800] 2 2 5 2 5 2
2 = +
= 348 000 7 7 7 7 7 7
5 2 2
So Kate earns $348 000 over the 12 years. +
7 7 7
(b) (i) y = x3 3x2 9x + 2 60
y l = 3x 2 6 x 9 =
343
y m = 6x 6
(ii) P (at least one W) = 1 P(NNN)
For stationary points, yl = 0
5 5 5
i.e. 3x2 6x 9 = 0 = 1
7 7 7
3(x 3)(x + 1) = 0 218
` x = 3, 1 =
343
When x = 3, y = 33 3] 3 g2 9] 3 g + 2
(b) x = 60, 180 60 (first, second quadrants)
= 25
= 60, 120
When x = 1, y = ] 1 g3 3 ] 1 g2 9 ] 1 g + 2
=7 (c) (i) v = # (6t + 4) dt
So (3, 25) and (1, 7) are stationary points. = 3t 2 + 4 t + C
At (3, 25), y m = 6(3) 6 When t = 0, v = 0
>0 (minimum point) ` 0 = 3] 0 g2 + 4 ] 0 g + C
At (1, 7), y m = 6(1) 6 =C
<0 (maximum point) ` v = 3t 2 + 4t
` (1, 7) is a maximum, (3, 25) minimum When t = 5,
stationary point v = 3 ] 5 g2 + 4 ] 5 g
(ii) For inflexions, y m = 0 = 95 cms 1
i.e. 6x 6 = 0 (ii) x = # (3t2 + 4t) dt
6x = 6
= t 3 + 2t 2 + C
x =1
When t = 0, x = 0
When x = 1, y = 13 3(1) 2 9(1) + 2
` 0 = ] 0 g3 + 2 ] 0 g2 + C
= 9
=C
` (1, 9) is a point of inflexion ` x = t 3 + 2t 2
(iii) When t = 2,
` x = ] 2 g3 + 2 ] 2 g2
= 16 cm

7. (a) (ii) DC = AB = 2 (opposite sides of < gram)


1
` DX = AD = 1 c DX = DC given m
2
` ADX is isosceles
+DAX = +DXA = (180c 60c ) ' 2
= 60c
` ADX is equilateral
ANSWERS 409

dP
(iii) +XCB = 180c 60c (ii) The curve is increasing so> 0.
dt
(+ADC, +XCB cointerior +s, AD < BC) d2 P
The curve is concave downwards so 2 < 0.
= 120c dt
CXB is isosceles
[XC = CB = 1, similar to part (ii)] 8. (a) (i) x 1 2 3 4 5
` +CXB = +CBX = (180c 120c ) 2 y 0 0.301 0.477 0.602 0.699
= 30c
# f (x) dx = 12 (b a) [f (a) + f (b)]
b
+AXB = 180c (60c + 30c ) (ii)
(+DXC straight angle) a

# log x dx = 12 (2 1) 6 f (1) + f (2) @


5
= 90c 10
` AXB is right angled 1
1
(iv) AX = 1 ( ADX equilateral) + (3 2) 6 f (2) + f (3) @
2
` c2 = a2 + b2 1
22 = 12 + BX2 + (4 3) [ f (3) + f (4)]
2
4 = 1 + BX2 1
+ (5 4) [ f (4) + f (5)]
3 = BX2 2
3 = BX 1 1
= (log 1 + log 2) + (log 2 + log 3)
2 2
(b) Sketch y = x2 and x + y = 3 as unbroken lines. 1 1
+ (log 3 + log 4) + (log 4 + log 5)
2 2
Z 1.73
(b) (i) When t = 0, P = 20
20 = P0 e0
= P0
` P = 20ekt
When t = 6, P = 100
100 = 20ek (6)
100
= e 6k
20
5 = e 6k
ln 5 = ln e6k
Substitute (1, 0) into y x2 = 6k ln e
0 12 (false) = 6k
Substitute (1, 0) into x + y 3 ln 5
=k
1+03 (true) 6
0.268 Z k
(ii) P = 20e0.268t
When t = 10,
P = 20e0.268 (10)
= 20e2.68
= 292 mice
(iii) When P = 500,
500 = 20e0.268t
500
= e0.268t
20
25 = e0.268t
ln 25 = ln e0.268t
(c) (i) = 0.268t ln e
= 0.268t
ln 25
=t
0.268
12 = t (i.e. after 12 weeks)

9. (a) (i) S = 160


` 2 r (r + h) = 160
160
r+h =
2 r
160
h= r
2 r
410 Maths In Focus Mathematics HSC Course

80 (b) (i) For real, equal roots, = 0


= r
r i.e. b2 4ac = 0
V = r h
2
] k 1 g2 4 ] 1 g ] k g = 0
80 k 2 2k + 1 4 k = 0
= r2 c rm
r k 2 6k + 1 = 0
= 80r r 3
b b2 4ac
(ii) Vl = 80 3r2 k=
2a
For max./min. volume, Vl = 0
( 6) ( 6) 2 4 (1) (1)
i.e. 80 3 r2 = 0 =
2 (1)
80 = 3 r2
80 6 32
= r2 =
3 2
80 64 2
=r =
3 2
2.91 Z r =32 2
V m = 6 r (ii) When k = 5,
When r = 2.91, V m = 6 (2.91) x2 + (k 1) x + k = x2 + 4x + 5
<0 (maximum) a >0
` r = 2.91 cm = b2 4ac
= 42 4 (1) (5)
(iii) When r = 2.91, V = 80 (2.91) ] 2.91 g3 = 4
= 206.4 cm3 ` <0
(b) 1996 to 2025 inclusive is 30 years. Since a > 0 and < 0, x2 + 4x + 5 > 0
A = 500 (1.1230) + 500 (1.1229) + 500 (1.1228) + . . . for all x
+ 500 (1.121)
= 500 (1.1230 + 1.1229 + . . . + 1.121) (c) (i) y = x2 + 2px + q
= 500 (1.121 + 1.122 + . . . + 1.1230) y q = x2 + 2px
y q + p2 = x2 + 2px + p2
1.121 + 1.122 + . . . + 1.1230 is a geometric series
y (q p2) = ^ x + p h2
a = 1.12, r = 1.12
a (rn 1) This is in the form (x h)2 = 4a (y k), where a is
Sn = the focal length and (h, k) is the vertex. h = p and
r1
1.12 (1.1230 1) k = q p2
S30 = ` vertex is ( p, q p2)
1.12 1
Z 270.29 (ii) 4a = 1
` A = 500 (270.29) 1
`a =
= $135 146.30 4
10. (a) (i) Substitute y = 4 into y = ex
4 = ex
` ln 4 = x (by definition of log)
` point of intersection is (ln 4, 4)
(ii)

1
Count up units for the focus
4
1
` focus is c p, q p2 + m
#
ln 4
4
A = Area of rectangle OABC e x dx
0 (iii) For P: x = m since it is vertically below
ln 4
= 4 ln 4 6 e x @ 0 ^ m, 3m2 + q h
= 4 ln 4 (e ln4 e 0) When x = m
= 4 ln 4 4 + 1 m 2 = 8y
= (4 ln 4 3) units 2 m2
=y
8
m2
So P = d m, n
8
ANSWERS 411

m2 (d) a2 (a 2) 4 (a 2)
Distance = 3m2 + q
8 = (a 2) (a2 4)
23m2 = (a 2) (a + 2) (a 2)
= +q
8 = ] a + 2 g ] a 2 g2
23m 2
= +q since m2 0 and q > 0 (e) 2 4 6 25 6
8
=22 6 5 6
(iv) m + q = 5
=4 6 5 6
` q =5m
= 6
23m2
D= +q
8 (f) loga 50 = loga (52 2)
23m 2
= loga 52 + loga 2
= +5m
8 = 2 loga 5 + loga 2
dD 46m = 2 1.3 + 0.43
= 1
dm 8 = 3.03
dD
For stationary points =0
dm x1 + x2 y1 + y2
46m (g) P = f , p
1=0 2 2
8
3 + 0 4 + 2
46m =d , n
=1 2 2
8
1
46m = 8 = c 1 , 1 m
2
8
m=
46 2.
4
=
23
4
So there is a stationary point at m = .
23
To determine its nature
d2 D 46
=
dm2 8
>0
So concave upwards.
` minimum turning point
4
When m =
23
23m2
D= +5m
8 1
4 2 (a) Substitute A c 5, 1 m into 3x + 4y 9 = 0
23 d n 2
23 4
= +5 1
8 23 LHS = 3 5 + 4 1 9
2
21 =0
=4
23 = RHS
21
So minimum distance is 4 units. ` A lies on the line
23
1
Substitute B c 1, 1 m into 3x + 4y 9 = 0
2
MathematicsPaper 2 1
LHS = 3 1 + 4 1 9
2
1. (a) (i) x 3 = 5
=0
x=8
= RHS
(ii) x 3 = 5
` B lies on the line
x = 2
` AB has equation 3x + 4y 9 = 0
(b) 5 x2 = 4 (b) 3x + 4y 9 = 0
9 x2 = 0 4y = 3x + 9
9 = x2 3 9
3 = x y= x+
4 4
5 3
(c) sin ` m1 =
6 4

= sin c m l is perpendicular to AB, so m1 m2 = 1
6

= sin (2nd quadrant)
6
1
=
2
412 Maths In Focus Mathematics HSC Course

3 y1 + y2
m = 1 y=
4 2 2
4 1
` m2 = y +1
3 1 2
Equation of l: 1 =
4 2
y y1 = m (x x1) 1 1
2 = y + 1
4 2 2
y 1 = (x 4)
3 4 = y
3y + 3 = 4 (x + 4) (f) So D = (0, 4)
= 4x + 16
dy
0 = 4x 3y + 13 3. (a) (i) = ulv + vlu
dx
(c) 4x 3y + 13 = 0 (1)
= 1$ cos x + (sin x) x
3x + 4y 9 = 0 (2)
= cos x x sin x
(1) 4: 16x 12y + 52 = 0 (3)
dy
(2) 3: 9x + 12y 27 = 0 (4) (ii) = 5e 5 x
(3) + (4): 25x + 25 = 0 dx
25x = 25 dy 4x
x = 1 (iii) =
dx 2x 2 1
Substitute x = 1 in (1): (3x 2) 5
4 1 3y + 13 = 0 (b) (i) +C
35
9 3y = 0 (3x 2) 5

9 = 3y = +C
15
3=y 1
So point of intersection is ( 1, 3). (ii) 3 cos 2x + C
2
(d) d = (x2 x1) 2 + (y2 y1) 2 3
= cos 2x + C
AB: 2

1 1 2 1 x 3
d= (5 1) 2 + c 1 1 m (c) < ex e F
2 2 1 0

= 42 + ( 3) 2 = 6 ex + e x @ 03
= 16 + 9 = [e3 + e 3] [e0 + e 0]
= 25 = e 3 + e 3 1 1
=5 = e 3 + e 3 2
CP: dy
(d) = # (18x 6) dx
d = (1 4) 2 + (3 1) 2 dx
= 32 + 42 = 9x 2 6 x + C
= 9 + 16 dy
At (2, 1), =0
= 25 dx
=5 0 = 9 ] 2 g2 6 ] 2 g + C
1 = 24 + C
A = bh
2 24 = C
1 dy
= 55
2 ` = 9x2 6x 24
dx
= 12.5 units2
y = # (9x2 6x 24) dx
x1 + x2 y1 + y2
(e) Midpoint AC = f , p = 3x3 3x2 24x + C
2 2
Substitute (2, 1):
1
1 + 1 1 = 3 ] 2 g3 3 ] 2 g2 24 ] 2 g + C
=
f 4 + 5, 2 p
2 2 = 36 + C
1 1 35 = C
= c , 1 m ` y = 3x3 3x2 24x + 35
2 4
Midpoint AC = midpoint BD
where D = (x, y) 4. (a)
x1 + x2
x=
2
1 x +1
=
2 2
1= x +1
0=x
ANSWERS 413

2 3 (c) (i)
(i) P (WW) =
5 8
3
=
20
2 5 3 3
(ii) P (WL) + P (LW) = +
5 8 5 8
19
=
40
(iii) P (at least 1W) = 1 P (LL)
3 5
=1
5 8
(ii) 1 < x < 2
5
=
8 5. (a) y = 2x3 9x2 + 12x 7
(b) (i) dy
= 6x2 18x + 12
dx
d2 y
= 12x 18
dx2
dy
(i) For stationary points, =0
dx
6x 18x + 12 = 0
2

x 2 3x + 2 = 0
(x 2) (x 1) = 0
x = 1, 2
1 When x = 1, y = 2 ] 1 g3 9 ] 1 g2 + 12 (1) 7 = 2
(ii) A= bh
2 When x = 2, y = 2 ] 2 g3 9 ] 2 g2 + 12 (2) 7 = 3
1
= 55 So (1, 2) and (2, 3) are stationary points.
2
= 12.5 units2 d2 y
At (1, 2) = 12 (1) 18 = 6
# 2 (x + 2) dx
3
or A = dx2
3
` (1, 2) is a maximum turning point
x2
=< + 2x F d2 y
2 2 At (2, 3) 2 = 12 (2) 18 = 6
( 2) 2 dx
32
=< + 2 (3) F = + 2 ( 2) G ` (2, 3) is a minimum turning point
2 2
d2 y
= 12.5 units2 (ii) For points of inflexion 2 = 0
(iii) y=x+2 dx
12x 18 = 0
` y2 = ] x + 2 g2
12x = 18
V = # y2 dx
b

a x = 1.5
= # (x + 2) 2 dx
3
When x = 1.5,
2
y = 2 (1.5) 3 9 (1.5) 2 + 12 (1.5) 7
(x + 2) 3 3

= = G = 2.5
13 2

(3 + 2) 3 ( 2 + 2) 3 Check concavity:
= = G
3 3 x 1.25 1.5 1.75
125
= < 0F
3 d2 y
125 3 0 3
= units3 dx2
3
1 2 Concavity changes, so (1.5, 2.5) is a point of
or V= r h
3 inflexion.
1 (iii) When x = 3,
= ] 5 g2 5
3 y = 2 ] 3 g3 9 ] 3 g2 + 12 ] 3 g 7
125 = 178
= units3
3 When x = 3,
y = 2 ] 3 g3 9 ] 3 g2 + 12 ] 3 g 7
=2
414 Maths In Focus Mathematics HSC Course

1
Substitute x = 2 in (2):
4
1 2
y = d2 n
4
1
=5
16
1 1
` Q = c2 , 5 m
4 16
(iii)

(b) (i)

(ii)
PQ: x 4y + 18 = 0
x + 18 = 4y
x 18
+ =y
4 4
1
2
c2 = a2 + b2 2ab cos C x 18
Area = #
4
c + x2 m dx
= 8502 + 12002 2 850 1200 cos 120 2 4 4
= 3 182 500 x2 18x x3 2
1
4

c = 3182500 =< + F
8 4 3 2
= 1784 R 2 V
S c2 1 m 1 1 3
18 c 2 m c 2 m W
So the plane is 1784 km from the airport. S 4 4 4 W
= SS + WW
(c) (cosec + cot ) (cosec cot ) 8 4 3
T X
= cosec2 cot2 ( 2) 2 18 ( 2) ( 2) 3
= + G
= 1 + cot2 cot2 8 4 3
=1 = 12.8 units2
dy
6. (a) (i) = 2x (b) (i) The particle is at the origin when x = 0,
dx i.e. at t1, t3 and t5
dy
At ( 2, 4) = 2 ( 2) dx
dx (ii) At rest, = 0 (at the stationary points,
dt
` m1 = 4 i.e. t2 and t4)
Normal is perpendicular to tangent (c) T = T0 e kt
` m1 m2 = 1 When t = 0, T = 97
4m 2 = 1 ` T0 = 97
1 T = 97 e kt
m2 =
4 When t = 5, T = 84
y y1 = m _ x x1 i 84 = 97 e k 5
1 84
y 4 = ]x 2g = e 5k
4 97
4y 16 = x + 2 84
ln = ln e 5k
0 = x 4y + 18 (1) 97
(ii) y = x 2
(2) = 5k ln e
Substitute (2) in (1): = 5k
0 = x 4x2 + 18 84
ln
4x2 x 18 = 0 97
=k
(x + 2) (4x 9) = 0 5
0.029 = k
x + 2 = 0, 4x 9 = 0
So T = 97 e 0.029t
x = 2, 4 x = 9
1 (i) When t = 15
x=2
4 T = 97 e 0.029 15
= 63
So the temperature is 63C after 15 minutes.
ANSWERS 415

(ii) When T = 20 (c) +CBE = 50 (base +s of isosceles )


20 = 97 e 0.029t +DCB = 50 + 50 (exterior + of CBE )
20 = 100
= e 0.029t
97 +ABC = 130 50
20 = 80
ln = ln e 0.029t
97 +DAB = 360 (100 + 80 + 80)
= 0.029t ln e (+ sum of quadrilateral )
= 0.029t = 100
20 ` +DAB = +DCB and +ABC = +ADC
ln
97 ` ABCD is a parallelogram (opposite +s equal )
=t
0.029
54.9 = t 2
8. (a) (i) & (ii) Amplitude = 3, period = =
So the temperature is 20C after 54.9 minutes. 2

7. (a) (i)

#
b
h (iii) 4 points of intersection, so 4 roots
f (x) dx Z
7 (y + y 4) + 4 (y 1 + y 3) + 2y 2 A
a 3 0 (b) 2 sin x 1 = 0
2 sin x = 1

# 4
tan x dx Z
16
3
; c tan 0 + tan m

4 sin x =
1
(1st, 2nd quadrants)
0 2
3 x = 30, 180 30
+ 4 c tan + tan m + 2 tan G
16 16 8 = 30, 150
Z 0.35 units 2
dy sin x (c) (i) logx 12 = logx ] 22 3 g
(ii) = = logx 22 + logx 3
dx cos x
= tan x = 2 logx 2 + logx 3
= 2q + p
(iii) #0
4
tan x dx = : ln (cos x) D 0
4
(ii) logx 2x = logx 2 + logx x
= q +1

= = ln c cos m G [ ln (cos 0)] (d) (i) 1 + 3 + 5 + . . . is an arithmetic series with
4
= 0.35 a = 1, d = 2
(b) (i) When n = 12
Tn = a + (n 1) d
T12 = 1 + (12 1) 2
= 23

a b So there are 23 oranges in the 12th row.


= (ii) Total number of oranges is 289, so Sn = 289
sin A sin B
n
AD 40 Sn = [2a + (n 1) d ]
= 2
sin 23 sin 110
n
40 sin 23 289 = [2 1 + (n 1) 2]
AD = 2
sin 110
578 = n [2 + 2n 2]
= 16.6 m
= n 2n
BD = 2n 2
(ii) sin 47 =
16.6 289 = n2
16.6 sin 47 = BD
289 = n
12.2 = BD
17 = n
So the height is 12.2 m So there are 17 rows of oranges altogether.
416 Maths In Focus Mathematics HSC Course

9. (a) (ii) y = ] x + 2 g2
` y2 = ] x + 2 g4
V = # y2 dx
b

= # (x + 2) 4 dx
1

(x + 2) 5 1

= = G
15 2

(1 + 2) 5 ( 2 + 2) 5
= = G
5 5
243
= < 0F
(b) The statement would only be true if there were equal 5
numbers of each colour. It is probably false. 243
= units3
(c) ln x2 = ln (2x + 3) 5
` x 2 = 2x + 3 d
(b) (i) s =
x 2 2x 3 = 0 t
(x 3) (x + 1) = 0 d
So t =
x = 3, 1 s
3000
But x 1 (ln 1 does not exist) =
s
so the solution is x = 3
Cost of trip taking t hours:
dy C = ] s2 + 7500 g t
(d) (i) = ex
dx 3000
= ] s2 + 7500 g
When x = k s
dy 7500 3000
= 3000s +
= ek s
dx
7500
= 3000 c s + m
So gradient m = ek s
(ii) When x = k, y = ek 7500
(ii) C = 3000 c s + m
y y1 = m (x x1) s
y ek = ek (x k) = 3000 s + 7500 s 1 g
]
= ek x kek dC
= 3000 (1 7500 s 2)
y = ek x kek + ek ds
= ek (x k + 1) 7500
= 3000 d 1 n
s2
(iii) Substitute (2, 0) into the equation
0 = ek (2 k + 1) dC
For minimum cost, =0
= ek (3 k) ds
3k=0 7500
3000 d 1 n=0
3=k s2
(e) 180 = radians 7500
1 =0
s2
1 = 7500
180 1=
s2
` 53 = 53 s2 = 7500
180
53 s = 7500 (speed is positive)
=
180 = 86.6 km/h
1 Check:
A = r2
2 d2C
1 53 = 3000 (15 000s 3)
= 72 ds2
2 180 15 000
= 22.7 cm2 = 3000 d n
s3
10. (a) (i) When s = 86.6
d2C 15 000
= 3000 d n
ds2 86.63
>0
Concave upwards
So minimum when s = 86.6
7500
(iii) C = 3000 d 86.6 + n
86.6
= 519 615 cents
= $5196.15

You might also like